Sei sulla pagina 1di 324

Dr.

Swamy PLAB Courses Ltd


PLAB 1 MOCK TEST: MOCK 1 TIME ALLOWED: 3HRS

SINGLE BEST ANSWER

1. A 22 year old patient presents with colicky pain which radiates from loin to groin. He complains of
similar episodes in the past. Investigation has been done and 7mm stone was found in the ureter. What is
the most appropriate management for this patient?

A. Per cutaneous nephron-lithotomy


B. Open surgery
C. Ureteroscopy or laser
D. Conservative treatment
E. Extracorporeal shock wave lithotripsy

2. The treatment of a woman with breast cancer will depend on which of the following factors?
A. Axillary nodes positive
B. Tumour grade
C. Type of tumour
D. Age of patient
E. General health of the patient

3. An 18 year old female has just received her A-Level exam results and she didn’t get into the University
of her choice. She was brought into the A&E department after ingestion of 24 paracetamol tablets. O/E:
patient is slightly confused and tired. All initial management has been done. Investigations done after 24
hours of admission showed normal FBC, ABG (pH 7.1), P.T (17 seconds), Bilirubin (4µmol/L) and
Creatinine (83µmol/L). What is the next step in management?

A. Keep under observation for the next 24 hours.


B. Refer to Psychologist.
C. Give N-Acetylcysteine.
D. Discharge with referral to Psychiatrist.
E. Arrange for liver transplantation

4. A 40 year old woman notices increasing lower abdominal distension with little or no pain. On
examination of the abdomen, a lobulated cystic mass is felt and it seems to be arising from the pelvis.
What is the most appropriate investigation?

A. CA 125
B. CA 153
C. CA 199
D. Carcino-embryonic antigens (CEA)
E. Alpha Fetoprotein (AFP)
5. A man with suspected active tuberculosis wants to be treated at home. What should be done to prevent
spread of the disease?

A. Immediate start of the treatment with Antitubercular treatment (ATT) drugs


B. All family members should be immediately vaccinated with BCG vaccine
C. Patient should be isolated in a negative pressure chamber in his house
D. Universal prevention application protocol
E. Barrier nursing

6. A 72 year old man fell while shopping and hurt his knee. His vitals are all fine. He speaks in a low
voice and is very slow to give answers. What is the most probable diagnosis?

A. Alzheimers
B. Vascular dementia
C. Transient ischaemic attack (TIA)
D. Pseudo dementia
E. Pick’s dementia

7. A young man complains of double vision on seeing to the right side. Which nerve is most likely to be
involved?

A. Left Abducens
B. Right Abducens
C. Left Trochlear
D. Right Trochlear
E. Right Occulomotor

8. A 56 year old man comes with a history of right sided weakness and left sided visual loss. Where is the
occlusion?

A. Anterior meningeal artery


B. Middle meningeal artery
C. Middle cerebral artery
D. Carotid artery
E. Anterior cerebral artery

9. A 50 years old chronic smoker came to outpatient department with complains of chronic productive
cough, shortness of breath and wheeze. Investigations results are: FBC: Increase in PCV
CXR: >6 ribs anterior ribs seen above diaphragm in mid clavicular line. ABG: pO2 is decreased. What is
the single most likely diagnosis?

A. Interstitial lung disease


B. Wegener’s granulomatosis
C. Ca bronchi
D. Chronic obstructive pulmonary disease
E. Amyloidosis
10. A 78 years old lady has been on Tamoxifen for 3 years. Now she presents with a heavy vaginal
bleeding but clinical examination shows no obvious abnormality. What is the most definitive
investigation for this lady?

A. Transvaginal scan
B. Pap smear
C. Endometrial sampling
D. Colposcopy
E. Hysteroscopy and biopsy

11. A patient presented with atrophy of the thenar eminence, difficulty in abduction of the thumb and
paresthesia on the palmar aspect of the lateral three and half fingers. Which nerve is affected?

A. Radial nerve
B. Ulnar nerve
C. Median nerve
D. Axillary nerve
E. Posterior Interosseous Nerve

12. A 28 years old male had an Accident and presented with head trauma. What choice of investigation
would you do to rule out hemorrhages?

A. CT scan
B. MRI
C. X ray skull
D. X ray spine
E. USG

13. A lady with depression has a bag full of medicines. She now presents with coarse tremors. Which
drug caused her symptom?

A. Lithium
B. Thyroxine
C. Amitriptyline
D. Sodium valproate
E. Tetrabenazine

14. A 24 year old man presents to Accident & Emergency after his friends noticed his sclera turning
yellow. This happened after he had been taking paracetamol for a few days for an upper respiratory tract
infection. What is the single best cause for jaundice in him?

A. Gilbert’s syndrome
B. Criggler-Najjar syndrome
C. Glucose 6 phosphate dehydrogenase deficiency
D. Hereditary Spherocytosis
E. Hereditary Elliptocytosis
15. A 56 year old woman with breast cancer and widespread lung metastases has non-productive cough.
She has been given codeine and the cough is not relieved. What is the next step in management?

A. Oral antibiotics
B. Salbutamol/ Nebulizers
C. Oxygen
D. Nebulization with 0.9% NaCl
E. Anaesthetic nebulizer

16. A 71 year old man presents with coarse tremor. He is on some medications. Which one can be the
reason for his tremors?

A. Lithium
B. Diazepam
C. Fluoxetine
D. Imipramine
E. Haloperidol

17. A 23 year old woman has been having pain at the base of her thumb, the pain is reproduced when
lifting her 3 months old baby or changing diapers and also with forceful abduction of the thumb against
resistance . What is the most likely cause?

A. Avascular necrosis of the scaphoid


B. Trigger finger
C. De Quervain’s tenosynovitis
D. Mallet finger
E. Gamekeepers Thumb

18. A patient with Parkinson’s disease has been on NG tube for the past ten days. He is still unable to
swallow. What would be the best management for him?

A. Gastrostomy
B. Colonostomy
C. Duodenostomy
D. Total parenteral nutrition
E. Ileostomy

19. A 56 years old lady presents with a 2 cm lump in her breast. Fine needle aspiration cytology (FNAC)
has been inconclusive. What is the next best step?

A. Open Biopsy
B. Core Biopsy
C. Stereotactic Biopsy
D. Mammography
E. USG
20. A patient underwent liver biopsy. What is the most important investigation before the biopsy?

A. Viral screen
B. Coagulation profile
C. Serum electrolytes
D. Liver function tests
E. Abdominal US

21. A 75 year old man collapsed while walking in his garden. He recovered fully within 30 minutes with a
BP of 110/80mmHg and a regular pulse of 70 bpm. He has a systolic murmur on examination. His
activities have reduced lately which he attributes to old age. What is the definitive diagnostic
investigation that will assist you with his condition?

A. ECG
B. Echocardiogram
C. 24hr ECG monitoring
D. 24 hour blood pressure measurement
E. Previous cervical intraepithelial neoplasia (CIN)

22. A young boy presents with fever and cough. His father was diagnosed with TB a week ago. The
parents do not want him to have a broncho-alveolar lavage under anaesthesia. Which other samples can
be taken for diagnosis?

A. Urine
B. Blood
C. Cerebrospinal fluid (CSF)
D. Gastric washings
E. Sweat

23. A 38 year old female presents with difficulty in looking upward and on examination she was found to
have lid lag as well. She also complains of her heart racing at times. Which test will help in the diagnosis?

A. Tensilon Test
B. 24 hr ECG
C. Thyroid Function test
D. Schimmer Test
E. Young Helmholtz Ophthalmoscopy

24. Patient presents with paracetamol overdose which is above the threshold level. What will be the
management?

A. Administer N-Acetyl Cysteine


B. Repeat serum level of Paracetamol in 4 hours time
C. Airway, Breathing, Circulation Assessment
D. Referral for liver transplant
E. Wait and see
25. A 32-year-old woman presents to the Emergency Department with headache and vomiting. She was
decorating her ceiling that morning when the headache began, felt mainly at the occiput with neck pain.
Some two hours later she felt nauseated, vomited and was unable to walk; she also noticed that her voice
had altered. She takes no regular medication and has no significant past history. On examination, her
acuity, fields and fundi are normal. She has upbeat nystagmus in all directions of gaze, with normal facial
muscles and tongue movements. Her uvula is deviated to the right and her speech is slurred. Limb
examination reveals left arm past-pointing and dysdiadochokinesis with reduced pinprick sensation in her
right arm and leg. Although power is normal, she cannot walk, as she feels too unsteady. Where is the
most likely site of her lesion?

A. Right medial medulla


B. Left medial pons
C. Left cerebellar hemisphere
D. Right lateral medulla
E. Left lateral medulla

26. A young man presents to Accident & Emergency Department with severe bloody diarrhoea,
abdominal pain and haematuria after eating meat at a barbecue party. FBC reveals schistocytes, ↓platelets,
↓Hb. Clotting tests are normal. What is the most likely diagnosis?

A. Autoimmune haemolytic anemia


B. Paroxysmal cold haemoglobinuria
C. Paroxysmal nocturnal haemoglobinuria
D. Micro angiopathic haemolytic anemia
E. ITP

27. A 20 years old pop star singer complains of inability to raise the pitch of her voice. She attributes this
to the thyroid surgery she underwent a few months back. What is the most probable diagnosis?

A. Thyroid Storm
B. Bilateral Recurrent Laryngeal Nerve Injury
C. Unilateral Recurrent Laryngeal Nerve Injury
D. External Laryngeal Nerve Injury
E. Thyroid cyst

28. A 35 year old man with BP 140/80mmHg is having micro-albuminuria. He has never been diagnosed
with hypertension. Diabetes has been ruled out. What is the most appropriate next step?

A. Renal biopsy
B. Renal scan
C. Refer to renal unit
D. Start ACE inhibitor
E. IVU

29. A patient presents with acute severe asthma attack. He is unable to speak complete sentences, RR is
30/min, and pulse is 115 bpm. He is unresponsive to oxygen, nebulised salbutamol and oral
hydrocortisone. What is the next management for him?
A. Ipratropium bromide
B. Intubation & ventilation
C. Magnesium sulphate
D. Doxapram
E. Aminophylline

30. A child suffering from cystic fibrosis developed pneumonia. Which organism is responsible for this
pneumonia?

A. Haemophilius influenza
B. Klebsiella
C. Staphylococcus aureus
D. Streptococcus pneumonia
E. Pseudomonas

31. A 24 year old male involved in road traffic accident. X-ray showed fracture neck of humerus. What is
the SINGLE most associated nerve injury?

A. Axillary nerve
B. Radial nerve
C. Median nerve
D. Ulnar nerve
E. Musculocutaneous nerve

32. A 51 years old Lady in A&E presents with severe chest pain following accident. On examination,
there are multiple rib fractures on both sides. How will you manage?

A. Stabilization of the ribs


B. Paracetamol (PCM) tablet
C. Oral morphine
D. TENS
E. NSAIDS

33. A 65 year presents with dyspareunia after sex. She is in her menopause. She complaints of bleeding
after sex. What is the most probable diagnosis?

A. Cervical Ca
B. Endometrial Ca
C. Ovarian Ca
D. Breast Ca
E. Vaginal carcinoma

34. A 56 year old man has been posted for colorectal surgery under general anaesthesia .What is the best
prophylactic antibiotic for this surgery?
A. IV Cefuroxime and IV metrogyl before induction of anaesthesia
B. IV metrogyl 24 hours before the surgery
C. Oral metrogyl for 1 week
D. Oral metrogyl for 24 hours
E. Sub cutaneous metrogyl 2 hours before surgery

35. A type 1 diabetic man presents with high cholesterol levels and micoalbuminuria. He is not
hypertensive. What drugs should he receive?
A. Statin + ACE inhibitor
B. Statin + metformin
C. Statin+ calcium channel blocker
D. Statin + pioglitazone
E. Statin + Beta Blocker

36. A 65 year old woman complaining of symptoms suggestive of Reynaud’s phenomena and difficulty in
swallowing. O/E she has painful lesions on her finger tips and facial telangiectasia. What is the single
most likely positive antibody?
A. Anti Jo 1
B. Anti Scl 70
C. Anti Ro
D. Anti Ds DNA
E. Anti Mi-2

37. A 60 year old woman was found by her son. She was confused and had urinary incontinence. She has
recovered fully after 6 hours with no neurological complaints. What is the likely diagnosis?
A. Stroke
B. Vestibular insufficiency
C. Transient ischaemic attack (TIA)
D. Intracranial haemorrhage
E. Benign intracranial hypertension

38. A 3 years old boy presents with difficulty in walking and skin lesions. He has a rash on the 3rd day
after the appearance of fever. What is the most likely causative agent?
A. Streptococcus pyogenes
B. Rubella virus
C. Parvovirus
D. Papova virus
E. Paramyxo virus

39. A 7 year old boy presents with perioral blisters and a hot face. Some are weeping and some crusted.
What is the most likely diagnosis?
A. Impetigo
B. Shingles
C. Chicken pox
D. Acne
E. HSV 1
40. A 2 day old baby has been brought by his deaf mother. She wants to know if her baby is deaf. What
test can be done?

A. Pure Tone Audiometry


B. Distraction testing
C. Brain stem evoked response
D. Rinne & Weber tuning fork test
E. CT Scan

41. A 56 year old male smoker has a white patch on his pharynx, which group of lymph nodes is most
likely to be enlarged?

A. Deep cervical
B. Submandibular
C. Sub mental
D. Parotid
E. Occiptal

42. A 56 year old female presents with complain of diplopia. She notices this only while parking her car
in her garage at home after coming back from work, where she has to constantly type and look at
monitors. She also says that she feels unusually tired after this. What is the most likely diagnosis?

A. Motor neuron disease


B. Myasthenia Gravis
C. Occulomotor nerve palsy
D. Parkinson’s disease
E. Refractory error

43. A woman has low mood, anhedonia, and refuses to eat or drink because she says she does not want to
live any more. What is the treatment?

A. ECT
B. Dosulpine
C. Fluoxetine
D. Moclobemide
E. Diazepam

44. A 24 year old man asks his GP for sick note from work. He says that he feels down, is lethargic and
has stopped enjoying playing the piccolo (his main hobby). He was admitted to a psychiatric hospital last
year following an episode of overspending, promiscuity and distractibility. What is the most probable
diagnosis for this patient?

A. Psychosis
B. Cyclothymia
C. Bipolar Affective Disorder
D.Depression
E. Seasonal Affective Disorder
45. A 22 year old girl is type I diabetic. She is prescribed long acting insulin in the morning & short
acting insulin before meals. She starts to become hypoglycaemic at about 4 pm. What should be the next
management for her?

A. Stop long acting insulin


B. Stop short acting insulin
C. Decrease dose of long acting insulin
D. Increase dose of short acting insulin
E. Decrease dose of short acting insulin

46. A 64 year old man who has urinary frequency for past 2 years now complains of anuria. He is taking
nifedipine and propranolol. What is the mechanism of his anuria?

A. Drug induced
B. Diabetes
C. Enlarged prostate gland
D. High blood pressure
E. Cancer of the bladder

47. A 45 years old woman diagnosed with Crohn’s disease. There was leaking faeces per vagina.
A diagnosis of fistula was made. From which part of the intestine is the faeces leaking from?

A. Caecum
B. Descending colon
C. Sigmoid colon
D. Jejunum
E. Duodenum

48. A 26 year old man presented with abdominal distension and pain. His stool was mucoid and
sometimes blood stained. What is the most appropriate investigation?

A. Stool culture and sensitivity


B. Colonoscopy and Biopsy
C. IgA against tissue transglutaminase
D. Barium meal
E. Jejunal biopsy

49. A 70 year old man presented with muscle weakness and inability to climb the stairs. Investigation
shows Creatinine phospokinase (CPK) is very raised. ESR 15mm/hour. What is the most likely
diagnosis?

A. Polymyositis
B. Polymyalgia rheumatica
C. Reactive arthritis
D. Rheumatoid arthritis
E. Duchenne's muscular dystrophy
50. What is the best postoperative pain management following anterior resection of rectum in a middle
aged man?

A. Oral morphine
B. PCA
C. Oral cocodomol
D. Epidural anesthesia
E. Fentanyl patch

51. A clinical picture of breast cancer originated from the mammary duct. Biopsy was done and there
were neoplastic cells found. Choose the histological picture of the cancer from the given options.

A. Neoplastic cells are arranged in small clusters occupying a space between collagen bundles
(Scirrhous carcinoma)
B. Spindle call neoplasm with margins, which infiltrate adjacent structure, fat was invaded ( Sarcoma
of breast)
C. Small cells with round nucleus and scant indistinct cytoplasm (Carcinoma of lobular origin)
D. Spindle cells
E. Large cells

52. A 46 year old man who is a heavy drinker is brought to the A&E in a drowsy state. He is responding
vaguely to questions asked. On examination he has nystagmus and hyper-reflexia. His MCV is
103fL.What is the most likely cause for his cognitive impairment?
A. B1 Deficiency
B. B12 Deficiency
C. Folate Deficiency
D. B6 deficiency
E. Alcohol withdrawal

53. A 30 year old woman is taking treatment for asthma. She has heart rate of 130beats/min and peak
expiratory flow rate of 400.What is the most appropriate management?
A. Atenolol
B. Digoxin
C. Review drugs
D. Propanolol
E. ACE inhibitor

54. A 56 year old man presents with headache and nausea since the past few months. On examination
there is right sided double vision. What is the site of the lesion?

A. Abducent nerve
B. Trochlea nerve
C. Oculomotor nerve
D. Optic nerve
E. Olfactory nerve
55. A 30 years old female presented with a 1.5cm discrete lump in the upper outer quadrant of her right
breast, which was tender. What is the initial investigation?

A. USG
B. FNAC
C. Excisional biopsy
D. Mammogram
E. Open biopsy

56. A 16 year old boy presents with acute pain in the right ear and little bleeding from the same ear. He
had been in a boxing match and had sustained a blow to the ear. There is little amount of blood in the
auditory canal and a small perforation of the eardrum. What is the most appropriate management?

A. Admission for parental antibiotics


B. Nasal de-congestant
C. Oral Amoxicillin
D. Outpatient clinic review
E. Packing of the ear

57. A woman comes to A& E 2 months after giving birth to her first child. She says she is having
overwhelming repetitive thoughts of harming her baby. She is very distressed by these thought and says
she is sure she won’t harm her baby. What is the diagnosis?

A. Post partum blues


B. Post partum depression
C. Post partum psychosis
D. Obsessive compulsive disorder
E. Delusions

58. A 7 years old child presented with chronic cough and is also found to be jaundiced on examination.
What is the most likely diagnosis?
A. Congenital diaphragmatic hernia
B. Congenital cystic adenomatoid malformation
C. Bronchiolitis
D. Respiratory distress syndrome
E. Alpha 1 antitrypsin deficiency

59. A lady underwent debulking surgery for ovarian carcinoma. Soon after the surgery she presents with
signs of intestinal obstruction. What is the Single most appropriate investigation?

A. Pelvic CT
B. CA 125
C. Laparotomy
D. Laparoscopy
E. Abdominal USG
60. A first year college student is brought into A&E unconscious with constricted pupils, redness around
the nasal alae, inflamed and swollen nasal mucosa and high fever. Toxicity with what substance is
suspected?

A. Cocaine
B. Heroin
C. Ecstacy
D. LSD
E. Marijuna

61. Pre auricular lymph nodes may be enlarged in pathology of which of the following organs?

A. Scalp
B. Lip
C. Forehead
D. Parotid
E. Neck

62. A 62 year old man has multiple liver metastases due to adenocarcinoma with an unknown primary. He
is deeply jaundiced and has ascites, with oedema of his legs up to the buttocks. He is now drowsy and his
family are worried that he is not drinking enough. His medication includes: Haloperidol 1.5 mg and
Lactulose 10 ml. Blood taken three days ago showed:

Normal electrolytes
Urea 6.5 mmol/L
Creatinine 89mmol/L
Calcium 2.04mmol/L
Albumin 17g/L
Total bilirubin 189 mmol/L
What is the SINGLE most appropriate management of his fluid intake?

A. Albumin infusion
B. Crystalloid fluids (continuous intravenous)
C. Crystalloid fluids (intermittent subcutaneous)
D. Fluids Via nasogastric tube
E. Oral fluids

63. A 25 years old male who recently noticed change in his shoe size, he is also constipated, he has
preference to hot weather, his skin is dry, has severe pain in wrist joint. Joint is red and swollen. What is
the most probable diagnosis?

A. Chondro-sarcoma
B. Lipo-sarcoma
C. Gout
D. Seronegative arthritis
E. Ankylosing Spondylitis
64. A 6 year old child has vomiting and diarrhoea for past 4 days. He is lethargic, has oliguria, dry mucus
membranes and decreased skin turgor. His brother had a similar illness a few days ago. What is the most
likely diagnosis?

A. Pyloric stenosis
B. Intestinal obstruction
C. Gastroenteritis
D. Gastro oesophageal reflux disease
E. Hirshsprungs

65. A 2 years old child who can run down steps, says mama and dada and understands what he says, and
also says one word. How is the development of this child?

A. Delayed motor milestones


B. Delayed speech
C. Normal
D. Global milestones delay
E. Delayed speech and motor milestones

66. A 6 year old child presents with oedema and mild proteinuria. There is no hematuria. What is the most
likely diagnosis?

A. Post streptococcal glomerulonephritis


B. Membranous glomerulonephritis
C. Minimal change glomerulonephritis
D. Rapidly progressive glomerulonephritis (RPGN)
E. Renal failure

67. A 28 years old man returned from holiday presents with pain in the superior and posterior part of the
scrotum. Which is the best investigation to confirm the diagnosis?

A. MSU culture and sensitivity


B. USG of the scrotum
C. Semen analysis
D. Serology
E. CT scan of the pelvis

68. A 13 year old girl presents with abdominal pain. There are no changes to her weight or bowel habits,
the pain does not hinder her daily activities and all examinations are normal. What is the single best
diagnosis?

A. Abdominal migraine
B. IBD
C. IBS
D. Hirschsprung’s disease
E. Coeliac Disease
69. A 70 year old man admits to asbestos exposure 20 years ago and has attempted to quit smoking. He
has noted weight loss and hoarseness of voice. Choose the single most likely type of cancer associated
with the risk factors presented.

A. Basal cell Carcinoma


B. Bronchial Carcinoma
C. Oesophageal Carcinoma
D. Nasopharyngeal Carcinoma
E. Oral carcinoma

70. A 17 years old girl had an episode of seizure. Contraction of muscles started from around the
interphalangeal joints, which spread to the muscles of wrist and elbow. Choose SINGLE most possible
type of seizure.

A. Grand mall
B. Tonic clonic
C. Myoclonic
D. Absent
E. Petit mal

71. A child suddenly squats while playing. He is on 70% O2. What is the most probable diagnosis?

A. Tetralogy of Fallot
B. Patent Ductus Arteriosus (PDA)
C. Pulmonary stenosis
D. Aortic stenosis
E. Mitral regurgitation

72. A 14 year old girl suffers from hay fever which bothers her school attendance and performance. What
is the best treatment option?

A. Oral steroids
B. Topical steroids
C. Oral Antihistamines
D. Topical antihistamines
E. Adrenaline IM
73. A 50 year old man has had hoarseness of voice and drooping eyelid for two months. A mass is
palpable in the right supra-clavicular fossa. He smokes 20 cigarettes a day for the last 30 years. What is
the most likely diagnosis?

A. Carcinoma Larynx
B. Carcinoma Thyroid
C. Carcinoma of the Right Bronchus
D. Mesothelioma
E. Pancoast Tumour
74. A 68 year old diabetic, hypertensive with 45 pack years smoking history, has left sided chest pain
increased with breathing. Examination shows miosis on left side and wasting of small muscles of left
hand. What is the Single most appropriate diagnosis?

A. Costochondritis
B. Lung cancer
C. Good pasture’s syndrome
D. Motor neuron disease
E. Progressive massive fibrosis (PMF)

75. A patient presents with complaints of a day-long diarrhoea. He has dry mucus membranes. ABG
show: pH- 7.1, paO2- 12 kPa, paCO2- 5.2 kPa, HCO3- 10 mEq/L
What is the electrolyte abnormality in this patient?

A. Metabolic acidosis
B. Metabolic alkalosis
C. Respiratory acidosis
D. Respiratory alkalosis
E. Mixed alkalosis/acidosis

76. A child developed jaundice a few days after birth. He is fully breast fed, growing well and has yellow
stools. What is the most probable diagnosis for this child?

A. Breast milk jaundice


B. Galactosemia
C. Biliary atresia
D. Congenital hypothyroidism
E. Cystic fibrosis

77. A 77 years old woman presents with vesicular rash on the left side of her face, vertigo, deafness and
Bell’s palsy. What is the most probable diagnosis?

A. Acoustic Neuroma
B. Trigeminal Neuralgia
C. Viral Labyrinthitis
D. Ramsay Hunt syndrome
E. Otitis Externa

78. A man with prosthetic heart valve underwent hemicolectomy and after some days complains of left
hypochondriac pain, fever and has a systolic murmur. What is the next investigation to ascertain the cause
of heart failure?

A. CT scan
B. Blood culture
C. ECG
D. Magnetic resonance imaging (MRI)
E. Radioactive thyroid scan
79. A patient with hepato-cellular carcinoma has raised levels of the ferritin. What is the most probable
cause?

A. Haemochromatosis
B. A1 antitrypsin deficiency
C. Cystic fibrosis
D. Wilsons
E. Primary Biliary Cirrhosis

80. A young woman complains of diarrhea, abdominal cramps and mouth ulcers. Abdominal X-ray shows
a distended transverse colon. There is goblet cell depletion on rectal biopsy. What is the most probable
diagnosis?

A. Crohn’s disease
B. Ulcerative Colitis
C. Bowel Cancer
D. Bowel obstruction
E. IBS

81. An 80 year old man presented with pain in his lower back and hip. He also complains of waking up in
the night to go to the washroom and has urgency as well as dribbling. What is the most likely diagnosis?
A. Benign prostatic hyperplasia (BPH)
B. Prostatitis
C. Urinary Tract Infection
D. Carcinoma of the Prostate
E. Carcinoma of the Bladder

82. A 4 day old baby presents with high fever. What is the next management?
A. Admit and observe
B. Admit, draw blood for cultures and sensitivities, start broad spectrum antibiotics
C. Intubate and ventilate
D. Discharge with paracetamol
E. Discharge with NSAIDs

83. A young boy presented in Out -Patient department 12 weeks after renal transplantation with fever and
pain in lower abdomen. Renal functions were deranged. Renal biopsy showed immune cell infiltrate and
tubular damage. What is the most probable diagnosis?

A. Pyelonephritis
B. Chronic graft rejection
C. Acute rejection
D. Drug toxicity
E. Graft host disease.

84. A young girl has a rash on her scalp. His mother and grandmother both have had similar rashes on
their scalps and extensor surfaces. Mother is worried if it’s infective. What advise will you give the
mother?
A. Tell her its infective
B. Advise her to keep combs, towels, soaps separate
C. Reassure her it’s not infective
D. Offer RAST test
E. Prescribe anti-histamine
85. A young woman was brought into hospital 10 hours after ingesting 10 tablets of paracetamol. What
should be the next step?

A. Measure serum paracetamol levels


B. N acetlycysteine
C. Gastric lavage
D. Activated charcoal
E. IV fluids

86. A 57 year old shipyard worker has a 4 months history of cough and left sided chest pain. He has a
pleural effusion on the left side. What will you do to improve his symptoms?
A. Therapeutic thoracocentesis
B. Chemotherapy
C. Pleural biopsy
D. Radiotherapy

87. A 30 year old man is suffering from fever, rash and photophobia. Doctors are suspecting he is
suffering from meningitis. Which is the best medication for this condition?

A. Ampicillin
B. Cefotaxime
C. Tetracycline
D. Acyclovir
E. Dexamethasone

88. A 50 year old patient admitted to the hospital for elective herniorraphy. Which of the following option
will lead to his postponement of the operation?

A. Systolic BP less than 90mmHg


B. Myocardial infarction two months ago
C. Hb 12g/dl
D. Pain around the hernia
E. Abdominal distension

89. A 64 year old anemic man presents with constipation. Barium enema demonstrates a stricture on the
right side, close to the appendix. What is the most likely diagnosis?

A. Crohn’s
B. Ulcerative Colitis
C. Caecal Ca
D. Rectal Ca
E. Sigmoid Ca
90. A 34 year old man from Asia presented with 5 months history of productive cough, night sweats and
weight loss. His CXR reveals some shadowing in the left upper zone. What is the single most
discriminating investigation?

A. Acid fast bacilli test for sputum


B. CXR
C. CT scan
D. Thyroid function test
E. Ultrasound of the abdomen

91. A 30 year old patient who is not taking any medications presents with blood pressure 200/100 mmHg,
sodium 140 mmol/L, potassium -2.2mmol/L, calcium 2.0 mmol/L. What is the most likely diagnosis in
this patient?
A. Primary hyperaldosteronism ( Conn’s disease)
B. Primary adrenal insufficiency (Addison’s disease)
C. Cushing syndrome
D. Cushing’s disease
E. Hypopituitarism

92. A 4 years old child presents with pain of spontaneous onset in his knee of 2 days duration. He has
developed mild fever in the second day. He can walk but has a limp. On examination, he has painful
restriction in the right hip. What is the most probable diagnosis?

A. Osteosarcoma
B. Septic Arthritis
C. Tuberculus Arthritis
D. Exostosis
E. Osteomyelitis

93. A 52 year old man is on antihypertensive medication. He comes to cardiovascular Out Patient Clinic
and complains about dry cough. Lab report shows Na : 139mmol/L, K: 6.7mmol/L, Ca : 2.55mmol/L,
Creatinine : 165µ/l. Which is the most probable medication causing the dry cough?

A. Ramipril
B. Losartan
C. Bendroflumethiazide
D. Atenolol
E. Verapamil

94. A 79 year old man stumbled and sustained a minor head injury two weeks ago. He has become
increasingly confused, drowsy and unsteady on his feet. He has a GCS (Glasgow coma scale) score of
13. He takes Warfarin for atrial fibrillation. What is the most probable diagnosis?

A. Extradural Hemorrhage
B. Cerebellar Hemorrhage
C. Epidural Hemorrhage
D. Subdural Hemorrhage
E. Subarachnoid Hemorrhage
95. A young boy presents with non blanching rash on extensor surfaces, proteinuria, haematuria,
diarrhoea and abdominal pain after an upper respiratory tract illness 2 weeks ago. His platelet counts and
clotting factors are normal. What is the single best diagnosis?

A. ITP (Idiopathic thrombocytopenic purpura)


B. TTP (Idiopathic thrombotic thrombocytopenic purpura)
C. Haemophilia
D. HSP (Henoch-Schönlein purpura)
E. Vitamin K deficiency

96. What is the maintenance fluid of choice in paediatric rehydration?

A. 0.45-5% dextrose
B. 0.9 % normal saline
C. 0.18 % normal saline
D. 0.45 % normal saline
E. Hartmanns solution

97. A man presented with a purplish swelling at the anal area. It is acutely painful and he complains of
constipation for the last 2 months. What is the most appropriate management?

A. Incision and drainage


B. Incision and drainage plus antibiotics
C. Reassure
D. Analgesia
E. Sclerotherapy

98. A 33 years old lady who is a drug addict wants to quit. She says she is ready to stop the drug abuse.
She is supported by her friends and family. What drug treatment would you give her?

A. Benzodiazepines
B. Diazipoxide
C. Lithium
D. Methadone
E. Disulfiram

99. A 66 year old male presents with painful swallowing. What is the most likely diagnosis/ organism?

A. Neisseria meningitides
B. Cryptococcus neoformans
C. Candida albicans
D. Isospora belli
E. Mycobacterium avium-intra-cellular

100. A 23 year old pregnant lady presents with tingling and numbness in her right hand. She has been
bothered by it but lately she has discovered that shaking her hand relieves the symptoms. The
neurosurgeon has offered her a surgical option. What structure would he want to cut?
A. Common flexor sheath
B. Flexor retinaculum
C. Flexor carpi ulnaris
D. Flexor carpi radialis
E. Extensor retinaculum

101. A 45 years old woman has recently been diagnosed with multiple sclerosis and has been started on
oral steroids. She is brought to the hospital after having ingested 100 Paracetamol tablets 4 hours ago. She
is refusing all medical treatment. What is the next best step for this patient? B
A. Observe
B. Refer to a psychiatrist to assess patient’s ability to refuse
C. Gastric Lavage
D. Activated charcoal
E. Refer to a social worker

102. A 25 year old woman has a recent cough, hoarseness and swelling in the neck. There are several
non-tender swellings on both sides of her neck. She has lost 13 kg in weight. She takes recreational
drugs. What is the most probable diagnosis?
A. Thyrotoxicosis
B. Hyperthyroidism
C. Vocal Chord Nodules
D. Carcinoma Bronchus
E. Tuberculosis

103. An 89 year old patient has lung cancer. His sodium is 122mmol/l. What is the treatment for it?
A. Demeclocycline
B. Vasopressin
C. Restrict fluids
D. Reassure

104. A 5 week old breast fed baby whose birth weight was 3.5 kg and is now 4.5kg is thriving well but is
deeply jaundiced. What is the most likely diagnosis?
A. Galactosemia
B. Breast Milk Jaundice
C. Thalassemia
D. Sickle Cell Disease
E. Congenital Storage Disorder

105. A 35 year old man presented with painful vesicles around the right ear with facial palsy on
examination of the ear there are vesicles in external ear extending to the tympanic membrane. The patient
also complains of hearing difficulty. What is the most probable diagnosis?
A. Ramsay-Hunt syndrome
B. Herpes simplex
C. Otitis externa
D. Acute otitis media
E. Parotitis
106. A 24 year old patient has come 4 times to the hospital. She lives alone, has no relationships and
always comes to the hospital in loud clothes, demanding for the same doctor. Her behaviour is quite
inappropriate, always giving the doctor compliments and praising him. What is the most likely diagnosis?
A. Schizophrenia
B. Mania
C. Histrionic Personality
D. Borderline personality
E. Munchausen’s

107. A 64 year old woman has difficulty moving her right shoulder on recovering from surgery of the
posterior triangle of her neck. What is the SINGLE most appropriate option?
A. Accessory nerve
B. Glosso pharyngeal nerve
C. Hypoglossal nerve
D. Vagus nerve
E. Vestibulocochlear nerve

108. A 12 year old child with episode of sudden bluish discoloration and brief loss of consciousness.
Examination reveals clubbing, central cyanosis and systolic thrill with systolic ejection murmur in 2nd
left inter costal space. What is the most probable diagnosis?

A. Tetralogy of Fallot D. PDA (Patent ductus arteriosus)


B. Atrial septal defect (ASD) E. Coarctation of Aorta
C. Ventricular septal defect (VSD)

109. A 10 year old girl has been referred for assessment of hearing as she is finding difficulty in hearing
her teacher in the class. Her hearing tests show:
Bone conduction normal
Symmetrical air conduction thresholds reduced bilaterally
Weber test shows no lateralization.
What is the single most likely diagnosis?

A. Chronic perforation of tympanic membrane


B. Chronic secretory otitis media with effusion
C. Congenital sensori neural deficit
D. Otosclerosis
E. Presbycusis

110. A 78 years old male, diabetic and hypertensive, had a fall and since then he is unable to walk. He
presents with deformity and tenderness over the right hip area. X-ray showed fracture of the neck of
femur. What is the SINGLE most associated nerve injury?
A. Sciatic nerve E. Femoral nerve
B. Gluteal nerve
C. Lateral peroneal nerve
D. Tibial nerve
111. A 6 year old patient comes with easy bruising in different places when she falls. Her FBC results are:
WBC 25 * 10 9, Hb: 10.9, platelet 45* 10 9. Her Paul Bunnel test is positive. What is the single most
likely diagnosis? A

A. Glandular fever
B. Idiopathic thrombocytopenia purpura (ITP)
C. Trauma
D. Non accidental injury (NAI)
E. Septicaemia

112. An 8 months old infant presented with failure to thrive and constipation. On examination he has
large tongue and he also has a history of prolonged neonatal jaundice. What is the most likely diagnosis?

A. Down’s Syndrome
B. Fragile X Syndrome
C. PraderWilli Syndrome
D. Di George Syndrome
E. Congenital Hypothyroidism

113. A patient diagnosed with metastatic cancer now presents with frequent falls, weakness,
hyperpigmentation, myalgias, hyponatremia and hyperkalemia. What is the most likely cause of her
symptoms?

A. Hyperaldosteronism
B. Adrenal insufficiency
C. Adrenal hyperplasia
D. Cushing’s disease

114. Which nerve is most likely to be injured in a mid-shaft femoral fracture?


A. Femoral nerve
B. Sciatic nerve
C. Tibial nerve
D. Superficial peroneal nerve

115. A 20 year old male smoker is noted to have intense rubor of the feet and absent foot pulse. On
examination he has an amputated right second toe. What is the most probable diagnosis?
A. Intermittent Claudication
B. Cardiovascular Syphilis
C. Buerger’s Disease
D. Embolism
E. Acute Limb Ischemia

116. A woman with history of rheumatoid arthritis has sudden pain, redness and hotness over her knee
joint. What test will lead to a diagnosis?

A. Joint aspirate for negatively birefringent crystals D. Plain X Ray


B. Joint aspirate for positively birefringent crystals
C. Joint aspirate for cytology and culture
117. A 60 year old smoker presents with cramp-like pain in the calves relived by rest and non-healing
ulcers. On examination, he has cold extremities with lack of hair around the ankles and absent distal
pulses. What is the most probable diagnosis?

A. Intermittent Claudication
B. Chronic Ischemia of the Limbs
C. Buerger’s Disease
D. Deep Vein Thrombosis
E. Diabetes Mellitus

118. A 70 year old hypertensive white British man on thiazide diuretics needs a second drug to control
his blood pressure. Which one of the following is the best choice for him?

A. Amlodipine (CCB)
B. Enalapril (ACE inhibitor)
C. Propanolol (B blocker)
D. Increase dose of diuretic
E. Prazocin (Alpha blocker)

119. A 5 year old boy presents with prolonged epistaxis. Investigations reveal a prolonged APTT. What
is the most probable diagnosis?

A. Haemophilia
B. Von Willibrand’s disease
C. ITP
D. Glanzmann’sthrombasthenia

120. A patient had a stroke. Now, there is left sided weakness and right-sided facial numbness. CT shows
ischaemic stroke. Which one would you prescribe?

A. Alteplase
B. Aspirin
C. Clopidogrel
D. Heparin
E. Warfarin

121. A schizophrenic patient is non-compliant with his medication. He also complains of tardive
dyskinesia. What is the best management for him? D/E
A. Oral Haloperidol
B. IM Fluphenazinedecanoate
C. IV Chlorpromazine
D. IM Risperidone
E. Tetrabenazine
122. A 35 year old man presents with history of dyspepsia. H. Pylori antibodies are negative. No
improvement is seen after one month of treatment. What is the next step?

A. Urea breath test


B. Gastroscopy
C. CT scan
D. MRI scan

123. A 68 year old retired shipyard worker presents with cough and increasing breathlessness. CT scan of
the chest reveals thickening of the pleura and effusion. What is the most likely diagnosis?
A. Mesothelioma
B. Pneumonia
C. Tuberculosis
D. Asthma

124. A 22 year old man has rushed into A& E asking for help. He describes recurrent episodes of
fearfulness, palpitations, faintness, hyperventilation, dryness of the mouth with peri-oral tingling and
cramping of the hands. His symptoms last 5 to 10 minutes and have worsened since their onset 3 months
ago. He is worried he may be having a heart attack. An ECG shows sinus tachycardia (100beats/
min).What is the SINGLE appropriate immediate intervention?

A. High flow oxygen


B. Intravenous (IV) sedation
C. Re-breathe into a paper bag
D. Refer for anxiety management course
E. Refer for urgent cardiology opinion

125. A 48 year old woman always socially withdrawn has stopped going out of the house. She is afraid
to socialize because she fears that people will criticize her. What is the most probable diagnosis?
A. Agoraphobia
B. Post Traumatic Stress Disorder
C. Social Anxiety
D. Obsessive Compulsive Disorder
E. Generalized Anxiety Disorder

126. A 68 year old man has had malaise for five days and fever for two days. He has cough and there is
dullness to percussion at the left lung base. What is the SINGLE most appropriate investigation?
A. Bronchoscopy
B. Chest x-ray
C. CT scan
D. Magnetic resonance imaging (MRI)
E. V/Q scan

127. A man on treatment for depression now complains of palpitations, dry mouth, urinary retention,
constipation and blurred vision. Which drug is responsible for his symptoms?
A. Citalopram
B. Imipramine
C. Fluoxetine
D. Paroxetine

128. A 10 year old boy presented with generalized swelling. This has been present for4 days and included
swollen ankles and puffiness of the face. It started a few days after he had a mild cold with runny nose.
His only past medical history was eczema. Urine analysis shows hematuria, proteinuria 10g/24hr, serum
creatinine 60µmol/L and serum albumin 15g/L. What is the single most likely diagnosis?

A. IgA nephropathy
B. Henoch-schonlein purpura
C. Minimal change nephropathy
D. Wilson’s disease
E. Cardiac failure

129. A 44 year old man comes with history of early morning headaches and vomiting. CT brain shows
ring enhancing lesions. What is the SINGLE most appropriate option?

A. Cytomegalovirus
B. Streptokinase
C. Toxoplamosis
D. Non Hodgkins Lymphoma
E. Pneumocystis jerovici

130. A 45 year old lady complains of expressablegalactorrhea, decreased libido and amenorrhea, weight
gain and easy fatigue. Her serum prolactin levels are 11000mu/l. What is the likely cause of
hyperprolactinemia?

A. Hypothyroidism D. Prolactin secreting pituitary tumour


B. Stress E. Polycystic ovarian syndrome
C. Pregnancy

131. A 22 year old man is being investigated for steatorrhea and flatulence. On jejunal biopsy there is
shortening of the villi and lymphocytosis. What is the most likely cause? B
A. Coeliac disease
B. Whipple's disease
C. Crohn's disease
D. Irritable bowel syndrome

132. A 44 years patient has sudden onset of breathlessness and stridor few minutes after extubation for
thyroidectomy. The patient had long standing goitre for which he had the surgery. What is the most
probable diagnosis?
A. Thyroid Storm
B. Hematoma
C. Unilateral Recurrent Laryngeal Nerve Injury
D. External Laryngeal Nerve Injury
E. Tracheo-malacia
133. A 32 years old woman presents with obesity, easy bruising and striae on the abdomen. The low dose
Dexamethasone suppression test shows no suppression of cortisol levels, but with higher doses of
Dexamethasone there is suppression of the serum cortisol. What is the most likely diagnosis?

A. Pseudo-cushing's D. Adrenal Adenoma


B. Pituitary Adenoma E. Exogenous Steroid
C. Ectopic Adrenocorticotropic Hormone (ACTH) source

134. Child took grandmother’s medication and presents with blurred vision and bradycardia with multiple
ectopics. What is the diagnosis?

A. Digoxin toxicity
B. Amitriptyline toxicity
C. Atropine toxicity
D. B blockers toxicity
E. ACE inhibitors toxicity

135. A young lady after a heavy bout of drinking late last night comes to A&E with dizziness, abdominal
pain, vomiting blood with peripheries cool. After initial resuscitation oxygen and fluids, she still
continues to bleed with pulse 130 beats per minute and blood pressure 86/ 58 mm Hg. What would be
your next best management?

A. Clotting screen
B. USG
C. CT scan
D. Endoscopy
E. Omeprazole

136. A new born baby has been brought in with pansystolic murmur at sternal border but the baby is not
cyanosed. What is the most probable diagnosis?

A. Ventricular septal defect (VSD)


B. Atrial septal defect (ASD)
C. Tetrology of Fellot (TOF)
D. Patent Ductus Arteriosus (PDA)

137. A four year old girl is taken by her mother to A/E department and complains of feeling unwell,
urinary urgency and temperature of 39˚ C. What is the single next best investigation?

A. Catheter catch of urine


B. Clean catch of urine
C. Ultrasound
D. Intra venous urogram (IVU)
E. Supra-pubic catch of urine
138. A 78 year old retired teacher was admitted for a hernioplasty procedure. After the operation he
became agitated, aggressive and confused. What is the most appropriate management ?

A. Diazepam
B. Chlordiazepoxide
C. Vitamin B
D. Clozapine
E. Thiamine

139. A 40 year old man complains of thirst and lethargy. His BP is 140/90mmHg and his calcium which
was corrected is 3.7mmol/l. What is the most appropriate management at this stage?
A. IV fluids
B. Prednisolone
C. IV hydrocortisone
D. Calcium preparation

140. A 40 year old woman, with breast cancer, has back pain which keeps her awake at night. She blames
it on a gym session she had two weeks ago. She now has difficulty in climbing the stairs. There is
tenderness over the right thoracic spine. She has diminished fine touch and temperature sensation in her
right foot. What is the SINGLE most appropriate investigation?

A. Bone density scan


B. Computed tomography (CT) scan of the head
C. Magnetic resonance Imaging (MRI) of the spine
D. Nuclear medicine (NM) bone scan
E. Plain X-ray of the thoraco-lumbar spine

141. An ECG of an elderly female who collapsed in A&E shows rapid ventricular rate of 220 per minute,
QRS duration is 140 milli seconds. What is the most probable diagnosis?

A. Atrial Fibrillation
B. Ventricular tachycardia
C. Supra ventricular tachycardia
D. Mobitz Type I Second Degree heart Block
E. Sinus Tachycardia

142. A 34 year old woman presents with severe jaundice, fever and pruritus. What is the best initial
management to control the pruritus in her?

A. IV 0.9 % normal saline


B. IV dextrose
C. Cimetidine
D. Cholestyramine
143. An 8 year old child, who is tall for his age and has a refractory error for which he wears glasses, has
presented with severe crushing chest pain. What is the most likely diagnosis?

A. Fragile X Syndrome
B. PraderWilli Syndrome
C. Di George Syndrome
D. Marfan’s Syndrome

144. A 14 year old girl is clinically obese. She has not started her periods yet and has severe acne. Among
her investigations, a high insulin level is found. What is the most probable diagnosis?
A. Cushing’s Syndrome
B. Grave’s disease
C. Acquired Hypothyroidism
D. Polycystic ovarian syndrome
E. Addison’s disease

145. A 23 year old man has been stabbed in the back and has shortness of breath. The trachea is not
deviated, he has engorged neck veins and absent breath sounds on the right. What is the most appropriate
diagnosis?

A. Tension pneumothorax
B. Cardiac tamponade
C. Simple pneumothorax
D. Haemothorax
E. Pleural effusion

146. A 40 year old woman presents to the GP with low mood. Of note, she has an increased appetite and
has gone up to two dress sizes. She also complains that she cannot get out of bed until the afternoon.
What is the most likely diagnosis for this patient?

A. Pseudo Depression
B. Moderate Depression
C. Severe Depression
D. Dysthymia
E. Atypical Depression

147. A 8 weeks old baby with a fever of 38˚C. How will you manage this neonate?

A. Admit for infection screen and start IV antibiotic


B. Give antipyretic and ask to come later
C. Observe for 24hrs
D. Admit for infection screening
E. Reassure

148. A 48 year old woman who has been taking medications for asthma for a long time has now presented
with decreasing vision. What is the most probable cause for her decrease in vision?
A. Inhaled Salbutamol
B. Inhaled Steroids
C. Aminophylline
D. Beta blockers
E. Oral Steroids

149. A patient presented with a stab injury to his left groin, he keeps bleeding profusely. pressure
has been applied to stop bleeding , BP 80/60mmHg. What is the next appropriate action?
A. I/V fluid
B. Blood Transfusion
C. Oxygen
D. Elevate limb

150. A woman presented with intermittent pain in her left breast radiating to her left arm. What is the
diagnosis?

A. Cyclical mastalgia
B. Fibroadenoma
C. Non cyclicalmastalgia
D. Duct Ectasia
E. Mondor’s disease

151. A woman presents with alternating episodes of low mood and happiness. When she feels happy she
goes on extravagant shopping sprees. What is the most likely diagnosis in this patient?
A. Depression
B. Bipolar affective disorder
C. Schizophrenia
D. Panic disorder

152. A 24 year old woman complains of pain over the base of her right thumb. She finds that she cannot
extend her right thumb. The pain is worse when she is changing her son’s nappies and subsides with rest.
She noticed it for the first time a few months after the birth of her son. What is the most likely diagnosis?

A. Trigger finger
B. Mallet finger
C. Gamekeepers thumb
D. Mommy’s thumb

153. A 36 year old woman has been treated with dexamethasone for cerebral oedema for the last 3
months. She has now started having recurrent falls, especially in the morning. What is the most like cause
of her falls?

A. Drug side effects


B. Autonomic Neuropathy
C. Adrenal Insufficiency
D. Cushing’s syndrome
E. Addison’s disease
154. A boy came presents with plenty of transparent sticky liquid coming from both the eyes. The mucosa
of the lids and the conjunctiva was intact and calm. What will you give him?
A. Antibiotics
B. Steroids
C. Antibiotic and steroids

155. A 19 year old female is admitted with Paracetamol overdose after an argument with her boyfriend.
Her paracetamol levels are not dangerously high and she does not need medical treatment. She regrets her
actions and is very apologetic saying that she will never do this again. What is the next management?
A. Discharge
B. Full psychiatric assessment
C. N acetylcysteine
D. Gastric lavage

156. A 3 year old child with severe diarrhoea and vomiting, looks lethargic, has sunken eyes and a feeble
cry. What is the choice of fluids for this condition?
A. 0.9 % Saline
B. 0.9% Saline plus 5% Dextrose
C. 0.45% Saline plus 5% Dextrose
D. 0.45% Saline

157. A 35 year old woman has butterfly rash on her face and she suffers symmetrical joint pains on knee
and elbow, ESR is raised. What is the most discriminative investigation for diagnosis?

A. Anti DNA antibodies


B. Anti Jo 1 antibodies
C. Anti nuclear antibodies
D. Anti centromere antibodies
E. Anti la antibody

158. A patient was admitted to hospital with severe pyelonephnritis. He was given IV antibiotics. A few
days later he develops bloody diarrhoea. Stool cultures reveal Clostridium difficile. What is the best
treatment option for this patient?

A. Erythromycin
B. Gentamicin
C. Penicillin
D. Metronidazole

159. A 6 year old came with full thickness burn. He is crying continuously. What is next step of
management?
A. Refer to burn unit
B. IV fluid start
C. Antibiotic
D. Analgesic
E. Dressing
160. The annual incidence of Down Syndrome is :
A. 7
B. 70
C. 700
D. 7000
E. 70000

161. A 5 year old boy is rescued from a burning building and is presented to the accident and emergency.
He had 5% partial thickness burns over the arms and legs and had soot in the mouth and nose. His
breathing has become noisy. What is the SINGLE most immediate management?

A. Nebulised Adrenaline
B. Nebulised Salmeterol and Oxygen
C. Needle Cricothyrodotomy
D. Oropharyngeal Airway
E. Intubation of Airway

162. A patient has diabetes for the past 10 years. Now he presents with difficulty in reading small prints.
Visual acuity test shows field defects. Which part of the eye is affected?

A. Lens
B. Aqueous humour
C. Retina
D. Vitreous humour
E. Sclera

163. A six year old child with history of recurrent jaundice. Between the episodes he is totally fine.
Mother gives history of jaundice being brought about by on-going infections. What is the most likely
diagnosis?
A. Hereditary Spherocytosis
B. G6PD Deficiency
C. Thalassemia
D. Sickle Cell Disease
E. Congenital Storage Disorder

164. A 34-year-old woman is referred to the Endocrine Clinic with a history of thyrotoxicosis. At her first
appointment she is found to have a smooth goitre, lid lag and bilateral exophthalmos with puffy eyelids
and conjunctival injection. She wants to discuss the treatment of her thyroid problem as she is keen to
become pregnant. What is the most likely treatment you would advise?

A. 18 months of carbimazole alone


B. 18 months of propyl thiouracil alone
C. A combination of anti-thyroid drugs and thyroxine
D. Radioactive iodine
E. Thyroidectomy
165. A 65 year old man, scheduled for a hernia surgery, has been informed that his surgery has been
postponed. What is the most likely reason for cancelling the surgery?
A. Hb<10
B. SBP<90 (Systolic Blood Pressure)
C. Previous myocardial infarction
D. Diabetes Mellitus DM
E. Seizure disorder

166. Where is the level of occlusion if popliteal, dorsalispedis and posterior tibial pulses are not palpable?
A. External iliac artery
B. Femoral artery
C. Internal iliac artery
D. Sciatic artery

167. A patient after his house fire came with haematemesis with erosion of oesophagus and on
examination there is 55% burn and on endoscopy there is gastric erosion and soot on mouth. What is the
treatment?
A. Proton Pump Inhibitors oral
B. Proton Pump Inhibitors IV
C. Proton Pump Inhibitors &Antibiotics
D. Helicobacter pylori test
E. Tracheal intubation

168. A 6 year old boy is brought to the hospital for a third episode of sore throat in one month, he is found
to be bleeding from gums and nose has pale conjunctiva. What is the single cell type from given options?

A. Clumped platelets
B. Microcytes
C. Granulocyte without blast cells
D. Blast cells
E. Mature lymphocyte

169. A known type 1 diabetic presents with drowsiness, vomiting, abdominal pain, fruity smell on his
breath and deep laboured breathing. His ABG’s show metabolic acidosis. What is the best rehydration
regimen for him?

A. Insulin
B. Insulin + potassium
C. IV dextrose + insulin + potassium
D. IV 0.9% normal saline + insulin + potassium

170. A one month old boy has been brought to the Emergency Department. He is conscious but with cool
peripheries and has heart rate of 22 beats/minute. He has been irritable and feeding poorly for about 24
hours. His chest x-ray shows a borderline enlarged heart with clear lung fields and ECG shows a regular
narrow complex tachycardia. It is difficult to identify p wave on the tracing. What is SINGLE most
appropriate immediate treatment?
A. Administer fluid bolus
B. Administer oxygen
C. Oral beta blocker
D. Synchronised DC cardio-version
E. Unilateral carotid sinus massage

171. A 22 years sexually active male came with 2 days history of fever with pain in scrotal area. On
examination scrotal skin is red, tender. What is the most appropriate diagnosis?

A. Torsion of testis
B. Orchitis
C. Inguinal Hernia
D. Epididymo- orchitis

172. An eight year old boy with a body mass index (BMI ) of 28 kb/m2 was admitted to a surgical ward
following a motor vehicle accident. He was found to have glycosuria. When he recovered from his injury
the glycosuria resolved. What is SINGLE most appropriate follow-up investigation?

A. Fasting blood glucose concentration


B. Glycosylated hemoglobin (HbA1c)
C. Oral glucose tolerance test
D. Random blood glucose concentration
E. Serum cortisol concentration

173. A 50 year old man came to the hospital a few months after he had myocardial infarction. On
examination everything was normal, his S1 and S2 sounds were heard on auscultation, but there is a new
pan-systolic murmur. What is the most appropriate investigation of choice?

A. ECG
B. 24 hour ECG
C. Echocardiogram
D. CXR
E. CT scan

174. A 14 years old boy has been diagnosed with Nephrotic Syndrome. Five days later he presents with
flank pain, hematuria and fluctuating urea levels. A diagnosis of renal vein thrombosis is made. What is
the most likely cause for renal vein thrombosis?
A. Protein C deficiency
B. Vasculitis
C. Loss of Antithrombin III
D. High estrogen levels
E. Stasis
175. A three years old boy has a sudden onset of fever, vomiting and bilateral face swelling. Few days
earlier general practitioner saw him for bilateral parotid pain and gave analgesics. What is the most
appropriate next step?
A. Analgesic
B. Antibiotic
C. Biopsy
D. Immediate Surgery
E. Reassurance

176. A 30 year old man presents in Accident and Emergency department with difficulty in breathing. He
has returned from India. Recently examination of his throat revealed grey membranes on the tonsils and
uvula. He has mild pyrexia. What is the SINGLE most relevant diagnosis?

A. Diphtheria
B. Infectious mononucleosis
C. Acute follicular tonsillitis
D. Scarlet fever
E. Agranulocytosis

177. A 55 year old man presents with an ulcer of the scrotum. Which of the following lymph node is
involved?
A. External iliac lymph nodes F. Sub-mental lymph nodes
B. Pre aortic lymph nodes G. Submandibular lymph nodes
C. Aortic lymph nodes H. Deep cervical lymph nodes
D. Inguinal lymph nodes
E. Iliac nodes

178. A 10 year old boy is brought to A&E 10 hours after injury to the foot. It was punctured with a metal
spike that passed through his shoe. What is the next best step?

A. Immunoglobulin
B. Immunoglobulin and vaccine
C. Vaccine only
D. Clean the wound
E. Antibiotics

179. A young man complains of bloody diarrhoea with mucus. He has colicky abdominal pain. What is
the most appropriate test for diagnosing his condition?
A. Barium enema
B. Stool culture
C. Lower GI endoscopy & biopsy
D. H.Pylori stool antigen test
180. A 32 year old lady complains that she hears everyone saying that she is an evil person. What type of
hallucinations is she suffering from?

A. Second person auditory hallucinations


B. Third person auditory hallucinations
C. Echo de la pense
D. Gedankenlautwerden

181. A 55 year old woman with a persistent cough and history of smoking develops left sided chest pain
exacerbated by deep breathing with fever and localized crackles. What is the Single most appropriate
diagnosis?

A. Dissecting aneurysm
B. Pericarditis
C. Pneumonia
D. Pneumothorax
E. Pulmonary embolism

182. A 2 year old girl presents with a 4-day history of fever. The illness started with a cough. Her
respiratory rate is 45/min, saturations 94% on air, temperature 38.9˚C and capillary refill time 1 sec.
There are crepitations at the left base on auscultation. Urine is negative on dipstick. What is the single
investigation most likely to lead to a diagnosis?

A. Blood for Culture and Sensitivity


B. ESR
C. Chest X Ray
D. Urine for Culture Sensitivity
E. CSF Analysis

183. A 7 years old boy is brought by his mother. There are multiple perioral and forehead vesicles. Some
vesicles are crusted and some are not. The face is hot. What is the most likely diagnosis?
A. Varecilla zoster
B. Herpez zoster
C. Fungal infection
D. Impetigo
E. Psoriasis

184. A sick child presents with drooling of saliva and fever. What is the most probable diagnosis?
A. Acute asthma
B. Bronchiolitis
C. Croup
D. Epiglottitis
E. Tonsilitis
F. URTI
G. Pharyngitis
185. A 55 year old woman complains of retrosternal chest pain and dysphagia which is intermittent and
unpredictable. The food suddenly sticks in the middle of the chest, but she can clear it with a drink of
water and then finish the meal without any further problem. A barium meal shows a ‘corkscrew
oesophagus’. What is the SINGLE most likely cause of dysphagia from the given options?

A. Oesophageal candidiasis
B. Oesophageal carcinoma
C. Oesophageal spasm
D. Pharyngeal pouch
E. Plummer Vinson syndrome

186. A 50 year old man presents with the complaints of recurrent urinary tract infection and occasional
blood in the urine stream. Some unusual cells have been seen in urine on routine examination. Which of
the following investigations would you like to carry out now?
A. Cystoscopy E. Full blood count
B. Urine culture and sensitivity
C. X ray KUB
D. USG

187. A 60 years old lady is on Tamoxifen for treatment of breast carcinoma. What is the worst side effect
you will look out for?
A. Hot flush
B. Vaginal dryness
C. Osteoporosis
D. Vaginal bleed
E. Weight gain

188. A 34 year old man with multiple sclerosis has taken an overdose of 100 tablets of Paracetamol with
intent to end his life. He has been brought to the Emergency department for treatment but is refusing all
interventions.
A. Assessment of suicide
B. Evaluate patient’s capacity to refuse treatment
C. Establish if patient has a previous mental illness

189. A five year old girl has had an upper respiratory tract infection (URTI) for three days and has been
treated with paracetamol by her mother. For the last 12 hours she has been hot and irritable with severe
pain in her right ear. What is the single most likely diagnosis?

A. Herpes zoster infection D. Otitis media


B. Impacted earwax E. Perforation of eardrum

190. A child had a patchy rash following treatment for sore throat and cervical lymph node enlargement
.What is the antibiotic that caused this rash?
A. Ampicillin
B. Erythromycin
C. Cefuroxime
D. Metronidazole
E. Tetracycline

191. A 6 month old baby had loss of consciousness after which he had jerky movement of hands and feet.
What is the most probable diagnosis?
A. Infantile spasm
B. Absence
C. Partial seizure simple
D. Atonic seizure
E. Partial complex

192. A 45 year old manual worker presented with a two hour history of chest pain radiating into his left
arm. His ECG is normal. What is the SINGLE most appropriate investigation?
A. Cardiac enzymes
B. Chest x-ray
C. CT scan
D. ECG
E. V/Q scan

193. All the following drugs do not cause bronchoconstriction except:


A. Atenolol
B. Salbutamol
C. Salmeterol
D. Ipratropium bromide
E. Cocaine

194. A 30 year old woman has injured her left lower chest in a road traffic accident. She has a blood
pressure of 80/50mmHg and a pulse rate of 120 beats / minute. On auscultation of the chest, bowel
sounds are heard. What is the SINGLE most likely diagnosis?
A. Diaphragmatic rupture
B. Flail chest.
C. Fractured ribs
D. Ruptured oesophagus
E. Tension pneumothorax

195. The body of a 65 year old man, who was treated for TB and bronchitis, was seen at autopsy. His legs
were swollen and his liver showed signs of a transudate fluid. What was the cause of the transudate?
A. Liver Cirrhosis
B. Alcoholic Liver disease
C. Cardiac failure
D. Budd-Chiari syndrome
E. Tuberculosis
196. A child is diagnosed to have vesico-uretric reflux. What would you tell his parents?
A. Requires antibiotic prophylaxis
B. In most, will require surgery
C. In most, kidneys will be scarred by age 5
D. Nothing can be done for him anymore
E. Reassure

197. A thin 18-year-old girl has bilateral parotid swelling with thickened calluses on the dorsum of her
hand. What is the single most likely diagnosis?

A. Bulimia nervosa
B. C1 esterase deficiency
C. Crohn’s disease
D. Mumps
E. Sarcoidosis

198. A 24 year old man has been complaining of difficulty in hearing his lectures. He also has been
feeling dizzy and now presents with a ringing sensation in her ears. What is the best treatment for this
patient?

A. Cyclizine
B. Metoclopramide
C. Hearing aids
D. Domperidone
E. Aspirin

199. A child playing with toys suddenly develops breathlessness and stridor. Which investigation will
lead to the diagnosis?

A. Laryngoscopy
B. Chest x ray
C. Peak flow meter
D. Arterial blood gas

200. A 79 year old man who is being treated with gonadotropin releasing hormone (GnRH) antagonist for
proven adenocarcinoma of the prostate attends for a follow up session. What is the most appropriate
investigation?

A. Serum AFP (Alpha-fetoprotein)


B. Serum PSA (prostate-specific antigen)
C. Serum acid phosphates concentration
D. Serum alkaline phosphatase isoenzyme concentration
E. Trans Rectal USG
PLAB 1 MOCK ANSWERS :Mock 1 Keys
1 E 41 A 81 D 121 E 161 E
2 B 42 B 82 B 122 B 162 C
3 E 43 A 83 C 123 A 163 B
4 A 44 C 84 C 124 C 164 B
5 D 45 C 85 A 125 C 165 B
6 A 46 A 86 A 126 B 166 B
7 B 47 C 87 B 127 B 167 E
8 D 48 A 88 A 128 A 168 D
9 D 49 A 89 C 129 C 169 D
10 C 50 B 90 A 130 D 170 D
11 C 51 A/C 91 A 131 A 171 D
12 A 52 A 92 E 132 E 172 A
13 A 53 C 93 A 133 B 173 C
14 A 54 A 94 D 134 A 174 C
15 D 55 A 95 D 135 D 175 E
16 A 56 C 96 A 136 A 176 A
17 C 57 D 97 B 137 B 177 B
18 A 58 E 98 D 138 B 178 D
19 B 59 C 99 C 139 A 179 C
20 B 60 A 100 B 140 C 180 B
21 B 61 D 101 B 141 B 181 C
22 D 62 A 102 E 142 D 182 C
23 C 63 D 103 A 143 D 183 D
24 A 64 C 104 B 144 D 184 D
25 E 65 B 105 A 145 A 185 C
26 D 66 C 106 C 146 E 186 A
27 D 67 B 107 A 147 D 187 D
28 B 68 A 108 A 148 E 188 B
29 C 69 B 109 B 149 B 189 D
30 E 70 C 110 A 150 C 190 A
31 A 71 A 111 A 151 B 191 A
32 C 72 C 112 E 152 D 192 A
33 B 73 E 113 B 153 C 193 A
34 A 74 B 114 A 154 C 194 A
35 A 75 A 115 C 155 B 195 C
36 B 76 A 116 C 156 A 196 B
37 C 77 D 117 B 157 A 197 A
38 B 78 B 118 A 158 D 198 A
39 A 79 A 119 A 159 D 199 A
40 C 80 B 120 B 160 C 200 B
Dr. Swamy PLAB Courses Ltd
PLAB 1 MOCK TEST: MOCK 2 TIME ALLOWED: 3HRS

SINGLE BEST ANSWER

1. A 23 year old woman presents with heavy menstrual bleeding. She complains of being tired all the
time and looks pale on examination. What is your initial investigation?

A. Endometrial sampling
B. Urea and electrolytes
C. Hysteroscopy
D. Pelvic ultrasound

2. A 6 year old child presents with swelling of face and feet and weight gain despite poor
feeding recently. What investigation should be carried out next?
A. Urinary albumin
B. Serum urea & creatinine
C. Full blood count
D. eGFR
E. Thyroid function tests

3. A patient presents with widespread ovarian carcinoma. She had bowel obstruction and severe colic
for 2 hours and was normal for few hours in between severe pain. What is the management?

A. S/C morphine, pain controlled analgesia (PCA)


B. Spasmolytics
C. Palliative colostomy
D. Oral Morphine
E. Laxatives

4. A 35 years old construction worker is diagnosed with indirect inguinal hernia. Which statement
below best describes it?

A. Passes through the superficial inguinal ring only


B. Lies above and lateral to the pubic tubercle
C. Does not pass through the superficial inguinal ring
D. Passes through the deep inguinal ring

5. A 56 years old lady has developed severe right sided headache which worsens whenever she comes
to bright light since the last four days. She feels nauseated but does not vomit. What is the most likely
diagnosis?

A. Subarachnoid Hemorrhage (SAH)


B. Brain Tumor
C. Migraine
D. Cluster Headache
E. Subdural Hematoma
6. A 4 year old boy presents with a sharp metal injury. He never had any vaccination as his
parents were worried about the side effects. What should be the next step?
A. Tetanus toxoid + full course of DPT vaccine
B. TT Immunoglobulin
C. A + B
D. No treatment

7. A 60 years old female presents with acute onset of bone and back pain following a rough journey in
car. On examination she has tenderness at mid-thoracic vertebrae with spasm, she feels better once
she bend forward. What is the SINGLE most probable diagnosis?

A. Osteoporotic fracture vertebrae


B. Myofacial pain.
C. Whiplash injury
D. Myocardial Infarction
E. Pancreatitis

8. A mother brings her child saying that he is having problems at school. His teachers say that he is
naughty, difficult to control and he can never sit still. It is also difficult to make him concentrate and
he is unable to do tasks that need him to work on a particular thing for a long time. What is the most
appropriate diagnosis?

A. Attention deficit hyperactivity disorder (ADHD)


B. Autism
C. Schizophrenia
D. Conduct disorder

9. Few weeks old child presents with profuse projectile vomiting. What is the most appropriate initial
action in this child?

A. USG
B. Check serum K level
C. ABG
D. Nasogastric tube
E. IV fluids

10. A 45 years old known hypertensive man presents with haematuria, proteinuria and oedema. What
is the definitive diagnostic test for him?

A. Urine protein
B. Renal biopsy
C. Renal function tests
D. Urine microscopy
E. Serum protein

11. A 7 year old boy is brought in by his mother. She is concerned that he does not have any
friends at school and is not doing well in his studies. He only likes to collect toy cars of
which he has 2000. He does not understand emotions and has poor interaction with his
siblings. He gets angry and throws tantrums if his daily routine is changed. What is the most
likely diagnosis?
A. Schizophrenia
B. Depression
C. Autism spectrum disorder
D. Tourette’s syndrome
E. Alice in Wonderland syndrome

12. A 20 year old man has a head on collision in a car. On presentation he is breathless and has chest
pain as he has fractured his 5 to 7 ribs. Chest x-ray shows fracture of 5th to 7th ribs. What is the
most appropriate initial action in this patient?
A. Antibiotics
B. Analgesia
C. Oxygen by mask
D. Physiotherapy
E. Refer to surgeon

13. A pregnant woman presents with knee pain on movements. The pain becomes worse at the end of
the day. Radiography shows decrease in joint space. Lab report shows C-reactive protein is 12mg/L.
What is the first line of medication for her?

A. Paracetamol
B. NSAIDs
C. Oral Steroid
D. Intra-articular steroid
E. Disease modifying anti-rheumatic drugs (DMARDs)

14. A 63 year old female with a history of osteoporosis suddenly fall on her outstretched hand while
shopping. X ray shows there is fracture at the distal radius with backward shift of the distal fragment.
What is the SINGLE most probable deformity?

A. Dinner fork deformity.


B. Coxavara.
C. Mallet finger
D. Cubitus valgus
E. Garden spade deformity

15. A 35 year old factory machine operator has been diagnosed with depression and prescribed
fluoxetine. What is the most important precaution to take?
A. Take it after meals
B. Take it empty stomach
C. Take it in the night
D. Take it in the morning
E. Not to combine it with analgesics

16. A young woman has lost 6 kg in 2 weeks. The weight loss was deliberate as she exercises
5 hours daily at the gym and is very concerned about her body. She admits to using laxatives
and inducing vomiting. Her BMI is 22. She also admits staying up until 3am cleaning her
apartment, spending lots of money on shopping and eating in very expensive restaurants.
What is most probable diagnosis?
A. Anorexia nervosa
B. Bulimia nervosa
C. Eating disorder not otherwise classified
D. Depression
E. Seasonal affective disorder
17. A 28 year old female presents with a 3 months history of diarrhoea. She complains of abdominal
discomfort and passing stool 20 times a day. On examination she is febrile. Barium enema shows
cobble stone mucosa. What is the most likely diagnosis?

A. Amoeba
B. Colon Ca
C. Gastroenteritis
D. Crohn’s Disease
E. Ulcerative Colitis

18. A 28 year old pregnant woman with poly-hydramnios and shortness of breath comes for an
anomaly scan at 31 weeks. On ultrasound, there is absence of gastric bubble. What is the most likely
diagnosis?

A. Duodenal atresia
B. Oesophageal atresia
C. Gastroschisis
D. Exomphalos
E. Diaphragmatic hernia

19. A 7 year old child is being investigated for tuberculosis. His parents do not agree on taking a
broncho-alveolar lavage sample under general anaesthesia (GA).What other sample will show growth
of the organism?
A. Blood
B. Throat swab
C. Gastric washings
D. Mantoux
E. Cerebrospinal Fluid (CSF)

20. A 49 year old woman presents to the Out Patient Clinic. Her oral glucose test, after 2 hours of
glucose intake, versus plasma level in 2 different tests are 6mmol/L and 10mmol/L. This situation can
be categorized as :

A. Impaired glucose tolerance


B. Impaired fasting glucose
C. Diabetes Mellitus Type I
D. Diabetes Mellitus Type II
E. Metabolic syndrome.

21. When prescribing an SSRI, what will you warn the patient about its time of action?
A. It will work within few days
B. It will within 1 week
C. It will work by 2-3 weeks
D. It will work by 2-3 months
E. It will work by 1 year

22. A 32 year old woman had progressive decrease in vision over 3 years. She is now diagnosed as
almost blind. What would be the mechanism?
A. Cataract
B. Glaucoma
C. Retinopathy
D. Uveitis
E. Keratitis
23. Patient in terminal stage of cancer, who is on morphine, presents with rattling breath sounds. What
medication will you administer?

A. Anti-Muscarinic
B. Oxygen
C. Steroid
D. Naloxone
E. Methadone

24. A 25 year old woman was brought to the A&E by her boyfriend. She has many superficial
lacerations. On examination there are many scars of old cut in her forearm. She is so distressed and
constantly says her boyfriend is going to end the relationship. She denies trying to end her life. What
is the most likely diagnosis?

A. Acute psychosis
B. Severe depression
C. Psychotic depression
D. Border line personality disorder
E. Schitzophrenia

25. A 57 year old female, who is suffering from hypertension, presented to the hospital with
complaints of recurrent falls when trying to get out of bed or get up from sitting. She is on some anti-
hypertensive therapy, with no other medical problems. What is the most likely cause of her fall?

A. Calcium channel blocker


B. Vertibrobasillar insufficiency
C. Bendrofluazide
D. Hypoglycemia
E. Infection

26. A 48 year old woman is admitted to the Accident and Emergency Department with a productive
cough and moderate fever. She often has central chest pain and regurgitates undigested food most
times but does not suffer from acid reflux. These symptoms have been present for the last three and
half months which affects both food and drink. A chest x-ray shows an air-fluid level behind a
normal-sized heart. Which is the single most likely diagnosis?

A. Pharyngeal Pouch
B. Hiatus Hernia
C. Bulbar Palsy
D. Achalasia
E. Tuberculosis

27. A 4 weeks old baby develops yellowing of skin and sclera. He is thriving well on breast
milk. On presentation he is playing well and active. What is the most likely cause of jaundice
in this child?
A. Breast milk Jaundice
B. Galactocaemia
C. ABO incompatibility
D. G6PD deficiency
E. CMV infection
28. A 15 years old boy who complains of pain in his legs. His pain has settled with aspirin. What is
the most probable diagnosis?
A. Leomyosarcoma
B. Liposarcoma
C. Painful Hip
D. Exostosis
E. Osteod Osteoma

29. An elderly patient who is known to be a diabetic presents to the hospital with drowsiness, tremors
and confusion. What investigation should be done next to help in further management?

A. Blood Sugar
B. ECG
C. Standing and lying blood pressure
D. Fasting Blood Sugar
E. CT Scan

30. A 67 year old man with dribbling of urine undergoes prostatic procedure. What serum osmolality
you would find in a patient after prostatic surgery?
A. Low Na
B. High K+
C. High Ca
D. High Na

31. A 6 month old baby boy has palmar grasp but cannot hold things with thumb and index
finger. He is not afraid of strangers, sits with support and vocalises. How is his
development?
A. Normal development
B. Delayed motor milestones
C. Refer for speech evaluation
D. Metabolic investigations
E. Neuroimaging

32. A young man has been brought to the hospital after being hit on the head by a metal rod. On
evaluation his GCS is 7. What is the best next step?
A. CT scan
B. MRI
C. PET scan
D. Call anaesthetist
E. Refer to a neurosurgeon.

33. A 55 year old woman was found collapsed at home, paramedics revived her but in the ambulance
she had a cardiac arrest and could not be revived. The paramedic’s report tells that the woman was
immobile lately due to hip pain and that they found ulcers on the medial side of ankle. She was a
diabetic and was on anti-diabetics. What is the cause of her death?

A. Acute myocardial infarction


B. Diabetic ketoacidosis
C. Pulmonary embolus
D. Acute pericarditis
E. Cardiac tamponade
34. A 61 year old man presents with headache. He also complains of dizziness and tinnitus. He has
recently realized he has visual problem. There is history of burning sensation in his fingers and toes.
On examination there is splenomegaly. He says he feels itchy after a hot bath. The lab results show
RBC
87 x104/L, Hb= 31.9/dL and platelet 796 x109. What is the most likely diagnosis?

A. Chronic myloid leukaemia


B. Chronic lymphoblastic leukaemia
C. Polycythaemia rubravera
D. Mylofibrosis
E. Non Hodgkin’s lymphoma

35. An Afro-Caribbean boy presented with fever and arthralgia. FBC shows:
Hb : 6 g/dL
Reticulocyte count : 40%
Peripheral Smear : Target cells, Howell-Jolly bodies. What is the most appropriate
diagnosis?
A. Haemophilia A
B. Thalassemia major
C. Sickle cell anaemia
D. Thalassemia minor
E. Acute lymphoblastic leukaemia

36. A 28 years old man presents with a temperature of 38.7˚C and neck stiffness. There is no rash. His
GP calls, asking what should be done. How would you advise the GP?

A. Give the patient IV Benzyl penicillin


B. Give the patient IV Doxycycline
C. Give the patient IV Ceftriaxone
D. Send for a blood culture
E. Arrange for a CT head

37. A patient presents with sudden onset of blindness in one eye. Flame shaped hemorrhage all over
retina and papilloedema on fundoscopy. He is hypertensive. What is the most probable diagnosis?

A. Central retinal vein occlusion


B. Retinitis Pigmentosa
C. Central retinal artery occlusion
D. Branch retinal vein occlusion
E. Branch retinal artery occlusion

38. A 37 years old lady has inflammatory cells on cervical smear. There is no dyskaryosis. One year
ago her smear was normal. On speculum examination the cervix was normal on appearance. What
will be the next step?

A. Repeat smear in 3 years


B. Repeat smear in 4 months
C. Repeat smear in 6 months
D. Colposcopy
E. Cervical biopsy
39. A 58 year old lady presented with urinary incontinence. She looks anxious for her condition.
Urine culture is sterile. Her urodynamic study is normal. What is the next step?

A. Antibiotics
B. Topical oestrogen
C. Systemic oestrogens
D. Duloxetine
E. Pelvic floor exercise

40. A mother is worried about cot death and asks about precautions to prevent it.
A. Lie the baby prone with feet at feet end of cot
B. Lie the baby prone with feet at head end of cot
C. Lie the baby supine with feet at head end of cot
D. Lie the baby supine with feet at feet end of cot
E. Lie the baby supine in the middle of the cot

41. A 34 year old man presents with breathlessness and tiredness. He is also pale and sweaty. His Hb
is 6.4 g/dL, MCV is 66fL. He is tachycardic and mildly hypotensive. What is the best management for
this patient?

A. Oral ferrous sulphate


B. IM ferrous sulphate
C. IV ferrous sulphate
D. Blood transfusion

42. A 24 year old male is admitted with acute severe asthma. Treatment is initiated with 100%
oxygen, nebulised Salbutamol and Ipratropium Bromide nebulizers and IV hydrocortisone. Despite
initial treatment there is no improvement. Which is the next step in management?

A. IV Aminophylline
B. IV Magnesium Sulphate
C. IV Salbutamol
D. IM Adrenaline
E. IV Adrenaline

43. A boy with a history of recurrent swollen tender joints on both knees and elbows and not able to
participate in sports. What is the investigation of choice to diagnose his case?
A. Rheumatic factor/ASO titre
B. Clotting factor
C. ESR

44. A man has a BP of 160/90mmHg, proteinuria ++. On ultrasound kidneys are equally reduced in
size with smooth borders and normal pelvicalyceal system. What is the cause of hypertension in this
man?
A. Chronic glomerulonephritis
B. Chronic Pyelonephritis
C. Bilateral Renal Artery Stenosis
D. Essential HTN
E. Polycystic Kidney
45. A 26 year old lady presents with a breast lump. On examination there is a non tender ,smooth,
firm mass in the upper outer quadrant of the left breast with a defined margin. What is the most
probable diagnosis?
A. Breast abscess
B. Mastitis
C. Fibro adenoma
D. Ca breast
E. Fibrocystic breast

46. How to perform CPR on a 5 month old child?


A. 15 chest Compressions with both hands
B. 30 chest compressions with single hand
C. 30 chest compressions with both thumbs
D. 15 chest Compressions with index and middle finger
E. 15 chest compressions with thumb and index finger

47. A 16 year old girl presents with dehydration, dry mucus membranes, tachypnoea and abdominal
pain. She is drowsy. Blood glucose is 18mmol/L. What is the next step?
A. ABG
B. Check BP
C. Serum Alcohol Levels
D. Blood Sugar levels
E. CT Scan

48. A lady 76 years old had loss of consciousness but was fine after 10 minutes. She was taking
Aspirin, Bendroflumethiazide and Simvastatin. All examinations were normal. CT scan was done.
What is the most appropriate management?

A. Add aspirin 300


B. Clopidogrel
C. Dipyramidamole
D. Clopidogrel +Dipyramidamole

49. A 55 year old male presents to the accident an emergency after an RTA with breathlessness,
engorged neck veins and a dull percussion note on the right side of his chest. On examination his
pulse is 140bpm and BP is 80/50mmHg, what is the most likely diagnosis?
A. Haemothorax
B. Haemopneumothorax
C. Tension pneumothorax
D. Simple pneumothorax

50. A 4 years old girl has had a temperature of 38.5 degree Celsius for the last two days
and refused to eat her food. Yesterday she developed sore throat and a small painful ulcer inside her
mouth. Today she has small blisters on the palms of her hands and soles of her feet which are painful
but not itchy. What is the single most likely underlying cause?

A. Coxsackie virus
B. Herpes Simplex virus
C. Staphylococcus aureus
D. Streptococcus pneumonia
E. Varicella zoster virus
51. A 12 year old boy with type 1 diabetes has poor long-term control. He is unconscious, hyper-
ventilating and dehydrated. His blood glucose is 28 mmol/L. What is the SINGLE most important
initial treatment?

A. Albumin (IV)
B. Bicarbonate (IV)
C. Insulin (IV)
D. Insulin (subcutaneous)
E. Saline 0.9% (IV)

52. A lady has c/o lower abdominal pain, low grade fever and vaginal discharge suggestive of pelvic
inflammatory disease (PID). She is not toxic. What is the appropriate empirical treatment for Pelvic
inflammatory disease?

A. Ceftriaxone
B. Metronidazole
C. Doxycycline
D. Erythromycin
E. Ciprofloxacin

53. A patient with proteinuria ++. What medication in this patient will prevent the progression of this
disease?
A. ACE inhibitors
B. Calcium channel blockers
C. Thiazide
D. Beta blockers
E. Alpha blockers

54. A young man presents with chest pain, palpitations, shortness of breath and perioral tingling
sensation after an argument with his girlfriend. ECG is normal. What is the best management for his
shortness of breath?

A. Amitryptilline
B. Diazepam
C. Rebreathe into a paper bag
D. Propanolol

55. A 17 years old girl comes to see her GP after having unprotected sex 2 days ago. She asks if her
GP can explain her how this prescribed procedure would work by helping her not to get pregnant.

A. It helps to prevent implantation


B. It helps in preventing or delaying ovulation
C. It causes an early miscarriage
D. It releases progestogen and stops ovulation
E. It causes local enzymatic reaction

56. A 73 year old man who was a smoker has quit smoking for the past 3 years. He now presents with
hoarseness of voice and cough since past 3 weeks. On X-ray a mass is visible in the mediastinum.
What is the best investigation to confirm the diagnosis?
A. Bronchoscopy
B. Thoracoscopy
C. USG
D. CT Thorax
E. Lymph node biopsy
57. A child presents with fever and maculo-papular rash on the back as shown in the given picture.
What is the most probable diagnosis?
A. Chicken pox
B. Kawasaki disease
C. Measles
D. Rubella
E. Scarlet fever

58. A 29 year old who is 38 weeks pregnant presents with a two hour history of constant abdominal
pain. She then passes 100ml of blood per vagina. On examination she has a rock hard uterus. What is
the next appropriate investigation?

A. Ultrasound scan
B. Cardiotocograph
C. Clotting screen
D. Haemoglobin
E. Kleihauer-Betke test

59. An 8 week pregnant lady is brought to the A and E due to severe vomiting. She was administered
IV fluids and oral anti-emetics. She still cannot tolerate anything orally. What is the next best
treatment for her?

A. Parenteral feeding
B. IV antiemetics
C. Termination of pregnancy
D. Proton Pump Inhibitor
E.IV Steroid

60. A child was brought to A&E with his parents. They say that he took his grand-dad’s medication.
There is extra systole in the ECG. Which drug was taken?
A. Digoxin
B. Amitriptyline
C. Atenolol
D. Ramipril
E. Bendroflumethiazide

61. A 15 year old girl was admitted with anemia, chest infection and thrombo-cytopenia. She was
treated and her symptoms regressed. She was brought again with fever and the same symptoms a few
days later. She also seems to have features of meningitis. What is the most likely diagnosis?

A. Acute Myeloid Leukaemia (AML)


B. Acute lymphoblastic leukemia (ALL)
C. Aplastic Anemia
D. Chronic myeloid leukaemia (CML)
E. Chronic lymphocytic leukaemia (CLL)

62. A 75 year old man comes in complaining of difficulty in passing urine, poor stream and dribbling
at the end of voiding urine and anorexia. Ultra sonography (USG) shows bilateral hydro-nephrosis.
What is the cause for these findings?
A. Benign prostatic hyperplasia (BPH)
B. Renal stones
C. Bladder stones
D. Prostatic cancer
E. Urinary tract infection

63. A 30 year old woman has experienced restlessness, muscle tension and sleep disturbance on most
days over the last six months. She worries excessively about a number of every day events and
activities, and is unable to control these feelings which are impairing her ability to hold down her job.
What is the most likely diagnosis?

A. Panic Disorder
B. Generalized Anxiety Disorder
C. Pheochromocytoma
D. Acute Stress Disorder
E. Social Phobia

64. A 20 year old lady is suffering from fever and loss of appetite. She has been diagnosed with
Toxoplasmosis. What is the treatment for Toxoplasmosis?
A. Pyrimethamine
B. Pyrimethamine plus Sulfadiazine.
C. Clindamycin
D. Spiramycin
E. Trimethoprim and Sulfamethoxazole

65. An 80 year old diabetic lady presents with redness and swelling over her right foot. It is tender to
touch, warm and glossy. Choose the most appropriate complications this patient might develop?
A. Meningitis
B. Sepsis
C. Ulcer
D. Gangrene

66. A six week old formula-fed baby boy is found at Child Health Surveillance to be deeply
jaundiced. His weight gain is poor and his stools are pale. What is the most likely diagnosis?
A. Galactosemia
B. Biliary Atresia
C. Glucose-6-phosphate dehydrogenase deficiency (G6PD)
D. Rhesus Incompatibility
E. Congenital Viral Infection

67. A 65 year old lady presents with dyspareunia. What will you give her for her condition?
A. Hormone replacement therapy (HRT)
B. Combined Oral Contraceptive Pill (COCP)
C. Oestrogen gel
D. Testosterone gel

68. Pregnant lady had her ante natal screening for HIV and Hepatitis B. What more antenatal infection
should she be screened for?
A. Rubella and syphilis
B. Toxoplasma and rubella
C. Syphilis and toxoplasma
D. Hepatitis C and E
E. Hepatitis A and C
69. A patient presents with vaginal discharge, unpleasant odour with pH more than 4.5 and mild
dysuria. What is the most likely diagnosis?

A. Trichomonasis
B. Gardenella
C. Gonorrhoea
D. Bacterioides

70. A patient’s cervical smear shows moderate dyskaryosis. What is your next step in her
management?
A. Colposcopy
B. Repeat cervical smear
C. Hysterectomy
D. Cervical biopsy
E. Large loop excision of the transformation zone (LLETZ)

71. A male patient presents with recurrent pain in feet and ankle joints. On X-Ray there were punched
out lesions, with erosions. What is the most probable diagnosis?
A. Pseudogout
B. Gout
C. Osteoarthritis
D. Rheumatoid arthritis

72. A 6 week old child presents with the following features of progressive cyanosis, poor feeding,
tachypnoea during the first two weeks of life and holo systolic murmur. What is the most appropriate
condition?

A. Atrial septal defect E. Tetralogy of Fallot (TOF)


B. Ventricular septal defect
C. Tricuspid atresia
D. Patent ductus arteriosus

73. A 32 year old woman of 39 weeks of gestation attends the antenatal day unit feeling very unwell
with sudden onset of epigastric pain associated with nausea and vomiting. Her temperature is 36.7˚C.
On examination, she is found to be tender over the right upper quadrant. Her blood results show mild
anaemia, low platelets, elevated liver enzymes and haemolysis. Select the most likely diagnosis?

A. Acute fatty liver of pregnancy


B. Acute pyelonephritis
C. Cholecystitis
D. HELLP syndrome
E. Acute hepatitis

74. A 10 years old boy is clinically obese and the shortest in his class. He had a renal transplant last
year and his mother is worried that he is being bullied. What is the most probable diagnosis?
A. Cushing’s Syndrome
B. Congenital Hypothyroidism
C. Pseudo Cushing’s Syndrome
D. Lawrence Moon Biedel Syndrome
E. Down’s syndrome
75. A patient with severe pre eclampsia has just delivered a baby at 37 weeks gestation. 20min after
delivery she has her first fit. Her blood pressure is 160/90 mmHg. An IV line is established. What
will you administer?
A. MgSO4 drip
B. Hydralazine
C. Methyldopa
D. MgSO4 bolus dose
E. Calcium Gluconate

76. A 30 year old patient presents with heavy bleeding. She is in a stable relationship and is not keen
for conception. USG shows normal uterus. What would you advise her?
A. Mirena coil
B. Combined Oral Contraceptive Pill (COCP)
C. IUCD
D. Progesterone only pill
E. Tranexamic acid

77. A woman with sickle cell disease complains of heavy menstrual blood loss. What is the most
appropriate treatment for her?
A. Combined Oral Contraceptive Pill (COCP)
B. Mirena
C. Depot provera
D. Copper T insertion
E. Transdermal patch

78. A 34 year old man with a white patch on the margin of the middle third of the tongue. Which is
the SINGLE most appropriate lymph node involved?

A. External iliac lymph nodes F. Submental lymph nodes


B. Pre aortic lymph nodes G. Submandibular lymph nodes
C. Aortic lymph nodes H. Deep cervical lymph nodes
D. Inguinal lymph nodes
E. Iliac nodes

79. A 32 years old man travelled to Thailand and Indonesia, did not have malarial prophylaxis,
presents after 2 weeks with fever, petechiae and malaise. What is the likely cause?
A. Dengue Fever
B. Malaria
C. Infectious Mononucleosis
D. Idiopathic thrombocytopenic purpura (ITP)
E. HIV

80. An 18 year old girl (sexually active) presented with heavy menstrual bleeding and
dysmenorrhoea. What is the best contraceptive method you would advise her?
A. Combined Oral Contraceptive Pill (COCP)
B. Depoprovera
C. IUCD
D. Mirena coil
81. A 35 year old female attempts suicide 10 times. There is no history of psychiatric problem and all
neurological examinations are normal. What will be the best treatment?
A. Problem focussed treatment
B. Cognitive behaviour therapy (CBT)
C. Antipsychotic
D. Antidepressant
E. Electro convulsive therapy (ECT)

82. A lady came in gynaecology assessment unit with a history of 8 weeks pregnancy and bleeding
per vagina for the last two days. On bimanual examination uterus is 8 week in size. On speculum
examination cervical os is closed. How do you confirm the viability of foetus?
A. Transvaginal USG Scan
B. Serum Β-HCG
C. Urinary B-HCG
D. Abdominal USG Scan
E. Per speculum examination

83. A 3 years old child had a high temperature for four days and he had not seen a doctor. Then
mother notices rashes on buccal mucosa and some around the mouth. What is the most appropriate
diagnosis?
A. Measles
B. RoseolaInfantum
C. Rubella
D. Chicken Pox
E. Impetigo

84. An obese 37 year old woman with family history of DVT, who smokes 20 cig/ day comes to the
sexual clinic with menorrhagia and advise on contraception. What is the best method of contraception
for her?
A. Mirena coil
B. IUCD
C. Combined Oral Contraceptive Pill (COCP)
D. Depoprovera

85. Which of the following electrolyte abnormalities is most likely to be seen in SIADH?
A. High serum Na, low serum osmolarity, high urine osmolarity
B. Low serum Na, low serum osmolarity, high urine osmolarity
C. Low serum Na, high serum osmolarity, high urine osmolarity
D. High serum Na, high serum osmolarity, low urine osmolarity
E. Normal Na, normal serum osmolarity, normal urine osmolarity

86. A 34 year old woman complains of low mood, loss of interest in everyday activities, loss of
appetite and weight. She also has trouble falling asleep and wakes up early in the morning. What is
the best treatment for her?
A. Fluoxetine
B. Minaprine
C. Moclobemide
D. Curcumine

87. A 35 year old pregnant woman has been having tingling and numbness of her thumb, index and
middle fingers for a while. She has been treated with local steroids but it has not helped much, and
now she has been planned to undergo a surgical procedure. Which of the following structures will be
incised?
A. Flexordigitorum profundus
B. Transverse carpel ligament
C. Palmar aponeurosis
D. Extensor retinaculum

88. A patient presents with facial oedema, raised JVP, shortness of breath and cough. A chest X ray
shows superior vena cave obstruction. What is the immediate management?
A. Furosemide
B. Dexamethasone
C. Radiotherapy
D. Stenting

89. A 41 year old woman, who has completed her family, has suffered from extremely heavy periods
for many years. No medical treatment has worked. She admits that she would rather avoid open
surgery. Scan shows a normal uterus. After discussion, you collectively decide on a procedure that
would not require open surgery or general anaesthetic. Select the most appropriate management for
menorrhagia in this case?
A. Endometrial ablation
B. Hysterectomy
C. Hysteroscopic laser/ resection of fibroids
D. Myomectomy
E. Uterine artery embolization

90. A 28 year old woman with 8 weeks amenorrhoea comes with vaginal bleeding. On examination os
closed, uterus corresponding to period of amenorrhea (POA) -12 weeks. What is the single most
appropriate diagnosis?
A. Threatened abortion
B. Inevitable abortion
C. Molar pregnancy
D. Ectopic pregnancy

91. A 27 year old man presents with abdominal pain. He says his urine is dark. On examination his
BP is 160/105mmHg. What is the most appropriate investigation?

A. Ultra sound
B. Renal biopsy
C. CT Scan
D. Urine protein
E. Urine microscopy

92. A 32 years old woman had normal vaginal delivery 10 days ago. Her uterus has involuted
normally. Choose the single most likely predisposing factor for postpartum hemorrhage from the
options given below?

A. Retained product
B. Disseminated intravascular coagulation (DIC)
C. Uterine infection
D. Von Willebrand’s disease
E. Primary postpartum

93. A 23 year old woman with painless vaginal bleeding in 36 weeks of pregnancy otherwise seems to
be normal. What should be done next?
A. Vaginal U/S
B. Abdominal U/S
C. Vaginal examination
D. Reassurance

94. A 30 years old woman has Pelvic inflammatory disease (PID), which was treated with
Metronidazole and Cephalosporin. It is getting worse. What is the next investigation?
A. Endocervical swab
B. Ultra sonography (USG)
C. Laparotomy
D. High vaginal swab

95. A pregnant patient with RH negative who hasn’t been previously sensitized delivers her first baby
without any problems. What would be the latest time to administer anti sensitization?
A. 6 hours post partum
B. 24 hours post partum
C. 48 hours post partum
D. 72 hours post partum
E. 5 days post partum

96. A 24 year old woman presents with deep dyspareunia and severe pain in every cycle. What is the
initial investigation?
A. Laparoscopy
B. Pelvic ultrasound
C. Hystroscopy
D. Vaginal swab

97. A drug addict has bitten a police officer. What would you advise the police officer?
A. HIV serology
B. Hepatitis & HIV serology
C. No test required at this stage
D. Tetanus toxoid

98. A 27 year old lady came to the A &E department 10 days ago with fever, suprapubic tenderness
and vaginal discharge. Pelvic inflammatory disease was diagnosed. She has been on the antibiotics for
the last 10 days. She presents with lower abdominal pain. The temperature is 39.5˚C. What is the most
appropriate next management?

A. Vaginal swab
B. Endocervical swab
C. Ultra sound
D. Abdominal X-ray
E. Laparoscopy

99. A 2 months old baby who has ambiguous genitalia presents to the A and E with vomiting. Lab tests
are as follows.
Na: 125mmol/L
K: 6 mmol/L What is the most likely diagnosis?
A. Fragile X Syndrome
B. Turner’s Syndrome
C. Noonan Syndrome
D. Congenital Adrenal Hyperplasia
100. A 25 year old woman presents with a painful shallow ulcer on the vulva. What investigation has
to be done?

A. HSV antibodies
B. Syphilis serology
C. Swab for Haemophilusducreyi
D. Urine culture
E. Blood culture

101. A 25 years old woman with type one diabetes has delivered a baby weighing 4.5 kg. Her uterus
is well contracted. Choose the single most likely predisposing factor for developing postpartum
hemorrhage from the options?
A. Atonic uterus D. Large placental site
B. Cervical/vaginal trauma E. Rupture uterus
C. Retained product

102. A young man presents complaining of seeing coloured halos when his eyes are closed. What
drug is most likely to be the cause of his symptoms?
A. Heroin
B. Cocaine
C. Ecstasy
D. Lysergic acid diethylamide (LSD)

103. A 39 year old woman will undergo tubal sterilization and she wants to know the failure rate of
this type of sterilization.
A. 1:50
B. 1:200
C. 1:500
D. 1:1000
E. 1:5000

104. A 28 year old woman presents with 18 weeks of amenorrhoea. A scan was done and it revealed
an ovarian mass on the right side with solid and cystic areas, with a bright nodular structure
suggestive of tooth. What is your most likely diagnosis?
A. Ovarian cancer
B. Endometriosis
C. Tubo-ovarian abscess
D. Dermoid cyst

105. A 34 years old woman presents with truncal obesity, easy bruising, hyperglycemia, high blood
pressure and depression. Which of the following investigations will be most helpful in localising the
cause for Cushing’s Syndrome?

A. Serum Cortisol
B. 24 hours Urinary Cortisol
C. Low dose dexamethasone suppression test
D. High dose dexamethasone suppression test
E. Overnight dexamethasone suppression test

106. A 67 year old female who had undergone a radical mastectomy, now comes with the complaint
of swelling and redness in her right upper limb. Involvement of which of the following structures
explain these symptoms?
A. Epi-trochlear lymph nodes
B. Cephalic vein
C. Subclavian artery
D. Axillary group of lymph nodes
E. Long thoracic nerve

107. A 27 year old man presents with jaundice. On examination there is hepatosplenomegaly.
Peripheral blood film shows occasional spherocytes. What is the next appropriate investigation?
A. Direct Coomb’s test
B. Bone marrow test
C. Indirect Coomb’s test
D. RBC life span
E. Osmotic fragility test

108. A 40 years old lady complains of 10 months amenorrhoea. She presents with hot flush, mood
swings, irritability and insomnia. What will be the single most effective diagnostic test for this lady?
A. Serum FSH and LH E. Serum cortisol
B. Serum FSH
C. Serum Progesterone
D. Serum estradiol

109. A homeless lady presents with cough and fever. She complains of night sweats and weight loss.
Chest X-ray has been done and shows opacity. What is the next appropriate management?
A. Acid fast bacilli
B. Mantoux test
C. Interferon gamma testing
D. Bronchoscopy
E. CT scan.

110. A woman with a spontaneous abortion at 14 and 23 weeks of gestational age presents at 8 weeks
gestational age with bleeding PV. What is the most appropriate intervention?
A. Bed rest
B. Aspirin
C. Reassurance
D. Trans-vaginal sonography (TVS)

111. A 28 year old woman at 39 weeks of gestation is in labour. She develops abdominal pain and
heart rate of 125 beats/min. blood pressure is 100/42mmHg, temperature 37.2oC and saturations 99%
in air. On examination, her lower abdomen is exquisitely tender. The CTG, which was previously
normal, now shows reduced variability, and late deceleration develops with slow recovery. She has
had one previous LSCS for a breech baby. Choose the most appropriate post C-Section complication
for this lady?
A. Endometriosis
B. Urinary tract infection
C. Urinary tract injury
D. Pleurisy
E. Uterine rupture

112. A 40 year old manual worker presents with a swelling in the groin. On examination the mass is
found to be just above and lateral to pubic tubercle. There is no cough impulse and it is reducible. On
applying the pressure on the internal ring there is no cough impulse seen. What is the most probable
diagnosis?
A. Direct Inguinal Hernia
B. Indirect Inguinal Hernia
C. Femoral Hernia
D. Strangulated Hernia
E. Femoral Aneurysm

113. A 37 years old woman believes that her neighbours have been using her shower while she is
away from home. Her 42 years old partner is convinced about this and calls the police. What term best
describes this situation?
A. Capgras Syndrome
B. Cotard Syndrome
C. Delusion of Persecution
D. Folie a Deux
E. Munchausen Syndrome

114. A 24 year old woman has eight weeks amenorrhoea, right sided pelvic pain and vaginal bleeding.
She is apyrexial. Peritonism is elicited in the right iliac fossa. Vaginal examination reveals right-sided
cervical excitation. What is the most probable diagnosis?
A. Ectopic Pregnancy
B. Salpingitis
C. Endometriosis
D. Ovarian Torsion
E. Ovarian tumor

115. A 30 years old female attends outpatient dept with c/o fever & dry cough. She says that she had
headache, myalgia & joint pain like one week ago. On examination Pulse: 100 b/min, Temp: 37.5ºC.
Investigations: CXR: bilateral patchy consolidation. What is the single most likely causative
organism?

A. Pneumococcal Pneumonia
A. Legionella
B. Mycoplasma
C. Klebsiella
D. Chlamydia pneumonia

116. A 28 year old woman presents with foul smelling discharge with pH < 4.5. What is the single
most likely diagnosis?
A. Trichomoniasis
B. Candidiasis
C. Bacterial vaginosis
D. Chlamydia

117. A 27 year old presents with abdominal pain, bleeding, vomiting and diarrohea. Her last
menstrual period was 7 weeks ago. On examination abdomen is tender and BP is 90/60mmHg. What
is the next appropriate management?

A. Immediate laparotomy
B. Laparoscopy
C. Salpingotomy
D. Salpingectomy
E. Metotrexate
118. A 65 year old male presents with dyspnoea and palpitations. On examination pulse 170 bpm, BP
120/80mmHg. Carotid massage has been done as first instance. What is the next step of
management?
A. Adenosine
B. Amlodipine
C. DC cardioversion
D. Lidocaine
E. Beta blocker

119. A patient presents with mild dyskaryosis. One year ago smear was normal. What is the most
appropriate next step?
A. Cauterization
B. Repeat smear
C. Swab and culture
D. Cone Biopsy
E. Colposcopy

120. A young girl presented in Gynae assessment Unit with lower abdominal pain and per vaginal
bleeding after a history of hysteron-salpingo-graph as a part of her infertility treatment. On
observation her BP was 90/50mmHg and pulse was 120bpm and examination revealed rigid abdomen.
What is the most appropriate next investigation?

A. CT abdomen
B. X-ray abdomen in erect and supine
C. Ultrasound abdomen
D. Coagulation profile
E. X-ray chest

121. A 62 year old female who had an episode of postmenopausal bleeding PV was sent for a TV
USG. Endometrial thickness was found to be 7mm. What is the next best step in management?
A. Total hysterectomy
B. Laparoscopy
C. Hysteroscopy and Endometrial biopsy
D. Pipelle’s endometrial sampling
E. MRI of pelvis

122. A patient presents with ectopic pregnancy. Which is the most important risk factor for it?
A. Polycystic ovary syndrome (PCOS)
B. PID with chlamydia,
C. Depo-provera
D. Instrumentation

123. A 30 years old patient came to outpatient dept: with complains of breathlessness and dry cough.
He has lost 5kg weight in last 2 months. He is an IV drug abuser. Investigation: CXR: Bilateral
interstitial shadowing. What is the single most likely causative organism?

A. Klebsiella
B. Mycobacterium Tuberculosis
C. Chlamydia pneumonia
D. Pneumocystitis pneumonia (PCP)
E. Chlamydia psitacci
124. An elderly gentleman who used to work in the shipyard industry presented with cough and
shortness of breath few weeks to months was given salbutamol nebulization and antibiotics and
admitted to the ward. He died 3 days later. His CT scan shows patchy infiltrates, pleural thickening
and pleural effusions. Why is this a Coroner’s case?

A. Patient got wrong diagnosis or wrong management


B. Patient died soon after admission
C. Death could be due to occupational illness

125. A 64 year old man complains of increasing shortness of breath and cough for the past 18 months.
He coughs up a tablespoon of mucopurulent sputum with occasional specks of blood. What is the
most likely underlying cause?

A. Acute bronchitis
B. Bronchiectasis
C. Chronic bronchitis
D. Lung cancer
E. Pneumonia

126. A 26 year old builder had a cement burn over his trunk. What is the best management for his
blisters?

A. Dressing
B. Cut the base of his blisters
C. Wash his blisters with 0.9% saline/Irrigation
D. Reassure
E. IV fluids

127. An 87 year old man was found collapsed at home. The ambulance crew has found the following
medications at his house. Simvastatin, Frusemide, Aspirin and Metformin. What electrolyte
abnormality is associated with Frusemide use?

A. Hyponatremia
B. Hypokalemia
C. Hyponatremia plus Hypokalemia
D. Hyperkalemia

128. A 3year old boy presents with a 1 day history of being unwell. He appears shocked and has 3
hours old rash made up of urticaria and purpural spots. His respiratory rate is 30/min, saturations 94%
on air, temperature 39.0ºC and capillary refill time 1sec. Urine is clear on dip stick. What is the single
investigation most likely to lead to a diagnosis?

A. Blood for Culture and Sensitivity


B. ESR
C. Chest X Ray
D. Urine for Culture Sensitivity
E. CSF Analysis

129. A 45 year old lady came to family planning clinic for contraception advice. She is not keen to be
pregnant for the next 3 years. Her recent USG report showed multiple small submucosal fibroid. What
is the best method of contraception for here from the list below?
A. Etonogestrol
B. Combined OCP
C. IUS (intrauterine system)
D. Progestogen only pill
E. IUCD (intrauterine contraceptive device)

130. Which of the following anaemia is seen in a hypothyroid patient?


A. B12 deficiency anaemia
B. Folic acid deficiency anaemia
C. Iron deficiency anaemia
D. Haemolytic anaemia

131. A young lady with primary amenorrhea has normal Leutinising hormone (LH), Follicle
stimulating hormone (FSH), Estradiol and Prolactin. Choose the single most likely diagnosis?

A. Polycystic ovary syndrome (PCOS)


B. Premature ovarian failure
C. Absent uterus
D. Absent ovaries
E. Turner’s syndrome

132. A 65 year old female patient was given Tamoxifen, which of the following side effect caused by
it will concern you?
A. Fluid retention
B. Vaginal bleeding
C. Loss of appetite
D. Headache and dizziness

133. A patient was admitted to A&E with prolonged hematemesis. Endoscopy revealed a bleeding
gastric ulcer which was then sutured. During the procedure, patient received 15 units of blood. The
next day, the drain contained 1 unit of blood. What is the next step in the management of this patient?

A. Urgent USG
B. Urea breath test
C. H.Pylori serology
D. Manometry

134. A 7 year old boy is brought by his mother who says that he was well at birth but has been
suffering from repeated chest and gastro intestinal infections since then. She also says that he is not
growing well for his age. What is the likely condition in this child?
A. Cystic fibrosis
B. Severe combined immunodeficiency (SCID)
C. Primary T cell immunodeficiency
D. Primary B cell immunodeficiency
E. Malabsorption

135. A 77 year old man is admitted to hospital for investigation of an irregular heart beat. The
conducting system of the heart includes which one of the following anatomical structures?
A. Atrioventricular node
B. Chordae tendineae
C. Fibrous skeleton of the heart
D. Interatrial septum
E. Phrenic nerve

136. A 44 year old woman with breast cancer had an extensive removal and lymph node clearance.
She needs an adjunctive treatment. Her mother had cancer when she was 65 years of age. Which of
the following factors will be against the treatment?

A. Family history
B. Tumour grading
C. Lymph node involvement
D. Her age

137. A 55 year old man presents with history of weight loss and tenesmus. He is diagnosed with rectal
carcinoma. Which risk factors help to develop rectal carcinoma except following?
A. Smoking
B. Family history
C. High fibre diet
D. Previous Carcinoma
E. High fat diet
F. Polyp

138. A 67 year old woman has presented with a hard, irregular, poorly defined 5 cm lump in her right
breast. She has a bruise on the surface of the right breast and there is no discharge. What is the most
likely diagnosis?

A. Fibro-adenosis
B. Fat Necrosis
C. Fibro-adenoma
D. Duct ectasia
E. Ca Breast

139. A 21 year old college student presents with neck stiff, headache, vomiting and photophobia. A
diagnosis of viral meningitis is made. What is the most prominent white cell most likely to be seen in
the CSF of this patient?

A. Monocytes
B. Lymphocytes
C. Polymorphs
D. Eosinophils
E. Granulocytes

140. A Caucasian patient with nephropathy now presents with hypertension. What is the best drug for
him?
A. Propranolol
B. Ramipril
C. Nifedipine
D. Losartan
E. Bendroflumathiazide

141. A 49 year old man complains of fullness in the left ear, recurrent vomiting and tinnitus. What is
the most appropriate medication for this condition?

A. Buccal Prochlorperazine
B. Oral Chlorpheniramine
C. Oral Fluphenazine
D. Buccal Midazolam
E. IV Ranitidine

142. A 27 year old woman had hit her neck in RTA but no complains of tingling or motor loss. What
is the most appropriate investigation?

A. MRI Scan
B. Plain X-ray
C. CT Cervical
D. Diagonal X-ray

143. A 24 years old lady had been low after the death of her husband and had stopped contacting her
family. She was started treatment on SSRIs and felt better after a few months. On discontinuing the
medications she starts feeling that she has developed cancer, just like her husband. What is the most
appropriate next step?

A. Continue SSRIs
B. Add TCAs
C. Neuropsychiatric analysis
D. Cognitive behavioural therapy (CBT)
E. Antipsychotics

144. A 33 year old female patient presents with a family history of anaemia. She complains of
weakness, tingling and numbness in her hands and feet. Lately she also has begun to stumble and fall
at night. She is taking antidepressants for low mood. What is the best treatment for her?

A. Oral ferrous sulphate


B. Oral folic acid
C. Oral B12
D. IM hydroxycobalamine

145. A 65 year old man presents with a 3rd degree haemorrhoid, what will be the single most
appropriate action?
A. Rubber band ligation
B. Sclerotherapy
C. Do nothing
D. Haemorroidectomy
E. Cryotherapy

146. A man had a soft mass on his mandible. Mass is freely mobile and has started growing
progressively over the past six months. The mass still moves freely. What is the best investigation for
this patient?
A. Fine-needle aspiration cytology (FNAC)
B. CT scan
C. X ray
D. Magnetic resonance imaging (MRI)
E. Erythrocyte sedimentation rate (ESR)

147. A 24 years old schizophrenic has been under antipsychotic treatment for the last 1 year and now
complains of erectile dysfunction. Which drug is most likely to have caused this effect?
A. Fluoxetine
B. Citalopram
C. Clozapine
D. Haloperidol
E. Risperidone

148. A 27 year old female was brought to the A & E by her friend from a movie theatre. She
complaints of sudden severe pain in the eye followed by vomiting and also was seeing coloured halos.
She gives a past history of on-and-off headaches which used to resolve spontaneously. On
examination a fixed, dilated, ovoid pupil could be seen. What is the initial investigation?
A. CT Head E. Applanation Tonometery
B. MRI Orbits
C. Blood culture and sensitivity
D. Toxicology screen

149. A 61 year old man has been referred to outpatients with frequent episodes of breathlessness and
chest pain associated with palpitations. He has a regular pulse rate of 60 beats/minute. His ECG
shows sinus rhythm. What is the most appropriate investigation to be done?
A. Cardiac Enzymes
B. Chest X Ray
C. ECG
D. Echocardiogram
E. 24 Hour ECG

150. A 40 year old male with slow growing mass at the angle of mandible. The mass is smooth, firm
and mobile. What is the most likely diagnosis?
A. Submandibular bone tumour
B. Pleomorphic adenoma of parotid
C. Parotid carcinoma
D. Lipoma
E. Submandibular lymph node

151. A 19 year old boy was brought in by his mother with complaint of lack of interest and no social
interactions. He has no friends, he doesn’t talk much, he has interest only in collecting cars/vehicles,
he has around 2,000 toy cars. What is the most appropriate diagnosis?
A. Borderline personality disorder
B. Depression
C. Schizoaffective disorder
D. Autistic spectrum behaviour disorder

152. A 52 year old woman speaks rapidly without any pause and ignores interruptions. She doesn’t
even pause to take enough breaths. What term best describes this kind of speech?
A. Flight of Ideas
B. Broca’s Aphasia
C. Wernicke’s Aphasia
D. Pressure of Speech
E. Verbal dysphasia

153. A 28 year old man presents with a maculo-papular rash over his trunk and palms. He also has
numerous mouth ulcers. He had penile ulcer which healed 2 weeks ago. What will you do to confirm
diagnosis?
A. Polymerase chain reaction (PCR) for treponema and non-treponemal antibodies
B. Dark ground microscopy from mouth ulcer
C. Blood culture for treponema
D. Dengue Fever

154. A four month old girl has severe failure to thrive and increasing jaundice which was first noticed
at one week of age. She has an enlarged liver and scratches on her skin. Her parents have been
unable to seek medical care. What is the most likely diagnosis?
A. Biliary atresia
B. Glucose-6-phosphate dehydrogenase deficiency(G6PD)
C. Gilbert syndrome
D. Hepatitis B
E. Spherocytosis

155. An old man presented with fever and weakness. On examination he was found to have rashes as
shown in the given picture. Choose the right option from the following options.
A. Plantar Petechiae
B. Lipoderma Sarcoma
C. Talengectasia
D. Cellulitis
156. A 60 year old man was brought in by his wife complaining of ataxia, urinary incontinence and
erectile dysfunction. He also complains of rigidity and slowing of movement with a pill-rolling tremor
of the hands. What is the most likely diagnosis?
A. Parkinson's Disease D. Huntington's disease
B. Idiopathic Parkinson's Disease
C. Shy-Drager Syndrome

157. A 24 years old woman presents with episodes of peri-oral tingling and carpo pedal spasms every
time she has to give a public talk. This also happens to her before interviews, exams and after
arguments. What is the best management strategy for this patient?
A. Diazepam
B. Rebreath into a paper bag
C. Desensitization
D. Buspirone
E. Propranolol

158. A 6 weeks old child presents with profuse projectile vomiting. What is the first thing you will
do?
A. USG
B. Check serum K level
C. Arterial blood gas (ABG)
D. Nasogastric (NG) tube
E. IV fluids

159. A 55 year old woman who attends the clinic has recently been diagnosed with a depressive
episode. She complains of unintentionally walking early in the morning, a recent disinterest in sex
and a loss of appetite, losing 5 kg in weight in the last month. She feels that her mood is worse at the
beginning of the day. What is the most likely diagnosis for this patient?
A. Mild Depression
B. Moderate Depression
C. Severe Depression
D. Low mood
E. Pseudo Depression

160. A man has reducible bulge below the pubic turbercle, and on occlusion of the deep inguinal ring,
cough impulse is present. What is the most likely diagnosis?
A. Direct Inguinal Hernia
B. Indirect Inguinal Hernia
C. Femoral Hernia
D. Spigelian Hernia
E. Lumbar Hernia
161. A 60 year old woman is admitted to hospital after a fall. She is noted to have poor eye contact.
When asked how she is feeling, she admits to feeling low in mood and losing enjoyment in all her
usual hobbies. She has also found it difficult to concentrate, feels that she is not good at anything,
feels guilty over minor issues and feels very negative about the future. What is the most likely
diagnosis?

A. Mild Depression
B. Moderate Depression
C. Severe Depression
D. Psychosis
E. Seasonal depression

162. A 35 years old woman undergoing treatment for Tuberculosis presents with a malar rash,
photosensitivity and haematuria. What is the single most likely positive antibody?

A. AntiDs DNA
B. Anti Sm
C. Anti Histone
D. Anti La
E. Anti Centromere

163. A 66 years old woman has been brought to the hospital on a number of occasions with a history
of loss of memory. Her past history is significant for a MI six years ago. It is noted that she has a step
wise worsening of her cognitive functions. What is the most likely diagnosis?

A. Alzhemier’s Disease
B. Vascular Dementia
C. Pick’s Dementia
D. Huntington’s Disease
E. Lewy Body Dementia

164. A 54 year old man with alcohol dependence has tremor and sweating three days into a hospital
admission for a fractured femur. He is apprehensive and fearful. What is the SINGLE most
appropriate treatment?

A. Acamprosate
B. Chlordiazepoxide
C. Lorazepam
D. Lofexidine
E. Procyclidine

165. A patient diagnosed with HIV has received no treatment for HIV. He now presents with dry cough and
shortness of breath. Chest x-ray showed bilateral peri hilar opacity. Pneumocystitis carini was
indicated. What is the appropriate treatment?
A. Amphotericin B IV
B. Trimethoprim
C. Sulfadiazine
D. Gancyclovir
E. Dexamethasone
F. Pentamidine Isetionate

166. A mother presents with her 6-month-old son who is vocalizing. She has noticed that he does not
respond to loud noises. His motor milestones are normal. What is the best management strategy?
A. Arrange Hearing test
B. Assess Developmental Milestones
C. Reassure
D. Refer to a Speech Therapist
E. MRI Brain

167. A type 2 diabetic patient is undergoing a gastric surgery. What is the most appropriate pre-op
management?
A. Start him on I.V insulin with glucose and potassium just before surgery
B. Stop his oral hypoglycaemics on the day of the procedure
C. Continue regular oral hypoglycaemics
D. Stop oral hypoglycaemicsthe previous night and start IV insulin with glucose and potassium before
surgery
E. Change to short-acting oral hypoglycemics

168. A woman diagnosed with Ca breast presents now with urinary frequency. Which part of the brain
has the metastasis spread to?
A. Brainstem
B. Pons
C. Medulla
D. Diencephalon
E. Cerebral Cortex

169. A 29 year old lady came to A & E with complaints of palpitations that have been there for the
past four days and also feeling warmer than usual. Upon examination her heart rate was 154 bpm and
rhythm was irregular. What is the treatment for her condition?
A. Amiadarone E. Flecainide
B. Beta blocker
C. Adenosine
D. Verapamil

170. A 70 year old man with hoarseness of voice for 3 months presented with breathing
difficulties. On chest x-ray there was a unilateral opacity in hilum. There is no history of
smoking. What is most probable diagnosis?
A. Bronchial carcinoma
B. Laryngeal carcinoma
C. Oesophageal carcinoma
D. Nasopharyngeal carcinoma
E. Sarcoidosis

171. A 48 years old woman presents with left sided severe headache. She also has a red, watering eye
and complains of seeing coloured haloes in her vision. What is the most appropriate next step?

A. Measure Intra ocular pressure D. CT Scan


B. Relieve pain with aspirin E. Relieve pain with Sumatriptan
C. 100% oxygen

172. A 31 year old man has epistaxis following 10 days after Polypectomy. What is the most likely
diagnosis?

A. Nasal infection
B. Coagulation disorder
C. Carcinoma

173. A 28 year old pregnant lady presents with severe lower abdominal pain with excessive per vaginal
bleeding at her 34 weeks of pregnancy. What should be the initial investigation of choice?

A. Coagulation profile
B. US scan of abdomen
C. CT pelvis
D. D-dimer
E. Kleiuher Test

174. An 8 year old child swallowed 12 tablets of paracetamol 4 hours ago. Serum paracetamol levels
when tested were at critical level. What would you do next?

A. Activated charcoal
B. IV N-acetylcystiene
C. Gastric lavage
D. Observation only

175. A 50 year old patient presents with squamous cell carcinoma of bronchus. What is
electrolyte imbalance?
A. Hypocalcemia
B. Hyperkalemia
C. Hypercalcemia
D. Hypokalemia
E. Hypermagnesemia

176. A 36year old woman was recently admitted to a psychiatric ward. She believes that the staff and
other patients know exactly what she is thinking all the time. What is the most likely symptom this
patient is suffering from?
A. Thought Insertion
B. Thought Withdrawal
C. Thought Block
D. Thought Broadcasting
E. Hallucination

177. A 27 year old patient met with a road traffic accident comes to the A&E. While the nasogastric
tube is passing the bowel sounds are heard in the chest. On the chest x-ray nasogastric tube is seen
curled. What is the diagnosis?
A. Diaphragm rupture
B. Aortic rupture
C. Splenic rupture
D. Bowel rupture
E. Liver rupture

178. A 67 year old man presents to the A&E with pain in his left groin. He suddenly collapses and he
is not able to move or lift his leg. He is on Alendronate. What is the diagnosis?
A. Fracture of neck of femur D. Pelvic base fracture
B. Posterior hip dislocation E. Peripheral vascular disease
C. Fracture of shaft of femur

179. A 35 year old, intra-venous drug abuser on Penicillin and Flucloxacillin for cellulitis, now
presents with jaundice, pale stools and dark urine. What is the single most likely diagnosis?
A. Hepatitis A
B. Cholestatic jaundice
C. Chronic active hepatitis
D. Primary billiary cirrhosis
E. Hepatitis B

180. A teenager presented to A and E with shortness of breath and chest pain. On examination his
chest was hyper resonant on one side. He is the tallest guy in his class
A. Simple Pneumothorax
B. Open pneumothorax
C. Close Pneumothorax
D. Tension pneumothorax
E. Refractory pneumothorax

181. A 32 year old man on psychiatric medications complains of inability to ejaculate. Which drug is
most likely to cause these symptoms?
A. Lithium
B. Haloperidol
C. Chlorpromazine
D. Fluoxetine
E. Clozapine
182. A 49 year old man lost his job and now he is homeless. He was found wandering in the park. He
is muttering that some people are after him. Alcohol was tested and it was negative. What will your
next step be?

A. Thiamine
B. Neuropsychiatric analysis
C. Mini mental state
D. CT head
E. MRI head

183. A patient diagnosed with DVT is taking warfarin. What is his cut –off-international normalised
ratio (INR) limit?
A. <1
B. 1-2
C. 2-3
D. 3-4

184. A 10 year old boy presents with nose bleed. What measure should be taken to stop the bleeding?
Choose the most appropriate measure:
A. Press base of the nose
B. Ice packs
C. Press soft parts of the nose
D. Start tranexemic acid
E. IV fluids

185. A 33 year old male came to the hospital with complaints of occasional left sided chest pain that
lasts less than 30 minutes, following exercise, which relieves upon taking rest. What is the most
probable diagnosis?
A. Unstable angina
B. Decubitus angina
C. Stable angina
D. Coronary spasm
E. Myocardial infarction

186. A 25 years old woman complains of dizziness, nausea, vomiting, visual disturbances and anxiety,
which keeps coming from time to time. Most of the attacks are associated with sudden change in
posture. What is the most likely diagnosis?
A. Panic Disorder
B. Carotid Sinus Syncope
C. Benign Positional Paroxysmal Vertigo
D. Vertebro basilar Insufficiency
E. Postural Hypotension
187. A 52 year old man underwent hemicolectomy. After a few days he complains of left ventricular
pain and fever. Echocardiogram has been done and shows a systolic murmur. What is the next
appropriate investigation?
A. CT scan D. Blood culture
B. Ultrasound E. Liver function test
C. Chest x-ray

188. A 30 year old woman has been feeling low and having difficulty in concentrating since her
mother passed away 2 months ago. She feels lethargic and tends to have breathlessness and tremors
from time to time. What is the most likely diagnosis?
A. Adjustment Disorder
B. Post-traumatic stress disorder (PTSD)
C. Panic Disorder
D. Generalized anxiety disorder (GAD).
E. Bereavement

189. A 65 year old man has incurable bronchial cancer. He is unable to cough up his secretions. This
is leading to a distressing cough. Which of the following drugs is most likely to help him?
A. Scopalamine
B. Xanomeline
C. Aceclidine
D. Pilocarpine
E. Cevimiline

190. A 29 year old female presents with intermittent episodes of unilateral headache. Headache is
always on right side. She also complains of associated watering of the eye and nasal congestion on the
same side. All these symptoms were worst particularly in the mornings. What is the most likely
diagnosis?
A. Sinusitis
B. Tension headache
C. Space occupying lesion
D. Cluster headache
E. Acute angle closure glaucoma

191. A 17 year old male complains of fever and generally feeling unwell. On examination widespread
lesions on trunk were noticed. Lesions did not look the same everywhere, they were in various stages,
some of them were scabbed. What is the most likely diagnosis?

A. Herpes zoster
B. Varicella zoster
C. Impetigo
D. Psoriasis
E. None of the above
192. A 5 month old child can't speak but makes sounds. She can hold things with palm not fingers.
She can't sit independently but can hold her head and sit when propped up against pillows. How is the
child’s development?
A. Normal development
B. Delayed speech
C. Delayed sitting up
D. Delayed motor development

193. A 43 year old lady is admitted with pyrexia, arthropathy, breathlessness and syncope. She was
recently diagnosed as having Pulmonary Emboli. There is early diastolic sound and a mid -diastolic
rumble. Her jugular venous pressure (JVP) is elevated with prominent a waves. What is the most
likely cause from below options?
A. Mitral regurgitation
B. Ventricular ectopics
C. Pulmonary regurgitation
D. Atrial Myxoma
E. Complete heart block

194. A teacher had respiratory infection for which she was prescribed antibiotics. After her antibiotic
course, when she re-joined the school, she had lost her voice completely. What is the SINGLEmost
appropriate diagnosis?
A. Recurrent Laryngeal nerve palsy
B. Angioedema
C. Laryngeal obstruction by the medications
D. Laryngitis
E. Functional dysphonia/vocal cord

195. A 75 year old woman presents to the breast clinic having noticed that she has had a blood stained
discharge from the left nipple, together with ‘dry skin’ over the left areola. On examination there
was a blood stained discharge with dry, flaky skin noted on the left areola. The nipple was noted to
be ulcerated. What is the most appropriate investigation?
A. Fine-needle aspiration biopsy (FNAC)
B. MRI scan
C. Punch Biopsy
D. Open Biopsy
E. Stereotactic Biopsy

196. A 46 years old woman has an offensive yellow discharge from one nipple. She had a history of
breast abscess 3 years ago. What is the possible diagnosis?
A. Duct papilloma
B. Duct ectasia
C. Duct fistula
D. Breast cancer

197. A child, suffering from asthma, presents with Temp. 39 ˚C and drooling saliva on to the mother’s
lap and is taking oxygen by mask. What sign will indicate that he is deteriorating?

A. Intercostal recession
B. Diffuse wheeze
C. Drowsiness

198. A patient who came from India presents with cough, fever and enlarged cervical lymph node. On
examination there is casseating granulomata found in lymph node. What is the most appropriate
diagnosis?
A. Lymphoma
B. TB adenitis
C. Thyroid Ca
D. Goitre
E. Thyroid cyst

199. A 52 year old man with a history of anterior MI 3 weeks ago developed sudden onset of
dyspnoea. On examination: BP 100/60mmHg, PR 100/min, SaO2: 88%
Chest: bilateral crackles.
Which investigation would you do to find the underlying cause?
A. Chest x-ray
B. ECHO
C. D-DIMERS
D. Radio-nucleotide lung scan
E. Troponin

200. A 60 years old man has a pathological rib fracture. He also complains of recurrent infection.
Bone marrow aspiration is done. The lab report shows calcium is 3.9mmol/l and alkaline phosphatase
level is 127µ/l. What type of cell would be found in abundance in the marrow smear?
A. Plasma cells
B. Myeloid cells
C. Bence Jones protein
D. Megakaryocytes
E. Reticulocytes
PLAB 1 MOCK ANSWERS : Mock 2
1 D 41 D 81 B 121 C 161 B
2 A 42 B 82 A 122 B 162 C
3 B 43 B 83 B 123 D 163 B
4 D 44 A 84 A 124 C 164 B
5 C 45 C 85 B 125 C 165 F
6 C 46 D 86 B 126 D 166 A
7 B 47 A 87 A 127 C 167 D
8 A 48 A 88 B 128 A 168 D
9 B 49 B 89 A 129 A 169 B
10 B 50 A 90 C 130 A 170 A
11 C 51 E 91 B 131 C 171 A
12 C 52 C 92 A 132 B 172 A
13 A 53 A 93 B 133 A 173 A
14 A 54 C 94 B 134 A 174 B
15 C 55 B 95 D 135 A 175 C
16 B 56 A 96 B 136 D 176 D
17 D 57 A 97 B 137 C 177 A
18 B 58 B 98 C 138 B 178 A
19 C 59 B 99 D 139 B 179 B
20 A 60 B 100 140 B 180 A
21 C 61 B 101 B 141 CYCLIZINE 181 D
22 C 62 D 102 D 142 B 182 B
23 A 63 B 103 B 143 C 183 C
24 D 64 B 104 D 144 D 184 C
25 C 65 B 105 D 145 A 185 C
26 D 66 B 106 D 146 A 186 D
27 A 67 C 107 E 147 E 187 D
28 E 68 A 108 B 148 E 188 A
29 A 69 B 109 A 149 E 189 A
30 A 70 A 110 B 150 B 190 D
31 A 71 B 111 E 151 D 191 B
32 D 72 E 112 B 152 D 192 A
33 C 73 D 113 D 153 A 193 D
34 C 74 A 114 A 154 A 194 E
35 B 75 D 115 B 155 A 195 C
36 A 76 A 116 A 156 C 196 C
37 A 77 C 117 A 157 C 197 A
38 C 78 G 118 A 158 B 198 B
39 E 79 A 119 E 159 B 199 B
40 D 80 A 120 C 160 C 200 A
Dr. Swamy PLAB Courses Ltd
PLAB 1 MOCK TEST: MOCK 3 TIME ALLOWED: 3HRS

SINGLE BEST ANSWER

1. A 32 years old married woman complains of feeling low and has had two episodes of drug
overdose. She has a family history of depressive disorders. She has reduced social contacts now.
Which risk factor greatly increases risk of suicide?

A. Lack of Social Support


B. Past history of overdose
C. Family history of depression
D. Married status
E. Female sex

2. A child presents with maculopapular rash and severe itching. Crops of vesicles were observed on
the back and trunk, some of which were weeping and some crusted. What is the most likely
diagnosis?
A. Rubella virus
B. Cytomegalovirus CMV
C. Parvovirus B-19
D. Varicella
E. Herpes Simplex

3. A young football player has collapsed during a game. During the initial evaluation his respiratory
rate is 14/min, pulse is 88/min and the blood pressure is 110/70 mmHg. He seems to be sweating and
muttering some incomprehensible words. What is the most important next step?
A. CT scan
B. Magnetic resonance imaging (MRI)
C. Blood Sugar
D. Check body temperature
E. IV fluids

4. A 17 year old man has acute pain and earache on the right side of his face. He has a temperature of
38.4˚C and has extensive pre- auricular swelling on the right, tender on palpation bilaterally. What is
the single most likely diagnosis?
A. Acute mastoiditis
B. Acute otitis externa
C. Acute otitis media
D. Mumps

5. An 18 year old previously well student is in his first year at university. He has been brought to the
A&E department in an agitated, deluded and disoriented state. What is the most probable reason for
his condition?
A. Drug Toxicity
B. Delirium Tremens
C. Infection Toxicity
D. Electrolyte imbalance
E. Head injury
6. A 63 year old woman underwent knee surgery and was kept on low molecular weight heparin
postoperatively. On the 6th post-operative day she developed sudden severe chest pain with
breathlessness. Her ECG and chest x-ray were normal. What is your next action?
A. Stop low molecular weight heparin and start Warfarin
B. Keep on low molecular weight heparin and do CT. pulmonary angiography.
C. Increase the dose of low molecular weight heparin and do CT. pulmonary angiography.
D. Keep on low molecular weight heparin, add warfarin and do CT. pulmonary angiography.
E. Change to unfractionated heparin and do CT. pulmonary angiography.

7. A 40 year old man has pain, redness and swelling over the nasal end of his right lower eyelid. The
eye is watery with some purulent discharge. The redness extends on to the nasal peri-orbital area and
mucoid discharge can be expressed from the lacrimal punctum. What is the SINGLE most
appropriate clinical diagnosis?
A. Acute conjunctivitis
B. Acute dacryocystitis
C. Acute iritis
D. Retrobulbar neuritis
E. Scleritis

8. A 60 years old man presents with a six month history of poor sleep and loss of interest. He has
become forgetful recently and answers most of the questions with “I don’t Know”.What is the most
likely diagnosis?
A. Bipolar affective disorder
B. Depression
C. Normal Ageing Process
D. Dementia
E. Dyslexia

9. A baby boy when examined a few hours after birth, was found to have a machinery murmur. On
examination, he had heart rate of 160 bpm, he was not cyanosed and was asymptomatic. His parents
were told that the murmur will disappear on its own after a few weeks. What is the most likely
diagnosis?
A. Coarctation of the aorta
B. Tetralogy of Fallot
C. Patent ductus arteriosus
D. Atrial septal defect
E. Ventricular septal defect

10. A 32 year old woman presents with attacks of numbness in the right hand and fingers followed by
left side headache. What is the most probable diagnosis?
A. Cerebral tumour
B. Epilepsy
C. Migraine
D. Temporal arteritis
E. Cervical spondylosis

11. A 3 years old child, who was wasted on examination, has a history of diarrhoea on and off. The
mother describes the stool as bulky, frothy and difficult to flush. What is the single investigation most
likely to lead to a diagnosis?
A. Sweat Chloride test
B. Anti Endomysial Antibodies
C. Liver Function tests
D. Ultrasound Abdomen
E. Thyroid Function Test
12. A child presented with right sided hearing loss. Tuning fork test was done and showed Air
conduction > Bone conduction. Weber test localised to left ear. What is the most appropriate
diagnosis?

A. Acute Otitis Media


B. Acute Otitis externa
C. Serous Otitis Media with effusion (glue ear)
D. Otosclerosis
E. Acoustic neuroma

13. A 44 year old patient comes with right hemiparesis. On examination he has left sided ptosis and
his left pupil is dilated. Where is the lesion?
A. Cerebral infarct
B. Cerebellar infarct
C. Medulla Oblongata
D. Pons
E. Midbrain

14. A 60 year old lady has severe chest pain. ECG shows changes of inferior wall MI. ECG also
shows progressive prolongation of PR interval until a P wave is dropped. What is the most probable
diagnosis?
A. Atrial Fibrillation
B. Ventricular tachycardia
C. SupraVentricular tachycardia
D. Mobitz Type I Second Degree heart Block
E. Mobitz Type II Second Degree heart Block

15. A lady is afraid to go out in crowded open places. She has extreme anxiety & has started avoiding
trains, supermarket & public places.

What is most likely diagnosis?


A. Claustrophobia
B. Agoraphobia
C. Social Phobia
D. Generalized anxiety
E. Obsessive compulsive disorder

16. 28 year old man on inhaled short acting bronchodilator and low dose corticosteroid complains of
disturbing cough and wheeze on exercise. He has good inhaler technique and no known allergens.
What is the best modification in his therapy?
A. Oral leukotriene antagonist
B. Inhaled regular long acting bronchodilator
C. Increase dose of inhaled corticosteroid
D. Sodium Chromoglycate
E. Oral corticosteroid

17. A 2 weeks old child presents with 70% O2 saturation on air. He is cyanosed and has a pan
systolic murmur. What is the most likely reason for this?
A. Tetralogy of Fallot (TOF)
B. Atrial septal defect (ASD)
C. Patent ductus arteriosus (PDA)
D. Pumonary Hypertension
E. Type 1 respiratory failure
18. A 72 year old presents with ipsilateral ptosis and contralateral hemiparesis. Which of the
following structure is affected?
A. Medulla D. Cerebellum
B. Cortex E. Pons
C. Midbrain

19. A 53 year old man felt jerking in his left hand which spread to the rest of the body. A few minutes
later he developed a generalized seizure with loss of consciousness. What is the Investigation of
choice is?

A. CT head
B. EEG
C. ECG
D. Carotid USG
E. 24 hrs EEG

20. A 34 years old man has been diagnosed with Syndrome of inappropriate antidiuretic hormone
secretion (SIADH). Choose the most appropriate electrolyte picture from the following options?
A. High serum Na, low serum osmolarity, high urine osmolarity
B. Low serum Na, low serum osmolarity, high urine osmolarity
C. Low serum Na, high serum osmolarity, high urine osmolarity
D. Low Serum Na, high serum osmolarity, low urine osmolarity
E. Normal Na, Normal serum osmolarity, normal urine osmolarity

21. MRI spine of a patient shows L3/ L4 spine compression. She complains of bladder control loss
and also has some eye problem. She has breast carcinoma in the past. What explains her symptoms
the best?
A. Metastasis in brain
B. Metastasis in spine
C. Sciatica
D. Disc prolapse

22. A 63 year old man continues to experience chest pain and has a temperature of 37.8° C two days
after an acute myocardial infarction. His electrocardiogram (ECG) shows widespread ST elevation
with upward concavity. What is the single most likely explanation for the abnormal investigation?
A. Acute Pericarditis
B. Cardiac Tamponade
C. Atrial Thrombus
D. Left Ventricular Aneurysm
E. Dressler Syndrome

23. A 23 year old female has repeated admissions to A& E department for self-harm. She denies
trying to commit suicide. What is the best treatment option for her?

A. Flooding
B. Desensitization
C. Token economy
D. Problem focused cognitive behavioural therapy
E. Thought stopping

24. A child was brought to A&E after being rescued from a house on fire. On examination he was
cyanosed and his nose hair were singed. Soot was seen in the naso pharynx and he was having stridor.
What is the immediate next step?
A. 100 % oxygen
B. Tracheal intubation
C. IV access
D. Epinephrine spray
E. GCS assessment

25. A 43 years old woman has been feeling lethargic and tired. Her blood pressure is
160/90 mmHg. She has the following blood values:
Na: 140 mmol/L
K: 3.1 mmol/L What is the most likely diagnosis?
A. Cushing’s syndrome
B. Conn’s syndrome
C. Hyperparathyroidism
D. Renal disease
E. Pheochromocytoma

26. A 34 year old housemaid presents with headaches in the back of her head since several days. She
also complains of pain on flexing her neck. What is the most likely cause?
A. Subdural haemorrhage
B. Cervical spondylosis
C. Subarachanoid haemorrhage
D. Meningitis
E. Cluster headaches

27. A 45 years old female looking pale, has bluish discoloration of hands whenever she goes out in
cold. She has also noticed some reddish spots on her body. She has symmetrical peripheral
arthropathy for last 1-2 years. What is the most probable diagnosis?
A. Rheumatoid Arthritis
B. Osteosarcoma
C. Limited Systemic Sclerosis
D. Diffuse Systemic Sclerosis
E. Chondrosarcoma

28. A 25 years old woman presents with single lump in the breast and axilla. The lump is mobile and
hard in consistency. The ultrasound, mammogram and fine needle aspiration turned out to be normal.
What is the most appropriate investigation to confirm the diagnosis?
A. Fine Needle Aspiration Cytology (FNAC)
B. Magnetic resonance imaging (MRI)
C. Punch biopsy
D. Genetic testing and counselling
E. Core biopsy

29. A 35 years old male presented with purulent nasal discharge from one side with headache and
fever , examination showed supra orbital and maxillary tenderness , what is the best diagnostic
investigation?
A. CT Scan
B. MRI
C. X-ray
D. USG
E. Swab for culture
30. A boy presents to the A&E department. He was playing with his pencil in the class. He scratched
his ear with the pencil and when he took out his pencil, he found that the eraser at the pencil tip was
missing. Choose the single most appropriate method of removal of foreign body for this person.
A. Removal with a hook
B. Syringing
C. Removal under GA
D. Putting oil
E. Putting alcohol

31. A 34 year old male suddenly collapsed due to cardiac arrest, Cardiopulmonary resuscitation was
done at a shop, the patient opened his eye just in response to painful stimuli and his GCS was 6. What
is your diagnosis?
A. Coma
B. Concussion
C. Sub-conscious
D. Vegetative state
E. Brain death

32. A 53 year old woman presented with pain in the eye, blurry vision and clumsiness for 3 months.
She had a history of difficulty in swallowing and weakness in her right upper limb 2 years ago. What
is the investigation of choice?

A. CSF analysis D. MRI of the brain


B. Electroencephalography (EEG) E. Visuality Evoked Response test (VER)
C. Electromyography (EMG)

33. A 15 year old boy presents with a limb and pain in the knee. On examination, the leg is
externally rotated and 2 cm shorter. There is limitation of flexion, abduction, and medial rotation. As
the hip is flexed external rotation is increased. Choose the single most likely diagnosis.
A. Juvenile rheumatoid arthritis
B. Osgood Schlatter disease
C. Reactive arthritis
D. Slipped femoral epyphisis
E. Transient synovitis of the hip

34. A 25 year old woman is presenting with diarrhoea and abdominal bloating over the last 4
months. On examination she has blistering rash over her elbows. Biochemical investigation showed
that she has low serum albumin, calcium and folate concentration. On jejunal biopsy there is
shortening of the villi and lymphocytosis. What is the most likely cause?
A. Coeliac disease E. Giardiasis
B. Whipple's disease F. Cystic fibrosis
C. Crohn's disease
D. Tropical Sprue

35. A 55 year old man returns for routine follow-up six weeks after a myocardial infarction. He gets
breathless when walking uphill. His ECG shows ST elevation in chest leads V1, V2, V3 and V4.
What is the single most likely explanation for the abnormal investigation?

A. Heart Block
B. Right Ventricular Strain
C. Atrial Thrombus
D. Left Ventricular Aneurysm
E. Dressler Syndrome
36. Asthma not controlled on salbutamol and low dose Beudesonide. Which is the next add on drug?
A. Salmetrol
B. Oral steroid
C. Aminophylline
D. High dose budesonide (inhaled)
E. Ipratropium Bromide

37. A woman with Hep B Ag positive gave birth to a baby boy. Now she is worried that she might
have passed Hepatitis B infection to the child. What is the most appropriate regime of immunization
for the baby boy?
A. Hepatitis B vaccine single dose only
B. Hepatitis B vaccine complete course only
C. Hepatitis B vaccine complete course + immunoglobulin
D. Immunoglobulin only
E. No immunization required

38. A 72 year old man is receiving chemotherapy for small cell lung cancer. He had his fourth
treatment eight days ago. He has a cough with some green sputum but feels well. He has a
temperature of 37.6° C. Chest examination shows a few coarse crepitations in the right base. His heart
rate is 92 beats/minute. A blood count shows:
Hb 12.5g/dl
WBC 1.1 X 109/L
Neutrophils 0.6 X 109/L
Platelets 89 X 109/L
Sputum, urine and blood cultures have been sent to the laboratory for microbiology. What is the
SINGLE most appropriate management?
A. Broad spectrum antibiotic (intravenous)
B. Broad spectrum antibiotic (oral)
C. Granulocyte colony stimulating factor
D. Postpone treatment until bacteriology results available
E. Reassure and send home

39. A two year old boy fell off his tricycle, hurting his arm. He got up and started to cry but before
there was any sound he went pale, unconscious and rigid. He recovered after 1-2 minutes but
remained pale. After an hour he was back to normal. His mother says she was afraid he was going to
die and that he had a similar episode three months previously after he fell down some steps. What
SINGLE investigation is indicated?

A. Computed tomography (CT) scan of the head


B. Electroencephalogram (EEG)
C. Full blood count `
D. None
E. Skeletal survey

40. A 59 year old man suddenly fell down and died. On post mortem report, the cause of death was
found to be due to abdominal aortic rupture. What is the most likely cause of rupture?
A. Atheroma
B. Intimal dissection
C. Polyarteritisnodosa
D. Syphilis
E. Hypertention
41. An old lady who is a nursing-home resident presents with itchy rash, white linear lesions on
wrists and elbows. What is the best treatment?
A. Permetrin
B. Antibiotics
C. Antifungals
D. Emollients
E. Steroids

42. A 60 year old diabetic patient on antidiabetic medication developed diarrhoea. What is the most
likely cause for his diarrhoea?
A. Autonomic neuropathy
B. Infective
C. Celiac disease
D. Crohn’s disease

43. A patient presented to A & E with polyphonic wheeze. He was unable to complete sentence, pulse
was >110 bpm, RR > 25/min and PEFR 30-50% of predicted. What is the most likely diagnosis?
A. Mild asthma
B. Moderate asthma
C. Severe asthma
D. Life threatening asthma
E. Exercise induced

44. A 6 week old child presents with deep jaundice. He is breast feeding well but not gaining weight
and passes pale stools and dark urine. What is the most likely diagnosis?
A. Breast milk jaundice
B. Biliary atresia
C. Physiological jaundice
D. Galactosemia
E. congenital hypothyroidism

45. A 12 year old patient presents with copious diarrhoea. On examination his urine output is low,
mucous membranes are dry and skin turgor is low. What is the single most initial management?
A. Antibiotic
B. Antimotility
C. Anti-emetic
D. Fluid replacement
E. Reassure

46. A 12 year old boy presents with painful swollen knee after a sudden fall. Which bursa is most
likely to be affected?
A. Semi-membranous bursa
B. Prepatellar bursa
C. Pretibial bursa
D. Suprapatellar bursa

47. Which of the following group of drugs will be treatment of choice for Trigeminal neuralgia?
A. Anti epileptics
B. Anti depressant
C. Opioids
D. Benzodiazepens
E. NSAIDS
48. A 15 year old male has bilateral ankle oedema. His blood pressure is 110/70mmHg and
urinalysis shows protein ++++. What is the most likely diagnosis?
A. Haemolytic uraemic syndrome
B. IgA nephropathy
C. Membranous glomerulonephritis
D. Minimal change glomerulonephritis
E. Nephrotic syndrome

49. A patient, from Africa, comes with nodular patch on the shin. It’s colour is reddish brown. What
is the most probable diagnosis?
A. Lupus vulgaris
B. Erythema nodosum
C. Pyodermagangrenosum
D. Erythema marginatum
E. Solar Keratosis

50. Two healthy parents had their first child with cystic fibrosis. They are expecting another baby and
are concerned about the probability of disease in next child. What is the probability of their next child
having cystic fibrosis?
A. 5%
B. 10%
C. 25%
D. 50%

51. A 46 year old woman is brought to A&E after falling in bathroom while painting. She is drowsy
and gives history of sudden severe headache at the back of the head prior to the fall. What
investigation would be best choice in this case?
A. Carboxy haemoglobin
B. MRI head
C. CT Head
D. Lumbar puncture
E. Skull X-ray

52. An otherwise healthy 13-year-old boy presents with recurrent episodes of facial and tongue
swelling and abdominal pain. His father has had similar episodes. What is the single most likely
diagnosis?
A. C1 esterase deficiency
B. HIV disease
C. Mumps
D. Sarcoidosis
E. Sjogren’s syndrome

53. A mother comes with her 15 month old child. Which of the following will bother you?
A. Shies away from strangers
B. Can walk but cannot run
C. Vocabulary consists of only two meaningless words
D. She can’t make a sentence
E. None

54. A patient comes to the A&E with chest pain within 1 hour of onset. He was given aspirin and
nitrates and he is fine now. Myocardial infarction was confirmed in ECG. What is your next
management?
A. Streptokinase and thrombolytics
B. Morphine
C. Percutaneous coronary intervention (PCI)
D. Oxygen

55. A 4 year old boy is brought by his parents with complains of wetting his bed at night and
whenever he gets excited. What would be the most appropriate management for this child?
A. Desmopressin
B. Oxybutynin
C. Behavioural Therapy
D. Tamsulosin
E. Restrict fluid intake

56. A 2 year child is severely dehydrated and needs immediate fluid resuscitation. 4 attempts to gain
IV access have failed. Which of the following access routes should be considered next?
A. Subclavian artery
B. Radial artery
C. Intraosseous
D. Internal jugular vein
E. Saphenous cut

57. A 35 year old woman had an uneventful laparoscopic cholecystectomy 18 hours ago. She has a
pulse rate of 108 beats/minute and a temperature of 37.8° C. There are signs of reduced air entry at the
right base but the chest x ray does not show any obvious abnormality. What is the most appropriate
management strategy?
A. Cefuroxime oral
B. Ceftriaxone I.V
C. Chlorpheniramine oral
D. Chest Physiotherapy
E. Reassure

58. A 21 year old student, living in a hostel, presented to A&E with symptoms of headache, fever and
vomiting. What is the best treatment?
A. Ceftriaxone
B. Gentamycin
C. Benzyl penicillin
D Ampicillin
E. Rifampicin

59. A 33 year old man is hit by a car. He loses consciousness but is found to be fine by the
paramedics. When awaiting doctor’s review in the A&E he suddenly becomes comatose. What is the
most likely diagnosis?
A. Subarachanoid haemorrhage
B. Subdural haemorrhage
C. Intracerebral haemorrhage
D. Extradural haemorrhage

60. A 50 year old man presents to A&E with acute back pain radiating down to thigh/legs. Pain
which is usually relieved by lying down and exacerbated by long walks and prolong sitting .What
investigation would be the best option?
A. Magnetic resonance imaging (MRI)
B. CT scan of spine
C. Plain x-ray of spine
D. Dual energy x-ray abruptiometry
E. Serum para protein electrophoresis
61. A 62 year old man presents with cough, breathlessness and wheeze. 24% oxygen, Salbutamol
and hydrocortisone were given. The symptoms have not improved and so nebulised bronchodilator
was repeated and IV Aminophylline was given. ABG showed pH: 7.31 and respiratory rate is 32.
What is the next appropriate management?

A. Nasal intermittent positive pressure ventilation


B. Intubation and ventilation
C. Long acting beta agonist
D. Toxapram
E. Amoxicillin oral

62. A mother brings her baby to the hospital complaining that the baby has stridor only upon breast-
feeding. On examination, omega shaped cartilage was seen in the neck. What is the most probable
diagnosis?
A. Laryngomalacia
B. Bronchiolitis
C. Croup
D. Epiglotitis
E. Tracheo oesophageal fistula

63. A 57 year old female presents with pain in right eye and proptosis in the same eye. What is the
Investigation of choice in this case?
A. CT scan of head
B. CT scan of eye
C. X-ray skull
D. Carotid study
E. MRI head

64. Man who has motor neuron disease and is wheelchair bound, finds it difficult to swallow food-
both liquids and solids. What would be the best option to provide nutrition?
A. Gastrostomy
B. Fine bore NG tube
C. Parenteral nutrition
D. Give steroids and continue oral nutrition

65. A Patient presents with sensory loss of upper limbs with motor loss in lower limbs. On
examination he was found to have bilateral loss of pain and temperature only, sensations of vibration
and proprioception being preserved. What is the likely diagnosis?
A. Syringomyelia
B. Syringobulbia
C. Cerebral cortex lesion
D. Midbrain lesion
E. Cerebellar lesion

66. A 25 year old man has been suffering from breathlessness and wheeze for 3 months. He has been
taking salbutamol 2 puffs –as- required. In the last 2 weeks his symptoms have worsened, and he has
to take salbutamol more frequently during the day time. He also complains of excessive dyspnoea at
night. What drugs or regimen would you like to add?
A. Prednisolone (oral)
B. Fluticasone + Salbutamol (inhaled)
C. Beclometasone (inhaled)
D. Montelukast (oral)
E. Salmetrol (oral
67. A 27 year old lady had an uncomplicated pregnancy so far. She came to the hospital 2 hours ago
after her water broke. The midwife is looking after her now. She has been having regular contractions.
P.V examination revealed cervix is 2cm dilated. Her vital signs are normal. What stage of labour is this
lady in now?
A. Second stage
B. First stage
C. Latent phase
D. Third stage
E. Active phase

68. A 35 year old African woman presents with cyclical menorrhagia since last 8 month. She is
nulliparous and has been trying to conceive for some years now. What do you suspect from the
history?
A. Dysfunctional Uterine Bleeding
B. Endometriosis
C. Endometrial polyp
D. Adenomyosis
E. Fibroids

69. A 30 year old professional athlete presents with amenorhea of 7 months duration. What is the
Single most likely cause from the following options? FSH 8, LH 6, Oestradiol 80, Prolactin 600
A. Premature ovarian failure
B. Hypothalamic failure
C. PCOS
D. Prolactinoma

70. A 28 year old lady with a family history of cystic fibrosis comes for genetic counselling and
wants the earliest possible diagnostic test for cystic fibrosis for the baby she is planning. She is not in
favour of termination. What would you recommend for her?
A. Chorionic villous sampling
B. Amniocentesis
C. Pre implantation genetic diagnosis
D. Chromosomal karyotyping
E. Maternal serum test
F. Reassure

71. An 8 weeks pregnant woman presents with persistent vomiting and weight loss. On examination
her heart rate is 110bpm. Dehydration was corrected with normal saline infusion with potassium. The
condition didn’t improve so intramuscular cyclizine was given. She is still vomiting. What is the next
appropriate management?
A. IV fluids
B. IV antiemetics
C. IV steroids
D. Terminate the pregnancy
E. Thiamine

72. A 32 years old woman presents with history of lower abdominal pain and vaginal discharge .She
had her menses 4 weeks ago. She has a temperature of 38.6 ˚C. What is the most suitable diagnosis?
A. Acute appendicitis
B. Acute pelvic inflammatory disease
C. Endometriosis
D. Ectopic pregnancy
E. Urinary tract infection
73. What is the single most important risk factor for development of ectopic pregnancy?
A. Twin pregnancy
B. Endometriosis
C. IUCD
D. Chlamydia

74. A 27 year old woman presented with amenorrhoea of 10 months duration. FSH – 28, LH –
Normal, Testosterone – 1.8, Estradiol – 80. What is the most probable diagnosis?
A. Hypothalamic
B. Polycystic ovary syndrome (PCOS)
C. Premature ovarian failure
D. Hyper androgenism

75. A 30 year old lady who already has one child through a previous caesarean section demands a
reversible contraceptive. She presently experiences heavy and painful periods. What is the most
appropriate contraceptive you will recommend for her?
A. Combined Oral Contraceptive Pill
B. Progesterone only pill
C. Implanon
D. Danazol
E. Mirena coil
F. Intrauterine Contraceptive Device (IUCD)

76. A 33 weeks pregnant woman presents with vaginal bleeding, low Hb, low platelets, increased
bilirubin, AST was normal, APTT, PT increased. What is the most likely diagnosis?

A. Preclampsia E. Ectopic pregnancy


B. Disseminated intravascular coagulation (DIC) F. Missed abortion
C. Placental abruption G. Ectropion
D. Placentra-previa

77. An asymptomatic 35 year old lady with family history of ovarian carcinoma is concerned about
her risk. Her 2 sisters had Ovarian Cancer between the ages of 40 and 45. What is next step in plan of
management?
A. Pelvic ultrasound
B. CA 125
C. Genetic testing and counselling
D. Return if symptomatic
E. Reassurance

78. A 25 year old had an LSCS 24 hours ago for foetal distress. She now complains of intermittent
vaginal bleeding. Observations include saturations 98% in air, blood pressure 124/82 mmHg, pulse 84
beats/min and temperature 37.8oC. The midwife tells you that she had a retained placenta, which
required manual removal in the operating theatre. Choose the most appropriate post C-Section
complication in this case?

A. Retained products of conception


B. Aspiration pneumonitis
C. Endometritis
D. Uterine rupture
E. Disseminated intravascular coagulation

79. A 16 year old female presents with Sickle cell anaemia and heavy menstrual bleeding. What is the
best treatment of choice?
A. Combined Oral Contraceptive Pill (COCP)
B. Inj Depoprovera
C. Mirena
D. IUCD
E. Implanon

80. A 21 years old female in her first pregnancy at 38 weeks was brought to the emergency with a
generalized tonic clonic seizure. IV MgSO4 was given but fits was not controlled, she is having fits
again. What is the single most important immediate management of this patient?
A. IV MgSO4
B. IV Diazepam
C. Immediate C section
D. IV Phenytoin
E. Give MgSO4 bolus
F. IV Lorezepam

81. A 40 year old female had history of use of COCP for 5 years and condom for a few years. She is
obese and is a heavy smoker. Smokes 20 cigarette/day .Had repeated episodes of chlamydial
infections. Her family history showed that her aunt died of carcinoma of the ovary at 48 years of age.
What is the major risk factor for development of ovarian cancer in this patient?
A. OCOP
B. Condom
C. Obesity
D. Smoking
E. Family history

82. A 20 year old pregnant, 32/40 weeks by date, presents to the antenatal clinic with a history of
painless per vaginal bleeding after intercourse. On examination: P/A – soft and relaxed, uterus =
dates, CTG is reactive. Choose the SINGLE most likely diagnosis?
A. Abruption of placenta secondary to pre- eclampsia
B. Ante partum haemorrhage
C. Placenta previa
D. Preterm labour
E. Placenta percreta

83. A 25 year old woman who is 11 weeks pregnant had central abdominal pain for 36 hours. The
pain is now colicky. There is no vaginal bleeding. She has vomited once and has had an episode of
loose motions. She looks ill, has a temperature of 37.8 °C and there is rebound tenderness in the right
iliac fossa. What is the most probable diagnosis?
A. Salpingitis
B. Pelvic Inflammatory Disease
C. Appendicitis
D. Ovarian Torsion
E. Uterine Fibroid

84. An African lady with sickle cell anaemia having heavy bleeding and she needs contraception also.
What will you advice her?
A. Mirena coil
B. Inj Depot Medroxy progesterone acetate
C. Combined Oral Contraceptive Pill (COCP)
D. IUCD

85. What is the most appropriate antibiotic to treat uncomplicated Chlamydia infection in a 21 year
old female who is not pregnant?
A. Erythromycin
B. Ciprofloxacin
C. Metronidazole
D. Cefixime
E. Doxycycline

86. A 36 week pregnant woman presents with sudden onset of uterine pain and bleeding, uterus is
tender, no previous LSCS. What is the most appropriate cause?
A. Preclampsia E. Ectopic pregnancy
B. Disseminated intravascular coagulation (DIC) F. Missed abortion
C. Placental abruption G. Ectropion
D. Placenta-previa

87. A 39 year old woman in her 36th week of gestation with acute abdominal pain is rushed for
immediate delivery. Her report shows:
B.P: 110/60 mmHg
Hb: low
Bilirubin: 22 meq/mol
AST: 35 U
Platelets: 60 x 10*9
APTT: 60
PT: 20 or 30
Fibrinogen: 0.6
What is the most probable cause?

A. Pregnancy induced hypertension


B. Disseminated intravascular coagulation (DIC)
C. HELLP Syndrome
D. Acute Fatty Liver
E. Obstetric Cholestasis

88. A female with Sickle cell anaemia with history of frequent sickle cell crisis wants contraception.
What is the best form of contraception for her?
A. Progesterone only pill (POP)
B. COCP
C. Intrauterine system (IUS)
D. IUCD
E. DMPA

89. A 20 year old primigravida woman is in her 8th week of gestation. She presents with severe
repeated nausea and vomiting. What is the initial intravenous fluid to be used?
A. Normal saline 0.9%
B. Glucose water 5%
C. Saline 0.45%
D. Dextrose 5%

90. A 32 year old female, who has had three previous miscarriages in the first trimester, now comes
with vaginal bleeding at 8 weeks of pregnancy. USG reveals a viable foetus. What would be the most
appropriate definitive management?
A. Admit
B. Aspirin
C. Bed rest 2 weeks
D. Cervical cerclage
E. No treatment
91. A 28 year old woman who is 32 weeks pregnant, in her third pregnancy, is diagnosed as a case of
placental abruptio. After all the effective measures she is still bleeding. What is the underlying
pathology?
A. Clotting factors problem
B. Clauser’s syndrome
C. Platelets problem
D. Succenturiate lobe

92. An African woman 32 year old comes to the sexual clinic with a history of heavy menstrual
bleeding and dysmenorrhoea. What is the most likely cause?
A. Fibroid
B. Ectopic pregnancy
C. Endometriosis
D. PID
E. Adenomyosis

93. A 36 Weeks pregnant patient has been admitted with bleeding per vaginum and severe constant
pain in abdomen. A diagnosis of abruptio placentae has been made. What will be your immediate next
step?
A. Check Clotting profile
B. Ultrasound
C. CTG
D. Caesarian section
E. Check BP

94. A 37 year old lady stopped taking COCP 18 months ago and she had amenorrhea for 12 months
duration. What is the most appropriate cause? FSH : 8, LH: 7 and Prolactin is 400 and
Estradiol : 500.
A. Hypothalamic amenorrhea
B. Polycystic ovary syndrome (PCOS)
C. Prolactinoma
D. Post pill amenorrhea
E. Premature ovarian failure

95. A 12 week pregnant lady presents with severe morning sickness. On examination she has dry
mucous membranes. What is the best management for this patient?
A. IV fluids
B. Encourage oral intake
C. Anti-emetics
D. Trans-vaginal ultrasound

96. A 20 year old female , 14 weeks pregnant, presents with severe intermittent right iliac fossa pain
with occasional vomiting . Examination showed pregnant uterus of 14 weeks . What is your
diagnosis?
A. Ovarian torsion
B. PID
C. Gastroenteritis
D. Acute appendicitis
E. Ectopic pregnancy

97. A 21 year old female with 38 weeks gestation presents with a fit. She is admitted and IV MgSO4
and IV Hydralazine is started. BP is 155/90mmHg. She has another fit. What is your next step in
management?
A. Rectal diazepam
B. Hydralazine bolus
C. MgSO4 bolus
D. IV Diazepam
E. Mg SO4 Drip

98. A 14 year old girl presents with primary amenorrhoea. She also complains of cyclic monthly
abdominal pain and sensation of lump in the pelvis. Her hormone profile is normal. What is the
diagnosis?
A. Turner’s syndrome
B. Di George syndrome
C. Haematocolpos
D. Nephroblastoma

99. A 26 year old lady presents with high fever, lower abdominal pain and purulent vaginal discharge.
She looks very unwell. What is the most appropriate management for this patient?
A. Tetracycline 250mg 6 hourly
B. Doxycycline 100 mg 12 hourly po and Metronidazole 400mg 12 hourly
C. Ceftriaxone 2g 24 hourly I/V with Doxycycline 100mg po
D. Ceftriaxone 2g 24 hourly I/V with Doxycycline 500mg po
E. Ofloxacin 400mg 12 hourly po and Metronidazole 400 mg 12 hourly po

100. A 27 year old lady after caesarean section, developed epigastric pain after 8 hours. What is the
appropriate investigation?

A. ABG
B. Coagulation profile
C. Liver enzyme
D. Liver biopsy

101. A 27 year old woman at 34 weeks gestation for her first pregnancy attends antenatal clinic. Her
blood results showed. Hb 10.6g/dl, MCV 95 and MCHC 350. What do you do for her?

A. Folate F. Explain this is physiologic haemodynamic anemia


B. Dextran G. Blood Transfusion
C. Ferrous sulphate
D. No treatment required
E. IV FeSO4

102. A 38 weeks gestation comes with fits. She was given MGSO4 but again she developed fits.
On examination, knee jerk is absent. Whats your next step?
A. MGSO4 IV bolus
B. Deliver the baby
C. Hydralazine
D. MgSO4 Drip
E. Inj Diazepam

103. A 37year old woman presents with heavy bleeding. Investigations show sub-serosal fibroid
measuring 4cm and intramural fibroid of 6cm.What is the most appropriate treatment for her?
A. Uterine artery embolization
B. Abdominal Hysterectomy
C. Hysteroscopic myomectomy
D. Vaginal Hysterectomy
E. Abdominal myomectomy
104. A 30 year old female had Trichomonas Vaginitis infection. What is the best treatment?
A. Ceftriaxone
B. Erythromycin
C. Metronidazole
D. Doxycycline
E. Ciprofloxacin

105. A 36 year old woman came with uterine bleeding. Vaginal USG reveals uterine thickness
12mm. What is the most probable diagnosis?
A. Cervical Ca
B. Endometrial Ca
C. Ovarian Ca
D. Breast Ca
E. Vaginal carcinoma

106. A 32 years old woman wants reversible form of contraception. She has one child which was
delivered by emergency Cesarean section. She also suffers from migraine and heavy periods. What is
the most suitable form of contraception for this lady?

A. Combined Oral Contraceptive Pill (COCP)


B. Mini pill
C. Intrauterine device containing hormone
D. Barrier method
E. Abstinence

107. A 34 year old female presents with a foul smelling discharge, what set of organisms are we
looking for to be treated here?

A. Chlamydia, Gonorrhoea
B. Chlamydia, Gardenella
C. Chlamydia, Gonorrhoea, Gardenella
D. Gonorrhoea, Gardenella
E. Gardenella only

108. A 35 year old woman presents with hot flushes and night sweats. She had amenorrhea for the last
12 months. What is the single best next step?
A. Serum FSH level
B. HRT therapy
C. Serum estradiol level
D. No investigations needed

109. Which of the following is NOT a physiological change during pregnancy?


A. Tidal volume 500ml.
B. Red cell volume 1.64L.
C. Cardiac output 6.5L/min.
D. Uterus weight 1.1Kg.
E. ESR up by 4 folds.

110. A 56 year old lady is a chronic smoker. She smokes 20 cigarettes per day. She has post-
menopausal symptoms like hot flushes, night sweats, vaginal dryness. What will you prescribe her?
A. Pregablin
B. Clonidine
C. Evening primrose oil
D. Estrogen containing HRT
111. A 25 year old woman , 38 weeks pregnant , presents with blood pressure of 160/100 and +++
proteinuria . Her AST was 35 and ALT of 40. HB =12.7 g/dl. WBC count of 8×109 Platelet count of
115×109. What is your diagnosis?
A. Eclampsia
B. Preeclampsia
C. HELLP
D. Acute hepatitis

112. A 57 year old woman presents with dysuria, frequency and urinary incontinence. She complains
also of dyspareunia. Urine culture has been done and the lab report shows it is sterile. What is the
most appropriate step for this patient?
A. Oral antibiotic
B. Topical antibiotic
C. Topical oestrogen
D. Oral oestrogen
E. Oral antibiotic and topical oestrogen

113. A 37 year old infertile lady with 5cm subserosal and 3 cm submucosal fibroid. She is trying to
get pregnant, management?
A. Clomifene therapy
B. IVF
C. Myomectomy
D. Hysterectomy
E. Intra uterine insemination

114. A 14 year old girl with short stature and no secondary sexual characteristics developed.
What is your most probable diagnosis?
A. Turners
B. Constitutional delay
C. Klinefilters syndrome
D. Testicular feminisation syndrome

115. A 31 year old woman who is 32 weeks pregnant attends antenatal clinic. Her blood result shows
Hb 10.7 and MCV 91. What is the most appropriate management for this patient?
A. Folate supplement
B. Ferrous sulphate 200mg/24 Po
C. Iron dextran
D. No treatment required

116. A lady who has been treated for PID is unfortunately not getting better. What will be the next
step in the management?
A. High vaginal swab
B. Endocervical swab
C. USS abdomen and pelvis
D. Diagnostic laparoscopy
E. Continue antibiotics

117. A 33 weeks pregnant lady has been admitted with a 2 day history of leaking fluid per vagina. She
now complains of fever, foul smelling vaginal discharge and on examination, fetal tachycardia is
recorded. What is the most likely diagnosis?
A. Fetal distress
B. Pelvic Inflammatory Disease
C. Preterm Pre-labour rupture of membranes
D. Chorio-amnionitis
118. A 32 year old woman has a history of spontaneous abortions at 6 weeks, 12 weeks and 20 weeks.
She is now keen to conceive again. Which one of the following would you prescribe for the following
pregnancy?
A. Magnesium Sulphate
B. Aspirin
C. Warfarin
D. Mefenamic Acid
E. Heparin

119. A 56 years old lady presents with hot flushes. She had hysterectomy 2 years ago. She smokes
cigarettes. What will you give her for her symptoms?
A. Oestrogen and progesterone HRT
B. Oestrogen only HRT
C. Raloxifen
D. Bisphosphonates
E. Clonidine

120. A lady has been on contraception for a long time. She has regular periods and uterus also is
normal. She wants a contraception with least failure rate and side effects. Among the options given,
which one will you choose?
A. Depot Medroxyprogesterone Acetate( DMPA) injections
B. Cu intrauterine contraceptive device (IUCD)
C. Mirena coil
D. COCP
E. Female condom

121. A pregnant woman has her anomaly scan at 20 weeks. She had poly-hydramnios with no gastric
bubble seen. What is the diagnosis?

A. Duodenal atresia
B. Gastroschisis
C. Esophageal atresia
D. Jejunal atresia
E. Biliary atresia

122. A 24 year old lady who is 37 weeks pregnant was brought to A&E Department. Her husband
says a few hours ago she complained of headache, visual disturbance and abdominal pain. On arrival
to A&E she had a fit. What is the next appropriate management for this patient?

A. 4mg MgSO4 in 100ml 0.9% Normal saline in 5 min.


B. 2mg MgSO4 IV as a bolus
C. 2mg MgSO4 in 500ml Normal saline in 1 hour
D. 4mg MgSO4 IV as a bolus
E. 10mg Diazepam in 500ml 0.9% Normal saline in 1 hour

123. A 23 year old woman has white discharge from her vagina. She had a 7 day course of broad
spectrum antibiotics before her dental procedure. What is the causative microorganism?

A. Chlamydia
B. Candida
C. Trichomonas
D. Gardnerella
E. Nesseria Gonorrhea
124. A pregnant woman on her 18 weeks pregnancy with BP of 160/100mmHg and proteinuria. What
is the cause for this?
A. Gestational hypertension
B. Chronic hypertension
C. Pre eclampsia
D. Eclampsia

125. A 28 years old woman has delivered with rotational forceps after an eight hours labour and a
three hours second stage. Choose the single most likely predisposing factor for postpartum
haemorrhage for this patient?
A. Atonic uterus
B. Cervical/vaginal trauma
C. Retained product
D. Pre term labour
E. Uterine infection

126. A patient had inflammatory changes on cervical smear. There is no vaginal discharge, no pelvic
pain, and no fever. What is the next step?
A. Repeat smear in 6 months
B. Take swab
C. Treat with antibiotics
D. Colposcopy
E. Cone biopsy

127. A 24 year old 18 weeks pregnant lady presents with pain in her lower abdomen for the last 24
hrs. She had painless vaginal bleeding. On examination abdomen is tender and os is closed. What is
the most probable diagnosis?

A. Threatened miscarriage
B. Inevitable miscarriage
C. Incomplete miscarriage
D. Missed miscarriage
E. Spontaneous miscarriage

128. An 18 years old lady in her 30th week of pregnancy is brought to the hospital in altered
sensorium. She is taking slow, shallow breaths and her breath has a fruity smell. An ABG analysis
shows a pH of 7.20. Urine shows the presence of ketones. What is the most probable diagnosis?

A. Hyperglycaemic Hyperosmolar Nonketotic Coma (HONK)


B. Diabetic Ketoacidosis (DKA)
C. HELLP syndrome (Hemolysis, elevated liver enzyme levels, and low platelet levels)
D. Pregnancy-Induced Hypertension (PIH)
E. Gestational Diabetes Mellitus

129. A woman had an emergency LSCS 7 days ago. She now presents with fever and excessive
vaginal bleeding. What is the single most likely diagnosis?

A. Endometriosis
B. Endometritis
C. Vaginal infection
D. Retained placenta
130. You are called to see a 20 year old woman 2 hours post LSCS. She has not passed urine since her
operation. She denies any urinary symptoms preoperatively. On examination, she appears unwell,
with a temperature of 37.5oC, blood pressure 94/73mmHg, pulse 116 beats/min and saturations 97%
in air. Her abdomen id distended, with tenderness in the left flank and supra-pubic region. Bowel
sounds are not audible. Choose the most appropriate post C-Section complication for this lady?
A. Urinary tract infection D. Acute pyelonephritis
B. Urinary tract injury E. Paralytic ileus
C. Pleurisy

131. A 44 years old woman complains of heavy bleeding per vagina. Transvaginal ultrasound was
done and was normal. Which of the following would be the most appropriate investigation for her?
A. Hysteroscopy
B. Endometrial biopsy
C. Full blood count
D. High Vaginal Swab
E. Anti-fibrinolytics

132. A pregnant lady on her 28 weeks pregnancy presents with BP 160/100mmHg and proteinuria of
0.2 gm/ day. What is your most probable diagnosis?
A. Gestational hypertension
B. Pre eclampsia
C. Eclampsia
D. Chronic hypertension

133. A woman 5 days post-operative for bilateral salpingo-oopherectomy and abdominal


hysterectomy has developed abdominal pain and vomiting with associated abdominal distension and
cannot pass gas. No bowel sounds heard, she is also hydrated. What is the most appropriate next
step?

A. Plain X-ray abdomen


B. Exploratory Laparoscopy
C. CT scan
D. USG
E. Barium Enema

134. A 54 years old patient 7 days after a total hip replacement presents with acute onset
breathlessness and a raised JVP. Which of the following investigations will be most helpful in leading
to a diagnosis?
A. CXR
B. CTPA
C. V/Q Scan
D. D Dimers
E. Doppler USG of legs

135. A 19 years old man presents with weight loss, increasing thirst and increased frequency of going
to the washroom. His father, grandfather and two sisters have been diagnosed with Diabetes Mellitus.
What is the most likely type of diabetes this patient is suffering from?
A. Insulin-dependent diabetes mellitus (IDDM)
B. Noninsulin-dependent diabetes mellitus (NIDDM)
C. Latent autoimmune diabetes of adults(LADA)
D. Maturity Onset Diabetes of the Young(MODY)
E. Diabetic ketoacidosis (DKA)
136. A 56 years old lady with lung cancer presents with urinary retention, postural hypotension,
diminished reflexes and a sluggish papillary reaction. What is the most likely explanation for her
symptoms?
A. Paraneoplastic syndrome
B. Progression of lung cancer
C. Brain Metastasis
D. Hyponatremia
E. Spinal Cord Compression

137. A 65 year old male patient presents with dysphagia, weight loss and is thought to have
oesophageal cancer. What will be the best diagnostic investigation for this patient?
A. Barium swallow
B. Endoscopy and Biopsy
C. CT chest
D. CXR
E. Serum Tumour Markers
138. A 13-year-old girl complains of a 2-day history of hoarseness of voice associated with dry
cough. She feels feverish. On direct laryngoscopy her vocal cords are grossly oedematous. What is
the single most appropriate investigation?
A. No investigation required. D. Bronchoscopy
B. Sputum for acid fast bacilli. E. Cervical spine X-ray.
C. Laryngoscopy.

139. A patient,who suffered from an acute myocardial infarction a day ago, suddenly developed rapid
pounding and fast breathing. What is the most probable cause of this?
A. Ventricular ectopics
B. Atrial fibrillation
C. Ventricular tachycardia
D. Complete heart block
E. Acute pericarditis

140. A 56 year old woman with history of breast cancer 10 years ago when radical mastectomy and
axillary lymph node removal were done for her now complains of swollen upper limb 3 weeks after
an insect bite. The bite site is better but gross oedema still present. What is the cause?
A. Lymphoedema
B. Breast Ca
C. Allergy
D. Filariasis
E. Infection

141. A 5 year old girl had ear ache and some yellowish foul smelling discharge, perforation at the
attic and conductive hearing loss. She has no past history any ear infections. What is the diagnosis?
A. Acute otitis Media
B. Otitis media with effusion
C. Acquired cholesteatoma
D. Congenital cholesteatoma
E. Otitis externa

142. An elderly 80 year old woman lives in a residential home presents with agitation, confusion and
abdominal pain. She has been diagnosed to have faecal impaction. What will be the first step in your
management?
A. High fibre diet
B. Phosphate enema
C. Sodium picosulphate
D. Senna
E. Bran
143. A 38 years old patient presented with tingling, numbness, paraesthesia, respiratory stridor and
involuntary spasm of the upper extremities. She has undergone surgery for thyroid carcinoma a week
ago. What is the most probable diagnosis?
A. Thyroid Storm
B. Hyperparathyroidism
C. Unilateral Recurrent Laryngeal Nerve Injury
D. External Laryngeal Nerve Injury
E. Hypocalcemia

144. A 54 years old woman has presented with episodes of abdominal ache, vomiting and postural
hypotension. She also has a dark pigmentation of her skin. A diagnosis of Addison’s disease was
made. What is the most likely electrolyte abnormality expected in this patient?
A. High Na, low K
B. Low Na, high K
C. Low Na, low K
D. High Na, high K
E. Low Na, normal K

145. A mother has come to the GP. She has 2 daughters, one of whom has cystic fibrosis. The other
daughter is normal. She would like to know what is the chance her other daughter will be a carrier.
A. 1 in 4
B. 2 in 4
C. 1 in 2
D. 2 in 2
E. 1 in 3

146. A 2 years old girl has frequency, urgency and burning micturition. She has some supra pubic
tenderness. Which one of the following is the most appropriate initial investigation for this patient?
A. Supra pubic aspiration of urine for Culture and Sensitivity
B. Clean catch of urine for Culture and Sensitivity
C. USG
D.IVU
E. Micturating cysto-urethrogram

147. A 55 year old male presents with a gradually increasing right sided scrotal swelling over 3
months. On examination, the swelling transilluminates and it is possible to palpate above it. What is
the most appropriate diagnosis?
A. Testicular tumour
B. Indirect inguinal hernia
C. Direct inguinal hernia
D. Hydrocele
E. Varicocele

148. A 50 year old man presents with decreased body mass, mixed iron and folic acid deficiency, and
diarrhoea after recent travelling abroad to Barbados. He is also stating that his stools are smelly and
difficult to flush away. What is most discriminative investigation?
A. Blood cultures
B. Thyroid function tests (TFTs)
C. Serum calcium
D. CT scan
E. Jejunal biopsy

149. A 63 years old woman underwent knee surgery and was kept on low molecular weight heparin
postoperatively. On the 6th post-operative day she developed sudden sever chest pain with
breathlessness. Her ECG and chest x-ray were normal. What is your next action?
A. Stop low molecular weight heparin and start Warfarin
B. Keep on low molecular weight heparin and do CT. pulmonary angiography.
C. Increase the dose of low molecular weight heparin and do CT. pulmonary angiography.
D. Keep on low molecular weight heparin, add warfarin and do CT. pulmonary angiography.
E. Change to unfractionated heparin and do CT. pulmonary angiography.

150. A 35 years old man who has served in the army presents with lack of interest in enjoyable
activities and feeling low. He doesn’t feel like reading the news or watching movies as he believes
there is violence everywhere. What is the most appropriate first line therapy?
A. Citalopram
B. Lofepramine
C. Cognitive Behavioural Therapy
D. Chlordiazepoxide
E. Desensitization

151. A 55 year old man was involved in a fight near his pub on Saturday night. He is bleeding from
his ear. He complains of ringing in his ear and deafness. What is the most important investigation for
his condition?
A. CT head
B. X-ray skull
C. Otoscopy
D. Do nothing
E. MRI Inner Ear

152. A man presents with anaemia and profuse watery diarrhoea. He is found to be deficient in iron
and Vit B12. What is the possible reason for his anaemia?

A. IBD
B. Villous atrophy
C. IBS
D. Alcohol
E. Bacterial Infection

153. A 46 year old man was found outside a shop. He was confused, had ataxia and opthalmoplegia.
What is the most likely condition the patient is suffering from?

A. Delirium tremens
B. Wernicke’s encephalopathy
C. Korsakoff’s syndrome
D. Hepatic encephalopathy
E. Meningitis
154. A 75 year old man was undergoing treatment of pneumonia during which he developed stroke,
for which thrombolysis was administered. Following this he developed bloody diarrhoea with passage
of clots. Colonoscopy revealed multiple ulcers in transverse colon which were inflamed. What could
be the possible cause of bleeding?
A. Crohn’s disease
B. Complication of thrombolysis
C. Ulcerative Colitis
D. Bowel ischaemia
E. Pseudo membraneous colitis

155. A 28 year old shipyard worker was admitted for pain in calf while at work which has been
increasing over last 3 months. There is no history of hypertension or diabetes but he is a smoker. On
examination, he has loss of posterior tibial and dorsalispedis pulsations along with a non-healing ulcer
at the base of right 1st MCP joint. What is the most probable diagnosis?
A. Thromboangitis Obliterans
B. Sciatica
C. Deep Vein Thrombosis
D. Baker’s cyst
E. Embolus

156. A pregnant woman wants to know the possibility of her unborn male child being affected by
Duchene muscular dystrophy. Her first son has the disease.
A. 25%
B. 50%
C. 75%
D. 100%
E. 33%

157. A 60 year old man is posted for rectal surgery. His vitals are stable and there is no
comorbidities. What is the best time to start him on prophylactic antibiotics?
A. Local therapy perioperative D. None required
B. Antibiotic 24 hours prior to surgery E. One week
C. Antibiotic at induction of anaesthesia

158. A 40 year old man has sustained a full and partial thickness burn over his forearm and upper arm
along with pain and swelling. What is the most appropriate initial treatment?
A. Escharotomy
B. Fasciotomy
C. Oral analgesia
D. Resuscitation with IV Fluids
E. Referral to Burns Unit

159. A 50 year old female has terminal stage ovarian cancer with ascites. She developed intestinal
obstruction and has spastic abdominal pain. What is the best treatment for the management of her
abdominal pain?
A. IV Morphine
B. Palliative colostomy
C. Hyoscine bromide
D. Radiotherapy
E. Chemotherapy
160. A 28 year old drug user presents to the A&E with collapse and anuria. His serum potassium
concentration is 7.5mmol/L. His chest x-ray shows early pulmonary edema. What is the next
appropriate management for this patient?
A. Urgent haemo-dialysis
B. IV calcium gluconate
C. IV insulin +dextrose
D. Furosemide
E. IV Normal Saline 0.9%

161. A 14 year old girl has been brought by her school teacher as the teacher is concerned about the
girl’s health. She has a BMI of 14.2 and has been having bouts of dizziness recently. What is the most
appropriate next step?
A. Psychiatric routine referral
B. Admission to psychiatric ward
C. Admission to medical ward
D. CBT
E. Diet plan

162. A 47 year old ex-soldier suffers from low mood and anxiety. He cannot forget the images he
faced before and has always had flash-backs. He is not able to watch the news because there are
usually some reports about war in the news. What is he suffering from?
A. Depression
B. Post traumatic stress disorder
C. Panic attack
D. Agoraphobia
E. General anxiety disorder

163. A man, who was drunk, left a pan on the hob and dozed off causing fire in his house. He was
drowsy. His house caught fire. He was eventually brought to the A &E with laryngeal oedema and
hoarse voice. What is the next best step?
A. Anaesthetics for help
B. I/V fluids
C. CT scan
D. MRI
E. X-RAY

164. A 45 year old man who has undergone a recent subtotal gastrectomy for peptic ulcer disease has
now developed anaemia with haemoglobin 6.8 , tiredness, fatigue and loss of vibration sensation in
legs. What is the underlying cause of his symptoms?
A. B12 deficiency
B. Iron deficiency
C. Folate deficiency
D. Vit C deficiency
E. Metastases

165. A child has hypothyroidism. What feature is associated with it?


A. Microglossia
B. Prolonged neonatal jaundice
C. Un-descended testis
D. Anal tag
E. Cleft soft palate
166. A lady with breast cancer has undergone axillary lymph nodes clearance. She develops arm
swelling after being stung by a bee. What is the most likely mechanism responsible for the swelling?

A. Lymhoaedema
B. Cellulitis
C. Hypersensitivity reaction
D. Deep venous thrombosis (DVT)
E. Fluid retention

167. A 67 year old woman with carcinoma of the bronchus presents with drowsiness, confusion and
nausea. Her Ca is 2.95, Creatinine 135, Urea 9.3. What is the best next step?
A. Oral steroids
B. IV Steroids
C. IV normal saline
D. IV dextrose
E. IV Calcium chloride

168. A 36 year old lady comes in for routine follow up. She has been diagnosed with peptic ulcer
disease and treated with triple therapy. What is the most appropriate investigation to confirm the
eradication of H. Pylori?
A. Urea breath test
B. Endoscopy
C. Ambulatory ph monitoring
D. Barium Meal
E. Blood Cultures

169. A 4 year old boy presents with fever, sore throat and lymphadenopathy. The diagnosis of
tonsillitis has been established. He had 3 episodes of the condition last year. What is the most
appropriate management for this patient?
A. Tonsillectomy
B. Paracetamol ibuprofen
C. Oral Penicillin V
D. IV Penicillin
E. No Treatment required

170. A 50 year old patient has been prescribed propranolol for a cardiac condition, but his GP now
wants to stop this drug. Which of the following is the most important reason for this decision?
A. Familial tremor
B. Partial AV heart block
C. Mild hypertension
D. Angina pectoris
E. Supraventricular tachycardias

171. A 30 year old woman presents with intermittent breast pain. On examination a tender smooth
mobile lump measuring approximately 4cm by 3cm is found, which is not attached or fixed to deep
tissues. What is the best method for diagnosis?
A. Mammogram
B. Ultrasound
C. CT scan
D. Fine Needle Aspiration
E. Core Biopsy
172. A 35 year old man was found to have hematuria, after 3 days of having sore throat, running nose,
burning and increased lacrimation of eyes and headache. Which of the following would explain best
the reason for hematuria?
A. Thrombotic Thrombocytopoenic Purpura
B. Haemolytic Uraemic Syndrome
C. Post streptococcal glomerulonephritis
D. Berger’s disease
E. Minimal Change disease

173. A 46 year Afro Caribbean man is found to have  blood pressure of 160/90mmHg on three
separate occasions. What is the best initial treatment?
A. ACE inhibitor
B. Beta-blocker
C. Angiotensin II receptor blocker
D. No treatment
E. Calcium channel blocker  

174. A 24 year old man is suffering from Generalized Anxiety Disorder. Which drug may be
preferred in the short term to treat this patient from the following options?
A. Zolpidem
B. Buspiron
C. Midazolam
D. Triazolam
E. Phenobarbital

175. A 34 years old African-Caribbean man, with a history of Sarcoidosis, has presented with bilateral
kidney stones. What is the most likely cause for this patient’s stones?
A. Hyper calcemia
B. Hyper uricemia
C. Diet
D. Recurrent UTIs
E. Hyperparathyroidism

176. A man is admitted in a ward and develops varicella zoster after two days. Which of the following
group below should get immunoglobulin only?
A. All staff in contact with the patient
B. All pregnant women who are tested negative for antibodies.
C. All family members
D. None
E. All children

177. A lady comes with severe liver disease and hematemesis. Her INR is more than 10. What should
she be given?
A. Fresh Frozen Plasma
B. Steroids
C. Whole blood
D. IV Fluids
E. Vitamin K

178. A diabetic patient on investigation has got impaired renal function, but USG shows normal sized
kidneys. What will be the most appropriate medication for this patient?
A. Propranolol
B. Ramipril
C. Nifedipine
D. Losartan
E. Bendroflumathiazide

179. A 22 year old boy, who fell from a ladder, presented with haematuria.His blood pressure and
pulse were normal and no bruises were found. What is the most appropriate investigation?

A. CT abdomen E. X ray abdomen


B. IVU
C. Arteriogram
D. Colposcopy

180. With prolonged use in epilepsy, which of the following drugs can cause coarsening of facial
features, hirsutism, gingival hyperplasia and osteomalacia?

A. Valproic acid
B. Carbamazepine
C. Ethosuximide
D. Phenytoin
E. Gabapentin

181. A 30 year old male presents with a left sided scrotal swelling and scrotal discomfort which
increases toward the end of the day and after long standing. What is the most appropriate diagnosis?
A. Inguinal hernia
B. Testicular tumors
C. Hydrocele
D. Varicocele
E. Testicular torsion

182. During a laparoscopicprocedure, a trochar is inserted halfway between the umbilicus and the
anterior superior iliac spine. What are the structures most likely to be pierced?
A. Rectus sheath
B. Linea alba
C. External oblique aponeurosis
D. Internal oblique and transverse abdominal
E. Both C and D

183. A 26 year old man presented with history of passing loose stools for the past 2 months. Stools are
mixed with blood and mucus and is associated with abdominal pain. He undergoes a colonoscopy
after which he was started on treatment. What is the treatment of choice for his condition?
A. Mesalazine
B. Corticosteriods
C. Infliximab
D. Cyclosporin
E. Azothiaprine

184. An 11 year old boy is being checked by the diabetic specialist nurse. His HBA1C was high and
he has been skipping meals recently. He has been unhappy at school. Which single member of clinical
team would you refer him to next?
A. Refer to GP
B. Refer to paediatrician
C. Refer to dietician
D. Refer to clinical psychologist
E. Refer to school nurse

185. A 35 year old patient is complaining of severe pain along the right side of the jaw and face. What
is the best choice for this neuralgic pain?
A. Methadone
B. Ibuprofen
C. Carbamazepine
D. Lorazepam
E. Amphetamine

186. A 44 year old man has been taking diclofenac and paracetamol over the counter for long standing
lower back pain. He is at risk of which side effect?
A. Diarrhoea
B. Vomiting
C. Upper GI bleed
D. Colon cancer
E. Acute Renal Failure

187. A 67 year old man with long standing constipation presents with sudden onset of confusion.
What is the treatment of choice?
A. Sodium picosulpahte
B. Danthron
C. Phosphate enema
D. Methylcellulose
E. Bran

188. A 55 year old man presents with history of headaches and vomiting. On examination he has right
sided double vision. What is the location of his lesion?

A. Abducent nerve
B. Trochlear nerve
C. Oculomotor
D. Optic nerve
E. Oculomotor nerve

189. A 30 year old man from Australia returned from a business trip to Indonesia 6 days ago,
presented with complaints of fever, joint and muscle ache and headache, in particular behind the eyes,
for the past 2 days. What is the most probable diagnosis?
A. Malaria
B. Chicken pox
C. Tuberculosis
D. Lyme's disease
E. Dengue

190. A 16 year old girl presents with heavy and painful vaginal bleeding. Choose the most
appropriate initial investigation from the following options.
A. Endometrial sampling
B. Trans vaginal ultra sound scan
C. Hysteroscopy
D. Pelvic ultrasound
E. Examination under anesthesia

191. Which of the following drugs produces respiratory depression and maybe reversed by the
administration of flumazenil?
A. Fentanyl
B. Propofol
C. Ketamine
D. Midazolam
E. Enflurane

192. A patient who is a known case of tuberculous abscess with the history of previous abscess
drainage presented with fever and tenderness between L2 L3 vertebra. Which is the best investigation
for this patient?
A. X-ray
B. CT
C. USG
D. MRI
E. Blood culture

193. A 6 hours old child born at 34 weeks of gestation presents with tachypnoea, nasal flaring and
grunting. His chest x-ray is below. What is the most probable diagnosis?

A. Transient Tachypnoea of Newborn


B. Meconium Aspiration
C. Bronchiolitis
D. Streptococcal Pneumonia
E. Respiratory Distress Syndrome

194. A 7 months old baby is irritable and fails to gain weight. His stools are pale, frothy and bulky.
What will be the most diagnostic test to confirm the condition?

A. Sweat Test D. Stool Culture


B. Jejunal Biopsy E. Liver biopsy
C. Endomysial Antibodies
195. A 56 years old woman with Multiple Sclerosis presents with drooping of the left side of her lips.
She also has loss of sensation over her face, hearing impairment and some in-coordination of her
movements. What is the most likely anatomical site affected?
A. Cerebellum
B. Cerebrum
C. Spinal Cord
D. Brain Stem
E. Optic Nerve

196. A 38 years old female returns from a holiday in Greece. Now has dry cough, bilateral
consolidated areas in the chest x-ray. What is the best antibiotic you would prescribe?
A. Clarithromycin
B. Ciprofloxacin
C. Amoxycillin
D. Cephalexin
E. Gentamycin

197. A 23 year old woman with neurofibromatosis and cafe au lait spots wants to know what the
chances of her child inheriting the condition are.
A. 1 in 2
B. 1 in 4
C. None
D. 1 in 1
E. 1in 3

198. A 70 year old male who underwent an anterior resection of the rectum 5 days ago, now presents
with sudden severe abdominal pain. He is afebrile with blood pressure of 110/70, pulse rate of 110
min. What is the most appropriate diagnosis?

A. Abdominal sepsis
B. Intra-abdominal hemorrhage
C. Anastomotic leak
D. Intestinal obstruction
E. Post-operative fistula formation

199. A 22 year old male came to the hospital after an injury in his hand while playing basketball.
Examination reveals there is avulsion of extensor tendon from the distal phalanx. What is the
SINGLE most probable deformity?

A. Dinner fork deformity.


B. Game keeper thumb.
C. Mallet finger.
D. Gun stock deformity.
E. Garden spade deformity.

200. A 21 year old girl who is sexually active presents with small painful multiple genital ulcers.
Which of the followings maybe useful topically as an initial treatment for her condition?
A. Acyclovir
B. Amantadine
C. Ritonavir
D. Trifluridine
E. Foscarnet
PLAB 1 MOCK ANSWERS : Mock 3
1 C 41 A 81 E 121 C 161 C
2 D 42 A 82 C 122 A 162 B
3 C 43 C 83 C 123 B 163 A
4 D 44 B 84 B 124 B 164 A
5 A 45 D 85 E 125 B 165 B
6 B 46 B 86 C 126 B 166 A
7 B 47 A 87 B 127 A 167 C
8 B 48 D 88 E 128 B 168 A
9 C 49 B 89 A 129 B 169 B
10 C 50 C 90 B 130 B 170 B
11 B 51 C 91 A 131 C 171 E
12 E 52 A 92 A 132 A 172 D
13 E 53 C 93 C 133 A 173 E
14 D 54 C 94 B 134 B 174 B
15 B 55 C 95 A 135 A 175 A
16 D 56 C 96 D 136 A 176 B
17 A 57 D 97 C 137 B 177 A
18 C 58 A 98 C 138 A 178 B
19 A 59 D 99 C 139 C 179 A
20 B 60 A 100 C 140 A 180 D
21 A 61 A 101 F 141 D 181 D
22 A 62 E 102 E 142 B 182 E
23 D 63 A 103 E 143 E 183 B
24 B 64 A 104 C 144 B 184 D
25 B 65 A 105 B 145 C 185 C
26 B 66 C 106 C 146 B 186 C
27 C 67 B 107 E 147 D 187 A
28 E 68 E 108 A 148 E 188 A
29 A 69 B 109 A 149 B 189 E
30 A 70 C 110 B 150 C 190 D
31 C 71 C 111 B 151 A 191 D
32 D 72 B 112 C 152 B 192 D
33 D 73 D 113 C 153 B 193 E
34 A 74 C 114 B 154 E 194 B
35 D 75 E 115 D 155 A 195 D
36 A 76 B 116 C 156 B 196 A
37 C 77 C 117 D 157 C 197 A
38 A 78 C 118 B 158 D 198 C
39 D 79 C 119 E 159 B 199 C
40 A 80 E 120 C 160 B 200 A
Dr. Swamy PLAB Courses Ltd
PLAB 1 MOCK TEST: MOCK 4 TIME ALLOWED: 3HRS
SINGLE BEST ANSWER

1. A 75-year-old woman who had a carcinoma of the distal rectum underwent an anterior resection.
The blood supply of the tumour arises from which of the following?
A. Coeliac artery
B. Ileocolic artery
C. Inferior mesenteric artery
D. Internal iliac artery
E. Superior mesenteric artery

2. A 34 year old patient presented with redness and pain in the eye. He is also suffering with back
pain that is worse in the morning. He is taking NSAIDs for back pain. What is the cause of red eye?
A. Ibuprofen
B. Steroids
C. Anterior Uveitis
D. Posterior Uveitis
E. Trauma

3. A 33 year old female with bipolar affective disorder had an acute depressive episode which was
controlled by antidepressants. Which drug will be best for prophylaxis in this patient?
A. Mood stabiliser
B. Selective serotonin reuptake inhibitors (SSRI)
C. Tricyclic Antidepressant (TCA)
D. Antipsychotic

4. A 15 year old boy presents with severe earache after having a swimming session. He does not allow
you to do an otoscopy and the ear is severely tender to touch. What is the most appropriate antibiotic
therapy for his condition?
A. Amoxicillin
B. Ciprofloxacin
C. Flucloxacillin
D. Meropenem

5. A young woman on combined oral contraceptive pills has been prescribed Amoxicillin for sinusitis.
What should be the advice given to her?
A. Increase dose of OCPS
B. Reduce dose of Amoxicillin
C. Use barrier methods for 2 weeks
D. No extra precautions needed
E. Barrier methods only if patient develops diarrhea

6. A middle aged man complains of a node which has been growing in his nose for several months.
Now it is firm with central depression. It’s 0.6cm in size. What is the SINGLE most likely diagnosis?
A. Basal cell carcinoma F. Kerato acanthoma
B. Squamous cell carcinoma G.Molluscum Contagiosum
C. Lymph node
D. Melanoma
E. Kaposi’s Sarcoma
7. A 7 year old boy presents with her mother to the GP surgery. His mother describes he had
presented this since 3 weeks ago. He had not experienced any trauma. No other symptoms associated
with the condition. On examination the boy has non tender swollen ankles bilaterally. There is no rash
or lesion. He is otherwise well. Which single test would be the best as an initial assessment?

A. Plasma electrolytes.
B. Album
C. Total serum protein
D. Anti-streptolysin

8. A lady comes in 6 weeks after her normal vaginal delivery for contraceptive advice. She is breast
feeding her child and requests for reversible contraception. She is planning another pregnancy in 6
months time and is scared of injections. What is the best option for her?
A. COCP
B. Cu IUCD (intrauterine contraceptive device)
C. Mirena coil
D. POP
E. Transdermal patch

9. A 68 year old man with diabetes and hypertension was noted to have a cholesterol level on
3.4mmol/l. He was also noted to have micro-albuminuria. What is the best drug to add to his regimen?
A. Ace inhibitors
B. Statins
C. Amylodipine
D. Biguanides

10. A 65 year old man on dexamethasone underwent a surgery. During and after the surgery, his
blood glucose level was around 17 – 19 mmol/l. What will you give the patient?
A. Insulin
B. Oral hypoglycaemics
C. Remove dexamethasone
D. IV saline
E. IV dextrose

11. Which statement best describes the planes of the abdomen?

A. The transpyloric plane lies halfway between the xiphoid and the symphysis pubis.
B. The transpyloric plane passes through the hilar of the kidneys.
C. The subcostal plane is at the level of the body of L2.
D. The iliac crests lie at the level of L5.
E. The umbilicus usually lies at the level of the L4/L5 disc.

12. A 55 year old man is brought in the A& E department for acute abdominal pain. He is found to
have a strangulated hernia for which he undergoes emergency surgery. 3 days post op, he is in a very
agitated state claiming to see spiders on the wall and crawling over his body. He is sweaty, has
palpitations and is confused. What is the most likely diagnosis?
A. Delirium tremens
B. Wernicke’s encephalopathy
C. Korsakoff’s syndrome
D. Dysthymia
E. Cyclothymia

13. A 5 year old child was admitted with history of feeling tired and lethargic all the time, bleeding
gums and sore throat since the last 3 months. On examination he is found to have hepato-
splenomegaly. What is the most probable diagnosis?

A. Acute lymphoblastic leukemia (ALL)


B. Acute myeloid leukemia (AML)
C. Chronic myelogenous (or myeloid) leukemia (CML)
D. Chronic lymphocytic leukaemia (CLL)
E. Lymphoma

14. An 18 month old boy has been brought to the Emergency department because he has been
refusing to move his left arm and crying more than usual for the past 24 hours. He has recently been
looked after by his mother’s new boyfriend while she attended college. Assessment shows multiple
bruises and a fracture of left humerus which is put in plaster. What is the SINGLE most appropriate
next step?

A. Admit under care of paediatrician


B. Discharge with painkiller
C. Follow up in fracture clinic
D. Follow up in paediatric outpatients
E. Follow up with general practitioner (GP)

15. An 8 year old child presents with recurrent abdominal pains, occasional headaches but she
maintains a good appetite. Examination is normal. FBC, U&E and other investigations are normal.
What would you do for her next?

A. Ultrasound abdomen
B. CT head
C. Reassure
D. Analgesics

16. A young child just diagnosed with chicken pox. Usually goes to day care. What is the most
appropriate advice?

A. Child should be admitted to hospital straight away


B. Isolate the child from parents and siblings at home
C. Advise him he can go back to nursery when the rash is crusted over

17. A lady presents with abdominal pain, dysuria, dyspareunia and vaginal discharge. What is your
next step?
A. Laparoscopy E. Ultra sound
B. High vaginal swab
C. Hysteroscopy
D. Laparotomy

18. A primiparous woman with no previous infection with Herpes zoster is 18 weeks pregnant. She
had a recent contact-history with a young patient, aged 21, having widespread chicken pox. What is
the most suitable management for the pregnant lady?
A. Acyclovir orally
B. IV Acyclovir +IV Immunoglobulin
C. IV Acyclovir
D. Reassure
E. IV Immunoglobulin
19. A three- month old baby was miserable and cried for two hours following his first routine
immunization with diphtheria, tetanus, polio, Haemophilus influenza- B (HiB) and meningitis. What
is the single most appropriate action?
A. Defer immunization for two weeks
B. Do not give vaccine
C. Give half dose of vaccine
D. Give Paracetamol with future doses of the same vaccine
E. Proceed with standard immunization schedule

20. A 55 year old lady with swelling on the abdomen below the umbilicus on the right side. What is
the SINGLE most appropriate lymph node?
A. External iliac lymph nodes F. Submental lymph nodes
B. Pre aortic lymph nodes G. Submandibular lymph nodes
C. Aortic lymph nodes H. Deep cervical lymph nodes
D. Inguinal lymph nodes
E. Iliac nodes

21. A 45 year old man presents to A&E with pain the chest and shortness of breath. He has no
recollection of what has happened to him. His breath smells of alcohol. What is the most probable
diagnosis looking at the given picture?

A. Shingles
B. Cellulitis
C. Blunt trauma
D. ITP
E. Hypersensitivity reaction

22. A 45 year old man diagnosed with Multiple Sclerosis is admitted after taking 100 Paracetamol
tablets. He seems well, is not vomiting and is alert. He is refusing all treatment. What should you do
next?

A. Gastric lavage
B. Activated charcoal
C. N-Acetyl cysteine
D. Methionine
E. Evaluate his capacity to refuse treatment

23. A child had three attacks of tonsillitis last year. What will be the most appropriate management for
this child?

A. Treat when ill


B. Surgery
C. Prophylactic Antibiotics
D. Refer to ENT
E. Do Nothing
24. A patient 10 days post-op for rectal carcinoma presents with high grade fever and mass in pelvis.
What is the most probable diagnosis?
A. Pelvic Abscess
B. Recurrence of rectal carcinoma
C. Metastasis
D. Pneumonia secondary to aspiration
E. Septicemia

25. A 28 year old woman presents with pain and an ulcer on labia major. What is the lymphatic
drainage of labia major?
A. Superficial inguinal
B. Deep inguinal
C. Para aortic
D. Iliac

26. A 19 year old girl is brought into A & E after an argument with her boyfriend. She has multiple
superficial lacerations on her forearms and wrists which have been attended to by the medical team.
She has attended A&E many times in the past with similar episodes of self-harm and drug overdoses.
What is the most likely diagnosis?

A. Obsessive compulsive personality


B. Schizoid personality
C. Borderline personality
D. Dependent personality

27. A man with dementia has an ulcerative lesion on forehead. He wants it removed so ‘it can help
improve his memory’. Wife says he is not fit to give consent. What will you do?
A. Get letter signed from the GP
B. Get letter signed from wife
C. Get letter signed from patient
D. Refer to psychiatrist to assess the mental capacity to give consent

28. A young woman is brought to the hospital after consuming 17 tablets of paracetamol to the A&E.
On evaluation it is noted that she is clinically depressed and has not eaten anything for four days. She
has been brought to the hospital after 9 hours on consuming the tablets. What is the most important
next step?
A. N acetyl cysteine
B. Paracetamol levels
C. Gastric Lavage
D. Activated Charcoal
E. Psychiatric Evaluation

29. A 6 year old child presented with drooling of saliva, and a severe stridor. He is febrile and sick
looking. X ray of the neck in extension shows a thumb sign. Choose the single most likely diagnosis
from the list of options given below.
A. Croup
B. Upper respiratory tract infection (URTI)
C. Diphtheria
D. Acute epiglottitis
30. A child of 24 months can walk, run, can make a pyramid, speaks only two meaningful
words such as mama and dada. How will you manage this child?
A. Reassure
B. Speech consultation
C. Physiotherapy
D. CT scan
E. Admit further assessment

31. A 65 year old man presented with hearing impairment and weight loss. He complains of hearing
buzzing sound and feeling dizziness that is becoming worse slowly. On examination facial numbness
has been noticed. What is the best investigation option for this problem?
A. MRI acoustic meatus
B. CT Head
C. Blood culture
D. Audiometry
E. Tilt Test

32. A 15 years old boy fell of his bicycle and hurt his abdomen. He presents with pain in abdomen, a
CT scan shows a small sub capsular spleen haematoma, his BP is 110/70mmHg and pulse is 80bpm.
What is the next step in his management?

A. Refer to surgeons for observation


B. Immediate Laparotomy
C. Immediate Laparoscopy
D. Discharge with advice
E. Blood transfusion

33. A 4 year boy has cough and arthritis followed by rash on legs and trunk which are non- blanching
on glass test. No history of fever.
PT: 13
APTT : 31
Hb : 12
WBC: 6.5
Platelet: 300x10
Choose the most likely diagnosis?
A. Meningitis septicaemia
B. Haemophillia
C. Henoch-SchoenleinPupura
D. Idiopathic thrombocytopenic purpura
E. Thrombotic thrombocytopenic purpura

34. A 30 year old woman on oral contraceptives presents with dilated tortuous veins crossing her
abdomen to join the tributaries of superior vena-cava. What is the single most likely cause?
A. Intra-abdominal malignancy
B. Ovarian cyst
C. Fibroids
D. Ascites
E. Deep vein thrombosis (DVT)

35. A 45 year old heroin addict was involved in a car crash and is now paraplegic. During the first
week of his hospital stay he cried everyday because he couldn’t remember the accident.
What is the most likely diagnosis?
A. PTSD
B. Severe depression
C. Organic brain injury

36. A 15 year old boy presents with testicular pain for 2 days. There is no history of trauma. On
examination he has a temperature of 38.5˚C and there is tenderness of the right hemi-scrotum. What is
the single most appropriate management?

A. Give antibiotics
B. Give analgesia
C. Reassure
D. USG of scrotum
E. Exploratory surgery

37. A 25 years old man presents with fever and pain in the right lower thigh of one month duration.
On examination the lower third of his thigh is red, hot and tender. The X-ray showed new bone
formation. What is the most probable diagnosis?

A. Osteosarcoma
B. Ewing’s Sarcoma
C. Tuberculus Arthritis
D. Exostosis
E. Fibrosarcoma

38. A 72 year old man had myocardial infarction a few weeks ago. Now he is on medication. He
comes to outpatient department and he complains of yellow colour in his vision. What can be the
cause of this?
A. Digoxin
B. Amitriptyline
C. Atenolol
D. Ramipril
E. Bendroflumethiazide
39. In a laproscopic mesh repair for hernia, when the trochar is inserted at midpoint between
umbilicus and ischial spine, what structure will be pierced?
A. Linea alba
B. Rectus muscle
C. Conjoint tendon
D. External and internal oblique muscles
E. Inguinal ligament

40. A 25 year old male had an injury to the knee while playing football. X ray reveals the condylar
fracture of tibia. What is the SINGLE most probable deformity?
A. Dinner fork deformity
B. Gibbus
C. Cubitus valgus
D. Garden spade deformity
E. Genu valgus
41. A 37 year old patient presented with hearing loss and offensive discharge from the ear. Otoscopy
is performed and crusted tympanic membrane was noticed. A pearly white mass was seen on the attic
when crust was removed. Choose the correct diagnosis from following options
A. Frunculosis
B. Basal cell carcinoma
C. Cholesteatoma
D. Acoustic Neuroma
E. Squamous cell carcinoma

42. A 62 years old lady presents with a pathological fracture of thoracic vertebra, this is found to
be associated with an underlying metastatic lesion, what is the most probable primary cancer
site?
A. Ovary
B. Lung
C. Breast
D. Cervix
E. Colon

43. A man with a family history of panic disorder is brought to the hospital with palpitations, tremors
and sweating and muscle tightness on three occasions in the last six weeks. He does not complain of
headache and his blood pressure on each presentation is within normal limits. What is the single most
appropriate long term treatment for this patient?

A. Diazepam
B. Olanzapine
C. Haloperidol
D. Fluoxetine
E. Alprazolam

44. A 37year old female working as a health care assistant in a nursing home, comes to the A&E with
complaints of severe itching all over her body. On asking she replies that she had applied cream on
the body of a resident in the nursing home who had similar itches. What is the mechanism of her itch?

A. Allergic reaction
B. Inflammation of keratinocytes
C. Allergic reaction developed due to use of topical steroid creams
D. Subcutaneous bleeding
E. None of the above

45. A 55 year old male patient had TIA which he recovered from. He has a history of stroke and upon
examination his heart rate was in sinus rhythm. He is already on aspirin 75mg and anti-hypertensive
drugs. What other action should be taken?
A. Add Clopidogrel only
B. Increase the doses of aspirin to 300mg
C. Add Warfarin
D. Add Clopidogrel and Statins
E. Add Statins only
46. A man presents with medial deviation of his right eye. Which nerve is affected?
A. Left trochlear
B. Left oculomotor
C. Right trochlear
D. Right abducens
E. Right oculomotor

47. A middle age male after myocardial infarction is feeling unwell. The ECG has been performed
and shows a prolonged QRS complex and T wave is tented Sodium-136 mol/L
Potassium- 6.2mmol/L, Urea- 5.0 mmol/L
What is the most appropriate management?
A. Calcium gluconate E. Calcium
B. Calcium resonium
C. Calcium with Vitamin D supplements
D. Vitamin D

48. A 57 year old man who had myocardial infarction a few months ago has been having low mood. A
diagnosis of moderate depression has been established. Which medication is the best treatment for
him?
A. Selective serotonin re-uptake inhibitors (SSRI)
B. TACs
C. Monoamine oxidase inhibitor (MAOI)
D. Benzodiazepams
E. Mood stabilisers

49. A 64 year old man presents with ipsilateral vertigo, tinnitus and left side hearing loss. On
examination Rinne's test is positive and Weber’s test is lateralised to the right ear. What is the most
appropriate investigation?
A. CT scan
B. Magnetic resonance imaging (MRI) of brain
C. X-ray
D. Audiometry
E. No investigation required
F. Caloric Testing
50. A 74 years old lady who has had a stroke in the past, had an indwelling catheter for 10 months.
She presents with bluish-purple discoloration of the catheter bag. What is the most likely explanation
for this change?
A. Normal Change
B. Catheter Degradation
C. Acidic Urine
D. Alkaline Urine
E. Bacterial Colonisation of the Urinary Tract

51. A patient on antihypertensive drugs presents to Accident and Emergency department complaining
of sudden loss of vision. On examination of the fundus, tortuous blood vessels and stormy sunset
appearance was seen. What is most likely diagnosis?
A. Papiloedema
B. CRAO
C. Anterior ischaemic optic neruropathy
D. CRVO
E. Retinal detachment
52. A four years old boy with a febrile convulsion lasting eight minutes has been given intravenous (IV)
Lorazepam to control the convulsion. What is the SINGLE most likely serious side-effect?
A. Amnesia
B. Anaphylactic shock
C. Diabetes
D. Bronchospasm
E. Cardiac arrhythmia

53. A 33 year old female complains of diplopia on upright gaze. On examination ptosis can be seen.
There are no other complains or any significant past medical history. What is the most appropriate
investigation for this patient?
A. Ophthalmoscopy
B. Visual field test
C.Thyroid function test
D. CT scan
E. Checking red reflex

54. A 57 year old man, who is a known hypertensive, is on oral anti-hypertensives. However, he is
finding it difficult to mobilise as he feels dizzy whenever he tries to get up. What is the most
appropriate investigation for him?
A. Ambulatory blood pressure
B. Electrocardiogram (ECG)
C. Magnetic resonance imaging (MRI)
D. Chest X ray
E. CT scan

55. A 6 year old boy fell in the playground. He has not been using his forearm. He complains of pain.
On examination there is no sign of deformity or swelling. However, there is minimal tenderness on
examination. What is the most appropriate diagnosis?

A. Fracture of mid radius


B. Fracture of mid ulnar
C. Fracture of neck of humerus
D. Fracture of shaft of humerus
E. Green stick fracture of distal radius.

56. A 72 years old woman has been taking loop diuretics for left ventricular failure. She now presents
with palpitations and muscle weakness. What is the SINGLE most likely serum electrolyte pattern to
be found?
A. Sodium 130 mmol/L, potassium 2.5 mmol/L
B. Sodium 130 mmol/L, potassium 5.0 mmol/L
C. Sodium 140 mmol/L, potassium 4.5 mmol/L
D. Sodium 150 mmol/L, potassium 2.5 mmol/L
E. Sodium 150 mmol/L, potassium 3.5 mmol/L

57. What is the best treatment for candidiasis in the throat?


A. Chlorhexidine mouthwash
B. Nystatin suspension
C. Oral Fluconazole
D. IV Metronidazole
58. A 45 year old man had recently started taking antihypertensive therapy. Six months later his
Random Blood Sugar was found to be 14.0 mmol/l. Which single drug is most likely to have caused
this?
A. Amlodipine
B. Bendroflumethiazide
C. Doxazosin
D. Losartan
E. Ramipril

59. A 24 year old woman has severe depression three months after the birth of her first child. She is
breastfeeding but is otherwise unable to look after the baby and is convinced that her family is likely
to kill her. She has no interest in anything and keeps crying. What is the most appropriate treatment?
A. Fluoxetine
B. Citalopram
C. Cognitive behavioural therapy
D. ECT
E. Haloperidol

60. A 31 years old woman presents 7-10 days following childbirth, with loss of feeling for the child,
loss of appetite, sleep disturbance and intrusive and unpleasant thoughts of harming the baby. What is
the best treatment for this patient?
A. Fluoxetine
B. Haloperidol
C. Cognitive Behavioural Therapy
D. Reassurance
E. Electro Convulsive Therapy

61. A girl presents with signs of hyperventilation. What is the most likely arterial blood gas (ABG)
derangement?
A. pH increased pCo2 increased
B. pH decreased pCo2 increased
C. pH increased pCo2 decreased
D. pH decreased pCo2 decreased

62. A 6 year old child fell on his nose 2 days ago. His parents have now brought him with difficulty in
breathing. On examination he has a fever and his nasal bones are straight. What is the single most
likely diagnosis?
A. Nasal polyp
B. Septal hematoma
C. Septal abscess
D. Deviated nasal septum
E. Fractured nose

63. A 56 year old man with atrial fibrillation is prescribed digoxin. What side effect will you make the
patient aware of?
A. Seeing visual haloes
B. Nausea
C. Vomiting
D. Diarrhoea
64. A pregnant woman with long-term history of osteoarthritis came to the antenatal clinic with the
complaints of restricted joint movement and severe pain in her affected joints. What is the choice of
drug?
A. Paracetamol
B. Steroid
C. NSAID
D. Paracetamol + Dihydrocodiene
E. Pethidine

65. A 54 year old lady presents with sudden, severe pain in the left half of her skull. She also
complains of pain around her jaw. What is the most likely next step?

A. CT scan
B. MRI
C. Fundoscopy
D. ESR
E. Temporal artery biopsy

66. A patient admitted due to repeated attacks of pancreatitis presents with dementia and loss of
proprioception in the legs. What is the most appropriate treatment?

A. Thiamine
B. Pyridoxine
C. Cobolamin
D. Lypase
E. Antibiotics
67. A 2 year old boy with 38.4°C fever had an episode of tonic clonic seizure at home that lasted for
15 minutes. In A&E patient is conscious and fit has stopped. When temperature was checked, it was
normal. What is most probable diagnosis?
A. Febrile convulsion
B. Infantile spasm
C. Absence seizure
D. Grandmal Epilepsy
E. Focal seizure

68. A 10 years old girl presents with pallor and features of renal failure. She has haematuria as well as
proteinuria. The serum urea and creatinine are elevated. These symptoms started after an episode of
bloody diarrhoea 4 days ago. What is the most probable diagnosis?

A. Thrombotic thrombocytopenia purpura (TTP)


B. Haemolytic-uremic syndrome (HUS)
C. Idiopathic thrombocytopenic purpura (ITP)
D. Henoch-Schonleinpurpura (HSP)
E. Acute renal failure (ARF)

69. A 32 years old woman has undergone a biopsy for a breast lump. The report says “a well
circumscribed lump with clear margins and separated from the surrounding fatty tissue”. What is the
most appropriate interpretation of this report?
A. Fibroadenosis
B. Carcinoma of the Breast
C. Mammary abscess
D. Fibroadenoma
E. Fat Necrosis

70. A West indian woman with anaemia and mild jaundice comes with limb pains and abdominal
pain. And the symptoms are precipitated by ongoing infections. On examination spleen is no more
palpable. What is your most probable diagnosis?
A. G6PD deficiency
B. Sickle cell disease
C. Beta thalassemia major
D. Sideroblastic anaemia
E. Malaria

71. A 19 year old man presents for the first time with a firm and unshakable belief that he is being
followed by terrorists who are plotting against him. What is the single best term for this man’s
condition?
A. Delusion of Persecution E. Delusion of Nihilism
B. Delusion of Grandeur
C. Delusion of Control
D. Delusion of Reference

72. A 29 year old young man complains of recurrent attacks of diarrhoea. He says his stools contain
blood and mucus. Sometimes she has low grade fever. What is the most appropriate investigation?
A. Stool culture
B. Plain abdominal x-ray
C. Per rectal examination
D. Barium Enema

73. A boy with partial thickness, burn increased. What is the next step?
A. Escharectomy
B. Dressing
C. Burst blisters
D. Local antibiotics
E. Refer to burn unit

74. A 64 years old diabetic has come for a routine eye check-up. Fundoscopy shows new vessels all
over the retina. What is the most appropriate management?
A. Strict Sugar Control
B. Regular eye check ups
C. Non urgent referral to a specialist
D. Laser photocoagulation
E. Insulin

75. A 45 year old man underwent emergency splenectomy following a fall from his bicycle. He
smokes five cigarettes a day. Post -operatively despite mobile, he developed swinging pyrexia and a
swollen painful left calf. His chest x-ray shows lungs atelectasis and abdominal USG demonstrates a
small sub-phrenic collection.What is the SINGLE most likely risk factor for deep vein thrombosis in
the patient?
A. Immobility
B. Intraperitonealhaemorrhage
C. Smoking
D. Splenectomy
E. Sub-phrenic collection

76. A young woman was brought to the hospital. On examination she has low temperature and tremor.
She says when her eyes are closed she can see colour. What drug has been used?
A. Methylenedioxymethamphetamine (MDA)
B. Lysergic acid diethylamide (LSD)
C. Cocaine
D. Heroine
E. Ecstasy

77. A mother brings her 1 week old baby to the clinic because she is concerned about a large mark on
his back. On examination there seems to be a large blue bruise like mark on the baby’s back. What is
the most likely diagnosis?
A. Non accidental injury
B. Mongolian spot
C. Accidental bruising
D. HSP
E. ITP

78. You are the SHO in the hospital and the lab report of a patient shows glucose: 4mmol/L, K:
5.2mmol/l, Na129 mmol/l. What is the most appropriate management for this patient?

A. Normal Saline 0.9%


B. Normal saline 4.5%
C. Normal Saline 0.9% + insulin
D. Insulin
E. Dextrose

79. A 10 year old male child was brought by his mother complaining that her child watches TV at
very high volume, doesn’t like to play out and instead has become more sincere with reading. She also
says that her son does not respond to her. What do you expect to see on otoscopy?

A. Flamingo pink Tympanic Membrane


B. Attic perforation
C. Bluish grey Tympanic membrane with air fluid levels
D. Inflamed tympanic membrane with cart wheel appearance of vessels
E. Red and inflamed tympanic membrane

80. A patient is on long standing indwelling catheter and now the catheter bag is turned purple.
Dipstick test shows nitrates –ve, blood+. What investigation would you do next?
A. Urine culture and sensitivity
B. None
C. Urine porphobilinogen
D. USG abdomen

81. A 60 years old man brought to A&E department with fracture of hip; he is deaf and has bilateral
pedal oedema. What is the SINGLE most probable diagnosis?

A. Paget’s disease
B. Osteoporotic fracture vertebrae
C. Secondary
D. Multiple myeloma
E. Spondylosis

82. A 28 years old lady presents with dyspareunia and dysmenorrhoea. She is very obese. She now
wants reversible contraceptive method. Which of the following will be most suitable for her?
A. Mirena
B. Combined Oral Contraceptive Pill (COCP)
C. Progestogen only pill (POP)
D. Copper-T
E. Barrier method

83. Patient with widespread ovarian carcinoma had bowel obstruction and severe colic for 2 hrs and
was normal in between. What is the most appropriate management?
A. S/C morphine (PCA)
B. Spasmolytics
C. Palliative colostomy
D. Oral Morphine
E. Laxatives

84. An 80 year old man with prostatic cancer has confusion, thirst, abdominal pain and an abnormal
electrocardiogram (ECG). What is the most appropriate investigation?
A. CT scan
B. Radionuclide bone scan
C. Skeletal Survey
D. Serum alkaline phosphatase iso-enzyme concentration
E. Serum calcium concentration

85. A 35 year old man presents with a headache that worsens on bending his head forward. What is
the most likely diagnosis?
A. Chronic Sinusitis
B. Subarachnoid Haemorrhage
C. Migraine
D. Cluster Headache
E. Tension Headache

86. A 19 years old male presents with runny nose, cough and generalised rash. Blood MCS is positive
for coagulase positive cocci. What is the best management?
A. Amoxicillin
B. Rifampicin
C. Gentamycin
D. Flucloxacillin
E. Co-amoxiclav

87. An 82 year old man has woken up with incoherent speech and difficulty in finding the right
words. Examination is otherwise normal and his comprehension is good. Which anatomical site is
most likely to be affected?
A. Broca’s Area
B. Wernicke’s Area
C. Midbrain
D. Parietal Cortex
E. Pons

88. A 9 year old child is diabetic and unhappy in school. What is your next step?
A. Refer to social worker
B. Clinical psychologist
C. Special school
D. Check child protection register
E. Give oral anti diabetics

89. A10 year old girl presents with hoarseness of the voice. She is a known case of bronchial asthma
and has been on oral steroids for a while. What is the most likely cause of hoarseness of voice in this
case?
A. Candidial laryngitis
B. Tonsillitis
C. Chronic laryngitis
D. Laryngeal carcinoma

90. A 60 year old man presents with dysphagia and pain on swallowing both solids and liquids. A
barium meal shows gross dilatation of the oesophagus with a smooth narrowing at the lower end of
the oesophagus. What is the SINGLE most likely cause of dysphagia from the given options?
A. Achalasia
B. Myasthenia gravis
C. Oesophageal carcinoma
D. Oesophageal web
E. Systemic sclerosis

91. A patient operated for gastric carcinoma is on oral morphine for pain relief. He has adequate pain
relief but is experiencing side effects due to high dose of morphine required. What is the most
appropriate next step in his pain management?
A. Patient controlled analgesia
B. Oral oxycodone
C. IV morphine
D. Fentanyl patches

92. A 50 year old man with a known history of stroke. He cannot remember anything about his life.
What is the single most likely defect?
A. Homonymous hemianopia
B. Homonymous upper quadrantanopia
C. Bitemporal hemianopia
D. Binasal hemianopia
E. Homonymous lower quadrantanopia

93. A 27 year old woman had pre eclampsia and was delivered by c – section. She is now complaining
of right upper quadrant pain different from wound pain. What investigation will you do immediately?

A. Coagulation profile
B. Liver function tests
C. Ultrasound liver
D. MRCP
E. None

94. A 37 year old woman presents with more tiredness than usual, she is found to have angular
stomatitis, no koilonychia. Choose the single cell type you will find on blood film.

A. Macrocytes
B. Microcytes
C. Granulocyte without blast cells
D. Blast cells

95. A woman came with the complaint of pain in her right arm when she abducts it. She has recently
moved to a new house. There is no history of trauma. What is the likely cause of her pain?

A. Rupture of the long head of the biceps


B. Sprain of the acromio-clavicular ligament
C. Tendinitis of the abductor sheet
D. Supraspinatus tendinitis
E. Shoulder dislocation

96. A man presents to A& E with pins and needles in his arms and legs, decreased sensation and
reduced reflexes in lower limbs. Blood tests reveal a macrocytic anaemia. He sees things other people
can’t see and has a long history of alcoholism. When asked simple questions he makes up nonsensical
stories. What is the management?
A. Diazepam
B. Chlordiazepoxide
C. Thiamine
D. Folic acid

97. A 56 years old patient, whose pain was relieved by oral Morphine, now presents with
progressively worsening pain relieved by increasing the dose of oral Morphine. However the patient
complains that the increased Morphine makes him drowsy and he is unable to carry out his daily
activities. What is the next step to manage this patient?
A. Oral Oxycodone
B. Oral Tramadol
C. Patient Controlled Analgesia
D. I.V. Fentanyl
E. Diamorphine
98. A 20 years old young lady comes to her GP for advice regarding cervical cancer. She is worried as
her mother past away because of this disease. She would like to know what is the best method of
contraception in her case ?

A. Progestrogen only pill (POP)


B. Barrier method
C. Intrauterine Contraceptive Device (IUCD)
D. Combined Oral Contraceptive Pill (COCP)
E. Intruterine system
F. Osteoarthritis

99. A six year old girl has had two short episodes of cough and wheeze over the last 12 months. These
two acute episodes responded quickly to bronchodilator, she has no symptoms or abnormal physical
signs. She has slight eczema and her mother has asthma. What is the SINGLE most appropriate
investigation?
A. Chest x-ray
B. Peak flow rate diary
C. Pulse oximetry
D. Spirometry
E. Sweat test

100. A pregnant patient present with tingling, numbness, pain and loss of sensation on middle and
index finger of right hand. Tinel sign is negative. What is the most likely diagnosis?

A. Carpal tunnel syndrome


B. Hammer hypothenar syndrome
C. Ulnar nerve injury
D. Radial nerve injury
101. A 22 years old man keeps having persistent and intrusive thoughts that he is a dirty thief. No
matter what he tries these thoughts keep coming to him. Any attempt to avoid these thoughts leads to
serious anxiety. What is the most likely diagnosis?

A. Schizophrenia
B. Obsessive Compulsive Disorder (OCD)
C. Post traumatic stress disorder (PTSD)
D. Mania
E. Psychotic Depression

102. A 32 year old woman, para 3 of 39 weeks of gestation, reports having had spontaneous rupture of
membranes 4 days ago. She did not attend the delivery suite, as she knew that would happen and had
already decided on a home birth. Today she feels very hot and sweaty. She thought that she was
starting to have labour pains, but she describes the pain as more constant. On examination, her uterus
is tender throughout. Blood tests show a raised CRP and white cell count. Select the most likely
diagnosis from the following options:

A. Round ligament stretching


B. Chorioamnionitis
C. Uterine rupture
D. Labour
E. Disseminated intravascular coagulation
103. Patient had all signs of cerebello-pontine angle (CPA) tumour facial numbness, tinnitus and
vertigo. What is the best investigation?
A. CT scan
B. MRI Brain
C. MRI Head
D. X-ray head

104. A man has a habit of checking the locks of his house again and again. He tries to resist but he has
palpitations. What is the most appropriate treatment?

A. CBT + fluoxetine
B. CBT only
C. Fluoxetine only
D. ECT

105. A 70 year old lady presents with fever since 3 days and confusion. There is no significant past
history. What is most probable diagnosis?
A. Delirium
B. Hypoglycemia
C. Alzheimer’s disease
D. Diabetic Ketoacidosis (DKA)

106. A 6 year old girl has been brought to the hospital after being stung by a bee. She is complaining
of generalized itching, rashes, redness and has developed stridor. What is the most appropriate
treatment?

A. Oral Chlorpheniramine
B. IV Chlorpheniramine
C. IM Adrenaline
D. IV Adrenaline
E. Reassurance
107. A child who has type 1 diabetes is sad and unhappy at school. He has no friends and his
glycaemic control is poor. What would you do next?
A. Refer to diabetologist
B. Refer to psychiatrist
C. Refer for developmental assessment
D. Increase insulin dose

108. A 27 year old man believes that the whole world is plotting to kill him. What is the best
treatment for him?
A. Haloperidol
B. Chlorpromazine
C. Risperidone
D. Thioridazone

109. A 70 year old patient presents with cough and shortness of breath. He stopped smoking cigarettes
2 years ago but he has been smoking for about 50 years before quitting. CXR shows consolidation and
bilateral and bihilar lymphadenopathy. What is the best investigation for this patient?
A. Lymphnode biopsy
B. Pleural fluid cytology
C. CT scan
D. Magnetic resonance imaging (MRI)
E. Ultrasound
110. A 12 year old boy with pain and swelling in multiple joints (small and large), has mild fever .
Which of the following change in his blood investigations will definitely be there?
A. RA factor positive
B. Increase in ESR
C. ANA – positive
D. Rise in serum uric acid level

111. A 23 year old lady was prescribed with Azithromycin 1gm for her chlamydial pelvic infection.
She has got a new boyfriend for the last 2 months. She has recently started contraception to avoid
conception. Which of the following contraception method will be affected by Azithromycin? 23RD
NOVEMBER 2014
A. Barrier
B. Intrauterine Contraceptive Device (IUCD)
C. Progestogen only pill (POP)
D. Combined Oral Contraceptive Pill (COCP)

112. A 55 year old man presents with complaints of claudication pain in his right thigh and calf. On
examination the femoral and popliteal pulses are absent on the right leg. Localise the site of
occlusion.
A. External Iliac Artery
B. Aorto-Iliac
C. Ilio-Femoral
D. Internal Iliac
E. Abdominal aorta

113. A child presents with projectile vomiting and is diagnosed with pyloric stenosis. What electrolyte
should be checked next?
A. Potassium
B. Calcium
C. Magnesium
D. Iron

114. A 27 year old man presents with chest pain and respiratory distress. On examination there is
tachycardia, hypotension and neck vein distension. Trachea is deviated to the left side, breathing
sound on the right side is absent and on the left side is diminished. What is the next appropriate
management?

A. A. Chest x-ray
B. B. Right side aspiration (16 G cannula)
C. C. Left side aspiration (16 G cannula)
D. D. Right side drainage with a small tube (12F)
E. E. Left side drainage with a small tube (12F)

115. A 45 year old woman has had severe epigastric and right hypochondrial pain for a few hours.
She has a normal full blood count; serum alkaline phosphatase is raised, normal transaminase. Three
months ago she had a cholecystectomy done. What is the most appropriate investigation?

A. Ultrasound of the Abdomen


B. Endoscopic Retgrograde Cholangiopancreatography
C. Magnetic Resonance Cholangiopancreatography
D. CT scan of the Abdomen
E. Upper GI Endoscopy
116. A 23 year old female presented with a swelling of her neck that moved upwards on protrusion of
tongue. What is the next appropriate investigation?
A. FNAC
B. Punch biopsy
C. Core biopsy
D. MRI neck
E. Radioactive thyroid scan

117. An elderly male who is immunocompromised has whitish plaques in the pharynx. He has
dementia, has stopped taking food due to the whitish plaques on the tongue. How will you treat this
elderly male?
A. Fluconazole suspension E. IV Fluconazole
B. Nystatin suspension
C. Oral Miconazole
D. Oral Fluconazole

118. Mother got infected with Hepatitis B during pregnancy. Her child is born. She is worried about
the risk of infection to the baby with Hepatitis B. What would you give to the baby?
A. HepB Immunoglobulin only
B. HepB full vaccine and Ig
C. HepB vaccination only once
D. Nothing until immune status checked
E. HepB vaccine once and Ig

119. A 65 year old man with difficulty in swallowing presents with an aspiration pneumonia. He has a
bovine cough and a fasciculating tongue. Sometimes as he swallows food it comes back through his
nose. Choose the SINGLE most likely cause of dysphagia from the given options?
A. Bulbar palsy
B. Oesophageal carcinoma
C. Pharyngeal pouch
D. Pseudobulbar palsy
E. Systemic sclerosis
120. Which one of the following is the major contributor to the development of physiological jaundice?
A. Breast feeding.
B. Immature liver
C. Low haemoglobin levels and longs life span
D. Immature gut flora
E. Entero-hepatic circulation

121. A 25 year old primigravida is found to have a blood pressure of 160/90mmHg and proteinuria of
+++. What is the most likely diagnosis?
A. Eclampsia
B. Pre-eclampsia
C. HELLP
D. Normal finding
E. Renal failure
122. Among the risk factors given, which one will most likely enhance a woman’s chances of getting
ovarian cancer?
A. Family history
B. Previous history of breast cancer
C. Environmental factor
D. Young age
E. Nulliparity

123. A 45 year old male complains of tremors in hands. On examination these tremors are absent at
rest, but present when arms are held outstretched and persist on movement. What is the most
probable diagnosis?
A. Parkinsonism
B. Benign essential tremor
C. Cerebellar disease
D. Liver failure
E. Stroke

124. A 17 year old secondary school girl presents with complaint of prolonged irregular menstrual
periods and heavy blood losses. What is the most appropriate treatment for her?

A. Mefenamic Acid E. Intrauterine Contraceptive Device (IUCD)


B. Combined Oral Contraceptive Pill (COCP) F. Mirena
C. Progesterone only pill
D. Specific Antigen (PSA)
E. Midstream Specimen of Urine (MSU)

125. Patient with terminally ill cancer. He is on oral morphine. He cannot tolerate it. What is the next
mode of treatment?
A. Intravenous
B. Subcutaneous
C. Change to other pain management

126. A 75 year old woman from a nursing home is brought to the A/E by her carer who says that she
has been confused for four days. She is on Warfarin for atrial fibrillation. Examination is
unremarkable except for multiple bruises on her body. Blood investigations reveal an INR of 7.3.
What is the most likely diagnosis?
A. Alcohol intoxication
B. Urinary tract infection
C. Subdural haematoma
D. Dementia

127. A 50 years old man presents with low mood, poor concentration, anhedonia and insomnia. He
has had two episodes of increased activity, promiscuity and aggressive behaviour in the past. He was
arrested 8 months ago for trying to rob a bank, claiming it as his own. Which drug is most likely to
benefit him?
A. Haloperidol
B. Citalopram
C. Desipramine
D. Carbamazepine
E. Ethosuximide
128. A 2 year old boy was brought by his mother with a swelling in the right side of the neck
extending from the angle of the mouth to the middle one third of the sternocleidomastoid muscle. The
swelling was on the anterolateral side of the sternocleidomastoid and was brilliantly trans-illuminant.
What is the likely diagnosis?
A. Lymphangioma
B. Branchial cyst
C. Thyroglossal cyst
D. Ranula
E. Thyroid swelling

129. A 19 year old boy complains of itching on the site of an insect bite. What is the SINGLE most
appropriate management?

A. Penicillin oral D. Oral Ciprofloxacin


B. Doxycycline oral E. Reassurance
C. Oral antihistamine

130. A 2 year old boy has had fever, sore throat, runny nose and lethargy for the past few days. Now
he presents at A &E, is sitting in mother’s lap with his body inclined forward with continuous stridor
and is drooling saliva. What is the most important next step in his management?
A. Call ENT specialist
B. High flow O2
C. Call Anaesthetist
D. Antibiotics IV
E. Immediate bronchoscopy

131. A young patient, 10 days after a tonsillectomy, presents with pain and profuse bleeding from the
operative site. What is the most important intervention to take care of the underlying problem?

A. Pack the tonsillar area


B. Cryotherapy
C. Antibiotics
D. Laser cautery
E. Surgical closure

132. A 24 year old college student presents with nausea, vomiting, headache, neck stiffness and a
fever of 38.4˚C. What is the most appropriate empirical antibiotic to be started?

A. Cefotaxime
B. Penicillin
C. Gentamicin
D. Tazobactam
E. Meropenem
F. Rifampicin
133. A 3 year old girl was playing with her brother when she fell off her bicycle. She was about to cry
but fell unconscious without making any sound. She regained consciousness within 10 seconds and
was normal. The mother says similar episodes have happened in the past and she was afraid her
daughter as going to die. What is the appropriate investigation?
A. 24 hour EEG
B. CT scan
C. No investigation needed
D. MRI
E. Neuropsychological examination

134. A 23 year old female was involved in a house fire 3 months ago. She now complains of
nightmares and flashbacks about the incident. This is interfering with her daily life. Whenever she
hears a fire engine, she starts having palpitations and extreme anxiety and gets images of people being
burnt alive. What is the most likely diagnosis?
A. Obsessive compulsive disorder
B. Adjustment disorder
C. Bipolar affective disorder
D. Eating disorder
E. Post traumatic stress disorder

135. A young man febrile with cough and sputum production with left lower lobe patchy
consolidation in CXR. Which is the best antibiotic therapy for this patient?

A. Ciprofloxacillin
B. Cotrimoxazole
C. Cefotaxime
D. Vancomycin
E. Amoxicillin

136. A child with inflammation in the orbit, fever, lid swelling and decreased eye mobility. Which is
the best investigation for his condition?

A. FBC
B. Blood cultures
C. CT scan of the orbits
D. Throat swab
E. Nasal Secretions

137. A 6 weeks old baby has blue mark near coccyx since birth. His mother is worried. What would
you do?

A. Reassure
B. Coagulation profile
C. Karyotyping
D. Skeletal survey
E. CT scan
138. Infection of which of the following areas in the face will lead to pre auricular lymph node
enlargement?

A. Mastoid area
B. Mandibular area
C. Pinna of the ear
D. Upper eyelid

139. A 9 year old girl presents with weekly abdominal pain and with occasional headaches but not
associated with vomiting and diarrhea. She has good appetite. Full blood count, urea and electrolytes
and other investigations are normal. On examination no abnormality was found and the abdomen was
soft and non-tender. What is the most likely next step?

A. Ultrasound scan of the abdomen


B. CT of thorax
C. Liver Function test
D. Reassure
E. Analgesia

140. An 8 year old girl presents with a large swelling on the side of her neck, there were associated
lymph nodes swellings in her neck and axilla. What is the most appropriate investigation?

A. Bone marrow biopsy


B. Lymph node biopsy
C. CT of the neck
D. MRI of the neck
E. Hb electropheresis

141. A 34 year old patient with small cell lung cancer of lung presents with Na: 119 and all other
values normal. Fluid restriction has failed to bring the Sodium up. What is the next most likely step?

A. Chemotherapy
B. Radiotherapy
C. Demeclocyclin
D. Somatostatin
E. Loop diuretics

142. A patient complains of shortness of breath, wheeze, cough and nocturnal waking. He has dry
scaly shin with rashes that are itchy. What is the single most likely diagnosis?

A. Scabies
B. Eczema
C. Rheumatitis
D. Dermatitis
E. Psoriasis

143. A woman has been prescribed long term steroids for polymyalgia rheumatic. What additional
drug should be added to prevent osteoporosis?

A. Calcium
B. Alendronate
C. Vit D
D. Calcium resonium
144. A young man who recently served in the Army, has been diagnosed with post traumatic stress
disorder. What is the best treatment for him?
A. Narrative exposure therapy + paroxetine
B. Flooding + sertraline
C. Response prevention+ citalopram
D. Systematic desensitization + Fluoxetine
E. Play therapy

145. A 28 year old male is complying with her asthma treatment of low dose inhaled corticosteroids
and salbutamol as required. Her inhaler technique is rechecked. She has frequent night cough and
mild exercise induced wheeze. Most appropriate management
A. Add leukotriene antagonist
B. Add oral theophylline
C. Add regular LABA
D. Increase dose of ICS
E. Short course of oral corticosteroid

146. A 24 years old woman is afraid to leave her house as whenever she goes out; she tends to have
shortness of breath and sweating. She has stopped going out except with her husband. What is the
most likely diagnosis?

A. Social Phobia D. Panic Disorder


B. Claustrophobia E. Agoraphobia
C. Depression

147. A 65 year old hypertensive man wakes up in the morning with slurred speech, weakness of the
left half of his body and drooling. Which part of his brain is affected?

A. Left parietal lobe


B. Right internal capsule
C. Right midbrain
D. Left frontal lobe

148. A known asthmatic adult on beta2 agonist inhaler, presents with worsening of breathlessness and
cough. What medication should be added further to the regime to control asthma symptoms?

A. Inhaled steroids
B. Oral steroids
C. Antibiotics
D. Increase beta2 agonist inhaler dose
E. Nebulized salbutamol

149. Patient was admitted with erectile dysfunction, reduced facial hair and galactorrhoea. What is
the most probable diagnosis?

A. Hyperprolactinemia
B. Cushing’s syndrome
C. Pheochromocytoma
D. Hyperthyroidism
E. Hypoparathyroidism
150. A 42 year old man presents with stroke, he is not able to walk straight and his speech is slurred.
What is the initial appropriate investigation?

A. CT scan brain
B. PET scan brain
C. MRI (brain)
D. Carotid angiography
E. Monitor for 24 hours

151. A 14 year old boy fell and hit his head in the playground school. He did not lose consciousness.
He has swelling and tenderness of the right cheek with a sub-conjunctival haemorrhage on his right
eye. What is the most appropriate initial investigation?

A. CT Brain
B. EEG
C. MRI
D. Skull X-Ray
E. Facial X-Ray

152. Diabetic man feels hot painful lump near the anal region. What is the most probable diagnosis?

A. Anal fissure
B. Abscess
C. Haematoma
D. Wart
E. External haemorrhoids

153. What is the most common post-operative complication following anterior resection of rectum?

A. Infection
B. Bleeding
C. Recurrence
D. Intestinal obstruction
E. Pain

154. A 43 year old woman is brought in by her husband to the A& E department. He says she has
been getting up repeatedly at night to wash her hands as she thinks they are dirty. She gets very
anxious if he tries to stop her. She admits washing her hands again and again but says she cannot
control her actions. What is the diagnosis?
A. Schizophrenia
B. Obsessive compulsive disorder
C. Generalised anxiety disorder
D. Panic disorder
E. Depression

155. A 31 year old man is single and lives with his mother. He usually drives from his home to his
work place. He always thinks when the traffic lights change his mother is calling him, so he drives
back to his house. What is the diagnosis?
A. Obsessive compulsive disorder
B. General anxiety disorder
C. Schizophrenia
D. Bipolar personality disorder
E. Cyclothymia
156. A 19 year old boy comes to A&E department. There is swelling, pain and tenderness 2cm distal
to Lister’s tubercle of radius. On examination proximal pressure on the extended thumb and index
finger is painful. The x-ray doesn’t show any fracture. What is the next appropriate management for
the patient?

A. Immobilization with a cast


B. Repeat the x-ray
C. Magnetic resonance imaging (MRI)
D. Surgery
E. No treatment required

157. A 35 year old female developed sudden severe chest pain with shortness of breath. She recently
underwent pelvic surgery. What is the most appropriate diagnostic investigation that should be done?

A. Serum D-dimer
B. Chest X-ray
C. ECG
D. CT pulmonary angiography
E. Arterial Blood Gas (ABG)

158. A 24 year old, male patient, presents with bizarre behavior. What single characteristic symptom
would confirm the diagnosis to be most likely to be schizophrenia?

A. Thought insertion
B. Visual hallucination
C. Aggressive, violent behavior
D. Low mood
E. Body dysmorphic image

159. A 36 years old man has been diagnosed with Diabetes Insipidus. What electrolyte picture is
expected to be seen?

A. High serum Na, low serum osmolarity, high urine osmolarity


B. Low serum Na, low serum osmolarity, high urine osmolarity
C. Low serum Na, high serum osmolarity, high urine osmolarity
D. High Serum Na, high serum osmolarity, low urine osmolarity
E. Normal Na, Normal serum osmolarity, normal urine osmolarity

160. A 79 year old man has a swelling of the right groin, which was clinically diagnosed to be indirect
inguinal hernia. What is the SINGLE feature of the hernia sac that would confirm the diagnosis?

A. Comes through femoral ring


B. Does not pass through deep inguinal ring
C. Lies below and lateral to pubic tubercle
D. Only passes through superficial ring
E. Passes through deep inguinal ring

161. A patient is admitted with fever, photophobia, and a non-blanching rash. His ABG values are as
follows:
pH : 7.5
PaCO2 : 2.2 KPa
PaO2 : 9.0 KPa
What is the most likely diagnosis?
A. Metabolic acidosis
B. Metabolic alkalosis
C. Respiratory alkalosis
D. Respiratory acidosis

162. A 34 year old man was walking along the country side when an insect bit him. After which he
started to complain of an annular rash spreading upwards. Which is the best treatment for this
patient?
A. Penicillin oral
B. Doxycycline oral
C. Gentamicin oral
D. Flucloxacillin oral

163. A 24 years old woman has been complaining of amenorrhea for the last 12 weeks. She also has
diarrhoea and weight loss of 8 kg over the last 3 months. What is the most important next step?

A. Serum Prolactin
B. Thyroid Function Test
C. Thyroid Antibodies
D. 24 hours urinary VMA (Vanillylmandelic Acid)
E. Admit under the MHA (Mental Health Act)

164. A man brings his wife to see the doctor. He says she has been talking about their private life
openly and is throwing tantrums. Her mood changes quickly and she becomes aggressive. Physical
examination is normal. There is no history of trauma and she is not on any medications. What is the
most likely diagnosis?
A. Huntington’s dementia
B. Lewy body dementia
C. Alzheimer’s dementia
D. Vascular dementia
E. Benign old age related dementia

165. How to keep a check on salbutamol's efficiency?


A. PEFR
B. Spirometry
C. Vital capacity
D. FEV/ FEV1 ratio
E. Lung function test

166. A 30 year old alcoholic woman presents with low mood, anhedonia, lack of appetite and sleep.
What is the best treatment for her?

A. Amitryptiline
B. Fluoxetine
C. Venlafaxine
D. ECT

167. A pregnant woman had hit her chest 3 weeks ago. Now she is 24 weeks pregnant and presents
with left upper quadrant mass with dimpling. What is the most probable diagnosis?
A. Breast Ca
B. Carcinoma
C. Fibroadenoma
D. Fibroadenosis
E. Fatty necrosis of breast
168. A lady who works at a nursing home presents with itching. On examination, she had linear tracks
in the wrists. She says that 2 days ago she had come in contact with a nursing home inmate with
similar symptoms. What is the mechanism of itching?

A. Infection
B. Destruction of keratinocytes
C. Allergic reaction
D. Immunosuppression
E. None

169. A 35 year old man presents with painless left testicular enlargement since past 6 months, which
is increasing in size. The swelling is 3 times larger than the right testis; there is no tenderness or
redness. What is the most likely diagnosis?

A. Testicular tumour D. Epididymo – orchitis


B. Hydrocele E. Reassure
C. Epididymal Cyst

170. A 50 year old woman complains of several months’ history of weakness and difficulty in
climbing stairs. O/E she has fissuring of the skin of her hands and her CXR shows pulmonary
fibrosis. What is the single most likely positive antibody?
A. Anti Jo 1
B. Anti Scl 70
C. Anti Ro
D. Anti Ds DNA
E. Anti Centromere

171. A 65 year old lady with IDDM for the last 20 years comes with a tender lump near the anal
opening. She says she also has fever. What treatment should she get?
A. Incision and drainage and antibiotics
B. IV antibiotics
C. Culture and sensitivity of aspirate from swelling
D. Painkillers
E. Cautery of the swelling

172. A 29-year-old woman presents to her GP with a history of weight loss, heat intolerance, poor
concentration and palpitations. Which of the following is most likely to be associated with a diagnosis
of thyroiditis associated with viral infection?

A. Bilateral exophthalmos
B. Diffuse, smooth goitre
C. Reduced uptake on thyroid isotope scan
D. Positive thyroid peroxidase antibodies
E. Pretibial myxoedema

173. A child was playing and suddenly developed breathlessness. What is the best investigation for
this child?

A. Nasal Endoscopy
B. Lateral neck X-Ray
C. CT scan
D. Pulse Oximetry
E. Spirometry
174. A 50 year old presented to the hospital with complain of chest tightness wheeze and chronic
cough. He has been smoking 20 cigarettes per day since last 35 years. A diagnosis of COPD was
made.What will be the picture of ABG for this patient?

A. Respiratory Acidosis
B. Respiratory alkalosis
C. Metabolic acidosis
D. Metabolic alkalosis
E. Normal pH

175. A known COPD patient presented to the hospital with complains of persistent cough, copious
sputum production and intermittent hemoptysis. On examination, finger clubbing and coarse
inspiratory crepitations were observed. What is the most probable diagnosis?
A. Lung cancer
B. Cystic fibrosis
C. Bronchiectasis
D. Pulmonary edema
E. Lung abscess

176. A 64 years old man presents with a history of left sided hemi paresis and slurred speech. He was
absolutely fine 6 hours after the episode. What is the most appropriate prophylactic regimen?

A. Aspirin 300 mg for 2 weeks followed by Aspirin 75mg


B. Aspirin 300 mg for 2 weeks followed by Aspirin 75mg and Dipyridamole 200mg
C. Clopidogrel 75mg
D. Dipyridamole 200mg
E. Aspirin 300mg for 2 weeks

177. A 44 years old obese patient presents with lab findings of:
Fasting blood sugar (FBS) 6mmol/L
Oral Glucose Tolerance Test (OGTT) 10mmol/L . What is the most likely diagnosis?

A. Impaired glucose tolerance


B. Diabetes Insipidus
C. Type 1 Diabetes Mellitus
D. Type 2 Diabetes Mellitus
E. Maturity Onset Diabetes of the Young (MODY)

178. A patient who does not eat regular meals presents with feeling faint and tired. Blood tests reveal
a deficiency of both iron and folic acid. What state best describes the reason for his anaemia?
A. Nutritional deficiency
B. Malabsorption
C. Pernicious anaemia
D. Haemolytic anaemia

179. A 32 year old female with axillary freckles and cafe au lait spots wants to know the chances of
her child also having similar condition.

A. 1 in 2
B. 1 in 4
C. No genetic link
D. 1 in 16
E. Depends on the genetic make-up of the partner
180. A 45 year old man has been diagnosed with moderate depression. His past medical history is
significant for an anterior wall MI 6 months ago. What is most appropriate treatment for him?

A. Benzodiazepine
B. ECT
C. SSRI
D. MAO inhaler
E. TCA

181. A 65 year old man presents with progressively worsening breathlessness and productive cough.
He has a history of treated tuberculosis and rheumatoid arthritis, he smoked 20 cigarettes/day since 35
years. When he was given a trial of salbutamol nebuliser, FEV1/FEV changed from 1.7/2.0 to 1.8/2.1
(predicted 3.0/3.5). What is the most probable diagnosis?

A. Chronic bronchitis
B. Bronchiectasis
C. Bronchial carcinoma
D. Asthma
E. Pulmonary fibrosis

182. A 56 year old lady presents has a pathological fracture of her T11 vertebra. There is found to be
an underlying metastatic lesion. What is her most common primary cancer?

A. Lung
B. Breast
C. Uterine
D. Brain

183. A 61 year old man who had stroke 2 years ago is on aspirin. He has Rheumatoid arthritis but
suffers from the pain and cannot tolerate it. He is taking senna for constipation. What is the best
medication to relieve his pain?

A. Disease-modifying anti-rheumatic drugs (DMARDs)


B. Ibuprofen
C. Co-codamol
D. Paracetamol

184. A man presented to the hospital with complain of productive cough and chest pain. His
temperature was 38.5° C. Lobar Consolidation was seen on x-ray with pleurisy. What is the best
management?
A. Clarithromycin
B. Amoxicillin
C. Tetracycline
D. Ciprofloxacilling
E. Imipenam

185. A man with a carcinoma and multiple metastasis presents with intractable nausea and vomiting.
He has become drowsy and confused. What is the most appropriate management?

A. Dexamethasone IM
B. Dexamethesone oral
C. Ondansetron IM
D. Ondansetron oral
E. Morphine oral

186. A 55 year old man is brought into A&E unconscious. He has a distended abdomen with caput
meduase and spider navi. He has a previous history of bleeding oesophageal varices. His Hb is 10
g/dL, MCV is 100 fL. His skin is pale and clammy. What is the reason for his loss of consciousness?

A. Alcoholic hypoglycaemia
B. Diabetic hypoglycaemia
C. Hepatic encephalopathy
D. Subdural haematoma

187. A 19 year old boy came with skin rash and decreased urine output. On investigations, he was
found to have raised urea and creatinine, and hematuria. There was history of bloody diarrhoea and
fever. What is the most likely diagnosis?

A. Thrombotic ThrombocytopoenicPurpura
B. Haemolytic Uraemic Syndrome
C. Post streptococcal glomerulonephritis
D. IgA Nephropathy
E. Minimal Change disease

188. During a basketball match, one of the players suddenly collapsed to the ground with coughing
and shortness of breath. What is the investigation of choice for this patient?

A. Chest x-ray
B. CT scan
C. Magnetic resonance imaging (MRI)
D. V/Q scan
E. CT pulmonary angiogram (CTPA)
189. A 63 year old woman came with uterine bleeding. Vaginal USG reveals uterine thickness 12mm.
What is the most probable diagnosis?
A. Cervical Ca
B. Endometrial Ca
C. Ovarian Ca
D. Breast Ca
E. Vaginal carcinoma

190. A 48 years old male with history of chronic cough presents with increased breathlessness
copious sputum with streaks of blood, examination shows finger clubbing. What is the initial
investigation?
A. Chest x-ray
B. CT chest
C. Bronchoscopy
D. Blood culture
E. ECG

191. A man has been diagnosed with metastatic carcinoma of the middle 1/3rd of the oesophagus.
What is the best initial management?
A. Chemotherapy
B. Celestin tube
C. Radiotherapy
D. Surgery
192. A man presents with inoperable carcinoma and back pain. His pain has been well controlled with
morphine, but he develops vomiting. Morphine was stopped and he was started on metoclopramide
and fentanyl patches. He then develops neck stiffness and fever. What is the cause of these
symptoms?
A. Metoclopramide
B. Fentanyl
C. Morphine
D. Meningitis
E. Metastasis

193. A 72 year old presents with polyuria and polydipsia. The fasting blood sugar is 8 and 10 mmol/L.
The level of cholesterol is 5.7and blood pressure is 130/80mmHg. There is micro-albuminuria. What
is the most appropriate next management?
A. ACE inhibitors and Sulfonylurea
B. Statin + biguanide
C. Statin + Glitazone
D. Insulin + ACEinhibitors
E. ACEinhibitors + Statin

194. A 56 year old male presents with intermittent vertigo, intermittent tinnitus and intermittent
hearing loss. What is the best drug treatment for this patient?

A. Buccal Prochlorperazine
B. Oral Flupenphenazine
C. Tricyclic antidepressants (TCA)
D. Gentamycin patch on the round window
E. No medical treatment available

195. A neonate’s chest X-ray shows double bubble sign. On examination the neonate has low set ears,
flat occiput. What is the most likely diagnosis?

A. Down’s Syndrome
B. Fragile X Syndrome
C. Turner’s Syndrome
D. Di George Syndrome

196. A patient having headache with sudden onset of loss of vision in one eye. What is the most
appropriate next step?

A. ESR
B. Temporal artery biopsy
C. Full blood count

197. A woman had widespread metastasis from a carcinoma. She presented with severe back pain.
Where do you expect the cancer to be?

A. Lungs
B. Cervix
C. Ovary
D. Uterus
E. Breast
198. A patient known to be HIV positive, has now got chicken pox in the maternity ward. What
precautions should be taken to prevent further spread of infection?

A. Vaccinate all the patients and staff in the ward


B. immunoglobulins to all the pregnant women in the ward
C. Immunoglobulins to all pregnant women who have negative serology
D. Start 7 days course of Acyclovir for all patients in the ward
E. No action required

199. An 18 year old female was brought into the A & E by her parents who are very concerned about
her. Her BMI is 15 kg/m2. She considers herself to be obese and has severely restricted her diet in
order to lose weight. She exercises 5-6 hours daily to further supplement her weight loss. She admits
to abusing laxatives and inducing vomiting. She has had no periods for 7 months but urine Beta HCG
is negative. She has fine lanugo hair covering her entire body and complains of being cold all the
time. What is the most likely diagnosis in her?

A. Bulimia nervosa
B. Anorexia nervosa
C. Hypothyroidism
D. Somatization disorder
E. Conversion disorder

200. A 34 year old male presents with history of headaches presents with ataxia, nystagmus and
vertigo. Where is the site of the lesion?
A. Auditory canal
B. Eight nerve
C. Cerebellum
D. Cerebral hemisphere
E. Brain stem
PLAB 1 MOCK ANSWERS : Mock 4
1 D 41 C 81 A 121 B 161 C
2 C 42 C 82 A 122 A 162 B
3 A 43 D 83 C 123 B 163 B
4 C 44 A 84 E 124 B 164 C
5 E 45 B 85 A 125 C 165 A
6 A 46 D 86 D 126 C 166 B
7 B 47 A 87 A 127 D 167 E
8 D 48 A 88 B 128 A 168 C
9 A 49 AB 89 A 129 C 169 A
10 D 50 E 90 A 130 C 170 A
11 B 51 D 91 B 131 C 171 A
12 A 52 A 92 B 132 A 172 C
13 A 53 AD 93 B 133 C 173 B
14 A 54 A 94 H 134 E 174 A
15 C 55 E 95 D 135 E 175 C
16 C 56 A 96 C 136 C 176 B
17 B 57 C 97 A 137 A 177 A
18 E 58 B 98 A/B 138 C 178 A
19 E 59 D 99 D 139 D 179 E
20 B 60 E 100 D 140 B 180 C
21 C 61 C 101 B 141 C 181 E
22 E 62 C 102 B 142 B 182 B
23 A 63 A 103 B 143 B 183 B
24 A 64 A 104 A 144 A 184 B
25 A 65 D 105 A 145 C 185 C
26 C 66 C 106 C 146 E 186 A
27 D 67 A 107 B 147 B 187 B
28 A 68 B 108 C 148 A 188 A
29 D 69 D 109 A 149 A 189 B
30 B 70 B 110 B 150 B 190 A
31 A 71 A 111 D 151 E 191 B
32 A 72 D 112 A 152 B 192 A
33 C 73 E 113 A 153 B 193 E
34 D 74 D 114 B 154 B 194 A
35 C 75 D 115 B 155 C 195 A
36 A 76 B 116 A 156 A 196 A
37 A 77 B 117 E 157 D 197 E
38 A 78 A 118 B 158 A 198 C
39 D 79 C 119 A 159 D 199 B
40 E 80 A 120 B 160 E 200 C
 

SAMSONPLAB  ACADEMY  LIMITED  


BOW  BUSSINESS  CENTRE  
BOW  ROAD  153-­‐159  
E3  2SE,  LONDON  
EMAIL:  info@samsonplab.co.uk  
Telephone:  07940433068/  
 
 
       SAMSONPLAB  ACADEMY  PLAB  1  MOCK  5  

1. A man having so many symptoms and presenting to the hospital


several times but all investigations are normal. Which kind of disorder?
a. Somatization disorder
b. Hypochondriasis
c. Disociative disorder
d. Nihilistic syndrome
e. Cotard syndrome
 
2.      A  woman  with  ovarian  cancer  has  had  5  cycles  of  chemotherapy.  
Each  one  of  them  have  been  associated  with  vomiting  and  poor  intake.  
She  develops  tingling  and  numbness  of  her  fingertips.  There  is  also  
decreased  sensation  to  touch.  There  are  no  other  neurological  deficits.  
What  is  the  most  likely  cause?  
a.  Cervical  vertebra  metastases  
b.  Chemotherapy  related  neuropathy  
c.  Hypocalcemia  
d.  Barium  meal  
e.  None    
 
 
3. A lady who is a heroin addict wants to quit. She says she needs
someone to motivate her. What is your management of this patient?
A. . refer to clinical psychologist
B. Refer to community mental health team
C. Admit to medical team
D. Admit under psychiatrist
 

E. refer to substance abuse team

 
4.  A  man  had  a  trip  to  West  Indies  and  comes  back  with  pain  in  both  ears.  
What  is    
       the  most  appropriate  management?  
a. Urgent  referral  
b. IM  gentamicin  
c. Oral  amoxicillin  
d. Routine  referral  to  ENT  
e. Reassure  
 

5. A  6  year  old  boy  who  travelled  from  New  york  to  united  kingdom  
with  a  fever.  He  presents  to  hospital  in  the  united  kingdom  with  
night  sweats.  On  examination  he  has  splenomegaly  and  his  
temperature  36.4.  What  is  the  single  most  likely  diagnosis?  
A. Tuberclosis  
B. Lymphoma  
C. Brucelosis  
D. Sarcoidosis  
E. Malaria  
 
 
6.  A  50  year  old  man  presents  with  pain  in  his  right  ear.  On  examination  
there  are        
       blisters  in  the  ear.  What  is  the  most  likely  cause?  
a. Herpes  simplex  
b. Acute  otitis  media  
c. Acute  otitis  externa  
d. Herpes  zoster  
e. H.  influenza  
 
7.  A  male  aged  80  years  wakes  up  in  the  morning  with  testicular  pain.  
What  is  the      
       most  appropriate  management?  
 

a. Surgical  exploration  immediately  


b. Antibiotics  
c. Ultrasound  scan  of  the  scrotum  
d. Analgesics  
e. Reassure  
 
8.  A  50  year  old  alcoholic  man  presents  with  ataxia,  confusion  and  
opthalmoplegia.      
       What  is  the  diagnosis?  
a. Delirium  tremens  
b. Wenicke’s  encephalopathy  
c. Korsakoff’s  psychosis  
d. Water  intoxication  
e. Acute  psychosis  
 
9.  A  male  adult  went  to  a  night  club  and  was  brought  to  the  hospital.  He  
was    
       commenting  on  a  friend’s  shirt  colour.  Which  drug  is  most  likely  
responsible  for    
       this?  
a. Ectasy  
b. Cocaine  
c. Opiate  
d. Heroin  
e. Methadone  
 
10.  A  70  year  old  man  presents  with  back  pain,  increased  calcium,  
anaemia  and    
             normal  creatinine.  What  is  the  most  likely  diagnosis?  
a. BPH  
b. Myeloma  
c. Amyloidosis  
d. Chronic  renal  failure  
e. Dehydration  
 
 

11.  A  30  year  old  lady  who  takes  combined  oral  contraception  pupils  
develops  chest  infection  and  she  wants  to  amoxicillin  for  7  days.  
What  is  the  single  most  appropriate  advice  for  her?  
 

A. Nothing  needs  to  be  done  


B. Continue  same  contraception  
C. Continue  same  contraception  with  another  for  7  days  
D. Continue  same  contraception  with  another  one  for  14  days  
E. Continue  same  contraception  with  another  for  3  days  
 
 
13.  A  boy  is  playing  with  toys.  He  can’t  breathe  and  immediately  starts  
coughing.    
             There  is  sudden  onset  of  shortness  of  breath,  which  is  still  persisting.  
What        
             investigation  will  you  do?  
a. Chest  X-­‐ray  
b. Laryngoscope  
c. Pulmonary  function  test  
d. Upper  GI  endoscope  
e. No  test  indicated  
 
14.  A  28  year  old  lady  presents  with  a  mlar  rash  on  the  both  sides  of  
her  cheeks.  She  also  complains  of  joint  pains.  What  is  the  single  
most  appropriate  investigation?  
A. Anti-­‐  DNA  
B. Anti-­‐jo  
C. Anti-­‐ANA  
D. Ant-­‐Ro  
E. Anti-­‐mitochomdrial  
 
 
 
 
 
 
 

15.  A  30  years  old  post  op  surgical  patient  comes  complains  of  sudden  
onset  of  chest  pain  and  shortness  of  breath.    
             What’s  the  most  definitive  investigation?    
a. CTPA  
b. Ventilation  perfusion  scan  
c. D  dimer  
d. Pulmonary  angiogram  
e. Doppler  ultrasound  
 
16.  A  45  year  old  patient  presents  with  COPD  and  has  worked  in  an  
asbestos  company.  He  now  has  shortness  of  breath  and  pleural  effusion.  
How  would  diagnose  the  fluids?  
a. Chest  drain  
b. Antibiotics  
c. CT  scan  of  the  chest  
d. Oxygen  
e. Pleural  effusion  
 
17.  A  7  year  old  lady  ingested  her  sister’s  methadone  by  mistake  and  
was  found        
           unconscious.  He  was  given  naloxone  and  regained  consciousness.  
Half  an  hour    
           later,  his  GCS  dropped  down  to  11/15.  What  is  the  cause  of  the  2nd  
unconscious    
           episode?  
a. Naloxone  metabolises  faster  than  methadone  
b. He  took  something  else  apart  from  naloxone  
c. Naloxone  is  a  partial  agonist  
d. Naloxone  works  better  in  an  adult  
e. Metabolism  of  naloxone  in  adult  is  low  
 
18.  A  45  year  male  had  mechanical  injury  to  his  arm.  The  X-­‐ray  is  normal.  
What  will  you    
           advise?  
a. High  arm  sling  for  2-­‐3days  
b. Broad  arm  sling  
c. Plaster  cast  
d. Analgesia  ad  bed  rest  
e. Analgesia  only  
 
 

19.  An  elderly  man  fell  in  the  garden.  On  examination,  there  is  a  murmur.  
What  is    
           the  most  appropriate  investigation?    
a. ABG  
b. ECG  
c. ECHO  
d. Chest  X-­‐ray  
e. Hb  
 
20.  A  45  year  old  man  was  involved  in  a  fight  and  sustained  an  injury  to  
his  right  ear.        
             He  has  bleeding,  ringing  and  deafness  in  his  right  ear.  What  is  the  
most      
             appropriate  investigation?  
a. CT  head  
b. Otoscopy  
c. X-­‐ray  
d. MRI  scan  
e. Refer  to  the  outpatient  clinic  
 
21.  A  45  year  old  man  was  trapped  under  a  rock  for  4  hours.  He  then  
presents  with    
           red  urine  and  renal  failure.  What  is  the  most  appropriate  treatment?  
a. Urgent  dialysis  
b. Fluid  challenge  
c. Glucose  and  insulin  
d. Dobutamin  
e. Antibiotics  
 
22.  A  30  year  lady  was  found  to  have  inflammatory  changes  on  routine  
cervical    
           smear.  What  is  the  most  appropriate  action?  
a. Do  colposcopy  
b. Repeat  smear  in  6  months  
c. Reassure  
 

d. Repeat  cervical  smear  immediately  


e. Take  biopsy  
 
23.  A  25  year  old  man  rarely  left  his  house  for  the  last  2  months.  He  
believes  that  his    
           neighbours  are  spying  on  him.  His  sleep  and  appetite  are  poor  and  he  
has  lost    
           weight.  Mental  state  examination  reveals  that  he  hears  derogatory  
voices  even    
           when  he  is  alone.  Neurological  examination  is  normal.  What  is  the  
single  most    
           appropriate  management?  
a. Diazepam  
b. ECT  
c. Fluoxetine  
d. Lithium  
e. Olanzapine  

24.  A  45  year  old  homosexual    who  has  multiple  partners  presents  
with  urethral  discharge.  He  describes  a  transiet  painless  ulcer  few  
weeks  back.  What  is  the  single  most  appropriate  diagnostic  
investigation?  
A. Ulcer  swabs  and  culture  
B. Buccal  mucosa  swab  
C. PCR  for  antigen  
D. Antibodies  to  treponema  pallidum  
E. Biopsy  of  the  ulcer  
 
 
25.  A  40  year  old  man  is  seen  with  a  hematoma  over  the  mastoid  after  
being    
             involved  in  a  fight.  Which  bone  is  fractured?  
a. Parietal  
b. Temporal  
c. Sphenoidal  
 

d. Mandibular  
e. Occipital  
 
26.  A  15  year  old  female  presents  with  colicky  pain  in  the  pelvis  region  
since  1  year    
           ago  which  has  now  increased.  The  pain  is  in  the  right  iliac  fossa.  What  
is  the    
           diagnosis?  
a. Ovarian  torsion  
b. Twisted  ovarian  cyst  
c. Appendicitis  
d. Ovarian  cancer  
e. Endometriosis  
 
27.  A  30  year  old  lady  had  blood  film  which  shows  a  picture  of  anaemia  
with  spherocytes.  What  will  you  do?  
           a.      Indirect  Coomb’s  test  
           b.      Bone  marrow  examination  
           c.      Measure  serum  ferritin  
           d.      Bilirubin  
           e.      Direct  Coomb’s  test  
 
28.  A  45  year  old  man    presents  with  pain,  loss  of  sense  and  reflexes  on  
both  legs.  Where  is  the  lesion?    
a. Aorta  
b. External  iliac  
c. Internal  iliac  
d. Common  iliac  
e. Femoral  
 
29.  A      45  year  old  man  presents  with  eye  pain  and  semi  dilated  pupil.  
What  is  the  single  diagnosis?  
             a.      Closed  angle  glaucoma  
             b.      Open  angle  glaucoma  
             c.        Uveitis  
 

             d.      Iritis  
             e.      Central  retinal  artery  
 
30.  A  40  year  old  lady    patient  with  rheumatoid  arthritis  presents  with  
redness  in  the  lateral  side  of  the  eye.  The  vision  is  normal.  What  is  the  
diagnosis?  
a. Uveitis  
b. Cataracts  
c. Posterior  uveitis  
d. Glaucoma  
e. Central  retinal  vein  occlusion  
 
31.  A  35  year  old  female  presents  with  5  percent  full  thickness  burns.  
What  is  the  most  appropriate?  
             What  will  you  do?  
a. Reassure  
b. IV  fluids  
c. Fresh  frozen  plasma  
d. Do  not  give  fluids  
e. IV  fluids  
 
32.  A  40  year  old  woman  with  diabetes  mellitus  has  high  ferritin  levels.  
What  cancer        
             is  she  predisposed  to?  
a. Liver  carcinoma  
b. Lung  carcinoma  
c. Renal  carcinoma  
d. Intestinal  carcinoma  
e. Pancrease  carcinoma  
 
33.  A  30  year  old  female  is  afraid  of  developing  appendicitis.    What  risk  
factor  is  likely  to  increase  the  risk  of  ectopic  pegnancy?  
a. Chlamydial  infection  
b. PCOS  
c. Smoking  
 

d. Bleeding  per  vaginum  


e. Multiple  pregnancies  
 
34.  A  30  year  old  man    has  had  diarrhea  for  the  past  4  days.  What  
metabolic  changes  will  you  see  in  the  blood?  
             a.      PH  increased,  bicarbonate  decreased  
             b.      Increased  bicarbonate,  increased  PH  
             c.      Decreased  PH,  decreased  bicarbonate  
             d.      Normal  PH,  normal  bicarbonates  
 
 
36.  A  60  year  old  man  who  is  hypertensive  and  on  bendroflumethiazide.  
What  electrolytes  changes  do  loop  diuretics  cause?  
           a.        Hyponatremia  and  hypokalemia  
           b.        Hypokalemia  
           c.        Hypercalcemia  
           d.        Hypocalcemia  
           e.        Hypokalemia  and  hyponatremia  
 
37.  An  18  year  old  young  female  is  losing  weight  despite  eating  3  times  a  
day.  The    
           skin  on  her  bottom  is  flaccid.  What  is  the  definitive  diagnosis?  
a. Jejuna  biopsy  
b. Duodenal  biopsy  
c. Endomysial  antibodies  
d. Anti-­‐reticulin  
e. Anti-­‐glutaminase  
 
38.  A  30  year  old  lady  who  is    pregnant  l  with  38  weeks  gestation  
presents  with  bleeding  per  vaginum  and    
           abdominal  pain.  What  is  the  diagnosis?    
a. Abruptio  placentae  
b. Placenta  previa  
c. Miscarriage  
d. Ectopic  pregnancy  
 

e. Cervical  ectroprion    

 39.  A  70  year  old  man  with  known  prostrate  cancer  which  has  grown  
outside  the    
             capsule  presents  with  increasing  confusion.  He  also  has  high  urea  
and  creatinine.    
             What  do  you  do  next?  
a. IVU  
b. Furosemide  
c. Ultrasound  scan  
d. MSU  
e. Cystoscopy  
 
40.  A  child  of  10  years  has  positive  anti-­‐hepatitis  A  IgE  and  is  hepatitis  C  
IgE  negative.    
             What  do  you  check  to  confirm  the  diagnosis?  
a. Hepatitis  B  antigen  
b. Hepatitis  A  IgG  
c. PCR  for  hepatitis  C  
d. Envelope  hepatitis  C  antigen  
e. Surface  antigen  
 
41.  A  20  year  old    man  presents  with  syphilis  ulcers.  What  investigation  
will  you  do?  
             a.      Antibody  for  treponema  
             b.      Antigen  for  treponema  
             c.        PCR  for  treponema  
             d.        Swab  from  lesions  
             e.  Syphillis  antigen  
 
 
42.  A    30  year  male  adult  had  sex  with  a  female  adult  and  developed  a  
painless  ulcer.  What  is  the  diagnosis?  
a. Primary  syphilis  
b. Herpes  simplex  
c. Secondary  syphillis  
d. Erythema  multiforme  
e. Drug  related  
 
 

43.  A  40  year  old  woman  presents  with  very  painful  ulcers  on  her  vulva.  
She  recently    
             changed  her  partner.  What  is  the  single  most  likely  cause?  
a. Syphilis  
b. Herpes  simplex  
c. Herpes  zoster  
d. Erythema  multiforme  
e. Chlamydia  infection  
 
44.  A  45  year  old  man  patient  developed  infected  peritonitis  after  an  
appendicectomy  for  perforated  gangrenous  appedicitis.  What  analgesia  
will  you  give?  
a. Morphine  IM  
b. Diclofenac  PR  
c. Paracetamol  PO  
d. Patient  control  analgesia  
e. Diamorphine  IM  
 
45.  A  12  year  old  boy  who  is    asthmatic  child  fell  off  the  bicycle  and  
sustained  a  fracture  of  the  tibia  and  fibula.  His  GCS  is  15/15.  What  is  the  
most  appropriate  analgesia  to  give?    
a. Apirin  
b. Paracetamol  
c. Opiods  
d. NSAID’s  
e. Patient  controlled  analgesia  (PCA)  
 
46.  A  65  year  menopausal  woman  presents  with  recurrent  UTI  and  a  
little  bleeding  PV.  Transvaginal  ultrasound  scan  was  normal.  What  is  the  
appropriate  management?  
a. Hormonal  replacement  
b. Vaginal  oestrogen  
c. Vaginal  lubricant  
d. Combined  contraception  
e. Progetserone  only  pill  
 

 
 
47.  A    70  year  old  man  has  lung  cancer  and  is  confused.  There  is  also  
decreased  sodium  on  blood  testing.  Examination  is  normal.  What  is  the  
diagnosis?  
a. SIADH  
b. Fluid  overload  
c. Cushing  syndrome  
d. Phaechromocytoma  
e. Parathyroid  hormone  
 
48.  A  40  year  old  female  on  sodium  valproate  5mg  twice  daily  for  
epilepsy  has  been    
           getting  attacks  more  frequently  than  usual.  On  examination  she  has  
mild  obesity.    
           What  is  the  most  appropriate  management?  
a. Increase  the  dose  
b. Change  the  medication  
c. Review  the  dose  
d. Check  the  serum  drug  levels  
e. Add  a  second  drug  
 
49.    An  alcoholic  with  history  of  major  drinking  wants  to  quit.  He  doesn’t  
want  to  get    
           unpleasant  feeling.  He  has  no  liver  damage.  What  is  the  most  
appropriate    
           management?    
a. Give  disulfiram  and  ask  him  to  come  back  later  
b. Reduce  drinking  and  go  to  the  GP  
c. Ask  him  to  stop  drinking  
d. Refer  to  self  help  group  
e. Acamprosate  
 
50.  A  60  year  old  lady  presents  with  ovarian  carcinoma.  Which  lymph  
drainage  is    
 

             involved?  
a. Internal  iliac  lymph  nodes  
b. Paraortic  lymph  nodes  
c. External  iliac  lymph  nodes  
d. Deep  inguinal  lymph  nodes  
e. Superficial  lymph  nodes  
 
51.  A  female  of  74  years  presents  with  repeated  falls.  She  is  diabetic,  
hypertensive    
             and  is  on  medication  for  diabetes.  She  is  also  on  
bendroflumethiazide  and  some    
             other  drug.  What  is  the  next  investigation?  
a. Ambulatory  ECG  
b. ECHO  
c. Tilt  table  BP  
d. Carotid  angiography  
e. Standing  and  lying  blood  pressure  
 
52.  A  74  year  old  male  who  is  a  known  diabetic  presents  with  falls.  He  is  
on  the    
           following  medication;  aspirin,  diclofenac,  glibenclamide  and  
metformin.  What  is      
           the  most  appropriate  investigation?  
a. Ambulatory  ECG  
b. ECHO  
c. Carotid  angiography  
d. Blood  glucose  
e.  Standing  and  lying  blood  pressure  
 
53.  A  24  year  old  woman  whose  last  menstrual  period  was  6-­‐8  weeks  
ago  presents    
           with  sudden  onset  of  right  sided  iliac  fossa  pain.  What  is  the  most  
likely  diagnosis?  
a. Miscarriage  
b. Ectopic  pregnancy  
 

c. Ovarian  cyst  
d. Torsion  of  ovarian  cyst  
e. Pelvic  inflammatory  disease  
 
54.  A  15  year  old  female  presents  with  right  iliac  fossa  pain  every  4-­‐
8weeks  which  is      
           very  severe  in  recent  weeks.  What  is  the  most  likely  diagnosis?  
a. Miscarriage  
b. Ectopic  pregnancy  
c. Ovarian  cyst  
d. Torsion  of  ovarian  cyst  
e. Pelvic  inflammatory  disease  
 
55.  A  19  year  old  presents  with  per  vaginal  bleeding.  She  was  treated  for  
Chlamydia    
           in  the  past.  She  has  6  weeks  of  amenorrhea.  Pregnancy  test  is  
positive.  What  is    
           the  most  appropriate  investigation?  
a. Pregnancy  test  
b. Transvaginal  ultrasound  scan  
c. Serum  beta  HCG  
d. Laparascopy  
e. Laparotomy  
 
56.  A  35  year  old  woman  with  5  children  presents  with  right  upper  
quadrant  pain.    
           She  also  has  fever  and  vomiting.  3  years  ago,  she  had  intermittent  
pain  in  the    
           right  upper  quadrant.  What  is  the  most  appropriate  management?    
a. Ultrasound  scan  
b. Midstream  urine  
c. Intravenous  urogram  
d. Laparoscopy  
e. Retrograde  pyelography  
 
 

57.  A  3  month  old  child  collapses  and  was  resuscitated  and  is  now  well.  
His  parents    
           are  asking  how  to  do  CPR  in  case  it  happens  again.  
a. 5:1  with  mouth  and  nose  together  
b. 5:1  with  just  mouth,  nose  pinched  
c. 15:2  with  mouth  plus  nose  
d. 15:2  with  mouth,  pinch  nose  
e. 30:2  with  one  hand  
 
58.  A  45  year  old  man  has  been  withdrawn  recently  and  feels  that  he  is  
responsible    
           for  the  floods  in  Thailand.  What  is  the  likely  diagnosis?  
a. Schizophrenia  
b. Depression  
c. Schophrenia  
d. Anxiety  
e. Mania  
 
59.  A  man  fell  off  his  roof  from  a  4m  height.  He  was  stable  and  
answering  questions.    
           Suddenly,  he  became  unconscious.  On  auscultation,  there  is  
decreased  air  entry    
           on  the  right  side  of  his  chest.  What  is  the  most  appropriate  
investigation?  
a. CT  scan  
b. IV  fluids  
c. Thoracocentesis  
d. Chest  X-­‐ray  
e. MRI  brain  
 
60.  An  8  year  old  girl  fell  down  and  fainted  at  the  school  assembly.  She  
went  pale    
           and  light  headed.  What  is  the  most  likely  diagnosis?  
a. Decreased  blood  sugar  
b. Vasovagal  syncope  
 

c. Anxiety  
d. Arrhythmia  
e. Heat  stroke  
 
61.  A  7  year  old  boy  with  epilepsy  is  admitted  to  hospital  and  is  presently  
on    
             valproate.  He  has  a  seizure  in  the  night.  What  is  the  initial  
investigation?    
a. Check  drug  seum  
b. CT  head    
c. EEG  
d. ECG  
e. MRI  scan  of  the  brain  
62.  A  20  year  old  female  presents  with  palpitations.  On  examination,  
there  is  no    
           abnormality  and  pulse  is  regular.  What  is  the  singe  most  diagnosis?  
a. Sinus  tachycardia  
b. VT  
c. SVT  
d. AF  
e. Atrial  flutter  
 
63.  A  38  year  old  woman  presents  with  pruritus  for  5  years.  What  is  the  
most    
             appropriate  investigation?  
a. FBC  
b. Serum  amylase  
c. Anti-­‐mitochondrial  antibodies  
d. Liver  ultrasound  scan  
e. Liver  function  tests  
 
64.  A  45  year  old  man  recently  returned  from  Malaysia.  For  the  past  2  
days,  he  has    
           developed  fever  and  upper  right  quadrant  pain  and  jaundice.  What  is  
the  most    
 

           appropriate  investigation?  
a. CT  abdomen  
b. Hepatitis  screen  
c. Serum  amylase  
d. Ultrasound  scan  of  the  abdomen  
e. Liver  function  tests  
 
65.  A  22  year  old  man  presents  with  bilateral  facial  swelling.  What  is  the  
most  likely    
           diagnosis?  
a. Parotid  calculi  
b. Submandibular  calculi  
c. Lympadenopathy  
d. TB  adenitis  
e. Mumps  
 
66.  A  6  week  old  girl  with  pyloric  stenosis  presents  with  vomiting.  What  
is  the  most    
           appropriate  test?  
a. Calcium  
b. Potassium  
c. Sodium  
d. HCl  
e. Magnesium  
 
 
67.  A  15  year  old  boy  has  had  his  sebaceous  cyst  removed.  He  is  a  known  
asthmatic.    
           What  is  the  most  appropriate  management?  
a. Paracetamol  
b. NSAID’s  
c. Morphine  
d. Codeine  
e. Aspirin  
 
 

68.  A  50  year  old  man  presents  with  sudden  loss  of  vision  in  his  right  eye.  
On    
           fundoscopy,  there  are  hard  exudates  and  flame  shaped  hemorrhages.  
What  is  the    most  likely    
           diagnosis?  
a. Central  retinal  vein  occlusion  
b. Central  retinal  vein  artery  
c. Retinal  detachment  
d. Optic  neuritis  
e. Branch  of  retinal  artery  occlusion  
 
 
69.  A  40  year  old  lady  while  walking  at  home  develop  sudden  onset  
of  vomiting  and  loss  of  consciousness.  In  accident  and  emergency  
department  her  GCS  is  5/15.  Initially  she  lost  consciousness  and  
later  she  was  ok  with  GCS  15/15.  What  is  the  single  most  
appropriate  investigation?  She  had  a  laceration  to  her  temple  area  
A. CT  scan  of  the  head  
B. Facial  X-­‐ray  
C. MRI  brain  
D. No  investigation  
E.  Skull  X-­‐ray  
 
 
70.  A  23  year  old  lady  who  is  pregnant  presents  with  hypertension  in  her  
pregnancy.    
           She  has  history  of  recurrent  UTI  as  a  child.  What  is  the  most  likely  
cause  of  her    
           hypertension?  
a. Pregnancy  induced  hypertension  
b. Kidney  damage  
c. Essential  hypertension  
d. Pre-­‐eclampsia  
e. Diabetes  
 
 

71.  A  45  year  old  lady  was  found  to  have  CIN  2  on  routine  test.  What  is  
the  most          
           appropriate  next  step?  
a. Smear  
b. Colposcopy  
c. Cervical  biopsy  
d. Transvaginal  ultrasound  scan  
e. Endometrial  carcinoma  
72.  A  67  year  old  lady  presents  with  tingling  and  numbness  in  the  index  
and  middle    
           finger.  Tinnel  test  is  negative.  What  is  the  most  appropriate  
investigation?  
a. Blood  glucose  
b. CT  spine  
c. Spine  X-­‐ray  
d. MRI  spine  
e. Hormone  levels  
 
73.  A  50  year  old  man  wakes  up  in  the  morning  with  food  on  the  pillow.  
What  is  the    
           most  likely  cause?  
a. GERD  
b. Pharyngeal  pouch  
c. Laryngeal  cancer  
d. Facial  nerve  palsy  
e. Nocturnal  vomiting  
 
74.  A  3  year  old  boy  develops  haematuria  2  weeks  after  upper  
respiratory   tract   infection.   What   is   the   single   most   likely   diagnosis?  
A. Post-­‐streptococcal  glomerulonephritis  
B. Ig  A  nephropathy  
C. Chronic  pyelonephritis  
D. Urinary  tract  infection  
E. Bladder  tumour  
 
 
 

75.  A  23  year  old  lady  presents  with  shortness  of  breath  and  light  
headedness.  She    
           has  vitiligo  and  raised  MCV.  What  is  the  most  likely  diagnosis?  
a. Hypothyroidism  
b. Pernicious  anemia  
c. Phenytoin  side  effects  
d. Iron  deficiency  anaemia  
e. Folate  deficiency  
 
76.  A  56  year  old  man  who  has  never  smoked  presents  with  
regurgitation  of  food    
           when  lying  down.  He  also  has  hoarseness  of  voice  for  few  months.  
Who  do  you    
           refer  to?  
a. ENT  surgeon  
b. GIT  surgeon  
c. General  surgeon  
d. Voice  therapist  
e. Speech  therapist  
 
77.  A  43  year  old  pregnant  woman  presents  with  tingling  and  numbness  
in  the  index    
           and  middle  finger.  What  is  the  lost  likely  diagnosis?    
a. Carpal  tunnel  syndrome  
b. Diabetes  in  pregnancy  
c. Hypertension  in  pregnancy  
d. Cushing  syndrome  
e. Acromegaly  
 
78.  A  40  year  old  man  presents  with  2  months  history  of  chronic  short  
breath  and  an    
           erythema  on  his  shin.  What  is  the  most  likely  cause  of  the  lesion?  
a. Erythema  nodosum  
b. Erythema  multiforme  
c. Urticaria  
 

d. Purpura  
e. Maculopapular  rash  
 
79.  A  60  year  old  woman  presents  with  bleeding  per  vaginum.  Her  last  
cervical  smear    
           was  3  years  ago.  What  is  the  most  likely  diagnosis?  
a. Cervical  cancer  
b. Endometrial  carcinoma  
c. Cervical  poly  
d. Vaginal  cancer  
e. Pelvic  inflammatory  disease  
 
80.  A  50  year  old  woman  presents  with  bleeding  from  one  of  her  
mammary  duct.    
           What  is  the  most  appropriate  investigation?  
a. Mammography  
b. Open  biopsy  
c. FNAC  
d. Ductography  
e. Core  biopsy  
 
81.  A  70  year  old  lady  with  severe  heart  failure  presents  with  stress  
urinary    
           incontinence.  She  has  got  a  feeling  of  dragging  sensation  in  her  
perineum.  What    
           is  the  most  appropriate  management?  
a. Pelvic  floor  exercise  
b. Ring  pessary  
c. Oxybutinin  
d. Bladder  neck  surgery  
e. Local  estrogen  
 
82.  A  1  year  old  child  presents  with  dehydration.  Which  of  the  following  
is  an    
           indication  for  intravenous  fluids?  
 

a. A  capillary  refill  >  4  seconds  


b. Sunken  eyes  
c. Loose  skin  turgour  
d. Crying  
e. Crying  without  tears  
 
83.  A  20  year  old  man  presents  with  severe  asthma.  Which  of  the  
following  is  a    
           criteria  for  severe  asthma?  
a. Silent  chest  
b. Cyanosis  
c. Not  speaking  in  full  sentences  
d. RR  >  27cycles/minute  
e. PEFR  <  30  
 
84.  A  25  year  old  lady  presents  with  sudden  loss  of  vision.  
Fundoscopy  is  normal  .  Patient  visual  acquit  is  finger  counting  .  
What  is  the  single  most  likely  diagnosis?  
A. Optic  neuritis  
B. Retrobulbar  neuritis  
C. Papilloedema  
D. Optic  atrophy  
E. Macular  degeneration.  
 
85.  A  23  year  old  who  is  15  weeks  pregnant  presents  with  excessive  
vomiting.  On    
           ultrasound  scan  the  uterus  is  18  weeks  and  there  is  a  hyperechogenic  
shadow.    
           What  is  the  diagnosis?  
a. Hyperemesis  gravidarum  
b. Molar  pregnancy  
c. Acute  pancreatitis  
d. Diabetes  mellitus  
e. Polyhydramnious  
 
 

86.  A  30  year  old  lady  who  is  27  weeks  pregnant  is  found  to  have  anemia  
of  10.  She    
             has  normal  MCHC  and  MCV.  What  is  the  most  appropriate  action?  
a. No  treatment  required  
b. Endoscopy  
c. Blood  transfusion  
d. Vitamin  B12  replacement  
e. Iron  and  folate  
 
87.  A  30  year  old  lady  in  her  30  weeks  of  pregnancy  presents  with  
sudden  onset  of    
           uterine  bleeding  and  a  tender  uterus.  She  has  no  previous  history  of  
caesarian    
           section.  Fetal  heart  sounds  are  increased.  What  is  the  most  likely  
diagnosis?  
a. Placenta  praevia  
b. Uterus  rupture  
c. Cervical  ectropion  
d. Trauma  
e. Placental  abruption  
 
88.  A  23  year  old  lady  in  her  34  weeks  of  pregnancy  presents  with  
seizures.  Her  BP  is    
           170/110mmHg.  What  is  the  most  appropriate  management?    
a. Intravenous  hydralazine  
b. Intravenous  methyldopa  
c. Intravenous  magnesium  sulphate  
d. Caesarian  section  
e. Induce  labor  
 
89.  A  35  year  old  lady  with  previous  history  of  pelvic  inflammatory  
disease  has  6    
           weeks  history  of  amenorrhea.  She  presents  with  right  sided  lower  
abdominal    
           pain.  What  is  the  most  likely  diagnosis?  
 

a. Miscarriage  
b. Exacerbation  of  pelvic  inflammatory  disease  
c. Ectopic  pregnancy  
d. Molar  pregnancy  
e. Acute  pancreatitis  
 
90.  A  70  year  old  woman  with  breast  cancer  has  had  it  spread  to  her  
lung,  liver  and    
           bones.  There  is  also  increasing  constipation,  weakness,  thirst  and  
anorexia  for  3    
           days.  She  is  taking  only  haloperidol  for  hiccups.  She  is  more  
disorientated  today.    
           What  is  the  most  likely  diagnosis?  
a. Cerebral  metastases  
b. Hypercalcemia  
c. Spinal  cord  compression  
d. UTI  
e. Chest  infection  
 
91.  A  30  year  old  man  after  an  appendicectomy  develops  abdominal  
pain  radiating    
             to  the  shoulder.  What  is  the  most  likely  nerve  to  be  involved?    
a. Phrenic  nerve  
b. T10  
c. Aortic  nerve  
d. T12  
e. Celiac  nerve  
 
92.  A  40  year  old  lady  has  had  appendicectomy.  She  presents  with  
numbness  on  her    
             umbilical  area.  Which  nerve  is  damaged?    
a. Phrenic  nerve  
b. Celiac  nerve  
c. T12  
d. T10  
 

e. T5  
 
 
93.  A  40  year  old  man  is  stabbed    in  the  chest  BP  is  60/0,  HR  140.  On  
examination  he  has  engonged    neck  veins  and    trachea  is  on  the  
midline.      What  is  the  single  most  likely  diagnosis?  
A. simple  pneumothorax  
B. Tension  pneumothorax  
C. Haemothorax  
D. Cardiac  temponade  
Aortic  laceration  
94.  An  89  year  old  lady  is  terminally  ill  with  advanced  cancer  and  is  in  
severe  pain.    
           The  pain  is  not  controlled  by  morphine.  She  also  has  nerve  root  pain.  
What  is  the    
           most  appropriate  management?  
a. Oral  morphine  
b. Paracetamol  
c. Gabapentin  
d. Increase  analgesia  
e. No  further  action  
 
95.  A  6  month  old  child  develops  fever  and  difficulty  in  breathing  and  
feeding.  What    
             is  the  most  likely  diagnosis?  
a. Pneumonia  
b. URTI  
c. Broncholitis  
d. Croup  
e. Epiglottitis  
 
96.  A  6  week  old  child  has  repeated  vomiting  after  feeding.  What  is  the  
most    
             appropriate  investigation?  
a. CT  abdomen  
b. USS  abdomen  
 

c. Endoscopy  
d. Barium  enema  
e. No  investigations  required  
 
97.  A  40  year  old  woman  with  family  history  of  cancer  on  routine  
screening  was        
           found  to  have  an  APC  gene.  Which  cancer  is  most  likely  at  risk?  
a. Colonic  adenocarcinoma  
b. Ovarian  carconima  
c. Endometrial  carcinoma  
d. Cervical  carcinoma  
e. Breast  cancer  
 
98.  A  30  year  old  lady  presents  with  fresh  blood  per  rectum  after  
defecation.  She  has    
           been  constipated  for  the  past  2  months.  She  has  no  pain.  What  is  the  
diagnosis?  
a. Anal  fissure  
b. Haemorrhoids  
c. Inflammatory  bowel  disease  
d. Colonic  cancer  
e. Rectal  cancer  
 
99.  A  25  year  old  lady  presents  with  intermittent  abdominal  pain  and  
bloody    
           diarrhea.  What  is  the  most  likely  diagnosis?  
a. Crohn’s  disease  
b. Ulcerative  colitis  
c. Irritable  bowel  syndrome  
d. Colonic  cancer  
e. Abdominal  migraine  
 
100.  A  76  year  old  man  suddenly  collapses  and  dies.  The  post  mortem  
examination    
 

               shows  a  retroperitoneal  hematoma  due  to  a  ruptured  aneurysm.  


What  is  the    
               most  likely  cause  of  this  aneurysm?  
a. Atheroma  
b. Cystic  medical  necrosis  
c. Dissecting  aneurysm  
d. Polyarteritis  nodosa  
e. Syphilis  
 
101.  A  27  year  old  lady  after  caesarian  section  develops  epigastric  pain  8  
hours  later.    
                 What  is  the  most  appropriate  investigation?  
a.    ABG  
b.    Coagulation  profile  
c.    Liver  enzymes  
d.    Liver  biopsy  
 
102.  A  70  year  old  man  with  previous  history  of    lung  cancer  which  
was  treated  with  surgery  and  radiotherapy  presents  with  
headache  and  swelling  of  the  upper  limb.    The  lower  limb  were  
normal.  What  is  the  single  most  appropriate  management?  
A. Dexamethesaone  
B. Radiotherapy  
C. Chemotherapy  
D. Prednisolone  
E. Manitol  
 
 
103.    A  3  year  old  child  presents  with  burns  on  his  shoulder  and  upper  
arm  covering    
                   4%  of  the  total  body  surface.  He  is  in  pain  and  screaming.  What  is  
the  most    
                   appropriate  management?  
a.      Analgesia  
b.      IV  fluid  
c.      Dressing  
 

d.      Antibiotics  
e.      Refer  to  burns  team  
 
104.    A  28  year  old  man  presents  with  abdominal  distension  and  pain.  
His  stools  have    
                   been  mucoid  and  sometimes  blood  stained.  What  is  the  most  
appropriate    
                   investigation?  
a.      Barium  meal  
b.      Gastroscopy  
c.      IgA  tissue  transaminase  
d.      Jejunal  biopsy  
e.      Stoll  culture  and  sensitivity  
 
 
105.    A  67  year  old  lady    presents  with  nausea  ,  vomiting  and  diarrhea  
for  3  days.  She  has  atrial  fibrillation  and  on  digoxin.  What  is  the  
single  most  likely  diagnosis?  
A. Clostridium  difficile  
B. Digoxin  
C. Gastroenteritis  
D. Inflammatory  bowel  disease  
E. Colonic  cancer  
 
106.  A  73  year  old  patient  has  last  stage  colon  cancer  and  is  suffering  
from  severe    
                 pain  although  he  has  reached  the  maximum  dose  of  opiates.  What  
would  you    
                 prescribe  to  improve  his  symptoms?  
a.    Subcutaneous  morphine  
b.    Fentanyl  patches  
c.    Oxycodone  
d.      Radiotherapy  
 
107.  A  25  year  old  man  presented  with  history  of  breathlessness.  A  
transthoracic    
 

                 echo  revealed  patent  foramen  ovale.  What  diagnostic  investigation  


would  you    
                 do?  
a.    Trans  esophageal  ECHO  
b.    Bubble  ECHO  
c.    Trans  thoracic  ECHO  
d.    ECG  
 
108.  A  4  year  old  child  comes  to  the  hospital  with  foot  sprain.  History  
reveals  that    
                 the  child  has  had  recurrent  admissions  to  the  hospital  due  to  severe  
asthma.    
                 What  is  the  most  appropriate  analgesia?  
a.    Diclofenac  
b.    Ibuprofen  
c.    Paracetamol  
d.    Codeine  
 
109.  An  80  year  old  man  who  is  on  treatment  for  atrial  fibrillation  with  
digoxin    
                 presents  with  vomiting  and  diarrhea  for  3  days  and  he  is  
dehydrated.  What  will    
                 you  do  to  find  the  cause?  
a.      ECG  
b.      ECHO  
c.      Serum  digoxin  levels  
d.      Serum  calcium  
e.      Stool  culture  
 
110.  A  35  year  old  lady  flew  from  South  Africa  to  United  Kingdom.  3  
days  later  she  presents  cough  with  blood  stained  sputum?  What  
is  the  single  most  appropriate  investigation?  
A. D-­‐dimmer  
B. Verntilation  perfusion  scan  
C. Pulmpnary  angiogram  
D. Chest  X-­‐ray  
 

E. MRI  of  the  chest.  


 
 
 
111.  A  50  year  old  man  with  epilepsy  took  an  overdose  of  his  medication  
1  hour  ago.    
               He  has  dry  mouth  and  dilated  pupils  but  is  not  drowsy.  What  is  the  
most    
               appropriate  management?  
a. Haemodialysis  
b. Activated  charcoal  
c. Gastric  lavage  
d. Forced  emesis  
e. Hyperbaric  oxygen  
 
112.  An  80  year  old  lady  presents  with  frequency,  weight  loss,  hematuria  
and  blood    
                 clots.  USS  of  the  abdomen  is  inconclusive.  What  is  the  most  
appropriate    
                 investigation?  
a.    IVU  
b.    CT  abdomen  
c.    Cystoscopy  
d.    X-­‐ray  KUB  
e.    Tumour  markers  
 
113.  A  22  year  old  man  presents  with  swelling  of  both  legs.  Urine  dipstix  
shows    
                 protein+++.  What  is  the  most  appropriate  investigation?  
a.    Renal  biopsy  
b.    USS  Kidneys  
c.    IVU  
d.    X-­‐ray  KUB  
e.    Bence-­‐Jones  protein  
 
 

 
114.    A  20  year  old  mother  who  had  been  alcohol  dependence  for  
over  20  years.    She  has  been  misusing  of  different  types  of    drugs.  
She  is  now  well  and  alert..  She  would  like  to  be  helped?  
A. Voluntary  admission  to  psychiatric  ward  
B. Compulsary  admission  to  psychiatry  
C. Refer  to  community  mental  healthy  team  
D. Methadone  
E. Refer  to  the  psychiatrist  
 
115.  A  35  year  old  lady  presents  with  painless  bleeding  from  a  single  
duct.  What  is    
                 the  most  likely  diagnosis?  
a.    Duct  ectasia  
b.    Paget’s  disease  
c.    Duct  papilloma  
d.    Intraduct  pailloma  
e.    Breast  carcinoma  
 
116.  A  25  year  old  man  was  involved  in  a  road  traffic  accident  and  got  his  
foot    
                 trapped  under  the  car  for  4  hours.  Dorsalis  pedia  is  not  palpable  
and  he  has  got    
                 tenderness  between  the  big  toe  and  the  second  toe.  What  is  the  
most  likely    
                 diagnosis?  
a.    Arterial  puncture  
b.    Compartment  syndrome  
c.    Arterial  embolus  
d.    DVT  
e.    Fracture  of  the  big  toe  
 
117.  A  57  year  old  man  is  on  treatment  for  hypertension  with  
bendroflumethiazide    
               and  amlodipine.  His  BP  is  140/80mmHg  and  pulse  is  38b/m.  What  is  
the  most    
 

               appropriate  investigation?  
a. ECG  
b. Ambulatory  ECG  
c. Ambulatory  BP  
d. ECHO  
e. BP  
 
118.  A  64  year  old  man  presnstw  ith  dyspepsia  and  epigastric  pain.  
He    has  no  pas  history  of  helicobacter  pylori.  What  is  the  single  
most  appropriate  investigation?  
A. Ultrasound  scan  abdomen  
B. Upper  GI  endoscopy  
C. H.  Pylori  serology  
D. Urea  breath  tests  
E. Barium  swallow  
 
119.  A  7  year  old  boy  fell  down  from  a  bicycle.  He  now  presents  with  
swelling  of  the    
               right  knee  and  is  unable  to  bear  weight.  What  is  the  most  
appropriate    
               management?  
a. X-­‐ray  of  the  knee  
b. Analgesia  
c. Immobilise  in  a  cast  
d. Refer  to  fracture  clinic  
e. Reassure  
 
120.  A  2  year  old  boy  fell  down  from  the  stairs.  He  cried  for  sometime  
and  suddenly    
               became  quiet.  When  the  mother  turned  to  look  at  him,  he  was  
unconscious  and    
               stopped  breathing.  After  1  hour,  the  child  was  completely  fine  and  
playing    
               actively.  The  mother  said  that  this  has  happened  in  the  past.  What  is  
the  most    
               likely  diagnosis?  
 

a. Epilepsy  
b. Anoxic  episodes  
c. Febrile  convulsions  
d. Brain  tumour  
 
121.  A  2  year  old  boy  has  had  urinary  infection.  Investigation  shows  
vesico-­‐ureteric    
                 reflux.  What  will  you  tell  the  parents?  
a.    Requires  antibiotic  prophylaxis  
b.    Requires  surgery  
c.    In  most  kidneys  will  be  scarred  by  the  age  of  5  
d.    No  further  action  required  
e.    It  will  resolve  on  its  own  
 
122.  A  70  year  old  man  presents  with  back-­‐pain,  anemia  and  raised  ESR.  
Corrected    
                 serum  blood  calcium  is  3.70.  He  is  already  on  intravenous  fluid.  
What  is  the    
                 most  appropriate  management?      
a.    Pamidronate  
b.    Rehydration  
c.    Calcitonin  
d.    Allopurinol  
e.    Parathormone  
 
 
123.  A  45  year  old  man  while  in  the  gym  had  sudden  onset  of  back  
pan.  He  presents  wot  accident  and  emergency  department  with  
lower  back  pain.  He  has  been  given  analgesia  and  now  feels  
better.  All  examination  is  normal  .    What  is  most  appropriate  
advice  to  him?  
A. Continue  moving  
B. Bed  rest  
C. Needs  MRI  scan  of  the  spine  
D. Needs  CT  scan  spine  
E. Bed  rest  and  analgesia  
 

124.  A  76  year  old  man  fell  down  and  hit  his  head.  Five  days  later  he  
comes  back  with  confusion.    Which  of  the  following  is  the  most  
likely  sight  of  bleeding?  
A. Middle  meningeal  artery  
B. Internal  cerebral  artery  
C. Cerebral  vein  plexes  
D.  Bery  aneuryms  
E. Inernal  carotid  vein  
 
125.  A  70  year  old  man  presents  with  back  pain,  anemia  and  raised  
calcium.  What    
                 finding  will  be  on  bone  marrow  aspirate?  
a.    Increased  plasma  cell  
b.    Increased  leucocytes  
c.    Increased  T-­‐cells  
d.    Reduced  plasma  cells  
e.    Increased  neutrophils  
 
126.  A  70  year  old  man  presents  with  progressive  headaches,  nystagmus,  
dizziness,    
                 slurred  speech  and  unsteadiness  for  the  past  4  months.  What  
investigation  is    
                 the  best  to  localize  the  affected  area?  
a.      CT  head  
b.      MRI  brain  
c.      CT  angiogram  
d.      USS  head  
e.      Skull  X-­‐ray  
 
127.  A  2year  old  boy  presents  with  redish  blistering  rash  on  the  face  
especially  around  the    mouth  and  nose.  What  is  the  single  most  
likely  diagnosis?  
A. Impetigo  
B. Shingles  
C. Chicken  pox  
D. Measles  
E. Herpes  simplex  
 

 
 
128.  An  elderly  woman  cannot  carry  on  her  household  chores  and  has  to  
sit  down    
                 every  few  minutes.  A  chest  X-­‐ray  shows  diffuse  reticular  nodular  
shadows    
                 especially   in   the   lower   lobes.   What   is   the   next   step   in   management?      
a.  Oxygen    
b.    Thymectomy  
c.      Chemotherapy  
d.      Change  occupation  
e.      Radiotherapy  
 
129.  A  4  year  old  boy  presents  with  frontal  headaches.  Physical  
examination  reveals    
               that  the  child  is  below  the  normal  percentile  for  height  and  weight  
for  his  age.  A    
               computerized  tomography  study  of  his  skull  reveals  a  cystic,  
suprasellar  mass    
               with  focal  areas  of  calcification.  There  is  an  enlargement  of  the  sella  
turcica.    
               The  lesion  is  most  likely  derived  from?          
a.  Rathke  pouch  remnants  
b.  Histiocytes  
c.  Chromophobe  cells  
d.  Cells  in  the  posterior  pituitary  
e.  Astrocytes  
 
130.    A  45  year  old  man  presents  with  knee  pain  to  accident  and  
emergency  department.      He  was  later  found  to  have  gout.  What  
is  the  single  most  appropriate  management?  
A. Allopurinol  
B. Non  steroid  anti-­‐inflammatory  drugs  
C. Colchicine  
D. Paracetamol  
E. Oral  morphine.  
 

 
 
131.  A  36  year  old  lady  with  long  standing  history  of  migraine  presents  
with  severe    
                 headache.  She  says  it  is  the  worst  headache  of  her  life.  What  is  the  
most    
                 appropriate  management?  
a.    Give  analgesia  
b.    CT  scan  of    the  head  
c.    Reassure  
d.    Observe  in  emergency  department  
e.    Admit  for  24  hours  
 
132.  A  boy  is  brought  to  the  A&E  with  feeling  of  being  unwell  for  a  few  
days  and    
                 vomiting.  His  pulse  is  100b/m  and  capillary  refill  is  3  seconds.  He  is  
confused    
                 and  his  arterial  blood  gas  results  are  shown  as  follows;  
                 Ph:    7.25  
                 PaCO2:    3.2kPa  
                 PaO2:    20kPa  
                 Base  excess:    -­‐15  
                 What  is  the  single  correct  interpretation  of  the  blood  gas  result?  
a.      Metabolic  acidosis  
b.      Metabolic  alkalosis  
c.      Normal  
d.      Respiratory  disorder  
e.      Respiratory  alkalosis  
 
133.  A  3  day  old  child  has  a  harsh  systolic  murmur  without  any  radiation.  
What  is  the    
                 likely  diagnosis?  
a.    VSD  
b.    Tetralogy  of  Fallot  
c.      ASD  
 

d.      Pulmonary  Stenosis  
 
134.  A  5  year  old  boy  complains  about  an  offensive  unilateral  discharge  
from  the  nose  for  6  months.  What  is  the  most  likely  cause?  
a.    Foreign  body  
b.    Allergic  rhinitis  
c.    Polyp  
d.    Sinusitis  
e.    ITP  
 
135.  A  25  year  old  man  was  brought  to  the  A&E  department  after  
partying  in  the  night  club.  He  is  hallucinated,  has  high  temperature  and  
his  pupils  are  dilated.  His    
               friends  would  not  admit  which  substance  he  took.  The  patient  said  
that  the    
               ‘‘orange  colored’’  doctor  is  talking  to  him.  What  drug  is  most  likely  
implicated?  
a.    LSD  
b.    Ecstasy  
c.    Cocaine  
d.    Alcohol  
e.    Heroin  
 
136.  An  old  woman  develops  pain  in  the  lower  part  of  the  knee  joint  
anteriorly  on    
                 sitting.  Which  bursa  is  affected?  
a.    Suprapatellar  
b.    Infrapatellar  
c.    Prepatellar  
 
137.  An  old  woman  develops  sudden  pain  in  the  upper  part  of  her  arm  
after  carrying    
                 heavy  loads.  Which  part  of  the  arm  is  affected?  
a.    Ruptured  supraspinatus  tendon  
b.    Posterior  capsulitis  
 

c.    Biceps  tear  
d.    Bursitis  
 
138.  A  lady  said  that  she  is  being  controlled  by  some  outside  force  who  
tells  her  to    
                 do  everything.  What  is  this  condition  that  she  is  having?  
a.    Delusion  
b.    Thought  insertion  
c.    Thought  broadcasting  
d.      Acute  anxiety  
e.      Passivity  phenomenon  
 
139.  A  40  year  old  patient  presents  with  dry  cough  and  confusion.  Blood  
tests  reveal    
                 lymphopenia  and  hyponatremia.  Chest  X-­‐ray  shows  right  lobar  
shadowing.    
                 What  is  the  single  most  appropriate  treatment?      
a.    Erythromycin  
b.    Cyclophosphamide  
c.    Tobramycine  and  carbenicillin  
d.    Plasmaphoresis  
e.    Ciprofloxacin  
 
140.  A  2  week  s  old  babay  has  been  diagnosed  with  breast  milk  
jaundice.  What  would  the  blood  test  investigation  show?  
A. Bilirubin  40,  conjugated  less  5.  
B. Bilirubin  40  conjugated  <10  
C. Bilirubin  40,  unconjugated………  
D. Bilirubin  40,  …..  
E.  Bilirubin……  
 
 
141.      A  40  year  old  man  presents  with  vertigo,  nystagmus  and  weakness.  
He  also  has    
                 tingling  sensation  in  his  right  arm  and  diplopia.  On  examination,  
there  is  loss  of    
 

                 central  vision,  nystagmus  and  ataxia.  What  is  the  most  likely  
diagnosis?  
a.    Vertebra-­‐basilar  ischemia  
b.    Multiple  sclerosis  
c.    Acoustic  neuroma  
d.    Lateral  medullary  syndrome  
 
 
142.  A  40  year  old  Afro-­‐Caribbean  woman’s  lung  function  tests  are  as  
follows:  
                 PET/FEV1    2.51,    FEV1/FVC    86%,  Serum  ACE  levels    -­‐High.  
                 What  is  the  single  most  likely  diagnosis?  
a.      Pneumonectomy  
b.      Pulmonary  fibrosis  
c.      Pulmonary  embolism  
d.      Panic  attack  
e.      Sarcoidosis  
 
143.    A  34  year  old  man  presents  with  right  upper  quadrant  pain  and  
was  later  found  to  have  acute  hepatitis.  What  would  be  the  
values  of  liver  function  tests?  
A. AST…..  
B. ALT…….  
C. Bilirubin…..400  
D. Alkaline  phosphatase  
E. AST  
 
 
144.  A  62  year  old  man  has  a  5  years  history  of  diabetes.  On  examination,  
his  blood    
               pressure  is  120/80mmHg.  Laboratory  tests  for  a  24  hour  urine  
collection  show  a    
               protein  level  of  200mg/l.  What  medication  should  be  added  to  this  
patient?  
a.  B-­‐blockers  
b.  Calcium  channel  blockers  
 

c.  ACE  inhibitors  
d.  Diuretics  
 
145.  A  woman  has  a  35  days  period.  On  what  day  should  her  
progesterone  level  be    
               measured  to  determine  ovulation?  
a.  Day  14th  
b.  Day  16th  
c.  Day  21st  
d.  Day  25th  
e.  Day  28th  
 
146.  A  patient  after  trauma  has  a  claw  hand.  What  clinical  test  should  
you  perform      
               to  prove  the  diagnosis?  
a.  Abduction  of  the  fingers  
b.  Extension  of  the  wrist  
c.  Flexion  of  the  wrist  
d.  Abduction  of  the  thumb  
e.  Opposition  of  the  thumb  
 
147.  A  middle  aged  woman  presents  with  chronic  cough  and  purulent  
sputum.    
               Recently  she  has  become  very  wheezy  and  dyspnoeic.  ECG  gives  a  
low  voltage    
               reading  and  chest  X-­‐ray  shows  a  large  heart  shadow.  What  is  the  
most    
               appropriate  management?    
a. Antibiotics  
b. Sputum  culture  
c. Pericardial  aspiration  
d. Streptokinase  
e. Furosemide  
 
 

148.  A  60  year  old  farmer  presents  with  fever,  cough  and  shortness  of  
breath.  He    
               was  forking  hay  that  morning.  Chest  X-­‐ray  shows  fatty  nodular  
shadows  in  the    
               upper  zone.  What  is  the  single  most  appropriate  treatment?  
a. Prednisolone  
b. Erythromycin  
c. Cyclophosphamide  
d. Tobramycin  and  carbenicillin  
 
149.  A  woman  after  road  traffic  accident  is  in  the  A&E  department.  Her  
blood    
               pressure  is  low  and  she  has  pain  in  the  left  part  of  her  abdomen.  
What  will  you    
               do  first?  
a.  Abdominal  Ultrasound  
b.  IV  fluid  
c.  Laparatomy  
d.  CT  abdomen  
e.  Blood  transfusion  
 
150.  A  child  has  a  dry  skin  on  the  arm.  Parents  tried  emollients  which  did  
not  help.    
               What  is  the  next  step  of  management?  
a.    1%  prednisolone  
b.    2%  hydrocortisone  
c.    0.1%  betamethasone  
d.    Antibiotics  
e.    Clobetasol  
 
 
 
 
 
 
 

 
151.  A  22  year  old  collapses  at  a  football  match  and  is  brought  in  by  the  
ambulance.    
               He  is  pale,  sweaty  and  has  a  Glasgow  coma  scale  of  10.  His  eyes  are  
open  to  pain,    
               he  also  localizes  to  painful  stimuli  and  is  speaking  inappropriate  
words.  His    
               blood  pressure  is  120/30mmHg,  pulse  rate  is  100b/m  and  the  
capillary  refill  time    
               is  less  than  2  seconds.  His  airways  are  also  patent.  What  is  the  single  
most    
               appropriate  initial  investigation?  
a.  Blood  ethanol  (alcohol)  level  
b.  Capillary  blood  glucose  
c.  Computed  tomography  scan  of  the  brain  
d.  Drug  screen  
e. Serum  urea  and  electrolytes  concentration  
 

152.    A  23  year  old  lady  who  10  weeks  pregnant  presents  to  
antenatal  clinic  and  she  is  worried  that  her  child  could  have  
cystic  fibrosis.  What  would  be  the  most  appropriate  investigation?  
A. amniocentesis  
B. Chorionic  villus  sampling  
C. Tripple  tests  
D. Anomaly  ultrasound  scan  
E. Prenatal  diagnosis.  
 
 
153.  An  elderly  woman  who  lives  alone  presents  with  pain  for  a  few  days  
in  the  6th    
               and  7th  intercostals  space.  What  is  the  most  likely  diagnosis?  
a.  Herpes  zoster  
b.  Broken  rib  
c.  Myalgia  
 

d.  Herpes  simplex  
e.  Staphylococcus  infection  
 
154.  A  child  after  viral  infection  has  otitis  media.  What  antibiotic  will  you  
prescribe?  
                 a.      Amoxicillin  
                 b.      Flucloxacillin  
                 c.      Gentamycin  drops  
                 d.      Gentamycin  tablets  
                 e.      Benzylpenicillin  
 
155.  A  patient  with  long  history  of  GERD  underwent  oesophago-­‐
duodenoscopy  and  Barret’s  oesophagus  was  diagnosed.  What  is  the  
single  most  important  term  to    
                 define  this  condition?  
a.    Squamous  metaplasia  
b.    Columnar  metaplasia  
c.    Dysplasia  
d. basal  cell  carcinoma  
e. Squamous  cell  carcinoma  
156.  A  76  year  old  man  presents  with  a  right  hemi-­‐paresis.  CT  scan  
shows  an    ischemic  stroke  and  Aspirin  300mg  is  commenced.  In  terms  of  
further  management  in  the  acute  phase,  which  one  of  the  following  
values  should  not    
               be  corrected?  
a.  Blood  pressure  210/110mmHg  
b.  Blood  glucose  9.4mmol/l  
c.  Oxygen  saturation  94%  
d.  Blood  glucose  2.5mmol/l  
e.  Temperature  38.5  C  
 
157.  A  35  year  old  woman  presents  with  thyrotoxicosis.  Which  one  of  
the  following    
               physical  signs  support  a  diagnosis  of  Grave’s  disease?  
a.  Opthalmoplegia  
 

b.  Lid  retraction  
c.  Tremor  
d.  Atrial  fibrillation  
e.  Symmetrical  goiter  
 
158.  A  woman  presents  with  a  lump  in  her  breast  with  
lymphadenopathy  and  possible  malignancy  suspected  after  a  fine  needle  
biopsy.  What  sit  the  next  step    
               of  investigation?  
a.  Core  biopsy  
b.  Mammography  
c.  Ultrasound  
d.  Open  biopsy  
e.  FNAC  
 
159.  A  4  year  old  girl  presents  with  dyusria,  frequency  and  fever.  Culture  
of  urine  shows  E.coli.  Which  antibiotic  will  you  give?  
a.  Rifampicin  
b.  Cefuroxime  
c.  Ciprofloxacin  
d.  Trimethoprim  
e.  Gentamicin  
 
160.  A  35  year  old  man  presents  with  acute  onset  of  red  eye,  pain  and  
blurry  vision.    
               He  has  peri-­‐orbital  edema  and  redness.  What  is  the  best  initial  
management?  
a.  Put  patient  in  a  dark  room  with  reduced  light  
b.  IV  morphine  and  routine  ophthalmological  referral  
c.  IV  morphine  and  emergency  ophthalmological  referral  
d.  IV  morphine  and  antibiotics  
161.  A    5  year  old  child  was  admitted  to  hospital  with  chest  infection.  5  
day  after  admission  he  was  found  to  be  MRSA    positive.  What  is  the  
most    
               appropriate  next  action?  
 

a.  Isolate  the  child  


b.  Take  swab  from  all  hospital  staff  
c.  Close  the  theatre  
d.  Close  the  ward  
e.  Vaccinate  all  hospital  staff  
 
162.  A  17  year  old  girl  took  12  tablets  of  paracetamol.  After  4  hours,  the  
paracetamol    
               levels  are  above  the  treatment  level  curve.  What  will  you  do?    
a.  Observe  and  give  basic  care  
b.  Acetylcysteine  
c.  Discharge  
d.  Refer  to  psychiatrist  
e.  Naloxone  
 
 
163.  A    50  year  old  man  who  was  painting  in  his  bathroom  suddenly  
lost  consciousness.  He  was  brought  to  accident  and  emergency  
department.  What  is  the  single  single  most  likely  diagnosis?  
A. Carbon  monoxide  poisoning  
B. CT  scan  head  
C. MRI  head  
D. Blood  culture  
E. Standing  and  lying  blood  pressure  
 
164.  A  35  year  old  woman  suddenly  fell  and  died.  Post  mortem  shows  
bleeding  from  a  berry    
               aneurysm.  What  type  of  hemorrhage  was  it?  
a.  Extradural  hemorrhage  
b.  Subdural  hemorrhage  
c.  Subarachnoid  hemorrhage  
d.  Intracranial  hemorrhage  
e.  Intraventricular  hemorrhage  
 
165.  A  40  year  old  man  after  an  operation  had  a  heavy  bleeding.  What  
abnormality  will  you  find?  
 

                 a.      Low  sodium  


                 b.      Low  potassium  
                 c.      High  magnesium  
                 d.      High  calcium  
                 e.      Low  calcium  
 
 
 
 
166.  A    35  year  old  woman  has  been  suffering  from  migraine  for  10years  
and  came  to  the  A&E  department  with  severe  headache.  Painkillers  do  
not  help  and  there  is  no    
               history  of  trauma.  What  will  you  do?  
a.  No  treatment  
b.  CT  head  
c.  MRI  head  
d.  Doppler  of  carotid  
e.  Full  blood  count  
 
167.  A  middle  aged  man  develops  constipation  with  overflow  diarrhea  
and  bright      red  bleeding  per  rectum.  Barium  enema  reveals  an  ulcerated  
stricture  in  the    
                 sigmoid  colon.  What  is  the  likely  diagnosis?    
a.    Sigmoid  carcinoma  
b.    Crohns  involvement  of  the  colon  
c.    Ulcerative  colitis  
d.    Ulcerative  pancolitis  
e.    Diverticular  disease  
 
168.  A  40  year  old  patient  presents  with  symptoms  of  depression  and  
mania  and  he  is  about  to    be  started  on  lithium.  Which  investigation  will  
you  do?    
a.    Urea  and  electrolytes  
b.    Parathyroid  hormone  
c.    Full  blood  count  
 

d.    ECG  
e.    ECHO  
 
169.  A  12  year  old  boy  presents  with  vomiting.  On  examination,  he  is  
dehydrated,    
                 has  cold  peripheries  and  deep  breathing.  His  blood  glucose  is  
28mmol/l.  What  is    
                 the  first  step  in  management?  
a.    Give  IV  insulin  
b.    Give  normal  saline  0.9%    
c.    Reassure  the  patient  
d.    Give  normal  saline  9%  
e.    Give  insulin  SC  
 
170.  An  18months  old  child  choked  on  food.  What  will  be  your  next  step?  
                 a.      Helmich  manoeuvre  
                 b.      Back  blows  with  chest  compression  with  child  on  parents  lap  
                 c.      Chest  compression  with  child  on  parents  lap  
                 d.      Induced  vomiting  
171.  An  elderly  man  had  pneumonia  and  then  suffered  myocardial  
infarction  for    
               which  he  was  thrombosed.  Then  he  was  found  to  have  multiple  
mucosal    
               hemorrhages  and  ulcer  15cm  in  diameter  at  the  splenic  flexure  of  
the  colon.    
               What  is  the  cause?  
a.  Thrombosis  
b.  Antibiotics  
c.  Ischemia  
d.  Malignancy  
172.  A  5  year  old  child  ws  involved  in  a  road  traffic  accident  with  her  
parents.  On  examination  she  has  noise  breathing?    What  is  the  
single  most  appropriate    management?  
A. Intravenous  fluid  
B. Secure  airway  
C. CT  scan  head  
 

D. MRI  chest  
E. MRI  brain  
 
 
173.  A  40  year  old  patient  who  has  been  complaining  of  pain  was  given  
paracetamol  and    ibuprofen  but  is  still  complaining  of  pain.  What  drug  
should  be  given  to  this    
                 patient  to  ease  his  symptoms?  
a.      Codydramol  
b.      Codeine  phosphate  
c.      Morphine  orally  
d.      Tramadol  
e.      IV  morphine  
174.      A  4  year  old  child  develops  chicken  pox.  The  parents  brings  her  
to  hospital  .  she  is  adviced  that  no  treatment  is  required.  The  
mother  says  when  can  the  child  go  back  to  school?  
A. When  rush  crusted  over  
B. After  2  days  
C.          All  the  times  
D.      Never  
E.            Intermittently  
 
 
175.      A  45  year  old  man  who  is  on  multiple  medication  presents  with  
tremors  of  the  hands  and  difficulty  to  initiate  movement.  Which  
of  the  following  medication  is  the  most  likely  cause?  
A. Anti-­‐depressant  
B. Anti-­‐epileptic  medication  
C. Anti-­‐psychotic  
D. Aspirin  
E. Simvastatin  
 
176.    A  45  year  old  man  travelled  to  Thailand  for  a  holiday  and  returned  
a  few  days    
               ago.  He  now  presents  with  fever  and  cough.  On  examination  he  has  
cervical    
               lymphadenopathy.  What  is  the  most  likely  diagnosis?  
a.  Estein  Barr  Virus  
 

b.  Tuberculosis  
c.  Herpes  simplex  
d.  Human  immunodeficiency  syndrome  
 
177.  A  70  year  old  lady  who  has  been  on  treatment  who  has  been  
treated  with    
               carbamezepine  for  aplastic  anemia.  She  now  presents  with  
shortness  of  breath    
               and  light  headedness.  Blood  results  show  MCV  70,  Hb  8.5,  WBC  2.0,  
               Neutrophils  1,  Platelets    50.    
a.  Hemorrhage  anaemia  
b.  Aplastic  anemia  
c.  Hereditary  spherocytosis  
d.  Pernicious  anaemia  
e.  Folate  deficiency  anemia  
 
178.  A  60  year  lady  fell  down  and  sustained  neck  of  femur  fracture.  
Which  artery  is  she  likely  to  bleed  from?  
a.  Femoral  artery  
b.  Circumferential  artery  
c.  Popliteal  
d.  Common  iliac  artery  
e.  External  iliac  artery  
 
179.  A  45  year  old  man  had  developed  lower  back  pain  radiating  down  
the  posterior    
                 aspect    of  his    legs  down  to  his  knees.  What  is  the  most  likely  
diagnosis?  
a.    Spinal  cord  compression  
b.    Disc  proplapse  
c.    Spine  malignancy  
d.    Tuberculosis  
e.    Musculoskeletal  pain.  
 
 
 

180.    An  11  year  old  girl  presents  with  meningococcocaemia.    


Intravenous  line  has  failed.  What  is  the  single  most  appropriate  
management?  
A. Intravenous  central  line  insertion  
B. Intravenous  osseous  
C. NGT  tube  
D. Subcutaneous  fluids  
E. Intramuscular    fluids  
 
181.  A  55  year  old  man  has  weight  loss,  dyspnoea  and  syncope.  He  
smokes  20    
               cigarettes  a  day.  Investigation  confirms  a  squamous  cell  carcinoma  
in  the  left    
               bronchus.  What  is  the  single  most  likely  biochemical  abnormality?  
a.  Hypercalcemia  
b.  Hyperkalemia  
c.  Hypernatremia  
d.  Hypocalcemia  
e.  Hypomagnesemia  
 
182.  A  25  year  old  female  has  a  blackout  while  watching  a  disco  show  on  
TV  with  the    
               strobe  lights  on.  What  is  the  most  likely  diagnosis?  
a.  Vasovagal  syncope  
b.  Stoke-­‐Adam’s  attack  
c.  Anxiety  
d.  Epilepsy  
e.  Hypoglycemia  
 
183.  A  middle  aged  lady  was  laying  the  bed  sheet  when  her  index  finger  
got  stuck  in    
               between  the  bed  and  mattress.  She  is  unable  to  extend  the  
proximal    
               interphalangeal  joint.  What  is  the  most  appropriate  management?    
a.  Splint  the  finger  
b.  Twin  plaster  the  finger  
 

c.  Cast  
d.  Sling  
e.  Open  reduction  and  internal  fixation  
 
184.  A  26  year  old  woman  has  been  promoted  at  work  to  a  job  that  
requires  a  lot  of    
               public  speaking.  She  is  worried  and  anxious  about  it  and  feels  that  
she  would    
               faint.  What  is  the  single  most  useful  intervention?  
a.  Cognitive  behavioural  therapy  
b.  Diazepam  
c.  Fluoxetine  
d.  Psychodynamic  psychotherapy  
e.  Relaxation  therapy  
 
 
 
 
185.  A  lady  has  developed  breast  abscess  after  delivery.  What  is  the  
most  likely    
                 causative  organism?  
a.    Staphylococcus  aureus  
b.    Staphylococcus  albus  
c.    Streptococcus  pyogenes  
d.    Streptococcus  faecalis  
e.    E.coli  
 
186.  An  old  man  suffers  from  end  stage  prostatic  cancer.  He  has  pain,  
anuria  and  bilateral  hydronephrosis.  His  daughter’s  wedding  is  in  3  
weeks  time  and  he  is  looking  forward  to  enjoying  the  ceremony.  What  is  
the  most  appropriate    
               palliative  treatment  for  him?  
a.  Bilateral  nephrostomy  
b.  Palliative  chemotherapy  
c.  Catheterization  
 

d.  Palliative  radiotherapy  
 
187.  A  10  year  old  child  has  got  progressive  bilateral  hearing  loss.  He  has  
started  to    
               increase  the  television  volume.  All  other  examinations  are  normal.  
What  is  the    
               most  likely  diagnosis?  
a.  Bilateral  otitis  media  with  effusion  
b.  Nasopharyngeal  carcinoma  
c.  Presbyacusis  
d.  Noise  induced  hearing  loss  
 
188.    A  34  year  old  man  present  with  mass  in  the  testis  which  has  
been  found  to    malignant.  To  which  of  the  following  lymph  nodes  
will  it  metastasis  first?  
A. Para-­‐aortic    
B. Internal  iliac  
C. Common  Iliac  
D. Superficial  inguinal  
E. Deep  inguinal.  
 
 
189.  Which  of  the  flowing  results  establish  a  diagnosis  of  diabetes  
mellitus?  
                 a.      Asymptomatic  patient  with  fasting  glucose  7.9mmol/l  on  one  
occasion.  
                 b.      Symptomatic  patient  with  fasting  glucose  6.8mmol/l  on  2  
occasions.  
                 c.      Asymptomatic  patient  with  random  glucose  22.0mmol/l  on  1  
occasion.  
                 d.      Symptomatic  patient  with  random  glucose  12.0mmol/l  on  1  
occasion.  
 
 
 
 
 

190.  A  45  year  old  man  presents  with  bi-­‐temporal  hemianopia  and  spade  
like  hands.    
                 What  is  the  definitive  test  to  confirm  the  diagnosis?  
a.    Early  morning  growth  hormone  
b.    Insulin  tolerance  test  
c.    OGTT  with  growth  hormone  measurements  
d.    Random  insulin  growth  hormone  measurements  
e.    Short  ACTH  test  
 
191.  A  26  year  old  man  with  history  of  hereditary  hemorrhagic  
telangiectasia  is  planning  to  start  a  family.  What  will  the  mode  of  
inheritance?  
a.    Autosomal  dominant  with  incomplete  penetrance  
b.    Autosomal  co-­‐dominant  
c.    Autosomal  recessive  with  incomplete  penetrance  
d.    Autosomal  dominant  
e.    Autosomal  recessive  
 

192.    A  67  year  old  man  was  diagnosed  with  cancer  of  the  
oesophagus  with  metastasis  to  the  liver  .  What  is  the  single  most  
appropriate    management?  
A. Chemotherapy  
B. Radiotherapy  
C. Resection  
D. Stenting  
E. Chemotherapy  and  radiotherapy.  
 
 
193.  A  34  year  old  man  with  multiple  sclerosis  has  taken  an  overdose  of  
100  paracetamol  tablets  with  intent  to  end  his  life.  He  has  been  brought  
to  the  A&E      for  treatment  but  is  still  refusing  interventions.  What  is  the  
most  appropriate    
                 management?    
a.    Assessment  of  suicide  
 

b.    Evaluate  patient’s  capacity  to  refuse  treatment  


c.    Establish  if  patient  has  a  previous  mental  illness  
d. Respect  her  wish  
 
194.      A  45  year  old  man  presents  with  malaise  and  weakness.  
Chest  X-­‐ray  shows  bilateral  shadowing  on  the  lung  bases.  
What  is  the  single  most  likely  diagnosis?  
A. legionella  
B. Mycoplasma  
C. Staphyllococus  
D. Pnumocysti  carinii  
E. Chlamydial  pneumonia  
195.  A  62  year  old  man  who  was  admitted  for  minor  surgery  3  ago  
suddenly      becomes  confused.  His  attention  span  is  reduced.  He  is  
restless  and  physically  aggressive  and  picks  at  his  bed  sheets.  What  is  
the  single  aspect  of  the  patient’s  history  recovered  in  his  notes  is  most  
likely  to  aid  in  making  a  diagnosis?    
a.    Alcohol  consumption  
b.    Head  trauma  
c.    History  of  anxiety  
d.    Prescribes  medication  
e.    Obvious  cognitive  impairment  
 
196.  A  34  year  old  man  from  Zimbabwe  is  admitted  with  abdominal  pain  
to  the  A&E.    An  abdominal  X-­‐ray  reveals  bladder  calcification.  What  is  
the  most  likely  cause?  
a.    Schistosomiasis  mansoni  
b.      Sarcoidosis  
c.      Leishmaniasis  
d.      Tuberculosis  
e.      Schistosomiasis  haematobium  
197.  A  woman  gives  birth  vaginally  to  a  term  baby  boy.  She  has  hepatitis  
B  and  is  concerned  about  the  risk  of  transmission.  What  is  the  most  
appropriate    
               management?    
a.  Hepatitis  B  vaccine  
 

b.  Hepatitis  B  antibody  test  


c.  Hepatitis  B  vaccine  +  Ig  
d.  Hepatitis  B  Ig  only  
e.  No  management  
 
198.  A  70  year  old  man  with  few  month  history  of  weight  loss  and  
dysphagia  presented  with  sandwich  stuck  in  his  throat.  What  is  the  most  
appropriate  next  step?  
a.  Barium  meal  
b.  Barium  swallow  
c.  Endoscopy  foreign  body  removal  and  biopsy  
d.  USG  
e. Chest  X-­‐ray  
 
199.  A  50  year  old  lady  with  weak  limbs  when  examined  was  found  to  
have  burn  marks  on  her  finger  tips,  is  wasted  and  has  weak  hand  with  
diminished  reflexes.  She  also  has  weak  spastic  legs  and  dissociated  
sensory  loss.  What  is  the    
               diagnosis?  
a.  Multiple  sclerosis  
b.  Syringomyelia  
c.  Motor  neuron  disease  
d.  Guillian  Barre  syndrome  
e.  Freidrieches  ataxia  
 
200.  A  56  year  old  man  was  recently  put  on  anti-­‐hypertensive  
medication  and  recent        
                 blood  biochemistry  on  2  occasions  show  the  following  results;  
                 Na:  132,    K:  7.6,    Urea:  11.3,    Creatinine:  112.  
                 Which  of  the  following  drugs  is  responsible  for  the  above  results?  
a.      Amlodipine  
b.      Bendroflumethiazide  
c.      Doxazosin  
d.      Atenolol  
e.      Ramipril  
 

   
SAMSONPLB  ACADEMY  LIMITED  
BOW  BUSSINES  CENTRE  
BOW  ROAD  153-­‐159  
E3  2SE  
CONTACT:  07940433068  
EMAIL:  info@samsonplab.co.uk  
 
MOCK  5  ANSWERS  
 
1.    A  
2B    
3E    
4B  EAR  DROP  GENTAMICIN    
5B  
6D    
7A    
8B    
9A    
10B    
11D    
12  NO  QUESTION    
13B    
14A    
15D    
16E    
17A    
18A    
19C    
20B    
21B    
22B    
23E    
24D    
25B    
26B    
27A    
28D    
29A    
30A    
31B    
32A    
33A    
34C    
35  NO  QUESTION  35    
36    E    
37B    
38A    

 
 

39C    
40B    
41A    
42A    
43B    
44D  
45C    
46B    
47A    
48C    
49E    
5OB    
51E    
52D    
53B    
54D    
55B    
56A    
57D    
58B    
59C    
60B    
61A    
62C    
63C    
64B    
65E    
66B    
67A    
68A    
69A    
70A    
71B    
72C    
73B    
74A    
75B    
76A    
77A    
78A    
79B    
80D    
 
 
81B    
82A    
83C    

 
 

84B    
85B    
86A    
87E    
88C    
89C    
90B    
91A    
92D    
93D    
94C    
95C    
96B    
97A    
98B    
99B    
100A    
101B    
102A    
103A    
104  INCOMPLETE  QUESTION    
105B    
106A    
107A    
108C    
109C    
110B    
111B    
112C    
113A    
114C    
115C    
116B    
117A    
118B    
119A    
120B    
121B    
122A    
123A    
124C    
125A    
126B    
127A    
128A    
129A    
130B    

 
 

131B    
132A    
133A    
134A    
135A    
136C    
137C    
138E    
139B    
140  INCOMPLETE  QUESTION    
141A    
142E    
143INCOMPLETE  QUESTION    
144C    
145E    
146A    
147E    
148A    
149B    
150C    
151B    
152B    
153A    
154A  
155B    
156A    
157A    
158A    
159D    
160C    
161C    
162B    
163A    
164C    
165B    
166B    
167A    
168A    
169B    
170B    
171C    
172B    
173B    
174A    
175C    
176A    
177B    

 
 

178B    
179B    
180B    
181A    
182D    
183A    
184A    
185A    
186C    
187A    
188A    
189D    
190C    
191D    
192B    
193B    
194C    
195A    
196E    
197C    
198C    
199B    
200E    
 
 
 
 
 

 
 

SAMSONPLAB  ACADEMY  LIMITED  


BOW  BUSSINESS  CENTRE  
BOW  ROAD  153-­‐159  
E3  2SE,  LONDON  
EMAIL:  info@samsonplab.co.uk  
Telephone:  07940433068  
 
 
       SAMSONPLAB  ACADEMY  PLAB  1  MOCK  6        10th  JUNE  2013  
 
1. A 30 year old lady would like to know if her baby will have down syndrome as
she her first child has down syndrome. She has decided to go for amniocentesis .
When would you perform amniocentesis?
a. 10-13 weeks
b. 15-18 weeks
c. 20-24 weeks
d. 8-12 weeks
e. 7-9weeks
 
2.  A  7  year  old  boy  ingests  his  sister’s  methadone  and  was  found  unconscious.  He    
       was  given  naloxone  and  improved.  After  some  time  he  is  found  unconscious  again.    
       What  is  the  cause  of  the  second  unconscious  episode?  
a. Naloxone  is  weaker  than  methadone  
b. He  took  something  else  apart  from  naloxone  
c. Naloxone  is  a  partial  antagonist  
d. Naloxone  has  a  short  acting  duration  compared  to  methadone  
 
3. A  70  year  old  man  is  presents  with  weight  loss.  On  examination  there  is  
abdominal  pain  and  masses  palpable.  Per  rectal  examination  is  normal  with  
no  is  blood.  But  occult  blood  is  positive.  Where  is  the  lesion  most  likely  
diagnosis?  
A. Rectum  
B. Transverse  colon  
C. Descending  colon  
D. Caecum  
E. Sigmoid  colon.  
 
4.  A  30  year  old  lady  who  is  in  her  32  weeks  of  her  pregnancy  is  found  to  have  a  
blood  pressure  of  165/105.  What  is  the  single  most  appropriate  treatment?  
A. Hydralazine  
B. ACE  inhibitor  
C. Labetalol  
D. Thiazide  diuretics  
 

E. Loop  diuretics  
 
5.  A  12  year  old  boy  presents  with  testicular  pain  without  a  history  of  trauma.  He  Is    
       afebrile  and  on  examination,  the  right  testis  is  slightly  higher  that  the  left  one.          
       What  is  the  diagnosis?  
a. Testicular  torsion  
b. Epidydymo-­‐orchitis  
c. Teratoma  
d. Seminoma  
e. Varicocele  
 
6.  A  59  year  old  post  menopausal  woman  present  with  a  month  history  of  bleeding    
       per  vaginum.  On  speculum  examination,  the  cervix  appears  normal  and  there  is  no    
       vaginal  discharge.  What  is  the  single  most  appropriate  diagnostic  tests?  
a. Cervical  smear  
b. Hysteroscopy  
c. Pelvic  ultrasound    
d. Serum  FSH  level  
e. Transvaginal  ultrasound  with  endometrial  sampling  
 
7.  An  80  year  old  woman  is  brought  in  by  her  family  as  they  are  concerned  that  she’s    
       been  getting  progressively  more  confused  over  the  past  few  months.  She  is    
       disoriented  to  time,  place  and  person  and  often  wanders  in  the  streets  aimlessly.  
       What  is  the  single  most  likely  diagnosis?  
a. Alzheimer’s  
b. Delirium  
c. Parkinson’s  
d. Subdural  hematoma  
e. Vascular  dementia  
 
8.  A  32  year  old  woman  has  increasing  white  vaginal  discharge.  She  is  7  weeks    
       pregnant.  Her  Chlamydia  swab  is  positive.  All  other  tests  are  normal.  What  is  the    
       single  most  appropriate  treatment?  
a. Amoxicillin  
b. Clindamycin  
c. Doxicycline  
d. Erythromycin  
e. Metronidazole  
 
9.  A  2  year  old  is  brought  in  by  his  parents  as  they  are  concerned  that  he  has  a  past    
       history  of  febrile  convulsions.  His  last  convulsion  was  6  months.  There  is  also  a    
 

       family  history  of  epilepsy.  What  is  the  single  most  appropriate  management?  
a. Arrange  EEG  
b. Arrange  MRI  
c. Arrange  sleep  studies  
d. Commence  regular  lorazepam  
e. Educate  parents  in  when  and  how  to  administer  rectal  diazepam  
 
10.  A  50  year  old  man  presents  with  right  sided  hemiparesis.  There  is  a  bruit  on  neck    
           auscultation.  What  is  the  single  most  appropriate  investigation?  
a. Blood  pressure  measurement  
b. Carotid  Doppler  
c. CT  head  
d. MRI  
e. Slit  lamp  examination  
 
11.  A  60  year  old  woman  is  recovering  from  an  operation  following  a  hysterectomy    
           and  anterior  vaginal  repair.  She  has  had  voiding  difficulty  and  has  been    
           catheterized  for  3  days.  A  catheter  specimen  of  urine  is  taken  due  to  a  low  grade    
           pyrexia  and  confirms  a  UTI.  Which  organism  is  the  causative  agent?  
a. E.coli  
b. Klebsiella  pneumonia  
c. Proteus  
d. Pseudomonas  
e. Staphylococcus  epidermidis  
 
12.  A  17  year  old  man  presents  to  the  A&E  department  with  a  traumatic  hip.  On    
           examination,  he  has  a  foot  drop  and  is  unable  to  dorsiflex  or  evert  over  his  left    
           ankle.  Sensation  is  lost  over  the  front  and  lateral  side  of  the  leg  and  dorsum  of    
           the  foot.  Which  nerve  is  most  likely  damaged?    
a. Common  peroneal  nerve  
b. Femoral  nerve  
c. Sciatic  nerve  
d. Superficial  peroneal  nerve  
e. Tibial  nerve  
 
13.  A  22  month  old  girl  has  severe  dehydration.  You  are  unable  to  gain  peripheral    
           venous  access.  What  is  the  single  most  appropriate  management?  
a. Administer  subcutaneous  fluids  
b. Arrange  for  a  PEG  
c. Inset  a  central  line  
d. Insert  an  NG  tube  
 

e. Intraosseous  fluid  injection  


 
14.  A  55  year  old  man  presents  with  sudden  onset  of  lower  back  pain.  Loss  of    
           perineal  sensation  and  urinary  incontinence.  What  is  the  single  most  appropriate    
           management?  
a. Admit  to  hospital  
b. Give  paracetamol  and  keep  active  
c. Strict  bed  rest  
d. Skeletal  traction  
e. Urgent  spinal  cord  decompression  
 
15.  A  24  year  old  woman  has  regular  painful  uterine  contraction  at  26  weeks    
           gestation.  She  is  2cm  dilated.  Her  membranes  are  intact.  The  CTG  is  reassuring.    
           What  is  the  single  most  appropriate  management  plan?      
a. Admit  and  administer  analgesics  and  syntocinon  
b. Admit  and  administer  antibiotics  and  intramuscular  steroids  
c. Admit  and  administer  antibiotics  and  tocolytics  
d. Admit  and  administer  tocolytics  and  intramuscular  steroids  
e. Reassure  and  send  home  
 
16.  A  45  year  old  businessman  returns  from  Nigeria  with  fever,  confusion  and  rigors.    
           What  is  the  single  investigation  most  likely  to  give  a  definitive  diagnosis?  
a. Blood  culture  
b. Blood  film  for  parasites  
c. CT  head  
d. 12  lead  ECG  
e. Urine  toxicology  screen  
 
17.  A  30  year  old  pregnant  woman  presents  with  numbness  and  tingling  in  her  right        
           hand  involving  the  lateral  3  fingers.  What  is  the  single  most  appropriate    
           treatment?  
a. Carpal  tunnel  release  surgery  
b. Local  steroid  injection  
c. NSAID’s  
d. Paracetamol  
e. Splinting  
 
18.  A  22  year  old  man  sustains  a  femoral  shaft  fracture  while  rollerblading  and  is    
             brought  to  the  A&E  confused  and  agitated.  What  is  the  single  most  likely    
           diagnosis?  
a. Epidural  hematoma  
 

b. Fat  embolism  
c. Hypovolemic  shock  
d. Pericardial  effusion  
e. Pulmonary  embolism  
 
19.  A  30  year  old  woman  reports  with  palpitations  at  rest.  The  resting  pulse  is  70b/m    
           and  regular.  Holter  ECG  monitoring  shows  irregularities  associated  with  the    
           palpitations.  What  is  the  single  most  likely  diagnosis?  
a. Atrial  fibrillation  
b. Atrial  flutter  
c. Heart  block  
d. SVT  
e. Ventricular  ectopic  
 
20.  A  55  year  old  man  complains  of  pleuritic  chest  pain  5  days  after  an  MI  and  is    
           found  to  have  widespread  ST  elevation  on  ECG.  Pain  is  worse  with  movement.    
           What  is  the  single  most  likely  diagnosis?  
a. Acute  pericarditis  
b. Acute  pulmonary  embolus  
c. Dissecting  aortic  aneurysm  
d. Dressler’s  syndrome  
e. Ventricular  rupture  
 
21.  A  35  year  mother  of  4  reports  urinary  incontinence.  Urometry  suggests  stress    
           incontinence.  What  is  the  single  most  appropriate  form  of  treatment?  
a. Bladder  neck  surgery  
b. Oxybutynin  
c. Pelvic  floor  exercises  
d. Ring  pessary  
e. Topical  vaginal  oestrogens  
 
22.  A  15  year  old  boy  presents  to  the  A&E  with  severe  asthma.  He  is  using  his    
           accessory  muscles  of  respiration  and  is  unable  to  speak  in  sentences.  What  is  the    
           single  most  appropriate  treatment?  
a. Inhaled  short  acting  B-­‐2  agonist  
b. Inhaled  long  acting  B-­‐2  agonist  
c. IV  aminophylline  
d. IV  hydrocortisone  
e. Nebulised  short  acting  B-­‐2  agonist  
 
 
 

 
 
23.  A  40  year  old  man  presents  with  bleeding  per  rectum,  constipation  and  diarrhea.    
           Sigmoidoscopy  reveals  an  ulcerated  stricture  in  the  sigmoid  colon.  What  is  the      
           single  most  likely  diagnosis?  
a. Angiodysplasia  
b. Crohn’s  disease  
c. Diverticular  disease  
d. Sigmoid  colon  cancer  
e. Ulcerative  colitis  
 
24.  A  27  year  old  man  has  had  mild  dysuria  for  1  week.  He  has  been  having  sex  with    
           his  current  girlfriend  for  4  weeks,  occasionally  using  condoms.  She  has  no    
           symptoms.  His  last  sex  with  his  previous  female  partner  was  3  months  ago.  There    
           is  a  slight  mucoid  discharge  at  the  urethral  meatus.    
           Which  single  organism  is  the  most  likely  cause?  
a. Chlamydia  
b. Mycoplasma  hominis  
c. Neisserria  gonorrhoea    
d. Trichomonas  vaginalis  
e. Ureaplasma  urealyticum  
 
25.  A  32  year  old  man  has  weight  loss  and  general  malaise.  He  takes  an  HIV  test    
           which  is  positive  and  his  CD4  count  is  180x10/L  (12%)  Normal  range  is  450-­‐  
           1600x10/L.  He  is  otherwise  well.  He  does  not  feel  ready  to  start  antiretroviral    
           therapy  straight  away,  but  wishes  to  stay  well  in  the  interim.    
           For  which  single  organism  should  he  be  offered  primary  prophylaxis?    
a. Cryptococcus  neoformans  
b. Mycobacterium  avium  intracellulare  
c. Mycobacterium  tuberculosis  
d. Pneumocystitis  jiroveci  
e. Toxoplasma  gondii  
 
26.  A  21  year  old  university  student  is  just  back  from  a  hiking  holiday  in    
           Massachusetts  and  presents  with  bilateral  facial  palsy  and  a  raised  pink  annular    
           lesion  on  her  lower  leg.  She  says  this  was  all  preceded  by  flu-­‐like  symptoms.    
           What  is  the  single  most  likely  diagnosis?  
a. Erythema  multiforme  
b. Erythema  nodosum  
c. Herpes  zoster  
d. Infected  insect  bite  
 

e. Lyme  disease  
 
27.  A  15  year  old  girl  with  neural  tube  defect  is  brought  in  by  her  mother  for  advice    
           on  urinary  incontinence.  What  is  the  single  most  appropriate  management?  
a. Desmopressin  
b. Incontinence  pads  
c. Intermittent  self-­‐catheterisation  
d. Oxybutinin  
e. Prophylactic  trimethoprim  
 
28.  A  38  year  old  married  working  mother  of  3  asks  for  contraception.  She  states    
           that  she  had  a  pill  failure  in  the  past  and  also  has  a  history  of  heavy  periods.  
           What  is  the  single  most  appropriate  form  of  contraception?  
a. Bilateral  tubal  ligation  
b. Condoms  
c. Depoprovera  injection  
d. Mirena  coil  
e. Norethisterone  POP  
 
29.  A  56  year  old  NIDDM  male  presents  with  persistently  elevated  BP  of    
           180/110mmHg.  Blood  tests  are  unremarkable.  
           What  is  the  single  most  appropriate  medication?  
a. Amlodipine  
b. Atenolol  
c. Bendroflumethiazide  
d. Furosemide  
e. Perindopil  
 
30.  A  40  year  old  man  returns  from  Africa  with  a  high  swinging  pyrexia  and  diarrhea.    
           On  examination,  there  is  tenderness  in  the  right  upper  quadrant  of  the  abdomen.    
           What  is  the  single  most  likely  diagnosis?  
a. Acute  cholecystitis  
b. Acute  hepatitis  
c. Amoebic  liver  abscess  
d. Falciparum  malaria  
e. Salmonella  poisoning  
 
31.  A  35  year  old  male  heavy  smoker  states  that  his  right  leg  toe  becomes  pale  and    
           painful  when  walking.  What  is  the  single  most  likely  diagnosis?  
a. Embolism  
b. Gout  
 

c. Peripheral  atherosclerosis  
d. Raynauld’s  phenomenon  
e. Thromboangitis  obliterans  
 
32.  A  55  year  old  male  presents  with  TIA  and  is  found  to  have  a  pulse  rate  of  100b/m    
           with  irregular  rhythm.  What  is  the  single  most  appropriate  treatment?    
a. Aspirin  
b. DC  cardioversion  
c. Heparin  
d. Streptokinase  
e. Warfarin  
 
33.  A  40  year  old  man  with  past  history  of  total  thyroidectomy    presents  with  
dizziness.  Blood  tests  shows  Hb10.2,  MCV  110,  What  is  the  single  most  
appropriate  investigation?  
A. Serum  folate  levels  
B. Serum  B12  
C. Thyroid  function  tests  
D. Upper  GI  endoscopy  
E. Ultrasound  scan  of  the  neck  
 
 
34.  A  36  year  old  naval  seaman  presents  with  a  rash  on  his  forehead.  He  states  that      
           he  has  had  the  rash  for  years  and  often  forgets  to  wear  his  cap  while  he  is  on    
           duty.  What  is  the  most  likely  diagnosis?  
a. Actinic  keratosis  
b. Acne  vulgaris  
c. Keratoacanthoma  
d. Lentigo  maligma  
e. Squamous  cell  carcinoma  
 
35.  A  30  year  old  pedestrian  was  involved  in  an  RTA  and  presents  with  loss  of    
           consciousness  in  the  A&E  department.  He  regains  consciousness  and  complains    
           of  headache.  He  also  has  a  stiff  neck  on  examination  with  a  deep  laceration  to  his    
           right  temple.  What  is  the  likely  diagnosis?      
a. Basal  skull  fracture  
b. Cerebral  haemorrhage  
c. Extradural  haemorrhage  
d. Subarachnoid  haemorrhage  
e. Subdural  hematoma  
 
 
 

 
 
 
36.  A  16  year  old  girl  has  painful  periods  for  6  months.  Her  periods  are  regular  and    
             lasts  for  3  days.  She  misses  a  couple  of  days  of  school  every  month  due  to  the    
           pain.   She   is   not   sexually   active.   What   is   the   most   appropriate   initial   management?  
a. COCP  
b. Gonadotrophin-­‐releasing  hormone  analogues  
c. Intra-­‐uterine  system  (Mirena)  
d. Mefanemic  acid  
e. Tranexamic  acid  
 
37.  A  20  year  old  cyclist  is  hit  by  an  oncoming  vehicle  and  now  presents  with  loss  of    
             consciousness.  On  examination,  there  is  blood  behind  the  ear  drum  and  peri-­‐  
             orbital  ecchymoses.  What  is  the  single  most  likely  diagnosis?      
a. Basilar  skull  fracture  
b. Extradural  hematoma  
c. Linear  skull  fracture  
d. Subarachnoid  hematoma  
e. Subdural  hematoma  
 
38.  A  35  year  old  woman  complains  of  severe  pain  on  defecation.  She  is  unable  to      
             tolerate  digital  rectal  examination.  She  also  has  a  history  of  constipation.  
             What  is  the  single  most  likely  diagnosis?  
a. Anal  fissure  
b. Crohn’s  disease  
c. Haemorrhoids  
d. Perianal  hematoma  
e. Pilonidal  cyst  
 
39.  A  70  year  old  woman  being  treated  for  temporal  arteritis  now  presents  with    
           fever,  severe  abdominal  pain  and  vomiting.    
           What  is  the  single  most  likely  diagnosis?    
a. Acute  cholecystitis  
b. Acute  pancreatitis  
c. Gallstone  peritonitis  
d. Perforated  peptic  ulcer  
e. Ruptured  aortic  aneurysm  
 
40.  A  55  year  old  man  presents  with  prostatic  symptoms.  On  digital  rectal    
             examination,  you  palpate  a  firm  nodule  on  the  prostate.  
 

             What  is  the  next  single  most  appropriate  nest  step  investigation?  
a. Acid  phosphatase  
b. IVU  
c. Scinti  scan  
d. Serum  PSA  levels  
e. Transrectal  USS  and  biopsy  
41.  A    45  year  old  man  was  admitted  for  the  treatment  of  alcohol  withdrawal  He  
has  been  treated  successfully  but  now  he  has  ataxia  and  ophthalmoplegia.  
What  is  the  most  appropriate  treatment?  
A. Oral  thiamine  
B. Intravenous  vitamin  B  complex  
C. Acamproste  
D. Disulfiram  
E. Intravenous  fluids  only  
 
 
42.  A  15  year  old  girl  is  brought  in  by  her  family  as  they  are  concerned  that  she  is    
           anorexic  and  depressed.  She  has  no  suicidal  ideation.    
           What  is  the  single  most  appropriate  treatment?  
a. Commence  fluoxetine  
b. CBT  
c. Refer  to  rapid  access  mental  health  team  
d. Section  and  admit  to  the  hospital  
e. Suggest  family  counseling  
 
43.  A  30  year  old  man  with  ankylosing  spondylitis  now  presents  with  eye  pain  and    
             redness.  What  is  the  single  most  likely  diagnosis?  
a. Acute  angle  closure  glaucoma  
b. Acute  conjunctivitis  
c. Herpetic  ulcer  
d. Iritis  
e. Scleritis  
 
 
44.  A  10  year  old  boy  has  shortness  of  breath  and  wheezing  when  playing  football.  
             What  is  the  single  most  appropriate  treatment?  
a. Inhaled  steroids  
b. Nebulised  salbutamol  
c. Sodium  chromoglycate  
d. Trial  of  leukotrene  receptor  antagonist  
e. Trial  of  short  acting  B-­‐2  agonis  
 
 

 
45.  A  60  year  old  man  presents  with  severe  shortness  of  breath  with  basal    
             crepitations  on  auscultation.  Blood  pressure  is  160/110mmHg  with  a  pulse  of    
             100b/m.  Chest  x-­‐ray  confirms  suspicions.  
             What  is  the  single  most  appropriate  treatment?  
a.  Amlodipine  
b.  Bendroflumethiazide  
c.  Furosemide  
d.  Nifedipine  
e.  Spironolactone  
 
46.  A  25  year  old  man  complains  of  bouts  of  alternating  intermittent  diarrhea  and    
           constipation  for  the  past  1  month.  He  describes  a  sensation  of  incomplete    
           evacuation  and  bloating.  Symptoms  are  relieved  by  defecation.  Sigmoidoscopy  is    
           normal.  What  is  the  single  most  likely  diagnosis?  
a. Diverticulitis  
b. Gastroenteritis  
c. Irritable  bowel  syndrome  
d. Pseudomembranous  colitis  
e. Ulcerative  colitis  
 
47.  A  50  year  old  man  post  operatively  for  cholecystectomy  presents  with  fever,    
             jaundiced  and  dark  urine.  What  is  the  single  most  definitive  management?    
a. Abdominal  USS  
b. CT  scan  
c. ERCP  
d. Laparatomy  
e. Percutaneous  transhepatic  cholangiography  
 
48.  A  26  year  old  man  with  NIDDM  and  a  history  of  haemochromatosis  now  presents    
           with  jaundice.  What  is  the  single  most  definitive  investigation?  
a. Abdominal  USS  
b. ERCP  
c. Liver  biopsy  
d. Serum  ferritin  levels  
e. Viral  hepatitis  serology  
 
49.  A  56  year  old  man  with  new  onset  dyspepsia  now  reports  with  no  relief  with    
             antacids.  What  is  the  single  most  appropriate  management?  
a. Barium  meal  
b. Urgent  endoscopy  
 

c. Test  for  H.pylori  


d. Lansoprazole  trial  
e. 24  hours  ambulatory  Ph  monitoring  
 
50.  A  69  year  old  man  with  history  of  intermittent  claudication  now  reports  with    
             severe  back  pain.  On  examination  the  blood  pressure  is  90/50mmHg.  
             What  is  the  single  most  definitive  investigation?  
a. Abdominal  USS  
b. Chest  X-­‐ray  
c. 12  lead  ECG  
d. Spiral  CT  
e. MRI  spine  
 
51.  A  30  year  old  man  presents  with  bloody  diarrhea  and  mucus.  Histology  shows    
             crypt  abscesses  and  granular  inflammatory  mucosa.  Examination  is  
unremarkable.  
             What  is  the  single  most  likely  diagnosis?  
a. Angiodysplasia  
b. Crohn’s  disease  
c. Diverticulitis  
d. IBS  
e. Ulcerative  colitis  
 
52.  A  50  year  old  man  is  brought  to  the  A&E  after  overdosing  on  his  pills.  No  label  is    
             found  on  the  bottle.  He  is  noted  to  be  in  heart  block  and  has  a  palpable  bladder    
             and  dilated  pupils.  What  is  the  single  most  likely  culprit?    
a. Coproxamol  
b. Fluoxetine  
c. Quinine  
d. TCA  
e. Venlafaxine  
 
53.  An  80  year  old  woman  notes  that  she  had  trouble  putting  up  her  curtains  at  night.    
             She  states  that  she  has  loss  of  balance  and  repeated  falls.  
             What  is  the  single  most  definitive  investigation?  
a. Cervical  X-­‐ray  
b. Advise  about  posture  
c. Hearing  test  
d. Measure  intraocular  pressure  
e. Visual  acuity  
 
 

 
 
54.  A  1  year  old  child  develops  swollen  lips  after  eating  peanuts.    
             What  is  the  single  most  appropriate  management?  
a. IM  Adrenaline  
b. IV  Hydrocortisone  
c. IV  Chlorphenamine  
d. Urgent  admission  
e. Oral  antihistamine  
 
55.  A  30  year  old  woman  develops  bilateral  conductive  hearing  loss  after  giving  birth.    
             On  examination,  the  tympanic  membranes  appear  normal.  
             What  is  the  single  most  likely  diagnosis?  
a. Acoustic  neuroma  
b. Acute  otitis  media  
c. Gentamicin  toxicity  
d. Meniere’s  disease  
e. Otosclerosis  
55.  A  2  year  old  boy  presents  with  gradual  swelling  of  both  feet.  Even  if  his  
feeding  has  been  poor  recently  he  has  gained  weight.  His  urine  is  dark  in  
colour.  What  is  the  single  most  appropriate  investigation?  
A. calcium  
B. Serum  albumin  
C. Serum  urea  and  creatinine  
D. Glucose  
E. Liver  function  tests  
 
 
56.  A  young  girl  presents  with  crusted  lesion  on  her  neck  which  has  spread  from  the    
           neck  to  her  face.  She  is  otherwise  normal.  Which  investigation  will  establish  the    
           diagnosis?  
a. Viral  culture  
b. Bacterial  culture  
c. Skin  biopsy  
d. Temperature  
e. ABG  
 
57.  A  coal  miner  presents  with  a  diagnosis  of  transitional  cell  cancer  of  the  bladder  
             What  is  the  single  most  important  risk  factor  for  his  condition?  
a. Coalmine  
b. Smoking  
c. Family  history  
 

d. Vitamin  
e. Alcohol  
a. Metoprolol  
59.  A  patient  has  cervical  smear  with  the  result  showing  mild  dyskaryosis.  What  is    
             the  next  appropriate  management?  
a. Refer  for  biopsy  
b. Refer  for  colposcopy  
c. Repeat  smear  in  3  months  
d. Repeat  smear  in  6  months  
e. Repeat  smear  in1  year  
 
60.  A  4  week  old  baby  girl  has  been  diagnosed  as  having  breast  milk  jaundice.  She  is    
             otherwise  well.  What  is  the  single  most  appropriate  management?    
a. Continue  breast  feeding  
b. Exchange  transfusion  
c. Increase  fluid  intake  
d. Phototherapy  
e. Stop  breast  feeding  
 
61.  An  8  year  old  boy  is  suspected  to  be  allergic  to  strawberries.  
             What  is  the  single  most  appropriate  investigation?  
a. Double  blind,  placebo  controlled  food  provocation  challenge  
b. Skin  prick  test  
c. Radio-­‐allergo  sorbent  test  (RAST)  
d. Tissue  transglutaminase  antibody  
e. Total  IgE  blood  test  
 
62.  A  60  year  old  married  woman  is  brought  in  by  her  husband  as  he  reports  that  she      
             is  behaving  oddly.  She  has  suddenly  been  making  inappropriate  sexual  remarks    
             in  public.  She  has  always  been  a  shy  person  and  is  now  extremely  extroverted.  
             What  is  the  most  likely  diagnosis?  
a. Alzheimer’s  
b. Delirium  
c. Picks  disease  
d. Subdural  hematoma  
e. Tourette’s  syndrome  
 
63.  A  50  year  old  man  complains  of  pain  on  defecation.  On  examination,  there  is  a    
             tender  reddish  blue  swelling  near  the  anal  verge.  What  is  the  likely  diagnosis?  
a. Fistula-­‐in-­‐ano  
b. Hemorrhoids  
 

c. Perianal  abscess  
d. Perianal  hematoma  
e. Pilonidal  cyst  
64.  A  30  year  old  banker  complains  of  headache  and  blurry  vision.  He  sits  in  front  of    
             his  VDU  screen  for  8  hours  a  day  checking  the  stock  market.  
             What  is  the  single  most  appropriate  investigation?  
a. ESR  
b. Intraocular  pressure  
c. Split  lamp  test  
d. Refer  to  optician  for  visual  acuity  testing  
e. Visual  field  test  
 
65.  A  77  year  old  man  present  s  with  difficulty  in  reading  the  newspaper  with  missing    
             words  and  seeing  straight  lines  as  wavy  and  with  an  empty  spot  in  the  centre  of    
             his  vision.  What  is  the  single  most  likely  diagnosis?  
a. Acute  glaucoma  
b. Age  related  macular  degeneration  
c. Cataract  
d. Central  retinal  vein  thrombosis  
e. Optic  neuritis  
 
66.  A  34  year  old  man  from  Asia  retuned  to  the  United  Kingdome  with  fever  ,  
night  sweats  and  splenomegaly.  What  is  the  single  most  appropriate  
diagnosis?  
A. Brucelosis  
B. Tuberculosis  
C. Typhoid  
D. Lymphoma  
E. Leukemia  
 
 
 
67.  A  55  year  old  woman  presents  with  plaque  like  lesion  on  her  lower  calf  and  a      
             painless  ulcer  on  the  sole  of  her  foot.  
             What  is  the  single  most  useful  investigation?  
a. ESR  
b. Fasting  blood  sugar  
c. Lyme  disease  serology  
d. Peripheral  arteriogram  
e. Skin  biopsy  
 
68.  A  50  year  old  woman  complains  of  tinnitus,  vertigo  and  deafness.  It  is  so        
 

             disturbing  that  she  is  house  bound.  The  deafness  however  improves  gradually.    
             What  is  the  single  most  likely  diagnosis?  
a. Acoustic  neuroma  
b. Meniere’s  disease  
c. Presbyacusis  
d. Otosclerosis  
e. Viral  labyrynthitis  
 
69.  An  80  year  old  female  resident  of  a  nursing  home  is  seen  in  the  casualty  for    
             generalized  bruising.  A  month  prior  to  this  time,  she  was  admitted  for  a  femoral    
             neck  fracture  sustained  while  falling  out  of  bed.  A  year  ago,  she  sustained  a    
             Colle’s  fracture  after  tripping  over  her  feet.    
             What  is  the  single  most  likely  diagnosis?    
a. Balance  disorder  
b. Non-­‐accidental  injury  
c. Osteoporosis  
d. Postural  hypertension  
e. Vitamin  K  deficiency  
 
70.  A  45  year  old  alcoholic  male  would  like  medication  to  help  him  abstain  from    
             drinking.  He  attends  an  AA  and  has  been  sober  for  1  week.  
             What  is  the  single  most  useful  treatment?  
a. Acamprosate  
b. Chlormethiazole  
c. Diazepam  
d. Disulfiram  
e. Lorazepam  
 
71.  A    31  year  old  lady  with  history  of  eczema  and  rheumatoid  arthritis  presents  
with  shortness  of  breath.  He  also  has  got  tremors,  palpitations  and  
hyperpigementation    lesion  on  his  leg.  He  smokes  20  cigarettes  a  day.  What  is  
the  single  most  likely  diagnosis?  
A. Sarcoidosis  
B. Asthma  
C. COPD  
D. Thyrotoxicosis  
E. Rheumatoid  arthritis  
 
72.  A  12  year  old  girl  with  physical  and  mental    disability    is  on  treatment  for  
some  condition.  She  then  develops  ataxia  and  on  examination  she  found  to  
have  nystagmus.  What  is  the  single  most  likely  cause?    
A. Anti-­‐psychotic  
B. Anti-­‐depressant  
 

C. Anti-­‐epileptic  
D. Metochlorpramide  
E. Cimetidine  
 
 
73.  A  4  day  old  breast  fed  baby  presents  with  jaundice.  The  mother  is  concerned  as      
             her  baby  has  also  lost  250g  in  weight.  The  stools  are  also  pale.    
             What  is  the  single  most  likely  diagnosis?  
a. Biliary  atresia  
b. Breast  feeding  jaundice  
c. Breast  milk  jaundice  
d. Galactosemia  
e. Rh  incompatibility  
 
74.  A  5  year  old  boy  is  noted  to  have  a  clumsy  gait  and  mild  scolisosis.  When  he  gets    
             off  the  floor,  he  uses  his  hands  to  climb  up  the  stairs.  
             What  is  the  single  most  likely  diagnosis?  
a. Duchenne’s  muscular  dystrophy  
b. Gullian-­‐Barre  syndrome  
c. Hunter’s  syndrome  
d. Motor  neuron  disease  
e. Poliomyelitis  
 
75.  A  5  year  old  girl  presents  after  a  fall  on  her  right  arm  after  falling  off  a  swing.  On    
             examination,  there  is  no  swelling  or  deformity  but  the  forearm  is  extremely    
             tender.  What  is  the  single  most  likely  diagnosis?  
a. Greenstick  fracture  
b. Non-­‐accidental  injury  
c. Pulled  elbow  
d. Smith’s  fracture  
e. Spiral  fracture  
 
76.  A  6  week  old  baby  is  brought  in  by  her  mother  who  is  concerned  that  the  baby  is    
             always  hungry  and  cannot  keep  food  down.  The  mother  denies  that  the  baby  has    
             been  vomiting  bile.  On  examination,  a  small  mass  is  palpated  in  the  upper    
             abdomen.  What  is  the  single  most  likely  diagnosis?  
a. Celiac  disease  
b. Gastroenteritis  
c. GERD  
d. Intussusception  
e. Pyloric  stenosis  
 

77.      A  5  year  old  boy  who  has  asthma  is  taking  inhaled  steroid  and  salbutamol  as  
required.  At  night  he  gets  short  of  breath  and  also  during  exercise.  What  is  the  single  
most  appropriate  management?  

A. Long  acting  beta  agonist  


B. Oral  steroid  
C. Leukotriene  antagonist  
D. Increased  inhaled  steroid  
E. Ipratropium  bromide  
 
 
78.  A  50  year  old  burns  victim  needs  to  be  commenced  on  IV  fluids.  
             What  is  the  single  most  appropriate  type  of  fluid?  
a. Colloid  solution  
b. Crystalloid  
c. Dextrose  water  
d. Normal  saline  
e. Plasmalyte  
 
79.  A  4  year  old  girl  presents  with  nasal  speech  and  snoring.  The  mother  is    
             concerned  as  the  child  is  somnolent  during  the  day  and  the  teacher  finds  her    
             inattentive.  What  is  the  single  most  appropriate  management?  
a. Prescribe  ephedrine  nasal  drops  
b. Reassure  mother  that  the  child  will  outgrow  this  
c. Refer  for  pure  tone  audiogram  
d. Refer  for  assessment  for  tonsillectomy  and  adenoidectomy  
e. Refer  to  child  psychiatry  
 

80.    A  17  year  old  man  who  is  a  known  asthmatic  presents  to  accident  and  
emergency  department  with  severe  shortness  of  breath  and  wheeze.    He  has  
been  given  nebulised  salbutamol  and  ipratropium  bromide  and  intravenous  
steroid.  Which  other  intravenous  medication  would  you  give?  

A. Magnesium  sulphate  
B. Hydrocortisone  
C. Salbutamol  
D. Aminophylline  
E. Ipratropium  bromide  
 
81.  A  female  driver  had  a  head  on  collision.  What  is  the  last  vertebrae  that  the  X-­‐ray    
             should  be  done?  
 

a. C5-­‐C6  
b. C7-­‐C6  
c. T1-­‐T2  
d. C7-­‐T1  
 
82.  A  40  year  old  man  has  got  a  hematoma  over  the  mastoid  area  after  being    
             involved  in  a  fight.  Which  bone  is  likely  to  be  fractured?  
a. Parietal  
b. Temporal  
c. Sphenoidal  
d. Mandibular  
e. Occipital  
 
83.  A  38  year  old  female  presents  in  the  morning  with  unconsciousness.  There  is  no    
             previous  history  of  such  but  there  has  been  some  headache  prior  to  presentation.  
             The  BP  is  140/90mmHg.  There  is  also  neck  stiffness  and  extensor  plantar  reflex.  
             What  is  the  most  appropriate  investigation?  
a. CT  scan  
b. Lumbar  puncture  
c. Blood  culture  
d. CSF  analysis  
e. Sputum  MCS  
 

84.    A  15  year  old  boy  who  has  asthma  gets  exacerbation  of  asthma  whenever  he  
plays  football.  What  is  the  single  most  appropriate  management?  

A. Review  the  medication  


B.  Long  acting  beta  agonist  
C.  Oral  steroid  
D. No  action  required  
E. Avoid  football  
 
85.  A  34  year  old  lady    6  hours    after  delivery  of  her  health  baby  presents  with  
fits.  What  is  the  single  most  appropriate  management?  
A. Lorazepam  
B. Magnesium  sulphate  
C. Hydralazine  
D. Diazepam  
E. Phenytoin  
 
86.    A    45  year  old  man  presents  with  fits  for  4  hours  continuously.  What  
medication  will  you  give?  
A. Intravenous  lorazepam  
 

B. Phenytoin  
C. Chlordiazepoxide  
D. Intravenous  diazepam  
E. Per  rectal  diazepam  
 
 

87.        A  69  year  old  man  is  found  to  have  pansystolic    murmur  4  days  after  suffering  
myocardial  infarction.  What  is  the  single  most  appropriate  investigation?  

A. Electrocardiogram  
B. Echocardiogram  
C. CT  scan  chest  
D. MRI  scan  chest  
E. Chest  X-­‐ray  
 
 
 
88.  A  67  year  old  man  is  scheduled  to  have  a  hemicolectomy.  He  is  presently  on        
             warfarin.  What  will  you  do?  
a. Change  warfarin  to  heparin  
b. Change  warfarin  to  aspirin  
c. Clopidogrel  
d. Reduce  the  dose  of  warfarin  
e. Continue  warfarin  
 
89.  A  70  year  old  man  presents  with  right  sided  weakness  and  loss  of  consciousness    
             for  10  minutes.  He  is  already  on  aspirin  75mg,  bendroflumethiazide  and  statin.  
             What  drug  will  you  add  to  his  medication?  
a. Clopidogrel  75mg  
b. Aspirin  300mg  
c. Dypiridamole  200mg  
d. Clopidogrel  300mg  
e. Tissue  plasminogen  activator  
 
90.  A  35  year  old  man  sustained  55%  burs  to  his  body.2  weeks  after  the  incident  
he  presents  with  upper  GI  bleeding.  On  endoscopy  he  is  found  tp  have  
multiple  ulcers  What  is  the  single  most  appropriate  management?  
A. Oral  proton  pump  inhibitor  
B. Intravenous  proton  pump  inhibitor  
C. Oral  antiacids  
D. H.  Pylori  eradication  
E. H2  antangonist    orally  
 

 
 
91.  A  78  year  old  man  with  inoperable  colorectal  carcinoma  presents  with  fatigue,    
             malaise  and  feels  generally  unwell.  He  also  passes  stools  2-­‐3  times  a  week.  
             What  is  the  appropriate  management?  
a. Admit  
b. Dexamethasone  
c. Palliative  care  nurse  
d. Laxatives  
e. Analgesia  through  PCA  
 
92.  A  7  year  old  child  presents  with  shortness  of  breath,  lethargy  and  dehydration.    
             What  is  the  initial  management?  
a. 0.8%  saline  
b. 0.9%  saline  
c. 0.45%  saline  
d. Insulin  
e. Dextrose  
93.  A  man  presents  with  a  swelling  in  the  groin  which  lies  above  and  lateral  to  
pubic  tubercle.  On  examination  it  is  found  to  be  an  indirect  hernia.  Through  
which  structure  will  the  content  come  through?  
A. Deep  ring  
B. External  ring  
C. Linea  alba  
D. Umbilicus  
 
94.  A  5  year  old  boy  with  eczema  presents  with  vesicles  on  top  of  the  
eczematous  lesions.  What  is  the  single  most    
56. appropriate  investigation?  
95. Antifungal  
96. Anti-­‐viral  
97. Antibiotics  
98. Oral  steroid  
99. Iv  steroid  
 
 
95.  A  60  year  old  man  has  developed  vesicles  on  the  maxillary  division  of  the    
             trigeminal  nerve.    
             What  is  the  most  likely  mucous  membrane  to  have  been  involved?  
a. Tonsils  
b. Palletal  
c. Submental  
d. Buccal  
 

e. Posterior  part  of  the  tongue  


 
96.  A  2  year  old  boy  has  got  increasing  pain  in  the  left  hip  for  2  days.  Now  he  cannot    
             move  at  all.  What  is  the  most  likely  diagnosis?  
a. Septic  arthritis  
b. Perthe’s  disease  
c. Slipped  Upper  Femoral  Epiphysis  (SUFE)  
d. Fracture  
e. Developmental  dysplasia  of  the  hip  
 
 
 
 
97.  A  30  year  old  female  teacher  has  upper  respiratory  tract  infection  for  a  week.    
             She  has  recovered  well  and  went  back  to  work  and  completely  lost  her  voice.  
             What  is  the  likely  diagnosis?  
a. Functional  dysphonia  
b. Laryngeal  cancer  
c. Laryngitis  
d. Thyroiditis  
e. Recurrent  laryngeal  nerve  palsy  
 
98.  A  72  year  old  woman  has  got  urinary  incontinence  when  she  coughs  and  sneezes.    
             Urodynamic  assessment  shows  genuine  stress  incontinence.  
             What  is  the  most  appropriate  treatment?  
a. Pelvic  floor  exercises  
b. Physiotherapy  
c. Bladder  neck  surgery  
d. Colporrhapy  
e. Stop  smoking  

 99.      An  11  year  old  girl  presents  with  recurrent  bilateral  joint  pains  .  He  also  
complains  of    general  malaise.  Which  of  the  following  test  will  be  positive?  

A. Rheumatoid  factor  
B. ESR  
C. White  cell  count  
D. Haemoglobin  
E. HLA  27  
 
 
 

100. A  75year  old  lady  with  lung  cancer  was  found  to  have  sodium  of  
125mmol/l,  potassium  3.5,  Urea  2.5mmol/l.  .  What  is  the  most  likely  cause  
for  electrolytes  abnormality  ?  
A. Syndrome  inappropriate  anti-­‐diuretic  hormone  
B. Tumour  lysis  syndrome  
C. Conn’s  syndrome  
D. Sqamous  cell  carcinoma  
E. Cushing  syndrome  
 
 
101.  A  74  year  old  gentleman  was  started  on  antihypertensive  therapy.  1  week  later    
                 he  comes  back  with  a  severe  cough.  
                 Which  drug  is  most  likely  to  be  responsible?  
a.    Enalapril  
b.    Amlodipine  
c.    Propanolol  
d.    Verapamil  
e.      Diltiazem  
 
102.  A  45  year  old  gentleman  3  hours  after  a  hemicolectomy  presents  with  a  BP  of    
                 90/40mmHg  and  heart  rate  of  110b/m.  
                 What  is  the  most  appropriate  next  step  in  management?  
a.    Take  to  the  theatre  
b.    CT  scan  
c.    Intravenous  fluids  
d.    Take  blood  for  U&E  
e.    USS  
 
103.  A  47  year  old  gentleman  has  got  blisters  in  the  trigeminal  root.  He  is  noted  to    
                 have  loss  of  sensation  on  the  middle  part  of  his  lower  lips  and  gums.  
                 Which  part  of  the  trigeminal  nerve  is  involved?  
a.    Submental  
b.    Lingual  
c.    Palatal  
d.    Mandibular  
e.    Ophthalmic  
 
104.  A  79  year  old  lady  has  had  a  change  in  bowel  habits  since  the  last  2  months  and    
                 she  has  noticed  that  her  stools  are  mixed  with  blood.  
                 What  is  the  most  appropriate  investigation?    
a.    Barium  enema  
b.    Colonoscopy  
 

c.    Sigmoidoscopy  
d.    USS  of  the  abdomen  
e.    MRI  of  the  abdomen  
 
105.  A  60  year  old  gentleman  presents  with  sensori-­‐neural  hearing  loss  and  corneal    
                 reflex  loss  on  the  left  side.  What  is  the  most  definitive  investigation?    
a.    CT  internal  auditory  meatus  
b.    Nuclear  imaging  of  the  brain  
c.    MRI  of  the  internal  auditory  meatus  
d.    MRI  pituitary  
e.    MRI  posterior  fossa  of  the  brain  
 
 
 
106.  A  40  year  old  gentleman  presents  with  athralgia  and  skin  lesions  on  his  shin.    
                 Calcium  is  raised.  What  will  be  the  finding  on  the  chest  X-­‐ray?  
a.    Bilateral  hilar  lymphadenopathy  
b.    Bilateral  interstitial  shadowing  
c.    Bilateral  consolidation  
d.    Absent  lung  markings  
e.    Increased  lung  markings  
 
107.  A  30  year  old  lady  7  weeks  pregnant  presents  with  vomiting.  She  is  unable  to    
                 eat  or  drink.  What  is  the  most  appropriate  treatment?  
a.    Nil  by  mouth  
b.    Intravenous  fluids  
c.    Intravenous  Vitamin  B12  
d.    Intravenous  Fe  
e.    Bed  rest  
 
108.  A  90  year  old  lady  with  a  history  of  MI  and  hypertension  presents  with  a  step    
                 wise  deterioration  in  memory.  What  is  the  most  likely  diagnosis?    
a.    Pseudodementia  
b.    Vascular  dementia  
c.    Alzheimer’s  disease  
d.    Lewy  body  dementia  
e.    Alcohol  
 
109.  A  5  year  old  boy  presents  with  abdominal  pain  and  diarrhea.  On  duodenal    
                 biopsy,  he  is  found  to  have  a  short  villy  crypt.  What  is  the  most  likely  diagnosis?  
a.    Peptic  ulcer  
 

b.    Celiac  disease  
c.    Hirshprung’s  disease  
d.    Malabsorption  
e.    Galactosemia  
 
110.    A  60  year  old  man  presents  with    pain  in  the  left  leg  when  walking.  What  is  the  
single  most  appropriate  management?  
A. Anti-­‐hypertensive  
B. Statin  
C. Lifesyle  modification  
D.      Bendroflumethiazide  
E.          Calcium  channel  blockers  
 
111.  An  8  year  old  girl  was  found  to  have  Vitamin  B12  deficiency  and  is  about  to  be    
                 started  on  oral  therapy.  What  is  the  most  appropriate  test  that  needs  to  be    
                 done  before  commencing  oral  therapy?    
a.    Vitamin  B1  
b.    Intrinsic  factor  
c.    Folic  acid  
d.    Endoscopy  
e.    Iron  
 
112.  A  14  year  old  boy  after  playing  football  presents  with  itching  and  redness  on  his    
                 arm.  What  is  the  most  appropriate  treatment?  
a.    IM  adrenaline  
b.    IV  hydrocortisone  
c.    Oral  prednisolone  
d.    Chlorpheniramine  
e.    IV  adrenaline  
 
 
 
113. A 40 year old lady returned from U.S few days ago and now has night
sweats and splenomegaly. Examination is unremarkable. What is the most
likley diagnosis?
a. Lymphoma
b. Tuberculosis
c. Pumonary embolism
d. Brucelosis
e. Malaria
 
114.  A  60  year  old  lady  with  confirmed  diagnosis  of  hypothyroidism  presents  with    
                 dizziness,  shortness  of  breath  and  light  headedness.  FBC  shows  low  Hb  and  high    
 

                 MCV.  What  is  the  most  likely  abnormality?  


a.    Iron  deficiency  anaemia  
b.    Leukemia  
c.    Pernicious  anaemia  
d.    Megaloblastic  anaemia  
e.    Haemorrhagic  anaemia  
 
115.  A  60  year  old  lady  is  worried  about  ovarian  cancer.  What’s  the  initial    
                 investigation  that  you  will  do  to  reassure  her?  
a.    Colposcopy  
b.    CA  125  
c.    Cervical  smear  
d.    Abdominal  USS  
e.    CT  Abdomen  
116.  A  40  year  old  lady  who  is  diabetic  and  ha  cholesterol    of  5.7.  Urine    shows  
microalbuminuria.  What  is  the  single  most  appropriate  medication  to  
reduce  the  risk  of  ischaemic  heart  disease?  
A. Statin  and  angiotensin  converting  enzyme  inhibitor.  
B. Angiotensin  converting  enzymes  and  metformin  
C. Glibenclamide  and  statin  
D. Metformin  double  dose  
E. Bendroflumethiazide  and  statin  
 
 
117.  A  4  year  old  boy  has  been  admitted  with  meningitis  and  treated  successfully.  He    
                 is  now  going  home.  What’s  the  most  appropriate  investigation  to  do  for  him?            
a.    CT  scan  head  
b.    Blood  culture  
c.    Audiometry  
d.    EEG  
e.    Lumbar  puncture  
118. A 30 year old lady had an argument with her boyfriend. And they have
decided to end therir relationship. She now feeling depressed and worthless,
what is the most likley outcome of this scenario if left untreated?
a. Suicide
b. Bipolar affective disorder
c. Generalised anxiety disorder
d. Anxiety phoboid disorder
e. Hysteria
 
 

119.    A  45-­‐year-­‐old  man  presents  with  a  chest  pain  for  more  than  20  minutes.  
ECG  shows  St  depression  in  II,  II,  aVF.  What  is  the  most  appropriate  treatment?  
 

A.  Aspirin+  heparin  
B.  Angioplasty  
C.  Streptokinase  
D.  Aspirin+  warfarin  
E.  Aspirin+  clopidogrel  
 
 
120.  A  60  year  old  lady  presents  with  per  vaginal  bleeding.  Her  last  menstrual  period    
                 was  5  years  ago.  What  is  the  most  appropriate  investigation  to  do?  
a.    Cervical  smear  
b.    Hysteroscopy  and  biopsy  
c.    Colposcopy  
d.    Per  vaginal  examination  
e.    Cone  biopsy  
 
121.  A  40  year  old  lady  presents  with  occasional  per  vaginal  spotting.  What  is  the    
                 most  appropriate  next  step  investigation  to  do?  
a.    Colposcopy  
b.    Cervical  smear  
c.    PV  examination  
d.    Hysteroscopy  
e.    Cone  biopsy  
 
122.  A  35  year  old  lady  presents  with  post  coital  bleeding.  Her  last  cervical  smear    
                 was  2  years  ago.  What’s  the  most  appropriate  investigation?  
a.    Colposcopy  
b.    Cervical  biopsy  
c.    Cervical  smear  
d.    Hysteroscopy  
e.    CT  scan  
 
123.  An  80  year  old  lady  has  had  constipation  for  the  past  2  weeks.  She  is  on  large    
                 dose  of  opiates  for  back  pain.  She  has  been  confused  for  the  past  24  hours  and    
                 also  has  fecal  incontinence.  What  is  the  most  appropriate  management?  
a.    Stimulant  laxative  
b.    Phosphate  enema  
c.    High  fibre  diet  
d.    Bulk  laxatives  
e.    Reduce  dose  of  opiate  
 
124.  A  5  year  old  boy  has  had  ear  ache  for  the  past  2  days.  Otoscopy  reveals  red    
 

                 tympanic  membrane  with  fluid.  What  is  the  most  appropriate  treatment?  
a.    Amoxicillin  
b.    Benzylpenicillin  
c.    Gentamicin  
d.    Ciprofloxacin  
e.    Flucloxacillin  
 
125.  A  70  year  old  lady  has  had  several  falls  especially  whilst  getting  up  from  bed  in    
                 the  morning.  She  has  got  history  of  hypertension  and  is  on  treatment  for    
                 bendroflumethiazide.  What  is  the  most  appropriate  investigation?  
a.    24  hour  ECG  
b.    ECHO  
c.    Standing  and  lying  blood  pressure  
d.    CT  head  
e.    Tilt  test  
 
126.  A  30  year  old  lady  who  is  36  weeks  pregnant  has  got  a  BP  of  170/110mmHg  and    
                 protein  +++  in  the  urine.  What  is  the  most  appropriate  treatment?  
a.    Atenolol  
b.    Oral  methyldopa  
c.    IV  hydralazine  
d.    IV  magnesium  sulphate  
e.    BP  monitoring  
 
127.  A  70  year  old  gentleman  presents  with  severe  unilateral  headache.  On  eye    
                 examination,  the  cornea  is  hazy  and  the  eye  is  congested.  What  is  the  most    
                 appropriate  management?  
a.    Intraocular  pressure  measurement  
b.    CT  head  
c.    Fundoscopy  
d.    FBC  
e.    ESR  
 

128.  31.  A  45-­‐year-­‐old  lady  has  tingling  and  numbness  in  her  thumb,  
index  and  middle  finger.  She  has  tried  splinting  and  steroid  
injections  both  of  which  have  failed.  She  has  now  decided  to  do  
surgery.  What  is  the  only  anatomical  structure  that  you  need  to  
divide?  
A.  Palmar  aponeurosis  
B.  Flexor  retinaculum  
 

C.  Pronator  sheath  
D.  Extensor  digitorum  profundus    
                                 E.  Extensor  digitorum  superfacialis  
 
 
 
 
 

129.      A  40-­‐year-­‐old  man  on  treatment  for  dyspepsia  has  duodenal  ulcers.  He  has  
had  triple  therapy  and  he  has  come  back  with  epigastric  pain  after  6  months.  
What  investigation  will  you  do?  
A.  Urea  breath  test  
B.  Endoscopy  
C.  24  hour  PH  monitoring    
D.  Fundoplication  
E.  H.  pylori  serology  
 
 
130.  A  40  year  old  female  is  diagnosed  with  Grave’s  disease.  She  is  on  treatment  but    
                 develops  a  sore  throat.  What  investigation  will  you  do  for  her?  
a.    ESR  
b.    Thyroid  function  tests  
c.    Urea  and  electrolytes  
d.    Ultrasound  
e.    White  blood  cell  count  
 
131.  A  60  years  old  man  complains  of  blurred  speech  and  weakness  with  tingling  of    
                 the  left  arm.  The  whole  episode  lasted  for  3  hours.    
                 What  is  the  single  investigation  likely  to  give  a  definitive  diagnosis?  
                 a.      CT  head  
                 b.      Doppler  carotid  
                 c.        EEG      
                 d.      MRI  scan  
                 e.      12  lead  ECG  
 
132.  A  15  year  old  female  presents  with  colicky  abdominal  pain  in  the  pelvic  region    
                 since  1  year  ago  that  has  now  increased.  The  pain  is  situated  in  the  right  iliac    
                 fossa.  What  is  the  diagnosis?  
a.    Ovarian  torsion  
b.    Twisted  ovarian  cyst  
c.    Ovarian  cancer  
 

d.    Endometriosis  
e.    Appendicular  mass  
 
133.  A  blood  film  shows  a  picture  of  anaemia  with  spherocyte.    
                 What  investigation  will  you  do?  
a.    Indirect  Coomb’s  test  
b.    Bone  marrow  examination  
c.    Measure  serum  ferritin  
d.    Liver  function  test  
e. Direct  coombs  test  
 
134.  A  68  year  old  man  presents  with  pan-­‐systolic  murmur  in  the  apex  which          
                 radiates  to  the  axilla.  What  is  the  likely  diagnosis?    
a.    Mitral  stenosis  
b.    VSD  
c.    Mitral  regurgitation  
d.    Tricuspid  regurgitation  
e.    Aortic  regurgitation  

135.      A  40-­‐year-­‐old  man  has  had  deafness  for  1  year.  He  now  presents  to  
accident  and  emergency  department  with  the  same  complaint.  Otoscopy  shows  
yellow  discharge  form  the  upper  drum.  What  is  the  most  likely  diagnosis?    
A.  Otosclerosis  
B.  Ramsay  Hunt  syndrome    
C.  Cholesteatoma  
D.  Acute  otitis  media  
E.  Otitis  externa    
 
136.    A  70  year  old  man  presents  with  urinary  retention.  He  has  been  
catheterised.  His  prostate  is  enlarged  but  he  has  no  other  symptoms  and  all  
blood  tests  are  normal.  What  is  the  most  appropriate  management?  
A.  Bladder  outlet  surgery  
B.  Fenesteride  
C.  TWOC  +fenesteride  
D.  TWOC  +  tamsulosin  
E.  No  action  required.
 
 
138.  A  30  year  old  lady  presents  with  generalized  tonic-­‐clonic  seizures.  
                 What  is  the  single  most  appropriate  treatment?  
a.    Clonazepam  
b.    Gabapentin  
 

c.    Phenytoin  
d.    Sodium  valproate  
e.    Vigabatrin  
 

139.      A  70-­‐year-­‐old  man  with  a  terminal  CA.  His  pain  was  well  controlled  with  
60  mg  of  oral  morphine,  now  he  cannot  swallow  due  to  recurrent  chest  infection  
from  aspiration.  What  is  the  most  appropriate  analgesia?  
A.  60  mg  morphine  subcutaneously  
B.  120  mg  morphine  subcutaneously  
C.  60mgmorphineIV  
D.  60  mg  morphine  IM  
E.  Continue  same  treatment.  
 
 
140.  A  50  year  old  inpatient  has  diarrhea  while  on  treatment  for  osteomyelitis.    
                 What  is  the  single  most  likely  organism  responsible?  
a.    Clostridium  difficile  
b.    Clostridium  perfringes  
c.    E.coli  
d.    Salmonella  
e.    Shigella  
 
141.  A  30  year  old  female  who  returned  from  Africa  1  month  ago  now  complains  of    
                 fever  and  malaise  and  is  noted  to  have  faint  erythematous  maculopapules  (rose    
                 spot)  on  her  torso.  What  single  investigation  will  give  the  highest  sensitivity?    
a.    Blood  culture  
b.    Bone  marrow  biopsy  
c.    Stool  culture  
d.    Typhoid  M  test  (IgG  and  IgM  antibodies  test)  
e.    Urine  culture  
 
142.  A  55  year  old  man  with  asthma  was  recently  started  on  an  antihypertensive    
                 drug  which  he  now  thinks  is  responsible  for  his  impotence.  
                 What’s  the  single  most  likely  culprit?  
a.    Amlodipine  
b.    Atenolol  
c.    Bendroflumethiazide  
d.    Frusemide  
e.    Peridopril  
 
143.  A  30  year  old  woman  exhibits  pressured  speech  and  flight  of  ideas  changing    
 

                 from  subjects  frequently  in  a  conversation.    


                 What  is  the  single  most  likely  diagnosis?  
a.    Dementia  
b.    Mania  
c.    Obsessive  Compulsive  Disorder  (OCD)  
d.    Paranoia  
e.    Schizophrenia  

144.  A  30  year  old  pregnant  lady  gets  in  contact  with  a  child  who  has  
chicken  pox.  Varicella  antibodies  are  negative.    What  is  the  most  
appropriate  management?    
   A.  Reassure  
   B.  Immuniglobulin  G  
                                   C.  Varicella  Vaccination    
                                   D.  Acyclovir  
                                   E  .  Antibiotics    
 
 
145.  A  50  year  old  woman  post  thyroidectomy  complains  of  tingling  sensation    
                 around  her  lips.  What  investigation  will  you  do?  
a.    CT  scan  of  the  head  
b.    Serum  calcium  
c.    Thyroid  function  tests  
d.    Urea  and  electrolytes  
e.    Ultrasound  scan  of  the  neck  
 
146.  An  80  year  old  woman  is  noted  to  have  unilateral  right  sided  tonsilar    
                 hypertrophy.  What  is  the  single  most  likely  diagnosis?  
a.    Lymphoma  
b.    Peritonsilar  cellulitis  
c.    Quinsy  
d.    Tonsilar  carcinoma  
e.    Viral  tonsillitis  
 
148.  An  80  year  old  man  resident  in  a  nursing  home  presents  with  urinary  retention.    
                 On  examination,  the  bladder  is  distended  and  the  sigmoid  colon  is  palpable.  
                 What  is  the  single  most  likely  diagnosis?  
a.    UTI  
b.    Sigmoid  colon  carcinoma  
c.    Fecal  impaction  
d.    Diverticulosis  
e.    BPH  
 

 
149.  A  55  year  old  alcoholic  man  complains  of  severe  upper  abdominal  pain  radiating    
                 to  his  back.  He  also  passes  loose,  pale  stools  and  has  weight  loss.    
                 What  is  the  most  likely  diagnosis?  
a.    Acute  pancreatitis  
b.    Chronic  pancreatitis  
c.    GERD  
d.    Oesophagitis  
e.    Pancreatic  carcinoma  
 

150.      A  10  year  old  boy  with  cerebral  palsy  has  urinary  retention.  2  times  urine  
culture  was  negative.    What  is  the  most  appropriate  management?  
A.  Intermittent  self  catheterisation    
B.  Permanent  catheterisation  
C.  Surgery  
D.  Mid  stream  urine  
E.  Antibiotics    
 
151.      10  year  old  girl  presents  with  polyuria,  ploydipsia,  and  weight  loss.  You  
suspect  diabetes.  
Which  of  the  following  would  confirm  diagnosis  of  diabetes?  
A.  Random  glucose  8  
B.  Random  blood  glucose  14  
C.  Urine  dip  stick  protein  +2  and  fasting  glucose  6.5  
D.  Fasting  glucose  5.2  
E.  Urine  Dipstick  protein  +2  and  fasting  blood  glucose
 
152.  A  30  year  old  heroin  addict  presents  with  purple  papules  in  his  mouth  and  on    
                 his  leg.  What  is  the  most  likely  diagnosis?  
a.    Henoch-­‐Schonlein  purpura  
b.    Kaposi  sarcoma  
c.    Lichen  planus  
d.    Malignant  melanoma  
e.    TTP  
 
153.  A  2  year  old  fire  fighter  presents  to  the  A&E  with  painful  throat  and  coughing  up    
                 of  black  sputum.  He  states  that  he  was  in  a  confined  space  in  a  burning  building    
                 for  20  minutes.  On  examination,  the  eyebrows  and  nose  hair  in  the  vestibules    
                 are  singed.  He  has  also  become  disoriented.    
                 What  is  the  single  most  appropriate  management?  
a.    Intubation  
 

b.    Analgesia  
c.    Arterial  blood  gas  and  carbon  monoxide  levels  
d.    High  flow  oxygen  
e.    Surgical  tracheostomy  
 
154.  A  20  year  old  lady  presents  with  intermenstrual  bleeding.  She  was  started  on    
                 combined  oral  contraception  4  months  ago  and  now  has  a  new  partner.  
                 What  is  the  single  most  relevant  investigation?  
a.    Endocervical  swab  
b.    Endometrial  sampling  
c.    Cervical  smear  
d.    Urine  for  Chlamydia  
e.    Transvaginal  USS  
 
155.  An  18  month  old  baby  is  brought  in  with  dehydration  from  diarrhea  and    
                 vomiting  for  the  past  72  hours.  On  examination  she  is  found  to  have  mild    
dehydration.    What  is  the  single  most  useful  treatment?  
a.    Water  by  mouth  
b.    0.9%  saline  
c.    Oral  rehydration  solution  (60mmol)  
d.    Oral  rehydration  solution  (90mmol)  
e. No  action  required  
 
156.  A  28  year  old  man  presents  with  a  2  week  history  of  painful  swollen  right  knee.    
                 On  examination,  he  is  found  to  have  iritis  and  non-­‐specific  urethritis.  A  week    
                 prior  to  this,  he  had  a  severe  bout  of  diarrhea.  What  is  the  single  most    
                 appropriate  treatment?  
a.    Allopurinol  
b.    Ciprofloxacin  
c.    Colchicines  
d.    Naproxen  
e.    Paracetamol  
 
157.  A  month  old  baby  is  brought  to  the  A&E  by  her  mother  who  is  concerned  that    
                 her  baby  is  inconsolable.  Her  baby  has  been  vomiting  and  cannot  keep  food    
                 down.  On  examination,  the  abdomen  is  distended  and  a  sausage  shaped  mass  is    
                 palpable.  What  is  the  most  definitive  investigation?  
a.    Abdominal  USS  
b.    Abdominal  X-­‐ray  
c.    Barium  enema  
d.    CT  scan  of  the  abdomen  
 

e.    Protoscopy  
 
158.  A  70  year  old  man  presents  with  swollen  right  knee.  He  is  on    
                 bendroflumethiazide  2.5mg.  What  is  the  diagnostic  investigation?    
a.    Blood  culture  
b.    C-­‐reactive  protein  
c.    D-­‐dimer  
d.    Plain  X-­‐ray  of  the  knee  
e.    Serum  uric  acid  
 

159.      38  year  old  lady  30  weeks  pregnant  presents  with  proteinurea  +++  and  BP  
160/100?  What  is  the  most  likley  diagnosis?  
A.  Pregnancy  induced  hypertension    
B.  Eclampsia  
C.  Pre  eclampsia  
D.  HELP  syndrome  
E.  Hyperemesis  gravidarum  
 
 
160.  A  53  year  old  Asian  woman  with  diabetes  mellitus  type  2  has  a  BMI  of  35.  She    
                 had  an  operation  for  ovarian  cancer  for  which  she  received  prophylactic  low    
                 molecular  weight  heparin.  5  days  later,  she  develops  pain,  redness  and  swelling    
                 in  the  left  calf.  What  is  the  single  most  appropriate  investigation?  
a.    C-­‐reactive  protein  
b.    D-­‐dimer  
c.    Doppler  ultrasound  
d.    Thrombophilia  screen  
e.    Venography  
 
161.  A  70  year  old  man  who  has  rheumatoid  arthritis  has  a  painful  wrist.  He  takes          
                 methotrexate,  NSAID’s  and  anti-­‐hypertensive  medication.  His  Hb  is  low.    
                 What’s  the  most  likely  anemia  you  will  find  in  the  blood?  
a.    Hemolytic  anemia  
b.    Aplastic  anemia  
c.    Anemia  of  chronic  disease  
d.    Haemolytic  uraemic  anaemia  
e.    Megaloblastic  anaemia  
 
162.  A  45  year  old  man  had  geastrectomy.  He  now  presents  with  lethargy,  weakness    
                 and  paraesthesia  of  the  limbs.  His  MCV  is  110.    
                 What  is  the  likely  cause  of  his  symptoms?  
 

a.    Folic  acid  deficiency  


b.    Vitamin  B12  deficiency  
c.    Iron  deficiency  
d.    Malabsorption  
e.    Somatic  neuropathy  
 
 
 
163.  A  62  year  old  man  is  found  to  have  squamous  cell  carcinoma  of  the  lung  after    
                 being  investigated  for  hemoptysis.  Which  one  of  the  following  would  be          
                 contraindicated  to  surgical  resection?  
a.    Finger  clubbing  
b.    Hypercalcemia  
c.    Hypertrophic  pulmonary  osteoathropathy  
d.    Pleural  effusion  
e.    Superior  vena  cava  obstruction  
 

164.  A  20  year  old  lady  with  6  weeks  history  of  amenorrhoea.  She  presents  
with  excessive  vomiting.  USS  finds  the  uterus  to  be  16  weeks  in  size.  
What  is  the  most  likely  cause  of  vomiting?  
A.  Molar  pregnancy    
B.  Twins  
C.  Ectopic  pregnancy    
D.  Polyhydrominous    
E.  Diabetes  
 
 
165.      20  year  old  lady  who  is  10  weeks  pregnant  presents  with  heavy  PV  
bleeding  and  she  is  passing  clots.  The  os  is  open.  The  most  appropriate  
management?  
A.  TV  scan  
B.  Laporoscopy    
C.  Laparotomy    
D.  No  treatment    
E.  Anti  D  
 
 

166.    20  year  old  lady  who  is  10  weeks  pregnant  presents  with  slight  PV  bleeding.  
The  os  is  open.  The  most  appropriate  management?  
A.  reassure  
B.  TV  scan  
 

C.  Abdominal  USS  
D.  Laparoscopy  
E.  Surgery  
 
 
167.  A  70  year  old  man  presents  with  a  few  months  history  of  weight  loss  and    
                 dysphagia.  What’s  the  appropriate  next  step?    
a.    Barium  meal  
b.    Barium  swallow  
c.    Endoscope  
d.    USS  
e.    Chest  X-­‐ray  
 
 
168.  A  45  year  old  man  presents  with  acute  testicular  pain.  One  testes  is  higher  than    
                 the  other.  MSU  shows  no  nitrate  and  white  blood  cells.  
                 What  is  the  most  appropriate  management?    
a.    Analgesia  
b.    IV  saline  and  observe  
c.    Reassure  
d.    Immediate  surgical  exploration  
e.    Antibiotics  
 
169.  A  54  year  old  man  was  moving  things  from  his  house  during  which  a  cupboard    
                 fell  on  his  legs  and  thighs.  The  cupboard  was  on  his  legs  for  3  hours.  Later  he    
                 presented  with  blood  in  the  urine.  What’s  the  cause  of  his  renal  problem?  
a.    Creatinine  phosphakinase  
b.    Haemoglobin  
c.    Myoglobin  
d.    Troponin  
e.    Blood  loss  
 
170.  A  79  year  old  lady  who  had  breasts  carcinoma  present  with  seizures.  What  is    
                 the  best  route  of  administration  of  the  anti-­‐epileptic  drug?  
a.    Oral  
b.    Intravenous  
c.    Intramuscular  
d.    Rectal  
e.    Subcutaneous  
 
 
 

171.      A  30-­‐year-­‐old  lady  with  painful  labial  ulcers.  Which  investigation  will  you  
do?  
A.  VDPL  
B.  Treponema  pallidum    
C.  HPV  
D.  Anti  HSV  antibodies    
E.  chlamydia  swab
 
 
172.  A  38  year  old  female  with  family  history  of  polycystic  kidney  disease  dies    
                 suddenly.  She  also  had  hypertension.  Her  bother  wants  to  know  the  exact    
                 reason  for  her  death.  
a.    Intracranial  haemorrhage  
b.    Epidural  haematoma  
c.    Subdural  haematoma  
d.    Subarachnoid  haematoma  
e.    Intracerebral  haemorrhage  
 

173.    30  year  old  lady  on  COCP  has  got  nodular  lesions.  What  is  the  most  likely  
cause?  
A.  Human  papilloma  virus
B.  Warts  
C.Herpes  simples  virus  
D.  Syphillis    
E.    Pyederma  gangrenosum  
 
174.    A  74-­‐year-­‐old  woman  experiences  dizziness  while  decorating  the  house.  
What’s  the  cause?  
A.  Carotid  artery  disease    
B.  Meniers’  disease  
C.  BPV  
D.  Vestibular  neuritis  
E.  Wax  
 
 

175.      An  8  year  old  girl  presents  with  fever,  low  abdominal  pain.  On  examination,  
she  has  splenomegaly,  all  blood  tests  are  normal.  Treated  with  antibiotics.  In  a  
few  days  she  comes  back  with  same  complains.  What  is  the  most  likely  diagnosis?  
A.  ALL    
B.  CLL  
C.  Aplastic  anaemia  
 

D.  Myelodysplastic  anaemia  
E.  AML  
 
 
175.  A  4  month  old  baby  is  brought  in  with  acute  nose  bleeding  but  is  now  stable.  
                 What  is  the  single  action  that  can  stop  the  nose  bleeding?  
a.    Ice  pack  
b.    Pinch  the  bridge  of  the  nose  
c.    Pinch  the  soft  tissue  of  the  nose  
d.    Anterior  packing  
e.    Reassure  
 
176. An 70 year old man presents with painless haematuria. What is the most
appropriate investigation?

A. Cystoscopy
B. Intravenous urogram
C. Ultrasound of the kidney
D. Mid stream urine
E. Renal biopsy.
 
 
177.  A  patient  with  long  standing  1cm  lesion  on  the  hand  that  has  started  to  bleed    
                 on  touching.  What  is  the  diagnosis?  
a.    Basal  cell  carcinoma  
b.    Kaposi  sarcoma  
c.    Malignant  melanoma  
d.    Sqaumous  cell  carcinoma  
 
178.  A  man  presents  with  a  lesion  on  his  thigh  with  central  clearing  which  has  been    
                 clearing  since  3  weeks  ago.  He  also  has  malaise  and  generalized  aches.  What  is    
                 the  test  to  diagnose  his  lesion?      
a.    Skin  biopsy  
b.    Skin  scrapping  and  culture  
c.    Lyme  serology  
d.    Antibacterial  culture  
e.    Blood  culture  
 
179.  A  patient  presents  with  acute  onset  of  red  eye,  pain  and  blurry  vision.  He  also        
                 has  peri-­‐orbital  edema.  What  is  the  best  initial  management?  
a.    Put  patient  in  a  dark  room  with  reduced  light  
b.    IV  morphine  and  routine  ophthalmological  referral  
c.    IV  morphine  and  emergency  ophthalmological  referral  
 

d.    IM  morphine  
e.    IV  morphine  and  antibiotics  
 
180.  A  60  year  old  man  presents  with  increasing  forgetfulness,  poor  sleep  and  loss  of    
                 interest.  He  shows  psychomotor  retardation  and  answers  every  question  with  ‘I    
                 don’t  know.’  What  single  diagnosis  explains  his  condition?  
a.    Depression  
b.    Dementia  
c.    Bipolar  affective  disorder  
d.    Normal  aging  process  
e.    Panic  disorder  
 
181.  An  84  year  old  lady  with  metastatic  breast  cancer  presents  with  shortness  of    
                 breath  due  to  pleural  effusion.  She  has  got  few  weeks  remaining  for  her  to  live.  
                 What  is  the  most  appropriate  management?  
a.    Insert  a  pleural  drainage  
b.    Pleural  aspiration  
c.    Pleurodesis  
d.    Home  oxygen  
e.    Analgesia  
 
182.  A  34  year  old  man  presents  unwell.  Her  blood  tests  show  hyponatremia  and    
                 hypokalemia.  What  is  the  likely  diagnosis?  
a.    Acute  gastroenteritis  
b.    Acute  renal  failure  
c.    Addison’s  disease  
d.    Crohn’s  disease  
e.    Respiratory  acidosis  
 
183.  A  51  year  old  man  after  hernia  repair  has  been  constipated.  Recently  she  has    
                 been  complaining  of  loose  stool.  What  is  the  most  appropriate  management?  
a.    Laxatives  
b.    Phosphate  enema  
c.    High  fiber  diet  
d.    Exercise  
e.    Intravenous  fluids  
 
184.  A  52  year  old  boy  presents  with  sudden  onset  of  testicular  pain  with  the  right    
                 testes  swollen  and  tender.  There  is  no  history  of  trauma  and  he  has  a    
                 temperature  of  38.5C.    
                 What  is  the  most  appropriate  next  step  in  his  management?    
 

a.    Give  antibiotics  
b.    USS  of  the  scrotum  
c.    Immediate  surgical  referral  
d.    Reassure  
 
 
185. A 45 year old woman who has breast cancer develop paresthesias and
sensory loss of the fingers after chemotherapy. What is the cause?

A. Metastasis to cervical spine

B. Hypocalcemia

C. Peripheral neuropathy induced by chemotherapy


D. Lympadenopathy
E. Lympoedema
 
186.  A  1  year  old  boy  is  brought  to  the  A&E  department  with  a  5  day  history  of    
                 diarrhea  and  vomiting.  For  the  past  24  hours  he  has  been  unable  to  tolerate  any    
                 oral  fluids.  In  the  last  hour,  he  has  become  drowsy,  his  skin  tugor  has  reduced    
                 and  his  skin  is  mottled  with  his  eyes  sunken.  His  heart  rate  is  180b/m  and    
                 capillary  refill  is  prolonged.    
               What  is  the  single  most  appropriate  initial  management?  
a.  IV  fluids  bolus  over  20  minutes  
b.  IV  rehydration  over  48hours  
c.  IV  fluids  over  4  hours  
d.  Nasogastric  rehydration  over  24  hours  
e.  Oral  rehydration  solution  over  48  hours  
 
187.  A  17  year  old  girl  presents  with  a  5  day  history  of  urinary  frequency  and  dysuria.    
                 She  has  a  weight  of  20kg  despite  a  good  appetite.  She  had  been  taking  over  the    
                 counter  preparation  of  premenstrual  tension  medication  for  several  months.    
                 Urinalysis  shows  no  glycosuria  but  is  positive  for  nitrates.    
                 What  is  the  single  most  likely  diagnosis?    
a.    Diabetes  insipidus  
b.    Diabetes  mellitus  
c.    Diuretic  therapy  
d.    Psychogenic  prophylaxis  
e.    Urinary  tract  infection  
 
188.  A  4  year  old  is  brought  to  the  A&E  department  by  ambulance.  His  mother    
                 reports  that  he  has  been  unwell  with  a  sore  throat  for  8  hours.  He  is  sitting  on    
 

                 his  mother’s  knee  and  is  tolerating  an  oxygen  mask  but  looks  unwell.  He  has    
                 constant  noisy  breathing  and  is  drooling  saliva.  His  temperature  is  39.C.    
                 What  is  the  single  most  important  diagnosis  to  rule  out?  
a.    Acute  asthma  
b.    Bronchiolitis  
c.    Croup  
d.    Epiglotitis  
e.    Tonsillitis  
 
189.  A  17  year  old    man  complains  of  severe  eye  pain  and  blurry  vision.  He  also  takes  
pain  killers    
                 for  his  long  standing  back  pain.  What’s  his  likely  clinical  condition?  
a.    Episcleritis  
b.    Iritis  
c.    Posterior  uveitis  
d.    Acute  closed  angle  glaucoma  
e.    Scleritis  
 
190. An 18-year-old boy has come back after working in a farm and now has
fever with splenomegaly, myalgia, bone pain.
What is the diagnosis?
A. Brucellosis

B. Lyme disease
C. Leishmania
D. Tuberculosis
E. Lymphoma
 
191.  A  62  year  old  man  has  multiple  liver  metastases  due  to  adenocarcinoma  with  an    
                 unknown  primary.  He  is  deeply  jaundiced  and  has  ascites  with  oedema  of  his    
                 legs  up  to  the  buttocks.  He  is  now  drowsy  and  his  family  are  worried  that  he  is    
                 not  drinking  enough.  His  medication  includes:  
                 Haloperidol    1.5mg,    Lactulose  10ml  
                 Blood  taken  3  days  ago  showed  the  following  results:  
                 Normal  electrolytes,  Urea  6.5mmol/l,  Creatinine    89mmol/l,      
                 Calcium  2.04mmol/l,  Albumin  17g/L,  Total  bilirubin  189mmol/l  
                 What  is  the  single  most  appropriate  management  of  his  fluid  intake?  
a.    Albumin  infusion  
b.    Crystalloids  fluids  (continuous  IV)  
c.    Crystalloids  fluids  (intermittent  subcutaneous)  
d.    Fluids  via  nasogastric  tube  
e.    Oral  fluids  
 

 
192.  A  14  year  old  develops  an  itchy  scaly  patch  on  her  scalp.  She  had  a  similar  patch    
                 on  her  scalp  that  had  healed  spontaneously  2  years  ago.  Her  aunt  has  a  similar    
                 undiagnosed  rash  on  the  extensor  aspects  of  her  elbows  and  knees.  What  is  the    
                 single  most  likely  diagnosis?  
a.    Eczema  
b.    Fungal  infection  
c.    Impetigo  
d.    Lichen  planus  
e.    Psoriasis  
 
193.  A  4  week  old  baby  was  brought  to  the  A&E  by  the  mother  with  a  condition  of    
                 expiratory  wheeze,  cough  and  nasal  discharge  and  also  vomits  after  feeding  for    
                 the  last  3-­‐5  days.  What  is  the  most  likely  diagnosis?    
a.    Bronchiolitis  
b.    Croup  
c.    Broncho  pulmonary  dysplasia  
d.    Bronchitis  
 
194.  A  woman  presents  with  increasing  forgetfulness,  lethargy  and  tiredness.  She    
                 has  given  up  her  job  as  a  teacher  in  the  university.  She  has  increasing  dyspnea    
                 and  breathlessness  on  exertion.  Her  only  medication  is  L-­‐thyroxine.  There  are    
                 no  neurological  symptoms.  What  single  test  will  help  to  establish  her  diagnosis?  
a.    MRI  scan  of  the  head  
b.    Erythrocyte  sedimentation  rate  
c.    Hb  concentration  
d.    Thyroid  function  test  
e.    Vitamin  B12  level  
 
 
195.  A  30  year  old  woman  was  treated  successfully  for  CIN2  some  years  back.  She    
                 was  a  heavy  smoker  and  discontinued  smoking  a  few  months  back.  She  was  on    
                 oral  contraceptives  all  these  years  but  now  her  partner  uses  condoms.  She  is    
                 anxious  as  her  aunty  died  of  ovarian  cancer  at  the  age  of  48.  The  single  most    
                 important  factor  which  puts  her  at  risk  of  ovarian  cancer  is?    
a.    Age  
b.    Family  history  
c.    Smoking  
d.    CIN  2    
e.    Oral  contraceptives  
 
 

196.  A    30  year  old  lady  presents  with  hyperventilation  and  classical  signs  of  panic  
attack.  To  accident  and  emergency  department.  
                 What  is  the  single  most  appropriate  management?  
a.    Rebreathe  into  an  airbag  
b.    Diazepam  
c.    Salbutamol  
d.    Morphine  
e.    Diclofenac  
 
197.  A  22  year  old  man  says  that  he  can  hear  the  voice  of  his  deceased  uncle  telling    
                 him  that  he  is  being  spied  on.  The  patient  is  distressed  by  this,  is  starting  to    
                 have  low  mood,  is  anxious  and  has  not  left  the  house  for  weeks.  He  is  starting                
                 to  drink  increasing  quantities  of  alcohol.  He  is  also  noticed  to  have  thought    
                 block  and  passivity  phenomena.    
                 What  is  the  single  most  suitable  medication  to  treat  his  symptoms?  
a.    Diazepam  
b.    Disulfiram  
c.    Fluoxetine  
d.    Lithium  
e.    Olanzapine  
 
198.  An  18  month  old  boy  has  been  brought  to  the  emergency  department  because    
                 he  has  been  refusing  to  move  his  left  arm  and  crying  more  than  usual  for  the    
                 past  24  hours.  He  has  been  recently  looked  after  by  his  mother’s  new  boyfriend    
                 while  she  attends  college.  Assessment  shows  multiple  bruises  and  a  fracture  of    
                 the  left  humerus  which  has  been  put  in  a  plaster.  
                 What  is  the  single  most  appropriate  next  step?  
a.    Admit  under  the  care  of  a  pediatrician  
b.    Discharge  with  painkiller  
c.    Follow  up  in  fracture  clinic  
d.    Follow  up  in  pediatric  outpatients  
e.    Follow  up  with  general  practitioner  
 
199.    A  75  year  old  woman  had  an  emergency  total  hip  replacement  following  
fracture  of  the  neck  of  femur.  6  days  post-­‐operatively  she  wakes  up  in  the  middle  of  
the  night  with  left  sided  chest  pain,  haemoptysis  and  breathlessness.      
                 What  is  the  single  most  appropriate  immediate  management?  
a.    Ampicillin  
b.    Furosemide  
c.    Heparin  
d.    Salbutamol  
 

e.    Warfarin  
 
200.  A  mother  has  brought  her  3  year  old  son  to  the  emergency  department  2  hours    
after  he  has  taken  an  unknown  quantity  of  paracetamol  suspension.  He  appear  well  
and  is  playing  normally.  What  is  the  single  most  appropriate  time  to  measure  his  
serum  paracetamol  levels?  
a.    Immediately  
b.    In  2  hours  
c.    In  4  hours  
d.    In  8  hours  
e.    In  24  hours  
 
 
 
 
GOOD  LUCK    FROM  DR  SAMSON  
 

 
 

 
 
 
 

 
 
 

SAMSONPLAB  ACADEMY  LIMITED  BOW  BUSSINESS  


CENTRE  
BOW  ROAD  153-­‐-­‐159  
E3  2SE  
EMAIL:  info@samsonplab.co.uk  
Mobile:  07940433068  
PLAB  1  MOCK  6  JUNE  2013  ANSWERS
 
 

 
 
 

 
 
 

 
 
 

 
 

1.B     30.C     59  .B     88.A     117.   C   146.D    


2.D     31.E     60.A     89.C     118.   A   147.  

3.D     32.E     61.C     90.B     119.   A   148.C  


4.C     33.C     62.C     91.C     120.   B   149.B  
5.A.     34.A     63.D     92.B     121.   B   150.A  
6.E     35.C     64.D     93.A     122.   A   151.B  

7.A     36.D     65.B     94.B     123.   B   152.B  


8.D     37.A     66.B     95.D     124.   A   153.A  
9.E     38.A     67.B     96.A     125.   C   154.A  

10.B     39.D     68.B     97.A     126.   C   155.C  


11.A     40.D     69.B     98.C     127.   A   156.D  

12.C     41.B     70.A     99.B     128.   B   157.C  


13.E     42.B     71.D     100.A     129.   A   158.E  

14.E     43.D     72.C     101.A     130.   E   159.C  


15.D     44.C     73.A     102C     131.   A   160.C  
16.B     45.C     74.A     103.A     132.   B   161.C  

17.E     46.C     75.A     104.B     133.   E   162.B  


18.B     47.C     76.E     105.C     134.   C   163D  
19.E     48.C     77.A     106.A     135.   C   164.A  
20.A     49.B     78.B     107.B     136.   D   165.E  

21.C     50.D     79.D     108.B     137.     166.B  


22.E     51.E     80.A     109.B     138.   D   167.B  
23.D     52.C     81.D     110.C     139.   A   168.D  

24.A     53.B     82.B     111.B     140.   A   169.C  


25.D     54.A     83.A     112.D     141.   D   170.B  
26.E     55.E     84.     113.A     142.   C   171.D  
27.C     55.B     85.A     114.C     143.   B   172.D  

28.D     56.B     86.A     115.B     144.   B   173.B  

29.E     57.B     87.B     116.A     145.   B   174.C  

 
 

175.E  
175.C  

176.A  
177.D  
178.C  
179.C  

180.A  
181.B  
182.A  

183.B  
184.A.  

185.C  
186.A  

187.E  
188.D  
189.D  

190.A  
191.A  
192.E  
193.A  

194.D  
195.C  
196.A  

197.E  
198.A  
199.C  
200.B  

 
 

SAMSONPLAB  ACADEMY  LIMITED  


BOW  BUSSINESS  CENTRE  
BOW  ROAD  153-­‐159  
E3  2SE,  LONDON  
EMAIL:  info@samsonplab.co.uk  
Telephone:  07940433068/  
 
 
       SAMSONPLAB  ACADEMY  MOCK  7  
 
1.  A  36  year  old  gravida  1  woman  with  gestational  age  of  30  weeks  was  brought  to              
         the  A&E  department  as  she  started  to  have  bleeding  per  vagina.  The  bleeding  was          
         fresh  and  painless  and  was  not  preceded  by  trauma.  On  examination,  her    
         abdomen  was  not  tender,  BP  was  90/60mmHg  and  pulse  rate  was  130b/m.    
         Ultrasound  showed  a  low  lying  placenta  and  CTG  showed  normal  fetal  heart    
         sounds.  How  best  will  you  manage  this  patient?  
a. IV  fluids  blood  transfusiondeliver  
b. Blood  transfusionIV  fluidsdeliver  
c. Deliver  baby  immediately  
d. InvestigationsIV  fluidsblood  transfusiondeliver  
e. IV  fluidsblood  transfusioninvestigationsdeliver  
 
2.  A  65  year  old  patient  who  had  MI  a  year  back  now  comes  to  the  A&E  complaining    
         that  his  neighbor  is  conspiring  against  him.  When  his  son  is  asked,  he  denies  it  and    
         also  narrates  that  sometimes  his  father  says  that  everybody  in  his  office  is  always    
         talking  about  him  which  is  not  the  case.    
         What  is  the  most  appropriate  medication  for  this  patient?  
a. Tricyclic  antidepressants  
b. Clozapine  
c. Olanzapine  
d. Lorazepam  
 
3.  A  28  year  old  lady  with  a  family  history  of  cystic  fibrosis  comes  for  genetic    
         counseling  and  wants  the  earliest  possible  diagnostic  tests  for  cystic  fibrosis  for    
         the  baby  she  is  planning.  She  is  not  in  favor  of  termination.    
         What  would  you  recommend  for  her?  
a. Chorionic  villous  sampling  
b. Amniocentesis  
c. Pre  implantation  genetic  diagnosis  
d. Chromosomal  karyotyping  
 

 
4.  A  45  year  old  man  was  involved  in  a  fight  and  sustained  injury  to  his  right  ear.  He      
         has  bleeding  with  ringing  and  deafness  in  his  right  ear.  
         What  is  the  most  appropriate  investigation?  
a. CT  head  
b. Otoscopy  
c. X-­‐ray  
d. MRI  scan  
e. Refer  to  outpatient  clinic  
 
5.  A  45  year  old  man  was  trapped  under  a  rock  for  4  hours.  He  then  presents  with    
         red  urine  and  renal  failure.  What  is  the  most  appropriate  treatment?  
a. Urgent  dialysis  
b. Fluid  challenge  
c. Glucose  and  insulin  
d. Dobutamin  
e. Antibiotics  
 
6.  A  30  year  old  lady  was  found  to  have  inflammatory  changes  on  routine  cervical    
         smear.  What  is  the  most  appropriate  action?  
a. Do  colposcopy  
b. Repeat  smear  in  6  months  
c. Reassure  
d. Repeat  cervical  smear  immediately  
e. Take  biopsy  
 
7.  A  25  year  old  man  has  rarely  left  his  house  for  the  past  12  months.  He  believes    
       that  his  neighbors  are  spying  on  him.  His  sleep  and  appetite  are  poor  and  he  has    
       lost  weight.  Mental  state  examination  reveals  that  he  hears  derogatory  voices    
       even  when  he  is  alone.  Neurological  examination  is  normal    
 
8.  A  housebound  78  year  old  man  with  severe  chronic  obstructive  pulmonary  disease    
         has  had  a  gradual  deterioration  over  recent  months  and  is  now  breathless  at  rest.    
         He  is  on  maximal  inhaled  therapy.  Results  show  the  following:  
         Ph:                  7.36  
         PaCO2:    5.9kPa  
         PaO2:          6.9kPa  
         What  is  the  single  most  appropriate  additional  treatment?  
a. Aminophylline  (oral)  
b. Angiotensin  converting  enzyme  (ACE)  inhibitor  (oral)  
c. Antibiotic  (oral)  
 

d. Oxygen  
e. Steroid  (oral)  
 
 
9.  A  78  year  old  man  with  past  history  of  ischemic  heart  disease  presents  with    
             yellow  haloes,  nausea  and  vomiting.  ECG  reveals  arrhythmia.  
             Which  of  the  following  medication  is  most  likely  responsible  for  his  symptoms?  
a. Amlodipine  
b. Digoxin  
c. Aminophyline  
d. Propanolol  
e. Diltiazem  
 
 
10.  A  70  year  old  man  has  been  posted  for  anterior  resection  following  a  diagnosis  of    
             carcinoma  of  the  sigmoid.  
             What’s  the  most  appropriate  prophylactic  antibiotics  pre-­‐operatively?  
a. Penicillin  
b. Broad  spectrum  antibiotics  
c. Antipseudomonas  
d. Amoxicillin  
e. Chloraphenicol  
 
 
 
11  A  75  year  old  woman  had  an  emergency  total  hip  replacement  following  a    
             fractured  neck  of  femur.  8  days  post  operatively,  she  has  a  sudden  onset  of  left-­‐        
           sided  chest  pain  and  an  episode  of  hemoptysis  associated  with  breathlessness.  
           What  single  investigation  is  most  likely  to  provide  the  definitive  diagnosis?  
a. Arterial  blood  gases  
b. Chest  X-­‐ray  
c. CT  pulmonary  angiogram  (CTPA)  
d. D-­‐dimer  
e. Electrocardiogram  (ECG)  
 
12.  A  20  year  old  woman  has  had  abdominal  pain  in  the  left  iliac  fossa  of  6  weeks    
           duration.  Over  the  past  48  hours,  she  had  had  severe  abdominal  pain  and  fever    
           of  39.C.  Pelvic  ultrasound  shows  a  complex  cystic  7cm  mass  in  the  left  iliac  fossa.    
           What  is  the  most  likely  diagnosis?  
a. Endometriosis  
b. Dermoid  cyst  
 

c. Ovarian  carcinoma  
d. Tubo-­‐ovarian  abscess  
e. Ectopic  pregnancy  
 
13.  A  59  year  old  man  has  sleep  disturbance,  loss  of  interest  and  weight  loss  of  2    
             months  duration  complicated  by  myocardial  infarction.  He  has  a  feeling  of    
             worthlessness  but  nil  suicidal  ideation.  
             What  is  the  single  most  appropriate  treatment?    
a. Diazepam  
b. Electroconvulsive  therapy  
c. Fluoxetine  
d. Imipramine  
e. Lithium  
 
14.  A  64  year  old  man  has  had  a  recent  onset  of  cough,  fever  and  weight  loss.  A    
             chest  X-­‐ray  shows  upper  lobe  cavitation.  Tuberculosis  is  a  possible  diagnosis.  He    
             is  sufficiently  unwell  to  require  hospital  admission  while  further  investigations    
             are  arranged.    
             What  is  the  single  most  important  immediate  step  to  take  in  order    
             to  reduce  the  risk  of  tuberculosis  spreading  among  hospital  patients  and  staff?  
a. Adopt  universal  precautions  
b. Arrange  immediate  bronchoscopy  to  establish  the  diagnosis  
c. Commence  anti-­‐tuberculous  treatment  without  delay  
d. Ensure  staff  have  received  Bacillus  Calmette-­‐Guerin  (BCG)  immunization  
e. Isolate  patient  in  a  negative  pressure  room  
 
15.  A  patient  has  been  having  influenza  infection  for  the  past  5  days  and  now    
             presents  with  very  painful  hemorrhagic  blisters  on  the  deep  meatal  skin.  
             What  is  the  most  appropriate  treatment?    
a. Oral  acyclovir  
b. IV  Acyclovir  
c. Supportive  treatment  only  
d. Gentamicin  
e. Flucloxacillin  
 
16.  During  the  past  3  years,  a  middle  aged  man  has  had  2  episodes  of  mania  and  3    
             episodes  of  depression  with  several  hospital  admissions.    
             What  is  the  best  management  in  this  case?    
a. Flupientixol  
b. Chlorpromazine  
c. Lithium  
 

d. Electroconvulsive  therapy  (ECT)  


e. Haloperidol  
 
17.  A  32  year  old  lady  has  recently  become  more  active,  sleeps  less  and  bought  a    
             house  and  2  new  cars.  What  is  the  most  likely  diagnosis?  
a. Bipolar  disorder  
b. Mania  
c. Hypomania  
d. Schizophrenia  
 
18.  A  45  year  old  woman  with  a  history  of  rheumatoid  arthritis  presents  to  the  A&E    
           department  with  a  2  day  history  of  hot,  painful  and  swollen  right  elbow  joint.  
           What  is  the  most  appropriate  management?  
a. Joint  aspiration  
b. Start  infliximab  
c. Oral  high  dose  
d. Short  course  of  methotrexate  
e. Dexamethasone  injection  
 
19.  A  24  year  old  unconscious  man  was  brought  by  his  friends  from  a  party  where  he    
           was  said  to  have  drank  vodka.  While  he  was  being  attended  to  by  the  doctor  in    
           the  emergency  department,  he  became  conscious  and  said  the  green  tie  the    
           attending  doctor  was  wearing  was  talking  to  him.  What  substance  could  this    
           patient  have  taken?  
a. Cocaine  
b. Alcohol  
c. Heroin  
d. Cannabis  
e. LSD  
 
20.  A  woman  present  s  with  a  history  of  poisoning  10  times  with  different  substances.    
             There  are  no  obvious  signs  of  depression  or  suicidal  behavior.  
             What  is  the  best  preventive  step?  
a. Open  access  to  A&E  
b. 24  hours  help  line  
c. CBT  
d. Antidepressants  
e. Insight  into  the  problem  
 
21.    A  23  year  old  woman  presents  with  offensive  vaginal  discharge.  Vaginal  Ph  is  4.5.  
               What  is  the  most  likely  causative  organism?  
 

a.  Gardenella  
b.  Trichomonas  
c.  Candida  
d.  Mycoplasma  
 
22.    An  18  year  old  woman  says  that  she  can’t  walk  around  as  she  is  very  big  for  that        
               room.  What  is  the  most  likely  hallucination?  
a.  Extracampine  visual  hallucinations  
b.  Lilliputian  visual  hallucinations  
c.  Alice  in  wonderland  syndrome  
d.  Hypnagogic  hallucinations  
 
23.    A  middle  aged  lady  presented  with  fever,  altered  sensorium,  bleeding  gums  and    
               jaundice.  Laboratory  results  show  deranged  LFT’s,  normal  PT/APTT,  fragmented    
               RBC’s  and  low  platelets.  What  is  the  likely  diagnosis?  
a.  Cholesterol  emboli  
b.  Hemolytic  uremic  syndrome  
c.  Thrombotic  thrombocytopenic  purpura  
d.  Hepatorenal  syndrome  
e.  Sepsis  
 
24.    A  child  comes  to  the  A&E  with  severe  asthma,  has  vomited  several  times  and  is    
               not  responding  to  salbutamol  nebulizer.    
               What  is  the  most  appropriate  management?  
a.    Salmetrol  
b.    Monteleuklaast  
c.    Prednisolone  
d.    Budesonide  inhaler  
e.    Oxygen  
f.    IV  Salbutamol  
 
25.    A  retired  shipyard  worker  has  pleural  effusion,  diffuse  shadow  and  is  breathless.          
               What  is  the  single  management  that  can  relive  his  symptoms?    
a.    Oxygen  therapy  
b.    Salbutamol  
c.    Radiotherapy  
d.    Aspiration  
 
 
26.    A  54  year  woman  was  found  to  have  glycosuria  at  a  routine  medical  examination.    
               She  has  a  body  mass  index  (BMI)  of  32kg/m2.  Her  fasting  blood  sugar  is  6mmol/l.    
 

               After  a  glucose  tolerance  test  2  hours  later,  her  plasma  glucose  is  9mmol/l.  
               What  is  the  single  most  likely  diagnosis?        
a.  Cushing’s  syndrome  
b.  Fanconi’s  syndrome  
c.  Impaired  glucose  tolerance  
d.  Type  1  diabetes  
e.  Type  2  diabetes  
 
27.    A  71  year  old  man  has  liver  metastases  from  an  unknown  primary.  He  has    
               cerebrovascular  dementia  with  some  right  sided  weakness  and  inability  to  talk.    
               He  is  alert  and  responsive.  He  has  stopped  taking  food  in  the  last  24  hours    
               having  previously  had  a  good  appetite  although  he  still  takes  his  medication.    
             There  are  white  patches  over  the  mucous  membranes  of  the  mouth  and  pharynx.  
a.    Amphotericin  intravenously  
b.    Chlorhexidene  mouthwash  
c.    Fluctoxacillin  suspension  
d.    Metonidazole  orally  
 
28.    A  4  year  old  boy  presents  with  a  febrile  convulsion  lasting  8  minutes  and  has    
               been  given  intravenous  lorazepam  to  control  the  convulsion.  
               What  is  the  single  most  likely  serious  side  effect?    
a.  Amnesia  
b.  Anaphylactic  shock  
c.  Bronchospasm  
d.  Cardiac  arrhythmia  
e.  Apnoea  
 
29.    A  36  year  old  man  had  hematemesis  and  was  found  at  endoscopy  to  have  a    
               duodenal  ulcer.  He  was  treated  with  H.pylori  eradication  therapy.  Some  months    
               later,  he  returns  with  epigastric  pain.    
               What  single  investigation  is  most  likely  to  review  eradication  of  H.pylori?  
a.  Barium  meal  
b.  C13  urea  breath  test  
c.  H.pylori  serology  
d.  Repeat  endoscopy  
 
 
30.    A  45  year  old  alcoholic  man  presents  with  tremors,  agitation  and  confusion.  He    
               has  been  given  chlordiazepoxide.  He  is  already  on  oral  thiamine.    
               What  is  the  most  appropriate  next  step  in  his  management?  
a.  Intravenous  fluids  
 

b.  Diazepam  
c.  IV  High  potency  vitamins  
d.  Oral  thiamine  
e. High  potency  vitamins  and  thiamine  
 
31.    A  20  year  old  woman  presents  with  a  lump  in  her  right  breast.  On  examination,    
               there  is  a  small  lump  in  the  right  breast  which  is  mobile,  firm  in  consistency  and    
               non-­‐tender.  What  is  the  most  likely  diagnosis?      
a.  Breast  cancer  
b.  Fibroadenosis  
c.  Fibroadenoma  
d.  Breast  abscess  
e.  Paget’s  disease  
 
32.    A  70  year  old  man  with  history  of  hypertension  and  on  treatment  with  different    
               antihypertensive  medication  presents  with  acute  pain  in  his  right  big  toe.    
               Which  one  of  the  following  medication  is  most  likely  to  be  responsible?  
a.  Beta  blocker  
b.  Calcium  channel  blocker  
c.  Lisinopril  
d.  Thiazide  
e.  Furosemide  
 
33.    A  40  year  old  woman  presents  with  haemoptysis  7  days  after  hysterectomy.    
               What  is  the  most  appropriate  diagnostic  investigation?  
a.  CT  head  
b.  Facial  X-­‐ray  
c.  MRI  scan  
d.  Venography  of  facial  veins  
e.  No  investigations  required  
 
34.    A  57  year  old  man  walks  into  the  bank  and  demands  money.  He  says  the  bank        
               belongs  to  him.  He  blames  the  bank  people  for  not  giving  him  his  money.  What    
               is  the  most  likely  diagnosis?  
a.  Delusion  of  grandiose  
b.  Mania  
c.  Paranoid  delusion  
d.  Nihilistic  delusion  
e.  Hypomania  
 
35.    A  60  year  old  woman  presents  with  per  vaginal  bleeding.  Her  last  cervical  smear      
 

               was  3  years  ago.  What  is  the  most  likely  diagnosis?  


a.  Cervical  cancer  
b.  Endometrial  carcinoma  
c.  Cervical  polyp  
d.  Vaginal  cancer  
e.  Pelvic  inflammatory  disease  
 
36.    A  man  with  anterior  resection  and  end  to  end  anastomosis  done  complains  of    
               severe  pain  in  the  chest  and  abdominal  distension.  What  is  the  most    
               appropriate  investigation  likely  to  review  the  cause  of  his  deterioration?  
a.  Plain  abdominal  X-­‐ray  
b.  Exploratory  laparoscopy  
c.  CT  scan  
d.  USG  
e.  Barium  enema  
 
37.    A  man  presents  with  scrotal  swelling.  The  swelling  is  cystic,  non-­‐tender  and            
               located  at  the  upper  pole  of  the  posterior  part  of  the  testes.    
               What  is  the  most  likely  diagnosis?  
a.  Epididymal  cyst  
b.  Hydrocele  
c.  Teratoma  
d.  Testicular  cancer  
 
38.    A  5  week  old  child  presents  with  vomiting.  H  feeds  readily  but  his  lips  are  dry    
               and  cracked.  What  is  the  investigation  of  choice?    
a.  USS  abdomen  
b.  Barium  study  
c.  Urea  and  electrolytes  
d.  CT  scan  abdomen  
 
39.    A  butcher  was  stabbed  accidentally  in  his  groin  and  bled  so  much  that  the  towel    
               was  soaked  in  blood.  His  BP  is  80/50mmHg  and  pulse  130b/m.      
               What  percent  of  circulatory  blood  did  he  loose?  
a.  Less  than  15%  
b.  15-­‐30%  
c.  20-­‐40%  
d.  40-­‐50%  
e.  Above  50%  
 
 
 

40.    A  45  year  old  woman  was  treated  for  persistent  pneumonia  for  2  weeks.  She    
               then  develops  diarrhea  with  blood  and  mucus  per  rectal.  What  is  the  most    
               appropriate  treatment?  
a.  Clarithromycin  
b.  Vancomycin  
c.  Tetracycline  
d.  Co-­‐amoxiclav  
e.  Rifampicin  
 
41.    A  25  year  old  woman  had  trauma  in  her  right  breast.  Later  she  develops  a  mass    
               in  the  right  breast.  What  is  the  most  likely  diagnosis?  
a.  Fibroadenoma  
b.  Breast  cancer  
c.  Fat  necrosis  
d.  Fibroadenosis  
e.  Duct  papilloma  
 
42    A  35  year  old  man  had  a  liver  biopsy  done  as  part  of  the  investigations  for    
               abnormal  liver  function  tests.  The  pathologists  states  that  special  stains    
               demonstrate  presence  of  a  very  large  amount  of  iron  pigment  within  the    
               hepatocytes.  
               What  is  the  single  most  likely  diagnosis?  
a.  Alpha-­‐1  antitrypsin  deficiency  
b.  Haemangioma  
c.  Haemochromatosis  
d.  Haemosiderosis  
e.  Wilson’s  disease  
 
 
43.    A  26  year  old  man  has  been  brought  to  hospital  by  his  parents.  He  has  runny    
               nose,  watery  eyes  and  muscle  cramps.  He  has  had  trouble  sleeping  for  the  past    
               3  days  and  appears  nervous.  He  has  been  attempting  to  stop  taking  drugs  for          
               the  past  3  days.    
               What  is  the  single  most  likely  substance  to  account  for  his  symptoms?  
a.  Alcohol  
b.  Cannabis  
c.  Cocaine  
d.  Heroin  
e.  Lysergic  acid  diethylamide  (LSD)  
 
44.    A  12  year  old  boy  with  type  1  diabetes  has  poor  long  term  control.  He  is    
 

               unconscious,  hyperventilating  and  dehydrated.  His  blood  sugar  is  26mmol/l.  


               What  is  the  single  most  important  initial  treatment?  
a.  IV    Albumin  
b.  IV    Bicarbonate  
c.  IV    Insulin  
d.  Subcutaneous  insulin  
e.  Saline  0.9%  
 
45.    A  50  year  old  woman  has  developed  a  breast  abscess  5  days  after  delivery  of  her        
               second  baby.  What  is  the  single  most  likely  causative  organism?  
a.  Group  b  beta-­‐haemolytic  streptococcus  
b.  Staphylococcus  albus  
c.  Staphylococcus  aureus  
 
 
46.    A  22  year  old  man  has  rushed  into  the  A&E  asking  for  help.  He  describes    
               recurrent  episodes  of  fearfulness,  palpitations,  faintness,  hyperventilation,    
               dryness  of  mouth  with  peri-­‐oral  tingling  and  cramping  of  the  hands.  His    
               symptoms  last  5  to  10  minutes  and  have  worsened  since  their  onset  3  months    
               ago.  He  is  worried  he  may  be  having  a  heart  attack.  An  ECG  shows  sinus    
               tachycardia  (100b/m).    
               What  is  the  single  immediate  appropriate  intervention?  
a.  High  flow  oxygen  
b.  Intravenous  sedation  
c.  Re-­‐breathe  into  a  paper  bag  
d.  Refer  for  anxiety  management  course  
e.  Refer  for  urgent  cardiology  opinion  
 
47.    A  26  year  old  man  with  vitiligo  presents  with  shortness  of  breath.  Full  blood    
               count  shows  high  MCV.  What  is  the  most  appropriate  diagnosis?  
a.  Alcohol  abuse  
b.  Pernicious  anemia  
c.  Folate  deficiency  
d.  Bullous  impetigo  
e.  Acute  leukemia  
 
48.    A  26  year  old  woman  with  ulcerative  colitis  presents  with  bleeding  per  rectum.    
               Full  blood  count  shows  Hb  of  8.4g/dl.  What  is  the  most  appropriate  treatment?  
a.  Steroid  
b.  Mesalazine  
c.  Methotrexate  
 

d.  Iron  sulphate  
e.  Colonoscopy  
 
49.    A  17  year  old  girl  was  brought  to  the  hospital  by  her  mother  due  to  the  fact  that    
             she  has  not  been  eating  well  in  the  last  couple  of  weeks.  She  is  lethargic  and    
             weak.  On  examination,  her  membranes  are  dry  and  she  is  feeling  thirsty.  The    
             patient  herself  is  not  concerned  as  she  is  busy  preparing  for  her  A  level  exams.  
             What  is  the  most  appropriate  management?  
a. Refer  to  psychiatrist  
b. Reassure  
c. Admit  in  medical  unit  
d. Refer  to  community  psychiatry  team  
e. Discharge  with  advise  
 
 
50.    A  45  year  old  woman  presents  with  incontinence  of  urine.  Which  part  of  the    
               colon  is  involved  in  urocele?      
a.  Sigmoid  
b.  Ascending  
c.  Descending  
d.  Rectum  
e.  Caecum  
 
51.    A  45  year  old  schizophrenia  patient  is  smashing  windows  in  the  hospital  after    
               being  admitted  to  a  psychiatrist  ward.    
               What  is  the  most  immediate  management  of  the  patient?  
a.  Haloperidol  
b.  Diazepam  
c.  Olanzapine  
d.  Risperidone  
e.  Chlorpromazine  
 
52.    A  38  year  old  man  was  involved  in  a  RTA.  Chest  X-­‐ray  shows  pneumothorax.    
               What  is  the  most  appropriate  treatment?    
a.  Chest  X-­‐ray  
b.  Needle  thoracocentesis  
c.  Chest  drain  
d.  Take  to  the  theatre  
e.  Observe  
 
53.    A  25  year  old  lady  who  is  15  weeks  pregnant  wants  to  know  if  her  baby  will  have    
 

               chromosomal  abnormality.  What  is  the  most  appropriate  investigation?  


a.  Villous  sampling  
b.  Amniocentesis  
c.  Cordocentesis  
d.  Ultrasound  scan  
e.  Peri-­‐implantation  diagnosis  
 
 
54.    A  40  year  old  woman  presents  with  some  inflammatory  changes  in  her  cervix.    
               What  is  the  most  appropriate  action?    
a.  Colposcopy  
b.  Smear  in  6  months  
c.  Repeat  smear  in  1  year  
d.  Repeat  smear  in  18  months  
e. Cervical  biopsy  
 
55.    A  35  year  old  woman  who  is  taking  Cerazette    (a  progesterone  only    
               Contraceptive  pill)  has  a  chest  infection  and  is  prescribed  amoxicillin.    
               Which  single  piece  of  advice  should  be  given  about  her  contraception?  
a.  No  additional  contraceptive  precautions  are  required  
b.  Use  additional  precautions  for  the  duration  of  the  antibiotic  course  
c.  Use  additional  precautions  for  the  duration  of  the  antibiotic  course  +2  days  
d.  Use  additional  precautions  for  the  duration  of  the  antibiotic  course  +7  days  
e.  Use  additional  precautions  for  the  remainder  of  the  current  packet  of  
cerazette  
 
 
56.    A  60  year  old  woman  is  recovering  post  operatively  following  a  vaginal    
               hysterectomy  and  anterior  vaginal  repair.  She  has  had  voiding  difficulty  and  has    
               been  catheterized  for  3  days.  A  catheter  specimen  of  urine  is  taken  due  to  a  low    
               grade  pyrexia  which  confirms  a  urinary  tract  infection  (UTI).    
               Which  single  organism  is  most  likely  to  be  causative?  
a.  Escherichia  coli  
b.  Klebsiella  pneumonia  
c.  Proteus  species  
d.  Pseudomonas  species  
e.  Staphylococcus  epidermidis  
 
57.    A  24  year  old  woman  has  had  an  abnormal  vaginal  discharge  for  the  past  week.    
               It  is  off-­‐white  and  non  itchy  with  an  offensive  odour.  She  has  had  1  sexual    
               partner  in  the  last  8  months  and  he  has  no  symptoms.  There  is  an  off  white    
 

               vaginal  discharge  of  Ph  6.4  pooling  in  the  posterior  fornix  with  no  inflammation    
               of  the  vulva  or  vagina.    
               Which  is  the  single  most  likely  finding  on  a  gram  stained  sample  of  the  vaginal    
               discharge?    
a. Gram  negative  intercellular  diplococci  
b. Gram  positive  and  negative  mixed  bacteria  
c. Numerous  lactobacilli  
d. Polymorphonuclear  leucocytes  
e. Yeast  cells  with  hyphae  
 
58.    A  72  year  old  woman  is  reviewed  following  a  course  of  oral  flucloxacillin  for  right    
               lower  limb  cellulitis.  The  local  protocol  suggests  oral  Clindamycin  should  be  used    
               as  the  next  line  drug.    
               Which  one  of  the  following  side  effects  is  most  important  to  warn  her  about?  
a.  Heart  burn  or  indigestion  
b.  Jaundice  
c.  Sore  throat,  bruising  or  lethargy  
d.  Avoid  any  food  or  drink  containing  alcohol  
e.  Diarrhea  
 
59.    A  50  year  old  obese  patient  has  hypertension  and  presents  with  daytime          
               headaches.  What  is  the  most  likely  cause?  
a.  Obstructive  sleep  apnea  
b.  Obesity  hypoventilation  syndrome  
c.  Narcolepsy  
d.  Stroke  
 
60.    An  80  year  old  diabetic  and  hypertensive  lady  presents  with  acute  onset  of        
               vomiting  and  vertigo  with  a  tendency  to  fall  to  the  right.  She  also  had  difficulty        
               swallowing.  Examination  shows  right  sided  partial  ptosis,  constricted  pupils  and        
               exophthalmus.  There  is  also  right  sided  soft  palatal  palsy  and  sensory  loss  of  the    
               face  on  the  right  side  and  left  half  of  the  lower  body.  The  diagnosis  is?  
a.  Weber  syndrome  
b.  Left  Middle  Cerebral  Artery  territory  infarct  
c.  Right  Posterior  Cerebral  Artery  infarct  
d.  Lateral  Medullary  Syndrome  
e.  Left  Anterior  Cerebral  Artery  infarct  
 
61.    A  woman  5  days  post  operative  for  bilateral  salphingo-­‐opherectomy  and    
               abdominal  hysterectomy  has  developed  abdominal  pain  and  vomiting  with    
               associated  abdominal  distension  and  cannot  pass  gas.  There  are  no  bowel    
 

               sounds  heard  and  she  is  also  dehydrated.    


               What  is  the  most  appropriate  next  step?          
a.  Plain  abdominal  X-­‐ray  
b.  Exploratory  laparoscopy  
c.  CT  scan  
d.  USG  
e.  Barium  enema  
 
 
 
 
 
62      A  10  year  old  child  presented  with  fracture  of  the  radius  which  was  treated  with          
                 a  plaster  and  now  complains  of  pain.  On  examination,  the  lower  limb  is  swollen      
                 and  well  perfused  and  the  pulses  are  present.  What  should  you  do  next?  
a.    Send  for  a  repeat  X-­‐ray  
b.    Remove  cast  
c.    Replace  cast  with  more  padding  
d.    Give  analgesia  
 
63.    A  34  year  old  man  sustains  a  fracture  to  the  shaft  of  the  femur  after  falling  from    
               the  roof  of  his  house.  On  examination,  his  distal  pulses  are  not  palpable.    
               Which  vessel  is  damaged?  
a.  Femoral  artery  
b.  Circumflex  femoral  artery  
c.  Profundus  femoral  artery  
d.  Popliteal  artery  
e.  Dorsalis  pedia  artery  
 
64.    A  5  year  old  boy  was  involved  in  a  road  traffic  accident  and  presents  with    
               abdominal  pain.  USS  abdomen  shows  a  small  hematoma  in  the  spleen.    
               What  is  the  most  appropriate  management?  
a.  Observe  
b.  Operate  immediately  
c.  Give  antibiotics  
d.  Discharge  home  
e.  Give  fluid  
 
65.    A  46  year  old  man  presents  to  the  emergency  department  after  being  hit  in  the    
               abdomen.  He  has  tenderness  on  the  episgastrium.    
               What  is  the  most  appropriate  management?  
 

a.  Observe  
b.  Operate  immediately  
c.  Refer  to  surgeon  
d.  Discharge  home  with  advice  
e.  Refer  to  GP  
 
 
 
 
 
 
 
66.      A  4  week  old  boy  has  been  brought  to  the  emergency  department  with    
                 vomiting  for  3  weeks.  His  mother  says  that  he  has  lost  weight  and  is  constantly    
                 hungry.  In  the  last  week,  the  vomiting  has  become  projectile  and  forceful.  An    
                 epigastric  mass  can  be  felt  during  feeding.    
                 What  is  the  single  most  urgent  investigation?  
a.      Full  blood  count  
b.      Liver  function  tests  
c.      Serum  calcium  concentration  
d.      Serum  electrolytes  concentration  
e.      Amino  acid  chromatography  
 
67.      An  18  year  old  woman  is  6  months  pregnant.  The  fetus  is  growing  on  the  10th    
               centile.  She  has  previously  had  2  early  miscarriages.  She  now  has  a  pulmonary    
               embolus.  Her  sister  has  a  thrombus  while  using  the  oral  contraceptive  pill.    
               What  is  the  single  most  likely  abnormality  on  screening  tests?    
a.  Anti-­‐phospholipid  antibody  
b.  Factor  V  Leiden  assay  
c.  Fibrinogen  degradation  products  
d.  Protein  C  and  S  
 
68.    A  70  year  old  man  had  a  right  hemi-­‐colectomy  for  caecal  carcinoma  6  days  ago.    
               He  now  has  abdominal  distension  and  recurrent  vomiting.  He  has  not  opened    
               his  bowels  since  surgery.  There  are  no  bowel  sounds.  He  has  a  white  cell  count    
               of  9x  10  m/l  and  a  temperature  of  37.C.    
               What  is  the  single  most  appropriate  next  management?  
a.  Antibiotic  therapy  invtravenously  
b.  Glycerine  suppository  
c.  Laparotomy  
d.  Nasogastric  tube  suction  and  IV  fluids  
 

e.  Total  parenteral  nutrition  


 
 
69.    A  30  year  old  lady  who  already  has  1  child  through  a  previous  caesarian  section    
               demands  a  reversible  contraceptive.  She  presently  experiences  heavy  and    
               painful  periods.  What  is  the  most  appropriate  contraceptive  that  you  will        
               recommend  for  her?  
a.  Combined  oral  contraceptive  pill  
b.  Progesterone  only  pill  
c.  Implanon  
d.  Danazol  
e.  Mirena  coil  
f.  IUCD  
 
70.    A  wife  is  concerned  that  her  neighbor  was  using  her  shower  when  nobody  was    
               at  home.  Her  husband  believed  her  and  called  the  police.  What  is  the  diagnosis?  
a.  Folie  a  trios  
b.  Folie  a  deux  
c.  Folie  a  quatre  
d.  Folie  de  fait  
 
71.    A  7  year  old  girl  with  allergy  became  acutely  unwell  while  visiting  a  friend’s    
               house  and  has  been  brought  immediately  to  the  emergency  department.  She  is    
               fully  conscious  but  has  stridor,  wheeze  and  generalized  erythematous  rash.  She    
               is  receiving  oxygen.  What  is  the  single  immediate  management?  
a.  Check  airway  patency  and  prepare  intubation  
b.  Give  0.25ml  in  100  epinephrine  (adrenaline)  by  IM  injection  
c.  Give  10mg  chlorphenaramine  by  IM  injection  
d.  Give  50ml  hydrocortisone  by  IM  injection  
e.  Obtain  secure  IV  access  
 
72.    An  89  year  old  man  has  been  breathless  at  rest  for  several  months.  His  current    
               medication  include  furesomide,  isorbibe  nitrate    and  digoxin.  He  is  in  sinus    
               rhythm  with  a  heart  rate  of  78b/m  and  a  BP  of  145/85mmHg.  His  serum  urea    
               and  electrolytes  are  normal.    
               What  is  the  single  most  appropriate  drug  to  use  at  this  stage?  
a.    Bendroflumethiazide  
b.    Doxasoxin  
c.    Nifedipine  
d.    Ramipril  
 
 

 
 
 
73.    A  30  year  old  woman  has  had  intense  itching  of  the  perineum  and  also  between    
               the  fingers  of  her  right  hand  for  2  weeks.  The  pruritus  is  more  pronounced  at    
               night  or  after  a  hot  shower.  Occasional  papulovesicular  lesions  with  excoriation      
               are  also  seen  on  the  perineum.  A  pelvic  examination  is  normal.  
               What  is  the  single  most  likely  organism  to  account  for  this  presentation?  
a.  Enterobius  vermicularis  
b.  Herpes  simplex  
c.  Pthrius  pubis  
d.  Sarcoptes  scabiei  
e.  Trichomonas  vaginalis  
 
74.    A  64  year  old  man  with  longstanding  alcohol  dependence  has  been  brought  to    
               the  emergency  department  having  been  found  wandering  unsteadily.  He  is    
               confused  and  ataxic  with  dry  mucus  membranes  and  bilateral  opthalmoplegia.    
               His  blood  pressure  is  115/85mmHg,  pulse  rate  is  96b/m,  temperature  of  36.8C    
               and  a  capillary  blood  glucose  of  3.5mmol/l.    
               What    is  the  single  most  urgent  intravenous  therapy?  
a.  Bolus  of  50%  glucose  
b.  Bolus  of  lorazepam  
c.  Bolus  of  thiamine  
d.  Infusion  of  0.9%  normal  saline  
e.  Infusion  of  5%  dextrose  solution  
 
 
75.    A  44  year  old  male  presents  with  painful  erection  with  curving  of  the  penis.          
               What  is  this  condition?  
a.  Balanitis  
b.  Phimosis  
c.  Priapism  
d.  Peyrone’s  disease  
e. Paraphimosis  
76.    A  32  year  old  lady  complains  that  she  hears  everyone  saying  that  she  is  an  evil    
               person.  What  type  of  hallucination  is  she  suffering  from?  
a.  Second  person  auditory  hallucination  
b.  Third  person  auditory  hallucinations  
c.  Echo  de  la  pense  
d.  Gedankenlautwerden  
 
 

77.    A  3  week  old  girl  has  been  brought  to  the  emergency  department  as  she  is    
               lethargic,  unwell  and  not  feeding.  Her  temperature  is  38.5C,  pulse  rate  is  90b/m    
               and  respiratory  rate  is  20b/m.  Her  skin,  chest,  abdominal,  ear  and  throat    
               examinations  are  normal.  What  is  the  single  most  appropriate  management?  
a.  Admit  for  full  infection  screening  
b.  Admit  for  full  infection  screening  and  start  broad  spectrum  antibiotic  
c.  Admit  for  serial  examination  and  observation  
d.  Discharge  with  broad  spectrum  antibiotic  
e. Discharge  with  oral  antipyretic  
 
78.    A  19  year  old  man  has  neck  pain  and  a  genralised  rash  for  12  hours.  He  is  not  on    
               any  medication  and  has  a  temperature  of  39.5C.  He  has  limitation  of  neck    
               flexion  and  no  lymphadenopathy.  The  rash  is  sparse,  mainly  on  the  trunk  and    
               looks  slightly  hemorrhagic  and  non-­‐blanching  on  pressure.    
               What  is  the  single  most  likely  diagnosis?  
a.  Henoch-­‐Schonlein  Purpura  
b.  Measles  
c.  Meningococcal  infection  
d.  Parvovirus  infection  
e.  Rubella  
 
79.    A  34  year  old  man  from  Zimbabwe  is  admitted  with  abdominal  pain  to  the    
               emergency  department.  An  abdominal  X-­‐ray  reveals  bladder  calcification.    
               What  is  the  most  likely  cause?  
a.  Schistosomiasis  
b.  Sarcoidosis  
c.  Leishmaniasis  
d.  Tuberculosis  
e.  Schistosoma  haematobium  
 
 
 
 
 
80.    A  74  year  old  female  presents  with  headache  and  neck  stiffness  to  the    
               emergency  department.  Following  a  lumbar  puncture,  the  patient  was  started    
               on  IV  ceftriaxone.  CSF  culture  grows  Listeria  monocytogens.  
               What  is  the  most  appropriate  treatment?  
a.  Add  IV  Amoxicillin  
b.  Change  to  IV  amoxicillin  and  gentamicin  
c.  Add  IV  Ciprofloxacillin  
 

d.  Add  IV  Co-­‐amoxiclav  


 
81.    A    45  year  old  presents  after  transurethral  prostatic  biopsy.    
                   What  electrolyte  imbalance  can  he  develop?  
a.      Hyperkalemia  
b.      Hyponatremia  
c.      Hypocalcemia  
d.      Hypernatremia  
e.      Hypercalcemia  
 
82.    A  man  was  brought  to  the  A&E  department  from  a  shopping  mall  after    
                   collapsing  there.  He  is  conscious  and  answering  questions.  His  ECG  shows    
                   irregular  rhythm.  What  is  the  choice  of  your  investigation?                
a.      CT  scan  
b.      MRI  
c.      24  hour  ECHO  
d.      ECHO  
 
83.    A  patient  is  on  morphine  but  cannot  tolerate  the  increasing  dose.    
                   What  will  you  give  to  this  patient?  
a.      Subcutaneous  morphine  
b.      Fentanyl  patches  
c.      Oxycodone  
d.      Methadone  
e.      Gabapentin  
f.      Bupremorphine  
 
84.    A  patient  is  on  morphine  but  this  alone  cannot  control  the  pain.  
                   What  will  you  give  this  patient?  
a. Subcutaneous  morphine  
b. Fentanyl  patches  
c. Oxycodone  
d. Methadone  
e. Gabapentin  
f.      Bupremorphine  
 
85.    A  patient  has  pain  controlled  but  needs  to  be  mobile.  
                   What  will  you  give  this  patient?  
a. Subcutaneous  morphine  
b. Fentanyl  patches  
c. Oxycodone  
 

d. Methadone  
e. Gabapentin  
f.        Bupremorphine  
 
 
86.    A  25  year  old  woman  has  her  first  routine  cervical  cytology  test  taken  as  part  of    
                   the  NHS  Cervical  Screening  Programme.  This  shows  mild  dyskaryosis,  CIN1  and    
                   she  is  advised  to  have  a  repeat  smear  performed  in  6  months  time.  She  has  had    
                   the  same  partner  for  18months  and  they  both  tested  negative  for  sexual    
                   transmitted  infections  at  the  start  of  the  relationship.  She  has  body  mass  index    
                   (BMI)  of  30kg/m  and  uses  a  progesterone  only  oral  contraceptive  pill.  She    
                   smokes  15  cigarettes  daily  and  drinks  approximately  25  units  of  alcohol  per    
                 week.  She  wants  to  know  if  there  is  anything  she  can  do  that  night  help  the    
                 abnormality  return  to  normal.    
                 What  single  action  that  she  can  advised  about  is  most  likely  to  decrease  her  risk?  
a.    Get  vaccinated  against  Human  Papilloma  Virus  (HPV)  infection  
b.    Give  up  smoking  cigarettes  
c.    Reduce  alcohol  consumption  
d.    Reduce  body  mass  index  
e.    Switch  to  an  alternative  contraceptive  pill  
 
 
 
 
87.    A  4  year  old  woman  requests  post-­‐coital  contraceptive.  Her  condom  broke  36    
                   hours  ago,  on  day  7  of  a  regular  29  day  cycle.  This  is  her  second  condom    
                   accident  in  2  months.  She  has  tried  the  oral  contraceptive  pill  but  stopped  it  6    
                   months  ago  because  of  concerns  about  weight  gain.  She  is  undecided  about    
                   future  contraceptive  use.  A  pregnancy  test  is  negative.    
                   Which  is  the  single  most  effective  form  of  post  coital  contraception  for  her?  
a.      A  combined  oral  oestrogen/progesterone  pill  
b.      A  progesterone  only  pill  
c.      Insertion  of  copper  containing  intrauterine  device  (IUCD)  
d.      Insertion  of  progesterone  containing  intrauterine  system  (IUS)  
e.      No  post  coital  contraception  is  required  
 
88.    A  27  year  old  man  has  had  mild  dysuria  for  1  week.  He  has  been  having  sex    
                   with  his  current  girlfriend  for  4  weeks,  occasionally  using  condoms  and  she  has    
                   no  symptoms.  He  last  had  sex  with  his  previous  female  partner  3  months  ago.    
                   There  is  a  slight  mucoid  discharge  at  the  urethral  meatus.    
                   Which  single  organism  is  the  most  likely  cause?    
 

a.      Chlamydia  trachomatis  
b.      Mycoplasma  hominis  
c.      Neiserria  gonorrhea  
d.      Trichomonas  vaginalis  
e.      Ureaplasma  urealyticum  
 
89.    A  15  year  old  previously  well  boy  has  had  ankle  and  facial  edema  for  1  month.    
                   His  blood  pressure  is  105/60mmHg.  His  eGFR  is  110ml/hour.  His  serum  albumin    
                   is  22g/l  and  urinalysis  shows  protein+++.    
                   What  is  the  single  most  likely  diagnosis?  
a.      Haemolytic  Uremic  Syndrome  
b.      IgA    Nephropathy  
c.      Membranous  glomerulonephritis  
d.      Minimal  change  glomerulonephritis  
e.      Post  streptococcal  glomerulonephritis  
 
90.    A  patient  with  a  known  history  of  alcohol  abuse  is  brought  to  the  A&E    
                   department  after  a  fall  in  which  he  hit  his  head.  His  GCS  is  9  and  the  CT  scan  for    
                   the  head  showed  a  crescent  shaped  hypo  dense  area.  
                   From  the  following  options  below,  choose  the  most  probable  diagnosis  for  the        
                   clinical  scenario  above.  
a.      Subarachnoid  hemorrhage  
b.      Epidural  hematoma  
c.      Subdural  hematoma  
d.      Diffused  axonal  injury  
e.      Brain  concussion  
 
 
91.    A  30  year  old  nurse  sustained  a  significant  needle  stick  injury  during  her  last    
                   shift  about  36  hours  ago.  The  patient  involved  is  HIV  positive  and  is  taking    
                   antiretroviral  therapy  with  his  last  viral  laod  being  1000copies/mL.  He  is    
                   hepatitis  B  virus  immune  and  negative  for  hepatitis  C  virus.  She  also  had    
                   unprotected  sex  early  in  her  current  menstrual  cycle  and  there  is  a  possibility    
                   that  she  may  be  pregnant.  What  is  the  single  most  appropriate  regarding  HIV    
                   post  exposure  prophylaxis  (PEP)?    
a.      It  is  already  too  late  for  her  to  start  taking  PEP  
b.      PEP  is  contraindicated  because  of  the  possibility  that  she  is  pregnant  
c.      She  does  not  need  PEP  as  the  patient’s  viral  load  is  so  low  
d.      She  should  start  PEP  without  further  delay  
e.      The  risks  associated  with  PEP  are  higher  than  the  risk  of  acquiring  HIV  
 
 

92.    A  37  year  old  woman  is  15  weeks  pregnant  and  requests  a  triple  test  to  rule  out    
                   Down’s  syndrome.  Which  is  the  single  most  appropriate  advice  to  give?  
a.      It  is  too  early  in  pregnancy  to  have  the  triple  test  
b.      It  is  too  late  in  the  pregnancy  to  have  the  triple  test  
c.      She  could  have  the  triple  test  arranged  today  
d.      The  triple  test  will  definitely  be  screened  positive  because  of  her  age  
e.      She  must  first  agree  to  have  an  amniocentesis  if  the  test  is  screen  psotive  
 
93    A  42  year  old  man  attends  a  genitourinary  clinic  asking  for  a  routine  check  for  a    
                   sexually  transmitted  infection.  He  has  no  symptoms  and  no  abnormal  clinical    
                   findings.  Serological  tests  for  syphilis  show  the  following:  
                   i.      Rapid  plasma  regain  (RPR)  positive  at  a  titre  of  1.64  
                   ii.    Treponema  pallidum  particle  agglutination  (TPPA)  assay  positive  
                   iii.  Flourescent  treponemal  antibody  absorption  test  (FTA-­‐ABS)  positive  
                   The  same  tests  were  negative  18  months  ago.  
                   Which  single  stage  of  syphilis  can  be  diagnosed?  
a.      Early  latent  
b.      Late  latent  
c.      Primary  
d.      Secondary  
e.      Tertiary  
 
94    A  50  year  old  man  is  experiencing  hoarseness  of  voice  a  day  after  having  a    
                   thyroidectomy.  What  is  the  most  likely  cause?    
a.      Hematoma  compression  
b.      Damage  to  vagus  nerve  
c.      Hypoglossal  nerve  
d.      Recurrent  laryngeal  nerve  palsy  
e.      Phrenic  nerve  palsy  
 
95    A  27  year  old  lady  who  is  16  weeks  pregnant  would  like  to  know  the  chance    
                   that  she  will  have  a  baby  with  Down’s  Syndrome  after  being  found  as  a  high    
                   risk  on  routine  blood  test.  What  is  the  single  most  appropriate  investigation?    
a.      Chorionic  villus  sampling    
b.      Tipple  test  
c.      Amniocentesis  
d.      Kleiur  test  
e.      Peri-­‐implantation  diagnosis  
 
96.    A  60  year  old  man  presents  to  the  emergency  department  with  left  sided  iliac    
                   fossa  pain  and  profuse  per  rectal  bleeding  .    
 

                   What  is  the  single  most  likely  diagnosis?  


a.      Gastroenteritis  
b.      Colonic  cancer  
c.      Rectal  cancer  
d.      Diverticulitis  
e.      Inflammatory  bowel  disease  
 
97.    A  47  year  old  man  presents  with  gradual  onset  of  forgetfulness.  His  uncle  had  a    
                   similar  condition.  What  is  the  most  likely  diagnosis?    
a.      Alzheimer’s  disease  
b.      Vascular  dememtia  
c.      Picks  disease  
d.      Huntington  disease  
e.      Prion  disease  
 
98.    An  83  year  old  man  has  been  admitted  to  the  hospital  after  having  diarrhea  for    
                   4  days.  He  has  been  constipated  for  the  past  2  months.  On  examination  she  is    
                   confused.  Her  carer  says  she  has  been  incontinent  of  urine  in  the  last  24  hours.    
                   What  is  the  most  appropriate  treatment?  
a.      Laxatives  
b.      Increase  water  intake  
c.      Intravenous  fluids  
d.      Antibiotics  therapy  only  
e.      Administer  a  phosphate  enema  
 
 
99    An  18  year  old  girl  has  been  diagnosed  with  anorexia  nervosa  and  mild    
                   depression.  She  has  cut  down  her  food  intake  and  her  BMI  is  15.  What  is  the    
                   appropriate  management?  
a.    Refer  to  eating  disorder  
b.    Refer  to  psychodynamic  assessment  
c.    Admit  in  medical  ward  
d.    Refer  to  psychiatrist  
e.    Admit  under  mental  act  
 
100.    A  35  year  old  lady  had  IUCD  inserted  8-­‐9  months  ago.  On  routine  follow  up  the    
                   same  thread  was  missing.  Uterine  ultrasound  shows  no  IUCD  in  the  uterus.    
                 What  is  the  best  investigation?  
a.    Laparoscope  
b.    Pelvic  CT  
c.    Laparotomy  
 

d.    Abdominal  X-­‐ray  
e.    Speculum  examination  
 
101    A  15  year  old  girl  presents  with  sticky  red  eyes  for  a  few  days.  There  is  no    
                   history  of  trauma.  What  is  the  appropriate  management?  
a.      Topical  antibiotic    
b.      Anti-­‐histamine  
c.      Steroids  
d.      IV  antibiotics  
e.      Analgesia  
 
102    A  5  year  old  boy  with  generalized  oedema  presents  with  ankle  swelling  and    
                   facial  puffiness.  He  has  hematuria  and  proteinuria  of  12-­‐24hours.  His  serum    
                   creatinine  is  60mmol/l  and  serum  albumin  is  15gm/l.  What  is  the  diagnosis?  
a.      IgA  nepropathy  
b.      HSP  
c.      Minimal  change  glomerulonephropathy  
d.      Post  streptococcal  glomerulonephritis  
e.      Membranous  glomerulonephritis  
 
 
103    A  43  year  old  man  is  due  to  have  a  resection  of  an  annular  carcinoma  of  the    
                   sigmoid  colon.  He  will  be  receiving  co-­‐amoxiclav  and  metronidazole  antibiotic    
                   prohylaxis.  What  is  the  single  optimal  drug  route?  
a.      Intramuscular  
b.      Intravenous  
c.      Local  instillation  
d.      Oral  
e.      Per  rectal  
 
104    A  45  year  old  man  has  developed  an  annular  rash  with  scaly  edge  on  his  thigh.    
                   The  rash  has  been  spreading  over  the  last  3  weeks.  He  has  some  general  aches    
                   and  pains.  What  is  the  single  most  useful  investigation?  
a.      Anti-­‐nuclear  antibodies  
b.      Biopsy  of  the  lesion  
c.      Lyme  antibodies  
d.      Skin  scrap  for  mycology  
e.      Skin  swab  for  bacteria  
 
105.    A  20  month  old  boy  has  developed  a  peri-­‐oral  blistering  rash.  The  size  of  the    
                   lesions  range  from  2-­‐10mm.  some  of  the  lesions  are  weeping  with  straw    
 

                   colored  fluid  while  others  have  formed  crusts  with  surrounding  redness.  He  has    
                   a  temperature  of  38.5C  but  is  otherwise  well.    
                   What  is  the  single  most  likely  diagnosis?  
a.      Bullous  impetigo  
b.      Chicken  pox  
c.      Hand,  foot  and  mouth  disease  
d.      Kawasaki  disease  
e.      Shingles  
 
106.    A  73  year  old  man  who  was  a  smoker  has  quit  smoking  for  the  past  3  years.  He    
                   now  presents  with  hoarseness  of  voice  and  cough  since  the  past  3  weeks.  On  X-­‐  
                   ray,  a  mass  is  visible  in  the  mediastinum.    
                   What  is  the  best  investigation  to  confirm  the  diagnosis?    
a.      Bronchoscopy  
b.      Thoracoscopy  
c.      USG  
d.      CT  thorax  
 
107.    A  9  month  old  girl  has  had  mild  diarrhea  and  vomiting  for  8  hours.  What  is  the    
                   single  most  appropriate  initial  investigation?  
a.      Flexible  cystourethroscopy  
b.      urea  and  electrolytes  
c.      Ultrasound  scan  of  the  renal  tract  and  pelvis  
d.      Urinalysis  for  nitrates  and  leucocytes  
e.      Uro-­‐dynamic  studies  
 
108    A  74  year  old  woman  is  found  to  have  a  single  large  gall  stone  in  her  gall    
                   bladder  on  ultrasound  scan  performed  for  left  sided  abdominal  repair.    
                   What  is  the  single  most  appropriate  treatment  for  the  gallstone?    
a.      Chemo-­‐deoxycholic  acid  
b.      Laparoscopic  cholecystectomy  
c.      Lithotripsy  
d.      Low  fat  diet  
e.      No  treatment  
 
109.    A  37  year  old  woman  has  a  cervical  smear  result  showing  inflammatory    
                   changes  and  no  dsykaryosis.  She  has  no  history  of  vaginal  discharge  and    
                   inspection  of  the  cervix  is  normal.  
                   What  is  the  single  most  appropriate  initial  management?    
a.      Endocervical  swab  
b.      No  action  required  
 

c.      Refer  for  colposcopy  


d.      Repeat  smear  in  6  months  
e.      Repeat  smear  in  3  years  
 
110.    A  65  year  old  lady  presents  with  dizziness  which  mostly  occur  on  exercise.    
                   What  is  the  most  appropriate  investigation?  
a.      ECG  
b.      CT  brain  
c.      Chest  X-­‐ray  
d.      Blood  glucose  
e.      CT  brain  
 
110    An  18  year  old  lady  collapsed  in  the  gym.  ECG  shows  minor  non-­‐specific    
                   abnormalities.  What’s  the  most  appropriate  diagnosis?  
a.      HOCM  
b.      Atrial  fibrillation  
c.      Ventricular  fibrillation  
d.      Echocardiogram  
e.      CT  scan  brain  
 
111    A  68  year  old  man  has  increasing  dysphagia  for  solid  food  for  3  months  and  has    
                   lost  5kg  in  weight.    
                   What  single  investigation  is  most  likely  to  lead  to  a  definitive  diagnosis?  
a.      Barium  swallow  
b.      Chest  X-­‐ray  
c.      CT  scan  
d.      Endoscopy  and  biopsy  
e.      Visuo-­‐fluoroscopy  
 
112    A  34  year  old  lady  presents  with  bleeding  per  vaginum  7  days  after  delivery.    
                   What  is  the  most  appropriate  treatment?  
a.      Reassure  
b.      Do  urine  dipstix  
c.      Antibiotics  
d.      Transvaginal  ultrasound  scan  
e.      No  investigation  required  
 
113    A  45  year  old  man  with  history  of  alcohol  abuse  presents  with  a  fall.  He  smells    
                   of  alcohol  and  his  blood  glucose  is  5mmol/l.    
                   What  is  the  most  appropriate  investigation?    
a.      Blood  glucose  
 

b.      CT  scan  of  the  brain  


c.      ECG  
d.      Chest  X-­‐ray  
e.      Echocardiography  
 
114.    A  30  year  old  man  who  is  an  intravenous  drug  abuser  presents  with  shortness    
                   of  breath  on  exercise.  What  is  the  most  likely  location  of  his  lesion?  
a.      Lungs  
b.      Atrioventricular  node  
c.      Triscupid  valve  
d.      Bundle  of  His  
e.      Aortic  valve  
 
115    A  45  year  old  man  presents  with  chest  pain  radiating  to  the  left  arm.  ECG    
                   shows  ST  elevation  in  lead  2  and  3.  He  also  had  a  myocardial  infarction    
                   treatment  with  thrombolysis  last  year.    
                   What  is  the  most  appropriate  management?  
a.      Streptokinase  
b.      Altepase  
c.      Percutaneous  coronary  intervention  
d.      Tissue  plasminogen  activator  
e.      Cardiac  passing  
 
116    A  47  year  old  lady  presents  with  palpitations.  ECG  shows  sinus  rhythm.  She  says    
                   that  she  has  lost  weight  despite  good  appetite.    
                   What  is  the  most  appropriate  investigation?  
a.      24  hour  ECG  
b.      Echocardiography  
c.      Thyroid  function  test  
d.      Standing  and  lying  blood  pressure  
e.      Review  medication  
 
117.    A  40  year  old  man  presents  with  sudden  onset  of  severe  epigastric  pain  and    
                   vomiting.  He  has  a  history  of  indigestion.    
                   What  is  the  most  likely  abnormality  on  chest  X-­‐ray?  
a.      Dilated  loops  of  bowel  
b.      Free  gas  in  the  mediastinum  
c.      Gas  under  the  diaphragm  
d.      Normal  
e.      Surgical  emphysema  
 
 

118.    A  22  year  old  woman  has  been  brought  to  the  emergency  department  having        
                   taken  35  tablets  of  paracetamol  following  an  argument  with  her  partner.  She    
                   has  no  previous  psychiatric  history  and  is  physically  well.    
                   What  is  the  single  most  appropriate  management?  
a.      Admit  to  medical  ward  
b.      Admit  to  psychiatric  ward  
c.      Discharge  home  with  advice  
d.      Refer  to  clinical  psychologist  
e.      Refer  to  social  worker  
 
 
119.    A  76  year  old  woman  on  paroxetine  has  become  increasingly  weak  and    
                   confused.  What  is  the  single  most  likely  reason  for  her  confusion?  
a.      Hypercalcemia  
b.      Hypernatremia  
c.      Hypocalcemia  
d.      Hypoglycemia  
e.      Hyponatremia  
 
 
120.    A  65  year  old  lady  presents  with  hypertension  without  any  history  of  cough  and    
                   renal  failure.  What  drug  should  be  prescribed  to  her?    
a.      Thiazide  diuretics  
b.      ACE-­‐Inhibitors  
c.      Beta  blockers  
d.      Calcium  channel  blockers  
e.      Alpha  blockers  
 
 
121    A  45  year  old  man  with  idiopathic  epilepsy  has  been  brought  to  the  emergency    
                   department  with  recurrent  seizures  over  40  minutes.  He  is  having  a  generalized    
                   tonic-­‐clonic  seizure  with  noisy  breathing  and  is  cyanosed.    
                   What  is  the  single  most  appropriate  immediate  management?  
a.      Diazepam  rectally  
b.      Oxygen  via  facemask  
c.      Paraldehyde  intravenously  
d.      Phenytoin  intravenously  
e.      Secure  airway  
 
122    A  56  year  old  man  comes  presents  with  a  history  of  right  sided  weakness  and    
                   left  sided  visual  loss.  Where  is  the  site  of  the  occlusion?  
 

a.      Anterior  cerebral  artery  


b.      Internal  carotid  artery  
c.      Posterior  cerebral  artery  
d.      Middle  cerebral  artery  
e.      Posterior  communicating  artery  
f.      Anterior  communicating  artery  
 
123    A  32  year  old  man  with  schizophrenia  and  a  history  of  violence  and  distressing    
                   auditory  hallucinations  was  admitted  to  the  ward  with  aggressive  behavior  and    
                   has  already  smashed  his  room.  He  is  refusing  oral  medication.    
                   What  is  the  single  most  appropriate  injection?  
a.      Flupenthixol  
b.      Fluphenazine  
c.      Haloperidol  
d.      Paraldehyde  
e.      Risperidone  
 
 
124    A  young  anxious  mother  of  a  10  month  old  boy  comes  to  you  and  requests  a    
                   test  for  cystic  fibrosis?  What  is  the  most  appropriate  investigation?  
a.      Sweat  test  
b.      Heel  prick  test  
c.      Breath  test  
d.      Chest  X-­‐ray  
 
125    A  67  year  old  man  being  managed  for  a  malignancy  develops  neutropenic  fever.    
                   He  has  been  commenced  on  ticacillin,  tazobactam  and  gentamicin.  He  has  also    
                   recently  commenced  metropenem  but  on  the  3rd  day  his  temperature  still        
                   remains  greater  than  39.C.  Two  blood  tests  and  urine  cultures  show  no    
                   organism.    
                   Investigation  results  are  as  follows;  
                   Hb  104g/dl,      WBC  <  0.5  x10,      Platelets  15x  10.  
                   What  will  you  do  next?    
a.      Continue  IV  antibiotics  and  add  oral  antifungals  
b.      Continue  antibiotics  and  add  IV  antifungals  
c.      Stop  antibiotics  
d.      Continue  only  present  antibiotics  
 
 
 
 
 

126    A  73  year  old  woman  with  skeletal  and  brain  metastases  from  breast  cancer    
                   has  worsening  low  back  pain  and  blurring  of  vision.  She  has  weakness  of  her    
                   legs,  minimal  knee  and  absent  ankle  tendon  reflexes  and  a  palpable  bladder.    
                   Her  power  is  2/5  at  the  hip,  3/5  at  the  knee  and  ankle  with  tenderness  over  the                    
                   second  lumbar  vertebrae.  There  is  reduced  sensation  in  the  perineum.  She  was    
                   started  on  dexamethasone  16mg  daily.    
                   What  is  the  single  most  likely  cause  of  her  weakness?  
a.      Paraneoplastic  neuropathy  
b.      Progression  of  a  brain  tumor  
c.      Prolapsed  intervertebral  disc  at  L2/L3  
d.      Spinal  cord  compression  
e.      Steroid  induced  myopathy  
 
127    A  62  year  old  woman  has  reversible  airways  obstruction.  She  is  on  salbutamol    
                   and  beclomethasone  inhalers  at  maximum  dosages  but  for  several  months  has    
                   been  waking  up  at  night  with  breathlessness  and  requiring  her  salbutamol.    
                   What  is  the  most  appropriate  management?  
a.      Oral  prednisolone  
b.      Magnesium  sulphate  
c.      Ipatropium  
d.      Long  acting  B  2  agonist  
e.      Salbutamol  nebulizer  
 
128    A  67  year  old  male  presented  with  2  months  of  abdominal  pain  and  watery    
                   diarrhea.  There  is  neither  blood  nor  mucous.  Patient  is  a  known  case  of    
                   diabetes  mellitus.  General  stool  examination  revealed  occult  blood.  A  contrast    
                   study  done  excluded  any  strictures  in  the  GIT.    
                   What  is  the  most  appropriate  investigation  to  reach  the  diagnosis?    
a.      FBC  
b.      Thyroid  function  test  
c.      Stool  culture  
d.      Capsule  endoscopy  
e.      Barium  enema  
 
129    A  22  year  old  male  law  student  presented  to  the  OPD  with  tremors,  dysphagia    
                   and  dystonia.  He  has  signs  of  dementia  and  is  depressed.  He  has  not  been    
                   performing  well  at  university.  On  examination  he  has  multiple  joint  tenderness.    
                   What  is  the  most  definitive  investigative  tool?  
a.      Serum  Anti-­‐trypsin  levels  
b.      Nicotinamide  levels  
c.      Liver  biopsy  
 

d.      24  hour  urinary  copper  excretion  


e.      MRI  scan  of  the  brain  
 
130      A  45  year  old  metal  worker  presents  to  the  emergency  department  with  pain    
                     in  his  right  eye.  He  says  he  had  felt  a  transient  pain  in  his  right  eye  at  work    
                     yesterday  when  he  was  cutting  metals.    
                     What  is  the  most  appropriate  investigation?  
a. Fluorescence  
b. X-­‐ray  orbit  
c. Fundoscopy  
d. Slit  lamp  examination  
e. Non  required  
 
131.    A  4  year  old  boy  with  Down’s  Syndrome  has  proven  middle  ear  effusion    
                   bilaterally  that  has  been  present  for  6  months.    
                   What  is  the  most  appropriate  management?  
a.      Adenoidectomy  
b.      Bilateral  mastoidectomy  
c.      Cochlear  implant  
d.      Bilateral  ventilation  tube  insertion  
e.      Watch  and  wait  for  3  months  
 
132    A  34  year  old  lady  presents  with  recurrent  miscarriages  at  12  weeks  gestation.    
                   What  is  the  most  appropriate  investigation?  
a.      Auto-­‐antibodies  
b.      Coagulation  screen  
c.      No  investigation  required  
d.      Lupus  anti-­‐coagulant  
e.      Chromosomal  analysis  
 
133    A  4  year  old  boy  presents  with  bleeding  from  the  nose.  Bleeding  time  is    
                   increased.  What  is  the  most  likely  diagnosis?  
a.      ITP  
b.      Haemophilia  
c.      HSP  
d.      Congenital  liver  disease  
e.      Von-­‐Willebrand  disease  
 
 
 
 
 

134    A  mother  brings  her  7  year  old  child  to  the  hospital.  She  would  like  to  know  if    
                   her  child  is  diabetic.  What  is  the  single  most  appropriate  investigation?  
a.      OGTT    
b.      Fasting  blood  glucose  
c.      HbAc  
d.      Random  blood  glucose  
e.      Auto-­‐antibodies  
 
135    A  70  year  old  man  presents  with  anaemia.  Hb  is  8.7  and  MCV  is  110.    
                   Which  of  the  following  is  the  most  likely  cause?  
a.      Haemorrhage  
b.      Alcohol  
c.      Vitamin  B12  deficiency  
d.      Aplastic  anaemia  
e.      None  of  the  above  
 
136    A  5  year  old  boy  presents  with  Iron  and  Folate  deficiency.    
                   Which  one  of  the  following  is  the  most  likely  cause?    
a.      Leukemia  
b.      Aplastic  anaemia  
c.      Celiac  disease  
d.      Liver  disease  
e.      None  of  the  above  
 
 
137.    A  45  year  old  man  with  prostate  cancer  has  just  finished  chemotherapy  and    
                   radiotherapy.  He  presents  with  rigors  and  pain  radiating  to  his  groin.    
                   What  is  the  most  appropriate  investigation?  
a.      IVU  
b.      USS  
c.      X-­‐ray  of  the  kidney,  ureters  and  bladder  (KUB)  
d.      Serum  calcium  
e.      Dietary  history  
 
 
138    A  46  year  old  man  is  about  to  undergo  colorectal  surgery.  
                   What  is  the  pre-­‐operative  antibiotics  that  you  will  give?  
a.      Cefuroxime  
b.      Cefuroxime  and  metronidazole  
c.      Amocixillin  
d.      Vancomycin  
 

e.      Penicillin  
 
137.    What  is  the  most  common  congenital  hernia  that  is  discovered  as  an  accidental    
                   finding  in  adults.  From  the  following  options  below,  choose  the  most  probable    
                   options  for  the  question  above.  
a.      Foramen  of  Morgagni  hernia  
b.      Rupture  of  the  diaphragm  
c.      Sliding  hiatus  hernia  
d.      Foramen  of  Bochdalek  hernia  
e.      Para  oesophageal  hiatus  hernia  
 
138    A  26  years  old  male  has  been  operated  for  abdominal  trauma  and      
                   splenectomy  was  done.  On  the  3rd  operative  day,  the  patient  developed  acute      
                   abdominal  pain  and  distension  in  the  upper  abdominal  area  with  hypotension.    
                   On  insertion  of  ryles  tube,  2  litres  of  coffee  ground  fluid  was  aspirated.    
                   From  the  following  options  below,  what  is  the  most  probable  diagnosis  of  the      
                   condition  above?  
a.      Acute  gastric  dilatation  
b.      Reactionary  haemorrhage  
c.      Subphrenic  abscess  
d.      Deep  vein  thrombosis  
e.      Left  lower  lobe  atelectasis  
 
 
 
 
 
139    A  1  year  old  child  has  a  small  protrusion  from  the  umbilicus  which  according  to    
                   the  mother  becomes  bigger  when  the  child  cries  but  regresses  back  to  its    
                   normal  size  when  the  child  stops  crying.    
                   What  is  the  single  most  appropriate  treatment?  
a.      Surgical  operation  
b.      Teach  the  mother  exercises  for  strengthening  the  abdomen  
c.      Reassure  
d.      Refer  to  a  specialist  
 
140    An  87  year  old  woman  with  a  history  of  hypertension  has  acute  breathlessness.      
                   She  has  a  respiratory  rate  of  32breaths/minute  with  widespread  lung  crackles    
                   and  a  pulse  rate  of  120b/m.  Her  blood  pressure  is  160/90mmHg  with  a  deep    
                   venous  pressure  and  peripheral  oxygen  saturation  (PO2)  of  85%.    
                   What  is  the  single  most  appropriate  initial  management?        
 

a.      IV  antibiotics  
b.      IV  furosemide  
c.      Nitrate  infusion  
d.      Nebulised  salbutamol  
e.      100%  oxygen  
 
141.    A  couple  have  not  been  able  to  conceive  despite  having  regular  sex  for  2  years.    
                   The  lady’s  menstrual  cycles  are  regular  and  of  28  days.  21  days  progesterone    
                   test  shows  low  levels.  What  is  the  best  treatment  for  this  clinical  scenario?  
a.      Bromocriptine  
b.      Clomiphene  
c.      IVF  
d.      Testosterone  
 
142.    A  79  year  old  woman  has  been  diagnosed  with  type  2  diabetes.  Her  body  mass    
                   index  (BMI)  is  22kg/meter  square.  Random  plasma  glucose  concentrations  are    
                   8mmol/l  and  10mmol/l.  Her  blood  pressure  is  130/80mmHg  and  her  fasting      
                   cholesterol  is  5.7mmol/l.  She  is  currently  symptom  free  but  has  micro-­‐  
                   albuminuria.    
                   What  is  the  single  most  appropriate  drug  management?    
a.      ACE  inhibitor  and  glibenclamide  
b.      ACE  inhibitor  and  metformin  
c.      Statin  and  ACE  inhibitor  
d.      Statin  and  glibenclamide  
e.      Statin  and  metformin  
 
 
 
 

143. A 30 year old lady who has been stopped OCPS 18 months ago
has Amenorhea since 9 months. Pregnancy test negative. What is
the most likely cause?

A. Hypothalamic amenorrhea

B. Prolactinoma

C. OCPS

D. Primary amenorrhoea
 

E. Menopause

144. A 45 year old obese man snores during the night and is sleepy
during the day time. What is the most appropriate investigation?

A. Overnight pulse oximetry

B. ABGs

C. Chest X-ray

D. CT scan chest

E. ECG  

145. On a second day postnatal rounds 30 year old lady is concered


that her son as might be deaf is deaf. What is the most appropriate
management?

A. Tuning fork.

B. Startle Reflux

C. Pure tone audiogram,

D. Auditory Brain stem Response.

E. Distracting

146. A 40 year old man presents with P ataxia, vertigo and nystagmus.
Where is the Site of lesion?

A. 8th nerve Auditory canal Cerebellum


 

B. Brain stem  

C.  

147. A 70 year old man man fell down in the garden and 24hr ECG was
taken which was normal. What would be the most appropriate next
step investigation?

A. CT brain

B. ECHO

C. MRI brain

D. ABG

E. Tilt test

148. A 25 year old man has had an irritating sticky red eye for 12
hours. What is the most appropriate management?

A. Antibiotic (topical)

B. Antibiotic (oral)

C. Pilocarpine drops

D. sterile saline

E. Anti-septic

149. A 30 year old woman taking COCP pills is on Amoxicillin tabs for
a week. What advice would you give to the patient?

A. Stop COCP and continue Amoxicillin

B. use condom for one week


 

C. Use condom for two weeks

D. No precaution required

150. A 70 year old man while he was moving his house cupboard fell
on his leg and he was stuck for 4 hours. Which enzyme will
measure?

A. creatine kinase  

B. LDH

C. Troponin

D. AST

E. ALT  

151. A 2 days old child is not cyanosed, but has Pan systolic
murmur and symptoms of heart failure

a. Tetrada of Fallot

b. VSD

c. Transposition of great vessels

152. A 30 year old man man returns to his hostel and gets
headache and lethargy. Now presents with fever and low lung
crepitation. What is the single most likely cause?
 

A. Legionella Pneumonia

B. Mycoplasma

C. Staphylococcus

D. Streptococcus

153. A 40 year old woman develops fever, jaundice and right


Hypochondria pain within 24 hours after ERCP in which a bile duct
stone was removed. What is the single most likely cause?

A. Infection

B. Obstruction

C. Perforation bile duct

D. Bud chiary syndrome

154. A 30 year old woman was treated successfully for CIN2 two years
ago. She was a heavy smoker, but stopped a few months ago. She
was on oral contraceptives all these years but her partner now uses
condoms. She is anxious as her anut died of ovarian cancer at the
age of 48. What is the single factor that puts her at risk of ovarian
cancer?

A. Age

B. Family history

C. CIN2

D. Smoking

E. Oral contraceptives  
 

155. A 45 year old mna presents with restrosternal chest pain radiating
to thethroat. What is the single most diagnostic investigation?

a. ECG

b. Troponin levels

C. Angiography

D. MRI scan chest

E. ECHO  

156. A 60 year old lady develops focal neurological symptoms a week


after head injury. Which vessel is she likely to be bleeding from?

a. Carotid

b. Middle meningeal

c. Basilar

d. cerebral vein

157. A 4 years old boy is severely dehydrated. 4 attemps of IV line


have been unsuccessful. What is next most appropriate step?

a. Internal jugular

b. Femoral vein

c. Saphenous
 

D. Cut down

E .interossei

158. A 45 year old woman has Electric shock like pain in left arm. What
is the single most appropriate management?

a. NSAIDs

b. Gabapentin  

c. Morphine  

d. Amitriptyline  

159. A 47 year old man has a temperature of 39.1 C and is delirious.


He has developed blisters mainly on his trunk. He last travelled five
months ago to Italy. What is the single most likely diagnosis?

A. Acute pemphigoid

B. Acute viral haemorrhagic fever

C. Chicken pox

D. Lyme disease

E. Shingles  

160. A 30 year old woman has a painless lump in the outer aspect
 

of her left breast. She has had a previous lump. Her grandmother
had breast cancer at 70 years of age. She has a 1cmx1cm smooth,
firm, discrete mobile lump in the outer quadrant of her left
breast. What is the single most likely diagnosis?

A. Breast abscess

B. Breast carcinoma

C. Breast cyst

D. Fibroadenoma

E. Sebaceous cyst

161. A 38 year old woman is in her first pregnancy at 16 weeks


gestation and is very anxious about the possibility of chromosomal
abnormalities in her baby. What is the single best test to give a
definitive diagnosis?

A. Amniocentesis

B. Chorionic villous sampling

C. Maternal alphafetoprotein

D. Nuchal fold thickness

E. Parental karyotype  

           162.      A 19 year old man has had fever, neck stiffness and generalized
rash for 12 hours. He is not on any medication. He has a
temperature of 39.5 C. He has limitation of neck flexion and no
lymphadenopathy. The rash is sparse and mainly on the trunk and
looks slightly haemorrhagic, non-blanching on pressure. What is
 

the single most likely diagnosis?

A. Henoch-Scholein purpura

B. Measles

C. Idiopathic thrombocytopaenic purpura

D. Meningococcal infection

E. Chicken pox

163. A 32 year old woman has recurrent breast abscesses. The


latest one was six weeks ago. There is an area of induration
between the junction on the areola which is discharging
pus.. What is the single most likely diagnosis?

A. Breast abscess

B. Duct ectasia

C. Mamary duct fistula

D. Fibroadenoma

E.  Mastitis  

164. A 40 year old woman who smokes 10 cigarettes a day has a


greenish discharge from both breasts. What is the single most
likely diagnosis?

A. Duct ectasia

B. Duct papilloma

C. Fibrocystic disease of the breast

D. Fibroadenoma
 

E. Intraduct papilloma

166. A 30 year old man has sudden onset of eye pain with a dilated
pupil. What is the management?

A. Analgesia

B. Analgesia and routine ophthalmology referral

C. Analgesia and urgent ophthalmology referral

D. Reassure and send home.

169. A man has pierced his foot with a metal spike 10 hours ago.
He has had 4 tetanus vaccinations with the last being 8 years
ago. What is the most appropriate management?

A. Tetanus toxoid

B. Tetanus toxoid + immunoglobulin (Ig)

C. Tetanus Ig

D. Tetanus toxoid + Ig + antibiotics

E. Tetatnus Ig + antibiotics.

170. A female who presented with cervical ectropion that did not
bleed on touch. What investigations?

A. ultrasound pelvis

B. no investigation needed

C. Antibiotics
 

E. cervical smea

171. 7 years old boy presents with epistaxis of two hour duration. The bleeding has
been controlled. His investigations are as follows:

Platelets 210,000
PT 13 seconds
APTT 42 seconds
Bleeding time: Normal

Which one of the following is the most likely diagnosis?

a.Hemophilia

b. Vonwilibrand disease
c. Idiopathic thrombocytopenic purpura
d. Vitamin k deficiency
e. Liver disease

172. A boy with recurrent swollen joints and not able to participate in sports. What
investigation will you do?
a. rheumatic factor
b. ASO titre
c. X-ray
 

A young man present with recurrent headaches and unable to look down to count
fingers. What is the diagnosis?
a. Migraine
b. Cluster headche
c. Brain tumur
d. Stress headache
e. Giant cell arhritis
 

173. A patient gets hurt while playing rugby. He has severe pain. On examination
there is just redness of the pinna and the tympanic membrane is normal. What
treatment will you give?
a. paracetamol
f. Amoxicillin
g. Gentamycin
 
 
 

A woman feel electric shock-like pains in the arm. What medication will you give?
a. Gabapentin
b. Morphine
c. Cotramadol
d. Fentanyl
 
 
174. A 2 years old girl has frequency and pain burning micturation.
She has some supra pubic tenderness.
Which one of the following is the most appropriate initial investigation for this patient?

A. Supra pubic aspiration of urine for culture and sensitivity


B. clean catch of urine for culture and sensitivity
C. USG
D. IVU
E. Micturating cystourethrogram
 
 
175.     An   85   year   old   woman   is   admitted   with   intestinal   obstruction.   Her   hemoglobin  
(Hb)   is   3.5   g/dl,   packed   cell   volume   (PVC)   48%   area   35   mm/dl   sodium   115mmol/l,  
potassium  5  mmol/l.  What  is  the  most  appropriate  initial  management?  
 
A. defer  operation  and  investigation  
B. fresh  frozen  plasma  
C. immediate  blood  transfusion  and  defer  operation  
D. immediate  blood  transfusion.  Stabilize  and  proceed  to  surgery  
E. preoperative  endoscopic  retrograde  cholangio  pancreatography  (ERCP)  
 
176.     A   45   year   old   premenopausal   woman   is   admitted   for   elective   hernia   repair.  
her….Is   8.4   g/dl   mean   cell   haemoglobin   (MDH)   22pg   MCH   concentration   28   g/dl,  
mean   cell   volume   (MCV)   70   her   white   blood   count   (WBC)   is   within   normal   limits.  
What  is  the  most  appropriate  initial  management?  
 
A. preoparative  fluid  load  
B. proceed  with  planned  surgery  
C. vitamin  K  injections  
D. immediate  blood  transfusion.  Stabilize  and  proceed  to  surgery  
E. preoperative  endoscopic  retrograde  cholangio  pancreatography  (ERCP)  
 
177.     A   65   year   old   man   has   haemetemesis.   Endoscopy   shows   a   duodenal   ulcer,  
which  was  injected.  He  has  a  further  haemetemesis  and  his  blood  pressure  drops  to  
100/60  mmHg.  His  Hb  is  6  g/dl.  MCH  25  pg,  MCH  concentration  33  g/dl,  MCV  85ft.  
What  is  the  most  appropriate  initial  management?  
 
A. defer  operation  and  investigation  
B. fresh  frozen  plasma  
C. proceed  with  planned  surgery  
 

D. vitamin  K  injections  
E. immediate  blood  transfusion.  Stabilize  and  proceed  to  surgery  
 
 
178.     A   60   year   old   woman   is   admitted   for   elective   cholecystectomy.   Her  
preoperative   electrocardiogram   (ECG)   shows   rapid,   irregular   rhythm   with   absent  
waves.  What  is  the  most  appropriate  initial  management?  
 
A. preoparative  fluid  load  
B. proceed  with  planned  surgery  
C. vitamin  K  injections  
D. fresh  frozen  plasma  
E. immediate  blood  transfusion  and  defer  operation  

 
 
 179.    A  35  year  old  woman  has  found  a  painless  lump  in  her  right  axilla.  She  has  no  
other  symptoms.  There  is  a  lump  in  the  upper  outer  quadrant  of  the  right  breast  and  
a  smooth  1cm  lump  which  is  mobile  in  the  right  axilla.  What  is  the  most  appropriate  
next  investigation?    
A.  Mammography  
B.  Ultrasound  scan  
C.  Fine  needle  biopsy    
D.  Core  biopsy  
E.  Lymph  nodes    
 
180.      A  70  year  old  woman  comes  to  the  clinic  complaining  of  a  lump  in  the  last  
breast.  There  is  a  4cm  hard,  irregular  lump  which  is  fixed  to  the  skin  and  chest  wall.  A  
mammogram,  ultrasound  scan  and  fine  needle  cytology  have  yielded  equivocal  
results,  but  there  is  a  suspicion  that  it  is  malignant.  What  is  the  most  appropriate  
next  investigation?    
A.  Mammography    
B.  Ultrasound  scan    
                             C.  Fine  needle  biopsy    
D.  Core  biopsy    
E.  Open  biopsy    
 
 
 181.    A  35  year  old  woman  has  found  a  painless  lump  in  her  right  axilla.  She  has  no  
other  symptoms.  There  is  a  lump  in  the  upper  outer  quadrant  of  the  right  breast  and  
a  smooth  1cm  lump  which  is  mobile  in  the  right  axilla.  What  is  the  most  appropriate  
next  investigation?    
 

A.  Mammography  
B.  Ultrasound  scan  
C.  Fine  needle  biopsy    
D.  Core  biopsy  
E.  Lymph  nodes    
 
182.      A  70  year  old  woman  comes  to  the  clinic  complaining  of  a  lump  in  the  last  
breast.  There  is  a  4cm  hard,  irregular  lump  which  is  fixed  to  the  skin  and  chest  wall.  A  
mammogram,  ultrasound  scan  and  fine  needle  cytology  have  yielded  equivocal  
results,  but  there  is  a  suspicion  that  it  is  malignant.  What  is  the  most  appropriate  
next  investigation?    
A.  Mammography    
B.  Ultrasound  scan    
C.  Fine  needle  biopsy    
D.  Core  biopsy    
E.  Open  biopsy    
 
 

183.    A  54  yr  old  man  with  metastatic  liver  disease  develops  exudative  ascites.  
Choose  the  single  most  likely  cause  of  ascites.  
A.  TB  pertionitis  
B.  Primary  hepatoma  
C.  Liver  cirrhosis  
D.  carcinomatosis  peritonei  
E.  Congestive  cardiac  failure  
 
184.    A  57  yr  old  man  has  a  history  of  recurrent  ascites  which  is  exudative  in  nature.  
On  abdominal  paracentesis  numerous  neutrophils  are  found  in  the  ascitic  fluid.  
Choose  the  single  most  likely  cause  of  ascites.  
A.  Liver  cirrhosis  
B.  Hepatocellular  carcinoma  
C.  TB  peritonitis  
D.  Congestive  cardiac  failure  
E.  Bacterial  peritonitis  
 
 
 
 
 
 
 
 

185.    An  immigrant  from  India  has  been  in  the  UK  for  10  yrs  but  regularly  goes  back  
to  her  country  for  summer  holidays.  She  frequently  develops  exudative  ascites,  and  
has  lost  considerable  weight.  Choose  the  single  most  likely  cause  of  ascites.  
A.  Budd-­‐Chiari  syndrome  
B.  Nephrotic  syndrome  
C.  Congestive  cardiac  failure  
D.  TB  peritonitis  
E.  Liver  cirrhosis  
 
186.    A  middle  aged  man  presents  with  bilateral  pitting  pedal  oedema,  ascites,  a  
raised  JVP.  The  ascitic  fluid  is  found  to  be  a  transudate.  Choose  the  single  most  likely  
cause  of  ascites.  
A.  TB  peritonitis  
B.  Congestive  cardiac  failure  
C.  Bacrerial  peritonitis  
D.  Liver  cirrhosis  
E.  Primary  hepatoma                
     
 187.    27  YEAR  old  lady  complains  of  visual  problems  and  weakness  of  both  legs,  she  
had  weakness  in  her  left  arm  3  months  ago  which  resolved.  Fundoscopy  reveals  
optic  atrophy.  What  is  the  diagnosis  

a)  bitemporal  hemianopia  

b)tunnel  vision  

c)increased  size  of  blind  spot  

d)amaurosis  fugax  

e)  cortical  blindness  

188.    A  54  yrs  old  woman  complains  of  coarsening  of  her  facial  features,  sweats  and  
headaches.  She  also  has  a  visual  deficit.  What  is  the  diagnosis  

a)  cortical  blindness  

b)  central  scotoma  

c)  visual  glare  

d)  uniocular  visual  loss  

e)  amaurosis  fugax  

 
 

 189.    A  65  yrs  old  womanis  blind  on  testing  but  denies  that  there  is  a  problem  with  
her  eyesight.  What  is  the  condition  called?  

a)  central  scotoma  

b)cortical  blindness  

c)  hysteria  

d)  uniocular  visual  loss  

e)  increased  size  of  blind  spot  

190.      A  70  yrs  old  man  presents  with  sudden  onset  of  mild  right  sided  weakness  and  
has  noticed  that  he  has  ben  bumping  into  things.  What  is  the  diagnosis?  

a) tunnel  vision  
b) b)  amaurosis  fugax  
c) right  homonymous  hemianopia  
d) uniocular  visual  loss  
e) hysteria  
 

191.      A  22  yrs  old  female  with  6  months  history  of  deteriorating  headaches.  On  
examination  she  is  obese  with  a  swollen  optic  disc.  What  is  the  diagnosis?  

a)  enlarged  blind  spot  

b)  cortical  blindness  

c)  central  scotoma  

d)  uniocular  visual  loss  

e)  right  inferior  quadrantinopia  

192.    A  42  yrs  od  male  with  galactorrhea,  pale  in  complexion  with  fine  wrinkling  of  
the  skin  and  lack  of  body  hair.  Breast  examination  reveals  galactorrhea  on  
expression  

a)  left  homonymous  hemianopia  

b)  left  inferior  quandrantinopia  

c)  enlarged  blindspot  
 

d)  cortical  blindness  

e)  bitemporal  hemianopia  

193.    A  2  yrs  old  female  with  sudden  onset  of  weakness.  Left  sided  weakness  with  
exaggerated  reflexes  on  the  left  arm  and  leg  and  an  extensor  plantar  response  

a)  uniocular  visual  loss  

b)  right  superior  quadrantinopia  

c)  enlarged  blind  spot  

d)  cortical  blindness  

e)  central  scotoma  

193.    A  62  yrs  old  male  presents  following  a  fit.  Ct  scan  reveals  a  right  parietal  mass.  
Where  is  the  lesion?  

a) right  superior  quadrantinopia  


b) right  inferior  quadrantinopia  
c) left  inferior  quadrantinopia  
d) left  superior  quadrantinopia  
e) left  homonymous  quadrantinopia  
 

194.      A  33  yrs  old  female  with  11yr  history  of  episodic  eye  pain,  dysarthria  and  
incordination.  Examinations  reveal  some  dysdiadokinesis,  nystagmus  and  temporal  
pallor  of  optic  disc  

a)  enlarged  blind  spot  

b)  uniocular  visual  loss  

c)  bitemporal  hemianopia  

d)  left  homonymous  hemianopia  

e)  cortical  blindness  

 
 

195.    A  30  yrs  old  woman  has  a  history  ofpalpitation  for  6  months.  Her  resting  ECG  
shows  shortened  PR  interval  and  delta  waves.  Holter  monitor  shows  evidence  of  
paroxysmal  SVT.  How  do  you  manage  

a)carotid  sinus  massage  

b)  adenosine  

c)  sotalol  

d)  digoxin  

e)  verapamil  

196.    A  50  yrs  old  man  was  admitted  with  an  acute  anterior  MI  earlier  in  the  day  2  
hours  after  thrombolysis  with  tpa  was  given  he  suddenly  started  feeling  faint.  His  
pulse  is  140/mins  and  BP  90/40.  Cardiac  monitoring  shows  a  long  run  of  VT  

a)  flecanaide  

b)  disopyramide  

c)  elective  D  conversion  

d)  emergency  DC  conversion  

e)  sotalol  

197.    A  24  yrs  old  woman  presents  to  casualty  complaining  of  dizziness.  Her  ECG  
shows  reentry  tachycardia  and  she  had  a  similar  episode  in  the  past  which  stoped  
spontaneously  and  she  is  on  no  medication.  She  is  31  weeks  pregnant.  

a)  carotid  sinus  massage  

b)  amiodarone  

c)  lignocaine  

d)  flecainide  

e)  sotalol  

198.    A  60  yrs  old  man  has  CPR  which  is  treated  with  CAPD.  He  has  a  low  grade  fever  
and  abdominal  pain  which  lasts  for  2  days.  He  has  noticed  that  the  diasylate  is  cloudy  
 

after  exchange.  He  suddenly  becomes  unwell  with  broad  complex  tachycardia  and  a  
BP  of  80/50  

a)  Elective  DC  

b)  Flecainide  

c)  Emergency  DC  

d)  digoxin  

e)  verapamil  

 
199.      A  20YR  OLD  MAN  HAS  COME  TO  THE  A&E  WITH  HEADACHE,  FEVER  AND  
VOMITTING.  WHAT  IS  THE  MOST  APPROPRIATE  TREATMENT?    
A) CEFTRIAZONE  
B) AMOXICILIN  
C) GENTAMYCIN  
D) CIPROFLOXACIN  
 
 
200.       A   35   YR   OLD   MAN   IS   FOUND   TO   HAVE   A   FIRM   MASS   ON   THE   TESTIS.  
WHAT’S  THE  MOST  APPROPRIATE  INVESTIGATION  
             A)  USS  
               B)  CT  
C) MRI  
D) MSU  
 
GOOD  LUCK  FROM  DR  SAMSON.  
 
 

BONUS  QUESTIONS:  
 
1.     A   5   YEAR   OLD   BOY   PRESENTS   WITH   EARACHE   FOR   2   WEEKS   AND  
TEMPERATURE  OF  39.  HE  HAD  SUDDEN  ONSET  OF  PURULENT  DISCHARGE  AND  
THE  PAIN  HAS  ABORTED.  WHAT  IS  THE  MOST  LIKELY  DIAGNOSIS?  
         ACUTE  OTITIS  MEDIA  
a. EAR  DRUM  PERFORATION  
b. VIRAL  OTITIS  MEDIA  
c. OTITIS  EXTERNA  
d. WARTS  
 
2.   A   45   YR   OLD   WOMAN   ONE   DAY   AFTER   SPLENECTOMY   COMPLAINS   OF  
SHORTNESS  OF  BREATH  AND  FEVER  OF  37.5.  SHE  IS  A  SMOKER  
e. CT    
f. MRI  
 

g. ECHO  
h. CHEST  XRAY  
 
3.     AN   18   YR   OLD   LADY   PRESENT   WITH   RECURRENT   HAY   FEVERS   WHICH  
INTERFERE  WITH  HER  NORMAL  LIFE  
A) ADRENALINE  
B) HYDROCORTISONE  
C)    PREDNISOLONE  
D) CHLOPHENERAMINE  
E) HLORAMPHENICOLA  
 
4. diabetic pattient was on long acting anti diabetic in the morning and
short acting in the evening.after lunch pt gets hypo .how uwill manage
this pt  
A)provide more food at lunch

B)decrease long acting in the morning (answer)

C)decrease short acting in the afternoon

D) decrease both dose

E)reassurance

 5.      pregnant  woman  who  is  in  her  last  trimester  of  pregnancy,  &  her  husband  are  
infected  with  varicella.  What  is  the  single  most  appropriate  management?    

A. Oral  acyliovir  
B. IV  acyclovir  
C. No  treatment  required  at  present  
D. Varicella  1g  
E. Quarantine  

6.      A  boy  who  is  taking  steroids  for  asthma  get  chicken  pox.  What  is  the  single  most  
appropriate  management?  

A. Oral  acyliovir  
B. IV  acyclovir  
C. No  treatment  required  at  present  
D. Varicella  1g  
E. Quarantine  

 7.      A  child  having  lymphoma/leukemia,  his  father  has  shingles.  What  is  the  single  
most  appropriate  management?  

A. Oral  acyliovir  
B. IV  acyclovir  
C. No  treatment  required  at  present  
D. Varicella  1g  
 

E. Quarantine  

8.      A  boy  whose  sister  has  just  had  renal  transplant  &  is  returning  from  hospital.  
What  is  the  single  most  appropriate  management?  

A. Oral  acyliovir  
B. IV  acyclovir  
C. No  treatment  required  at  present  
D. Varicella  1g  
E. Quarantine  

9.      80-­‐year-­‐old  man  develops  ophthalmic  shingles.  What  is  the  single  most  
appropriate  management?  

A. Oral  acyliovir  
B. IV  acyclovir  
C. No  treatment  required  at  present  
D. Varicella  1g  
E. Quarantine  

10.      A  boy  has  developed  vesicles  all  the  over  the  body  &  also  has  high  fever.    What  
is  the  single  most  appropriate  management?  

A. Oral  acyliovir  
B. IV  acyclovir  
C. No  treatment  required  at  present  
D. Varicella  1g  
E. Quarantine  

11.      A  woman  with  vesicles  on  small  part  of  the  chest.  What  is  the  single  most  
appropriate  management?  

A. Oral  acyliovir  
B. IV  acyclovir  
C. No  treatment  required  at  present  
D. Varicella  1g  
E. Quarantine  

12.      A  29-­‐year-­‐old  man  with  human  immunodeficiency  virus  (H/Y),  who  is  taking  
antiretroviral  drugs  and  has  a  falling  T  cell  count,  presents  with  a  two-­‐day  history  or  
herpes  zoster  rash  over  the  C8  dermatome.  What  is  the  single  most  appropriate  
management?  

A. Acyclovir  (intravenous)  
B. Acyclovir  (oral)  
C. Amitryptiline  (oral)  
D. Co-­‐amoxiclav  (oral)  
 

E. Dihydrocodeine  (oral)  
 

13.    A  19-­‐year-­‐old  woman  presents  with  a  herpes  zoster  rash  of  the  L4  dermatome.  
The  rash  is  itchy  but  not  painful.  What  is  the  single  most  appropriate  management?  

A. Acyclovir  (intravenous)  
B. Acyclovir  (oral)  
C. Amitryptiline  (oral)  
D. Co-­‐amoxiclav  (oral)  
E. Dihydrocodeine  (oral)  
 

14.      A  59-­‐year-­‐old  man  present  with  a  herpes  zoster  rash  affecting  all  three  divisions  
of  the  trigeminal  nerve.  He  has  severe  conjunctivitis  and  cannot  close  his  eyes.    What  
is  the  single  most  appropriate  management?  

A. Inoxuridine  paint  
B. Paracetamol  (oral)  
C. Prednisolone  (oral)  
D. Symptomatic  treatment  
E. Urgent  referral  to  ophthamologist  

15.    An  80-­‐year-­‐old  woman  presents  with  a  painful  herpes  zoster  rash,  which  
appeared,  yesterday  on  her  truck.  What  is  the  single  most  appropriate?  

A. Acyclovir  (intravenous)  
B. Acyclovir  (oral)  
C. Prednisolone  (oral)  
D. Symptomatic  treatment  
E. Urgent  referral  to  ophthamologist  

 16.      A  55-­‐year-­‐old  woman  present  with  severe  post  herpetic  neuralgia  following  an  
episode  of  ophthalmic  herpes,  which  occurred  six  months  ago.  Co-­‐codamol  tablets  
are  not  relieving  the  pain.  She  is  desperate  for  something  to  control  the  pain  at  night.  
What  is  the  single  most  appropriate  management?  

A. Acyclovir  (intravenous)  
B. Acyclovir  (oral)  
C. Amitryptiline  (oral)  
D. Co-­‐amoxiclav  (oral)  
E. Dihydrocodeine  (oral)  
 
 
17.      A  20  year  old  female  presents  with  palpitations.  On  examination,  there  is  no    
           abnormality  and  pulse  is  regular.  What  is  the  singe  most  diagnosis?  
a. Sinus  tachycardia  
 

b. VT  
c. SVT  
d. AF  
e. Atrial  flutter  
 
   18.      A  28  year  old  man  presents  with  abdominal  distension  and  pain.  His  stools  
have    
                   been  mucoid  and  sometimes  blood  stained.  What  is  the  most  appropriate    
                   investigation?  
a.      Barium  meal  
b.      Gastroscopy  
c.      IgA  tissue  transaminase  
d.      Jejunal  biopsy  
e.      Stoll  culture  and  sensitivity  
19.    A  2  week  s  old  babay  has  been  diagnosed  with  breast  milk  jaundice.  
What  would  the  blood  test  investigation  show?  
A. Bilirubin  40,  conjugated  less  5.  
B. Bilirubin  40  conjugated  <10  
C. Bilirubin  40,  unconjugated………  
D. Bilirubin  40,  …..  
E.  Bilirubin……  
 
 
20.      A  34  year  old  man  presents  with  right  upper  quadrant  
pain  and  was  later  found  to  have  acute  hepatitis.  What  
would  be  the  values  of  liver  function  tests?  
A. AST…..  
B. ALT…….  
C. Bilirubin…..400  
D. Alkaline  phosphatase  
E. AST  
 
21.    A  65  year  old  man  who  is  an  intravenous  drug  abuser  presents  with  weight  
loss  and  macopapular  rash.  What  is  the  single  most  likely  cause?  
a. Cryptococcus  infection  
b. HHV  8  
c. Pneumocystic  carinii  
d. Human  papilloma  virus  
e. Cryptosporidium  
 
22.    A  27-­‐year-­‐old  woman  at  34  weeks  gestation  in  her  first  pregnancy  attends  
ANC.  Her  blood  results  showed  Hb:  10.6,  MCV:  95,  MCHC:  350.  
What  should  you  do  for  her?  
 

a. Folate  
b. Dextran  
c. Ferrous  sulphate  
d. None  
 
 
23.    A  smoker  with  several  ulcers  in  the  mouth  and  lower  surface  of  the  tongue  
and  also  with  white  striae  on  the  mouth.  
a. Apthous  ulcer  
b. Lichen  planus  
c. Kaposi  sarcoma  
d. Cancer  of  the  mouth  
 
24.     A   35-­‐year-­‐old   man   skidded   on   a   wet   road   while   riding   his   motorbike   at   a  
speed   of   70mph.   He   has   a   large   hematoma   on   the   temporal   scalp,   some   bruises  
on  the  chest  wall  and  abdomen  and  a  deformed  thigh.  His  GCS  is  11/15.  He  has  
been  given  high  flow  oxygen  via  facemask.  Which  is  the  single  most  immediate  
radiological  investigation  required  during  the  initial  resuscitation  phase?  
a. CT  abdomen  
b. CT  brain  
c. X-­‐ray  of  the  chest  
d. X-­‐ray  of  the  abdomen  
e. X-­‐ray  of  the  femur  
 
25.    A  22  year  old  woman  says  she  has  taken  about  40  tablets  of  paracetamol  3  
hours  ago.  Her  heart  rate  is  110beats/minute,  BP  110/80mmHg  and  RR  22c/m.  
Which  is  the  single  most  appropriate  initial  management?  
a. Give  activated  charcoal  
b. Give  N-­‐acetylcysteine  
c. Induce  vomiting  
d. Perform  gastric  lavage  
e. Wait  for  the  4  hour  paracetamol  level  
 
 
26.    Which  of  the  following  is  the  side  effect  of  lithium  in  therapeutic  levels?  
A. corse  tremor  
B. Thirst  
C. Renal  failure  
D. Hyponatremia  

 
 

 
 

 
 
 
 

SAMSONPLAB  ACADEMY  LIMITED  BOW  BUSSINESS  


CENTRE  
BOW  ROAD  153-­‐-­‐159  
E3  2SE  
EMAIL:  info@samsonplab.co.uk  
Mobile:  07940433068  
PLAB  1  MOCK  7  JUNE  2013  ANSWERS
 
 

 
 
 

 
 
 
 

 
 
 

 
 
 

 
 

1.A     30.C     59  .A     88.A     117.C     144.A    


2.C     31.C     60.D     89.D     118.A     145.D  

3.C     32.D     61.A     90.C     119.E     146.  


4.B     33.B     62.D     91.D     120.A     147.B  
5.B     34.A     63.A     92.C     121.E     148.A  
6.B     35.B     64.A     93.D     122.D     149.C  

7.A     36.C     65.C     94.D     123.C     150.A  


8.D     37.A     66.D     95.C     124.A     151.B  
9.B     38.C     67.B     96.D     125.B     152.D  

10.B     39.C     68.D     97.D     126.D     153.A  


11.C     40.B     69.A     98.C     127.D     154.B  

12.D     41.C     70.B     99.C     128.C     155.B  


13.C     42.C     71.B     100.D     129.D     156.D  

14.E     43.D     72.A     101.A     130.B     157.E  


15.C     44.E     73.D     102C     131.D     158.B  
16.C     45.C     74.C     103.B     132.D     159.C  

17.C     46.C     75.D     104.C     133.E     160.D  


18.A     47.B     76.B     105.A     134.B     161.A  
19.E     48.B     77.B     106.A     135.C     162.D  
20.E     49.C     78.C     107.B     136.C     163.C  

21.  A     50.D     79.E     108.B     137.A     164.A  


22.C     51.A     80.B     109.D     138.B     165.  
23.B     52.C     81.B     110.  ECHO   137.C     166.C  

24.F     53.B     82.D     111.D     138.B     169.B  


25.D     54.B     83.     112.C     139.C     170.B  
26.C     55.A     84.C     113.B     140.E     171.E  
27.E     56.A     85.B     114.C     141.B   172.C    172.  A  

28.E     57.B     86.B     115.B     142.C   173.A  173.  A  

29.B     58.E     87.C     116.C     143.B     174.B  


 

175.D  
175.  

176.  DEFER  +  INVESTIGATE  


177.  E  
178.  DEFER  +  INVESTIGATE  
179.A  

180.D  
181.A  
182.D  

183.D  
184.E  

185.D  
186.B  

187.  UNIOCULAR  VISUAL  LOSS  


188.  BITEMPORAL  HEMIANOPIA  
189.B  

190.C  
191.A  
192.E  
193.  HOMONYMOUS  HEMIANOPIA  

194.B  
195.B  
196.D  

197.A  
198.C  
199.A  
200.A    

 
 

BONUS  QUESTIONS  
1.A  
2.H  
3.D  
4.B  
5.D  
6.B  
7.D  
8.E  
9.B  
10.C  
11.A  
12.A  
13.B  
14.E  
15.A  
16.C  
17.C  
18.ULCERATIVE  COLITIS  
19.NO  ANSWER  
20.NO  ANSWER  
21.B  
22.C  
23.A  
24.B  
25.B  
 

26.B  
 
 

 
 
 
 

 
 
 

 
 
 
 

 
 
 

 
 
 
 

 
 
 

 
 
 

Potrebbero piacerti anche